Feat: genere DS pour les TST
All checks were successful
continuous-integration/drone/push Build is passing
All checks were successful
continuous-integration/drone/push Build is passing
This commit is contained in:
parent
615ddbc735
commit
5b0de2d51d
262
TST/DS/DS_21_04_07/TST1/01_210407_DS8.tex
Normal file
262
TST/DS/DS_21_04_07/TST1/01_210407_DS8.tex
Normal file
@ -0,0 +1,262 @@
|
||||
\documentclass[a4paper,10pt]{article}
|
||||
\usepackage{myXsim}
|
||||
|
||||
% Title Page
|
||||
\title{DS8 \hfill AIOUAZ Ahmed}
|
||||
\tribe{TST}
|
||||
\date{\hfillÀ render pour le Mercredi 7 avril}
|
||||
|
||||
\xsimsetup{
|
||||
solution/print = false
|
||||
}
|
||||
|
||||
\begin{document}
|
||||
\maketitle
|
||||
|
||||
\begin{exercise}[subtitle={Automatismes}]
|
||||
\textit{Toutes les questions de cette exercice sont indépendantes et peuvent être répondus séparément}
|
||||
\begin{enumerate}
|
||||
\item De janvier à septembre, une quantité a augmenté de $20\,\%$. Faire un schéma pour représenter la situation puis calculer le taux d'évolution moyen mensuel.
|
||||
\item Une quantité augmente de $20\,\%$ par ans. En 2020, elle est de 110\euro. Quelle était sa valeur en 2019? Faire un schéma pour représenter la situation.
|
||||
\item Déterminer l'équation de la droite \\
|
||||
\begin{tikzpicture}[xscale=0.8, yscale=0.5]
|
||||
\tkzInit[xmin=-5,xmax=5,xstep=1,
|
||||
ymin=-5,ymax=5,ystep=1]
|
||||
\tkzGrid
|
||||
\tkzAxeXY
|
||||
\tkzFct[domain=-5:5,color=red,very thick]%
|
||||
{3.0*\x -3};
|
||||
\end{tikzpicture}
|
||||
\item Résoudre l'équation $2 \times 0.07^x = 20$
|
||||
\end{enumerate}
|
||||
\end{exercise}
|
||||
|
||||
\begin{solution}
|
||||
\begin{enumerate}
|
||||
\item On veut partager cette évolution en 8 évolutions.
|
||||
\[
|
||||
\left(1 + \frac{20}{100}\right)^{\frac{1}{8}} = 1.0231
|
||||
\]
|
||||
Donc le taux d'évolution moyen est
|
||||
\[
|
||||
t_m = 1.0231 - 1 = 0.0230999999999999
|
||||
\]
|
||||
\item Coefficient multiplicateur pour revenir en arrière
|
||||
\[
|
||||
CM = (1 + \frac{20}{100})^{-1} = 0.8333
|
||||
\]
|
||||
On en déduit la quantité en 2019
|
||||
\[
|
||||
110 * 0.8333 = 91.66300000000001
|
||||
\]
|
||||
\item L'équation de la droite est
|
||||
\[
|
||||
y = 3.0 x -3
|
||||
\]
|
||||
\item Il faut penser à faire la division à par $2$ avant d'utiliser le log car sinon, on ne peut pas utiliser la formule $\log(a^n) = n\times \log(a)$.
|
||||
|
||||
\[x = \frac{\log(10.0)}{\log(0.07)}\]
|
||||
\end{enumerate}
|
||||
\end{solution}
|
||||
|
||||
\begin{exercise}[subtitle={Restaurant}]
|
||||
Un \emph{food truck}, ouvert le midi et le soir, propose deux types de formules :
|
||||
|
||||
\setlength\parindent{10mm}
|
||||
\begin{itemize}
|
||||
\item la formule \emph{Burger} ;
|
||||
\item la formule \emph{Wok}.
|
||||
\end{itemize}
|
||||
\setlength\parindent{0mm}
|
||||
|
||||
\medskip
|
||||
|
||||
Le gérant a remarqué que 9\,\% de ses ventes ont lieu le midi. Le quart des ventes du midi correspondent à la formule \emph{Burger}, alors que 3\,\% des ventes du soir correspondent à la formule \emph{Wok}.
|
||||
|
||||
Le gérant se constitue un fichier en notant, pour chaque vente, la formule choisie et le moment de cette vente (midi ou soir).
|
||||
|
||||
On prélève une fiche de façon équiprobable. On définit les quatre évènements suivants:
|
||||
|
||||
\begin{enumerate}
|
||||
\item $M$ : \og la fiche correspond à une vente du midi\fg{} ;
|
||||
\item $S$ : \og la fiche correspond à une vente du soir\fg {};
|
||||
\item $W$ : \og la fiche correspond à une formule \emph{Wok} \fg{} ;
|
||||
\item $B$ : \og la fiche correspond à une formule \emph{Burger} \fg.
|
||||
\end{enumerate}
|
||||
\setlength\parindent{0mm}
|
||||
|
||||
\medskip
|
||||
|
||||
\begin{enumerate}
|
||||
\item Recopier puis compléter l'arbre pondéré
|
||||
|
||||
\begin{center}
|
||||
\begin{tikzpicture}[sloped]
|
||||
\node {.}
|
||||
child {node {$M$}
|
||||
child {node {$W$}
|
||||
edge from parent
|
||||
node[above] {...}
|
||||
}
|
||||
child {node {$B$}
|
||||
edge from parent
|
||||
node[above] {...}
|
||||
}
|
||||
edge from parent
|
||||
node[above] {...}
|
||||
}
|
||||
child[missing] {}
|
||||
child { node {$S$}
|
||||
child {node {$W$}
|
||||
edge from parent
|
||||
node[above] {...}
|
||||
}
|
||||
child {node {$B$}
|
||||
edge from parent
|
||||
node[above] {...}
|
||||
}
|
||||
edge from parent
|
||||
node[above] {...}
|
||||
} ;
|
||||
\end{tikzpicture}
|
||||
\end{center}
|
||||
|
||||
\item Calculer la probabilité de l'évènement $M \cap W$. Interpréter ce résultat dans le contexte de l'exercice.
|
||||
\item Montrer que la probabilité que la fiche choisie corresponde à une formule \emph{Burger} est égale à $0.9052$.
|
||||
\item On a prélevé une fiche correspondant à la formule \emph{Burger}. Quelle est la probabilité, arrondie au millième, que la vente ait eu lieu le soir?
|
||||
\end{enumerate}
|
||||
\end{exercise}
|
||||
|
||||
\begin{solution}
|
||||
\begin{enumerate}
|
||||
\item
|
||||
\begin{center}
|
||||
\begin{tikzpicture}[sloped]
|
||||
\node {.}
|
||||
child {node {$M$}
|
||||
child {node {$W$}
|
||||
edge from parent
|
||||
node[above] {$0.75$}
|
||||
}
|
||||
child {node {$B$}
|
||||
edge from parent
|
||||
node[above] {$0.25$}
|
||||
}
|
||||
edge from parent
|
||||
node[above] {$0.09$}
|
||||
}
|
||||
child[missing] {}
|
||||
child { node {$S$}
|
||||
child {node {$W$}
|
||||
edge from parent
|
||||
node[above] {$0.03$}
|
||||
}
|
||||
child {node {$B$}
|
||||
edge from parent
|
||||
node[above] {$0.97$}
|
||||
}
|
||||
edge from parent
|
||||
node[above] {$0.91$}
|
||||
} ;
|
||||
\end{tikzpicture}
|
||||
\end{center}
|
||||
\item On calcule la probabilité que la vente soit un wok et ait eu lieu à midi
|
||||
\[ P(M\cap W) = P(M) \times P_M(W) = 0.09 \times 0.75 = 0.0675 \]
|
||||
\item Probabilité que la vente soit un burger.
|
||||
\[
|
||||
P(B) = P(M\cap B) + P(S\cap B) = 0.09 \times 0.75 + 0.91 \times 0.03 = 0.9052
|
||||
\]
|
||||
\item On cherche à calculer la quantité $P_B(S)$. Pour cela on utilise la formule de Bayes
|
||||
\[
|
||||
P_B(S) = \frac{P(B\cap S)}{P(B)} = \frac{P_S(B) \times P(S)}{P(B)} = \frac{0.97\times 0.91}{0.9052} = 0.975143614670791 \approx 0.975
|
||||
\]
|
||||
\end{enumerate}
|
||||
\end{solution}
|
||||
|
||||
\begin{exercise}[subtitle={Continent plastique}]
|
||||
\textit{Les quantités évoqués dans cette exercice sont générés au hasard et sont donc complètement farfelus.}
|
||||
\medskip
|
||||
Le \og continent de plastique\fg{} est la plus grande des plaques de déchets plastiques évoluant sur les océans. Elle occupe actuellement dans l'océan Pacifique une surface dont l'aire est évaluée à plus de $1,6$ million de km$^2$, entre Hawaï et la Californie.
|
||||
|
||||
En 2017, des scientifiques ont estimé qu'il y avait $14$ millions de tonnes de déchets plastiques qui était déversé chaque année dans les océans et que cette quantité augmentait de $28\n\%$ par chaque année.
|
||||
|
||||
On modélise l'évolution de la masse de ces déchets plastiques déversée chaque année, si rien n'est fait pour la réduire, par une suite géométrique $\left(u_n\right)$. L'arrondi au centième du terme $u_n$ représente la masse de ces déchets déversée chaque année, exprimée en million de tonnes, pour l'année $(2017 + n)$.
|
||||
|
||||
\medskip
|
||||
|
||||
\begin{enumerate}
|
||||
\item Expliquer pourquoi la suite $u_n$ est géométrique?
|
||||
\item Calculer $u_1$ et $u_2$.
|
||||
\item Exprimer $u_n$ en fonction de $n$.
|
||||
\item Au début de l'année 2017, il y avait $300$ millions de tonnes de déchets plastique. Calculer la quantité totale de déchets plastiques en 2030.
|
||||
\item On souhaite déterminer en quelle année la masse totale de ces déchets plastiques aura pour la première fois augmenté de $50$\,\% par rapport à sa valeur de 2017.
|
||||
\begin{enumerate}
|
||||
\item Recopier et compléter l'algorithme ci-dessous pour que la variable $N$ contienne la réponse au problème posé.
|
||||
|
||||
\begin{center}
|
||||
\begin{tabularx}{0.4\linewidth}{|X|}\hline
|
||||
$N = 2017$\\
|
||||
$U = 14$ \\
|
||||
$S = 300 + U$ \\
|
||||
while $S < 450$: \\
|
||||
\hspace{1cm} $N = \ldots$\\
|
||||
\hspace{1cm} $U = \ldots$\\
|
||||
\hspace{1cm} $S = \ldots$\\
|
||||
\hline
|
||||
\end{tabularx}
|
||||
\end{center}
|
||||
\item Que contiennent les variables $S$, $U$ et $N$ après exécution de cet algorithme ?
|
||||
|
||||
Interpréter les résultats dans le contexte de l'exercice.
|
||||
\end{enumerate}
|
||||
\end{enumerate}
|
||||
\end{exercise}
|
||||
|
||||
\begin{solution}
|
||||
\begin{enumerate}
|
||||
\item Une augmentation de $28\,\%$ revient à multiplier la quantité par $1.28$. La suite est donc bien géométrique. Son premier terme est $u_0 = 14$ et sa raison est $q = 1.28$
|
||||
\item
|
||||
\[
|
||||
u_1 = u_0 * 1.28 = 17.92
|
||||
\]
|
||||
\[
|
||||
u_2 = u_0 * 1.28^2 = 22.9376
|
||||
\]
|
||||
\item
|
||||
\[
|
||||
u_n = u_0 \times q^n = 14 \times 1.28^n
|
||||
\]
|
||||
\item On calcule la quantité totale déversée entre 2017 et 2030.
|
||||
\[
|
||||
\sum_{n = 0}^{13} u_n = u_0 \times \frac{1-q^{13}}{1-q} = 14 \times \frac{1 - 1.28^{13}}{1 - 1.28} = 1187.94
|
||||
\]
|
||||
On en déduit la quantité totale de déchets en 2030
|
||||
\[
|
||||
300 + 1187.94 = 1487.94
|
||||
\]
|
||||
\item
|
||||
\begin{enumerate}
|
||||
\item ~
|
||||
\begin{center}
|
||||
\begin{tabularx}{0.4\linewidth}{|X|}\hline
|
||||
$N \gets 2017$\\
|
||||
$U \gets 14$ \\
|
||||
$S \gets 300 + U$ \\
|
||||
Tant que $S < 450$ \\
|
||||
\hspace{1cm} $N \gets N + 1$\\
|
||||
\hspace{1cm} $U \gets U * 1.28$\\
|
||||
\hspace{1cm} $S \gets S + u$\\
|
||||
Fin Tant que\\\hline
|
||||
\end{tabularx}
|
||||
\end{center}
|
||||
\item \textit{Pas de correction automatisé}
|
||||
\end{enumerate}
|
||||
\end{enumerate}
|
||||
\end{solution}
|
||||
|
||||
\end{document}
|
||||
|
||||
%%% Local Variables:
|
||||
%%% mode: latex
|
||||
%%% TeX-master: "master"
|
||||
%%% End:
|
262
TST/DS/DS_21_04_07/TST1/02_210407_DS8.tex
Normal file
262
TST/DS/DS_21_04_07/TST1/02_210407_DS8.tex
Normal file
@ -0,0 +1,262 @@
|
||||
\documentclass[a4paper,10pt]{article}
|
||||
\usepackage{myXsim}
|
||||
|
||||
% Title Page
|
||||
\title{DS8 \hfill BAHBAH Zakaria}
|
||||
\tribe{TST}
|
||||
\date{\hfillÀ render pour le Mercredi 7 avril}
|
||||
|
||||
\xsimsetup{
|
||||
solution/print = false
|
||||
}
|
||||
|
||||
\begin{document}
|
||||
\maketitle
|
||||
|
||||
\begin{exercise}[subtitle={Automatismes}]
|
||||
\textit{Toutes les questions de cette exercice sont indépendantes et peuvent être répondus séparément}
|
||||
\begin{enumerate}
|
||||
\item De janvier à septembre, une quantité a augmenté de $28\,\%$. Faire un schéma pour représenter la situation puis calculer le taux d'évolution moyen mensuel.
|
||||
\item Une quantité augmente de $28\,\%$ par ans. En 2020, elle est de 138\euro. Quelle était sa valeur en 2019? Faire un schéma pour représenter la situation.
|
||||
\item Déterminer l'équation de la droite \\
|
||||
\begin{tikzpicture}[xscale=0.8, yscale=0.5]
|
||||
\tkzInit[xmin=-5,xmax=5,xstep=1,
|
||||
ymin=-5,ymax=5,ystep=1]
|
||||
\tkzGrid
|
||||
\tkzAxeXY
|
||||
\tkzFct[domain=-5:5,color=red,very thick]%
|
||||
{1.0*\x -2};
|
||||
\end{tikzpicture}
|
||||
\item Résoudre l'équation $7 \times 0.54^x = 18$
|
||||
\end{enumerate}
|
||||
\end{exercise}
|
||||
|
||||
\begin{solution}
|
||||
\begin{enumerate}
|
||||
\item On veut partager cette évolution en 8 évolutions.
|
||||
\[
|
||||
\left(1 + \frac{28}{100}\right)^{\frac{1}{8}} = 1.0313
|
||||
\]
|
||||
Donc le taux d'évolution moyen est
|
||||
\[
|
||||
t_m = 1.0313 - 1 = 0.031300000000000106
|
||||
\]
|
||||
\item Coefficient multiplicateur pour revenir en arrière
|
||||
\[
|
||||
CM = (1 + \frac{28}{100})^{-1} = 0.7812
|
||||
\]
|
||||
On en déduit la quantité en 2019
|
||||
\[
|
||||
138 * 0.7812 = 107.8056
|
||||
\]
|
||||
\item L'équation de la droite est
|
||||
\[
|
||||
y = 1.0 x -2
|
||||
\]
|
||||
\item Il faut penser à faire la division à par $7$ avant d'utiliser le log car sinon, on ne peut pas utiliser la formule $\log(a^n) = n\times \log(a)$.
|
||||
|
||||
\[x = \frac{\log(2.57)}{\log(0.54)}\]
|
||||
\end{enumerate}
|
||||
\end{solution}
|
||||
|
||||
\begin{exercise}[subtitle={Restaurant}]
|
||||
Un \emph{food truck}, ouvert le midi et le soir, propose deux types de formules :
|
||||
|
||||
\setlength\parindent{10mm}
|
||||
\begin{itemize}
|
||||
\item la formule \emph{Burger} ;
|
||||
\item la formule \emph{Wok}.
|
||||
\end{itemize}
|
||||
\setlength\parindent{0mm}
|
||||
|
||||
\medskip
|
||||
|
||||
Le gérant a remarqué que 57\,\% de ses ventes ont lieu le midi. Le quart des ventes du midi correspondent à la formule \emph{Burger}, alors que 49\,\% des ventes du soir correspondent à la formule \emph{Wok}.
|
||||
|
||||
Le gérant se constitue un fichier en notant, pour chaque vente, la formule choisie et le moment de cette vente (midi ou soir).
|
||||
|
||||
On prélève une fiche de façon équiprobable. On définit les quatre évènements suivants:
|
||||
|
||||
\begin{enumerate}
|
||||
\item $M$ : \og la fiche correspond à une vente du midi\fg{} ;
|
||||
\item $S$ : \og la fiche correspond à une vente du soir\fg {};
|
||||
\item $W$ : \og la fiche correspond à une formule \emph{Wok} \fg{} ;
|
||||
\item $B$ : \og la fiche correspond à une formule \emph{Burger} \fg.
|
||||
\end{enumerate}
|
||||
\setlength\parindent{0mm}
|
||||
|
||||
\medskip
|
||||
|
||||
\begin{enumerate}
|
||||
\item Recopier puis compléter l'arbre pondéré
|
||||
|
||||
\begin{center}
|
||||
\begin{tikzpicture}[sloped]
|
||||
\node {.}
|
||||
child {node {$M$}
|
||||
child {node {$W$}
|
||||
edge from parent
|
||||
node[above] {...}
|
||||
}
|
||||
child {node {$B$}
|
||||
edge from parent
|
||||
node[above] {...}
|
||||
}
|
||||
edge from parent
|
||||
node[above] {...}
|
||||
}
|
||||
child[missing] {}
|
||||
child { node {$S$}
|
||||
child {node {$W$}
|
||||
edge from parent
|
||||
node[above] {...}
|
||||
}
|
||||
child {node {$B$}
|
||||
edge from parent
|
||||
node[above] {...}
|
||||
}
|
||||
edge from parent
|
||||
node[above] {...}
|
||||
} ;
|
||||
\end{tikzpicture}
|
||||
\end{center}
|
||||
|
||||
\item Calculer la probabilité de l'évènement $M \cap W$. Interpréter ce résultat dans le contexte de l'exercice.
|
||||
\item Montrer que la probabilité que la fiche choisie corresponde à une formule \emph{Burger} est égale à $0.3592$.
|
||||
\item On a prélevé une fiche correspondant à la formule \emph{Burger}. Quelle est la probabilité, arrondie au millième, que la vente ait eu lieu le soir?
|
||||
\end{enumerate}
|
||||
\end{exercise}
|
||||
|
||||
\begin{solution}
|
||||
\begin{enumerate}
|
||||
\item
|
||||
\begin{center}
|
||||
\begin{tikzpicture}[sloped]
|
||||
\node {.}
|
||||
child {node {$M$}
|
||||
child {node {$W$}
|
||||
edge from parent
|
||||
node[above] {$0.75$}
|
||||
}
|
||||
child {node {$B$}
|
||||
edge from parent
|
||||
node[above] {$0.25$}
|
||||
}
|
||||
edge from parent
|
||||
node[above] {$0.58$}
|
||||
}
|
||||
child[missing] {}
|
||||
child { node {$S$}
|
||||
child {node {$W$}
|
||||
edge from parent
|
||||
node[above] {$0.49$}
|
||||
}
|
||||
child {node {$B$}
|
||||
edge from parent
|
||||
node[above] {$0.51$}
|
||||
}
|
||||
edge from parent
|
||||
node[above] {$0.42$}
|
||||
} ;
|
||||
\end{tikzpicture}
|
||||
\end{center}
|
||||
\item On calcule la probabilité que la vente soit un wok et ait eu lieu à midi
|
||||
\[ P(M\cap W) = P(M) \times P_M(W) = 0.58 \times 0.75 = 0.435 \]
|
||||
\item Probabilité que la vente soit un burger.
|
||||
\[
|
||||
P(B) = P(M\cap B) + P(S\cap B) = 0.58 \times 0.75 + 0.42 \times 0.49 = 0.3592
|
||||
\]
|
||||
\item On cherche à calculer la quantité $P_B(S)$. Pour cela on utilise la formule de Bayes
|
||||
\[
|
||||
P_B(S) = \frac{P(B\cap S)}{P(B)} = \frac{P_S(B) \times P(S)}{P(B)} = \frac{0.51\times 0.42}{0.3592} = 0.5963251670378619 \approx 0.596
|
||||
\]
|
||||
\end{enumerate}
|
||||
\end{solution}
|
||||
|
||||
\begin{exercise}[subtitle={Continent plastique}]
|
||||
\textit{Les quantités évoqués dans cette exercice sont générés au hasard et sont donc complètement farfelus.}
|
||||
\medskip
|
||||
Le \og continent de plastique\fg{} est la plus grande des plaques de déchets plastiques évoluant sur les océans. Elle occupe actuellement dans l'océan Pacifique une surface dont l'aire est évaluée à plus de $1,6$ million de km$^2$, entre Hawaï et la Californie.
|
||||
|
||||
En 2017, des scientifiques ont estimé qu'il y avait $9$ millions de tonnes de déchets plastiques qui était déversé chaque année dans les océans et que cette quantité augmentait de $26\n\%$ par chaque année.
|
||||
|
||||
On modélise l'évolution de la masse de ces déchets plastiques déversée chaque année, si rien n'est fait pour la réduire, par une suite géométrique $\left(u_n\right)$. L'arrondi au centième du terme $u_n$ représente la masse de ces déchets déversée chaque année, exprimée en million de tonnes, pour l'année $(2017 + n)$.
|
||||
|
||||
\medskip
|
||||
|
||||
\begin{enumerate}
|
||||
\item Expliquer pourquoi la suite $u_n$ est géométrique?
|
||||
\item Calculer $u_1$ et $u_2$.
|
||||
\item Exprimer $u_n$ en fonction de $n$.
|
||||
\item Au début de l'année 2017, il y avait $300$ millions de tonnes de déchets plastique. Calculer la quantité totale de déchets plastiques en 2030.
|
||||
\item On souhaite déterminer en quelle année la masse totale de ces déchets plastiques aura pour la première fois augmenté de $50$\,\% par rapport à sa valeur de 2017.
|
||||
\begin{enumerate}
|
||||
\item Recopier et compléter l'algorithme ci-dessous pour que la variable $N$ contienne la réponse au problème posé.
|
||||
|
||||
\begin{center}
|
||||
\begin{tabularx}{0.4\linewidth}{|X|}\hline
|
||||
$N = 2017$\\
|
||||
$U = 9$ \\
|
||||
$S = 300 + U$ \\
|
||||
while $S < 450$: \\
|
||||
\hspace{1cm} $N = \ldots$\\
|
||||
\hspace{1cm} $U = \ldots$\\
|
||||
\hspace{1cm} $S = \ldots$\\
|
||||
\hline
|
||||
\end{tabularx}
|
||||
\end{center}
|
||||
\item Que contiennent les variables $S$, $U$ et $N$ après exécution de cet algorithme ?
|
||||
|
||||
Interpréter les résultats dans le contexte de l'exercice.
|
||||
\end{enumerate}
|
||||
\end{enumerate}
|
||||
\end{exercise}
|
||||
|
||||
\begin{solution}
|
||||
\begin{enumerate}
|
||||
\item Une augmentation de $26\,\%$ revient à multiplier la quantité par $1.26$. La suite est donc bien géométrique. Son premier terme est $u_0 = 9$ et sa raison est $q = 1.26$
|
||||
\item
|
||||
\[
|
||||
u_1 = u_0 * 1.26 = 11.34
|
||||
\]
|
||||
\[
|
||||
u_2 = u_0 * 1.26^2 = 14.2884
|
||||
\]
|
||||
\item
|
||||
\[
|
||||
u_n = u_0 \times q^n = 9 \times 1.26^n
|
||||
\]
|
||||
\item On calcule la quantité totale déversée entre 2017 et 2030.
|
||||
\[
|
||||
\sum_{n = 0}^{13} u_n = u_0 \times \frac{1-q^{13}}{1-q} = 9 \times \frac{1 - 1.26^{13}}{1 - 1.26} = 663.76
|
||||
\]
|
||||
On en déduit la quantité totale de déchets en 2030
|
||||
\[
|
||||
300 + 663.76 = 963.76
|
||||
\]
|
||||
\item
|
||||
\begin{enumerate}
|
||||
\item ~
|
||||
\begin{center}
|
||||
\begin{tabularx}{0.4\linewidth}{|X|}\hline
|
||||
$N \gets 2017$\\
|
||||
$U \gets 9$ \\
|
||||
$S \gets 300 + U$ \\
|
||||
Tant que $S < 450$ \\
|
||||
\hspace{1cm} $N \gets N + 1$\\
|
||||
\hspace{1cm} $U \gets U * 1.26$\\
|
||||
\hspace{1cm} $S \gets S + u$\\
|
||||
Fin Tant que\\\hline
|
||||
\end{tabularx}
|
||||
\end{center}
|
||||
\item \textit{Pas de correction automatisé}
|
||||
\end{enumerate}
|
||||
\end{enumerate}
|
||||
\end{solution}
|
||||
|
||||
\end{document}
|
||||
|
||||
%%% Local Variables:
|
||||
%%% mode: latex
|
||||
%%% TeX-master: "master"
|
||||
%%% End:
|
262
TST/DS/DS_21_04_07/TST1/03_210407_DS8.tex
Normal file
262
TST/DS/DS_21_04_07/TST1/03_210407_DS8.tex
Normal file
@ -0,0 +1,262 @@
|
||||
\documentclass[a4paper,10pt]{article}
|
||||
\usepackage{myXsim}
|
||||
|
||||
% Title Page
|
||||
\title{DS8 \hfill BALLOFFET Kenza}
|
||||
\tribe{TST}
|
||||
\date{\hfillÀ render pour le Mercredi 7 avril}
|
||||
|
||||
\xsimsetup{
|
||||
solution/print = false
|
||||
}
|
||||
|
||||
\begin{document}
|
||||
\maketitle
|
||||
|
||||
\begin{exercise}[subtitle={Automatismes}]
|
||||
\textit{Toutes les questions de cette exercice sont indépendantes et peuvent être répondus séparément}
|
||||
\begin{enumerate}
|
||||
\item De janvier à septembre, une quantité a augmenté de $12\,\%$. Faire un schéma pour représenter la situation puis calculer le taux d'évolution moyen mensuel.
|
||||
\item Une quantité augmente de $12\,\%$ par ans. En 2020, elle est de 126\euro. Quelle était sa valeur en 2019? Faire un schéma pour représenter la situation.
|
||||
\item Déterminer l'équation de la droite \\
|
||||
\begin{tikzpicture}[xscale=0.8, yscale=0.5]
|
||||
\tkzInit[xmin=-5,xmax=5,xstep=1,
|
||||
ymin=-5,ymax=5,ystep=1]
|
||||
\tkzGrid
|
||||
\tkzAxeXY
|
||||
\tkzFct[domain=-5:5,color=red,very thick]%
|
||||
{2.6666666666666665*\x -4};
|
||||
\end{tikzpicture}
|
||||
\item Résoudre l'équation $8 \times 0.54^x = 18$
|
||||
\end{enumerate}
|
||||
\end{exercise}
|
||||
|
||||
\begin{solution}
|
||||
\begin{enumerate}
|
||||
\item On veut partager cette évolution en 8 évolutions.
|
||||
\[
|
||||
\left(1 + \frac{12}{100}\right)^{\frac{1}{8}} = 1.0143
|
||||
\]
|
||||
Donc le taux d'évolution moyen est
|
||||
\[
|
||||
t_m = 1.0143 - 1 = 0.01429999999999998
|
||||
\]
|
||||
\item Coefficient multiplicateur pour revenir en arrière
|
||||
\[
|
||||
CM = (1 + \frac{12}{100})^{-1} = 0.8929
|
||||
\]
|
||||
On en déduit la quantité en 2019
|
||||
\[
|
||||
126 * 0.8929 = 112.50540000000001
|
||||
\]
|
||||
\item L'équation de la droite est
|
||||
\[
|
||||
y = 2.6666666666666665 x -4
|
||||
\]
|
||||
\item Il faut penser à faire la division à par $8$ avant d'utiliser le log car sinon, on ne peut pas utiliser la formule $\log(a^n) = n\times \log(a)$.
|
||||
|
||||
\[x = \frac{\log(2.25)}{\log(0.54)}\]
|
||||
\end{enumerate}
|
||||
\end{solution}
|
||||
|
||||
\begin{exercise}[subtitle={Restaurant}]
|
||||
Un \emph{food truck}, ouvert le midi et le soir, propose deux types de formules :
|
||||
|
||||
\setlength\parindent{10mm}
|
||||
\begin{itemize}
|
||||
\item la formule \emph{Burger} ;
|
||||
\item la formule \emph{Wok}.
|
||||
\end{itemize}
|
||||
\setlength\parindent{0mm}
|
||||
|
||||
\medskip
|
||||
|
||||
Le gérant a remarqué que 94\,\% de ses ventes ont lieu le midi. Le quart des ventes du midi correspondent à la formule \emph{Burger}, alors que 37\,\% des ventes du soir correspondent à la formule \emph{Wok}.
|
||||
|
||||
Le gérant se constitue un fichier en notant, pour chaque vente, la formule choisie et le moment de cette vente (midi ou soir).
|
||||
|
||||
On prélève une fiche de façon équiprobable. On définit les quatre évènements suivants:
|
||||
|
||||
\begin{enumerate}
|
||||
\item $M$ : \og la fiche correspond à une vente du midi\fg{} ;
|
||||
\item $S$ : \og la fiche correspond à une vente du soir\fg {};
|
||||
\item $W$ : \og la fiche correspond à une formule \emph{Wok} \fg{} ;
|
||||
\item $B$ : \og la fiche correspond à une formule \emph{Burger} \fg.
|
||||
\end{enumerate}
|
||||
\setlength\parindent{0mm}
|
||||
|
||||
\medskip
|
||||
|
||||
\begin{enumerate}
|
||||
\item Recopier puis compléter l'arbre pondéré
|
||||
|
||||
\begin{center}
|
||||
\begin{tikzpicture}[sloped]
|
||||
\node {.}
|
||||
child {node {$M$}
|
||||
child {node {$W$}
|
||||
edge from parent
|
||||
node[above] {...}
|
||||
}
|
||||
child {node {$B$}
|
||||
edge from parent
|
||||
node[above] {...}
|
||||
}
|
||||
edge from parent
|
||||
node[above] {...}
|
||||
}
|
||||
child[missing] {}
|
||||
child { node {$S$}
|
||||
child {node {$W$}
|
||||
edge from parent
|
||||
node[above] {...}
|
||||
}
|
||||
child {node {$B$}
|
||||
edge from parent
|
||||
node[above] {...}
|
||||
}
|
||||
edge from parent
|
||||
node[above] {...}
|
||||
} ;
|
||||
\end{tikzpicture}
|
||||
\end{center}
|
||||
|
||||
\item Calculer la probabilité de l'évènement $M \cap W$. Interpréter ce résultat dans le contexte de l'exercice.
|
||||
\item Montrer que la probabilité que la fiche choisie corresponde à une formule \emph{Burger} est égale à $0.2728$.
|
||||
\item On a prélevé une fiche correspondant à la formule \emph{Burger}. Quelle est la probabilité, arrondie au millième, que la vente ait eu lieu le soir?
|
||||
\end{enumerate}
|
||||
\end{exercise}
|
||||
|
||||
\begin{solution}
|
||||
\begin{enumerate}
|
||||
\item
|
||||
\begin{center}
|
||||
\begin{tikzpicture}[sloped]
|
||||
\node {.}
|
||||
child {node {$M$}
|
||||
child {node {$W$}
|
||||
edge from parent
|
||||
node[above] {$0.75$}
|
||||
}
|
||||
child {node {$B$}
|
||||
edge from parent
|
||||
node[above] {$0.25$}
|
||||
}
|
||||
edge from parent
|
||||
node[above] {$0.94$}
|
||||
}
|
||||
child[missing] {}
|
||||
child { node {$S$}
|
||||
child {node {$W$}
|
||||
edge from parent
|
||||
node[above] {$0.37$}
|
||||
}
|
||||
child {node {$B$}
|
||||
edge from parent
|
||||
node[above] {$0.63$}
|
||||
}
|
||||
edge from parent
|
||||
node[above] {$0.06$}
|
||||
} ;
|
||||
\end{tikzpicture}
|
||||
\end{center}
|
||||
\item On calcule la probabilité que la vente soit un wok et ait eu lieu à midi
|
||||
\[ P(M\cap W) = P(M) \times P_M(W) = 0.94 \times 0.75 = 0.705 \]
|
||||
\item Probabilité que la vente soit un burger.
|
||||
\[
|
||||
P(B) = P(M\cap B) + P(S\cap B) = 0.94 \times 0.75 + 0.06 \times 0.37 = 0.2728
|
||||
\]
|
||||
\item On cherche à calculer la quantité $P_B(S)$. Pour cela on utilise la formule de Bayes
|
||||
\[
|
||||
P_B(S) = \frac{P(B\cap S)}{P(B)} = \frac{P_S(B) \times P(S)}{P(B)} = \frac{0.63\times 0.06}{0.2728} = 0.13856304985337245 \approx 0.139
|
||||
\]
|
||||
\end{enumerate}
|
||||
\end{solution}
|
||||
|
||||
\begin{exercise}[subtitle={Continent plastique}]
|
||||
\textit{Les quantités évoqués dans cette exercice sont générés au hasard et sont donc complètement farfelus.}
|
||||
\medskip
|
||||
Le \og continent de plastique\fg{} est la plus grande des plaques de déchets plastiques évoluant sur les océans. Elle occupe actuellement dans l'océan Pacifique une surface dont l'aire est évaluée à plus de $1,6$ million de km$^2$, entre Hawaï et la Californie.
|
||||
|
||||
En 2017, des scientifiques ont estimé qu'il y avait $15$ millions de tonnes de déchets plastiques qui était déversé chaque année dans les océans et que cette quantité augmentait de $19\n\%$ par chaque année.
|
||||
|
||||
On modélise l'évolution de la masse de ces déchets plastiques déversée chaque année, si rien n'est fait pour la réduire, par une suite géométrique $\left(u_n\right)$. L'arrondi au centième du terme $u_n$ représente la masse de ces déchets déversée chaque année, exprimée en million de tonnes, pour l'année $(2017 + n)$.
|
||||
|
||||
\medskip
|
||||
|
||||
\begin{enumerate}
|
||||
\item Expliquer pourquoi la suite $u_n$ est géométrique?
|
||||
\item Calculer $u_1$ et $u_2$.
|
||||
\item Exprimer $u_n$ en fonction de $n$.
|
||||
\item Au début de l'année 2017, il y avait $300$ millions de tonnes de déchets plastique. Calculer la quantité totale de déchets plastiques en 2030.
|
||||
\item On souhaite déterminer en quelle année la masse totale de ces déchets plastiques aura pour la première fois augmenté de $50$\,\% par rapport à sa valeur de 2017.
|
||||
\begin{enumerate}
|
||||
\item Recopier et compléter l'algorithme ci-dessous pour que la variable $N$ contienne la réponse au problème posé.
|
||||
|
||||
\begin{center}
|
||||
\begin{tabularx}{0.4\linewidth}{|X|}\hline
|
||||
$N = 2017$\\
|
||||
$U = 15$ \\
|
||||
$S = 300 + U$ \\
|
||||
while $S < 450$: \\
|
||||
\hspace{1cm} $N = \ldots$\\
|
||||
\hspace{1cm} $U = \ldots$\\
|
||||
\hspace{1cm} $S = \ldots$\\
|
||||
\hline
|
||||
\end{tabularx}
|
||||
\end{center}
|
||||
\item Que contiennent les variables $S$, $U$ et $N$ après exécution de cet algorithme ?
|
||||
|
||||
Interpréter les résultats dans le contexte de l'exercice.
|
||||
\end{enumerate}
|
||||
\end{enumerate}
|
||||
\end{exercise}
|
||||
|
||||
\begin{solution}
|
||||
\begin{enumerate}
|
||||
\item Une augmentation de $19\,\%$ revient à multiplier la quantité par $1.19$. La suite est donc bien géométrique. Son premier terme est $u_0 = 15$ et sa raison est $q = 1.19$
|
||||
\item
|
||||
\[
|
||||
u_1 = u_0 * 1.19 = 17.849999999999998
|
||||
\]
|
||||
\[
|
||||
u_2 = u_0 * 1.19^2 = 21.2415
|
||||
\]
|
||||
\item
|
||||
\[
|
||||
u_n = u_0 \times q^n = 15 \times 1.19^n
|
||||
\]
|
||||
\item On calcule la quantité totale déversée entre 2017 et 2030.
|
||||
\[
|
||||
\sum_{n = 0}^{13} u_n = u_0 \times \frac{1-q^{13}}{1-q} = 15 \times \frac{1 - 1.19^{13}}{1 - 1.19} = 678.67
|
||||
\]
|
||||
On en déduit la quantité totale de déchets en 2030
|
||||
\[
|
||||
300 + 678.67 = 978.67
|
||||
\]
|
||||
\item
|
||||
\begin{enumerate}
|
||||
\item ~
|
||||
\begin{center}
|
||||
\begin{tabularx}{0.4\linewidth}{|X|}\hline
|
||||
$N \gets 2017$\\
|
||||
$U \gets 15$ \\
|
||||
$S \gets 300 + U$ \\
|
||||
Tant que $S < 450$ \\
|
||||
\hspace{1cm} $N \gets N + 1$\\
|
||||
\hspace{1cm} $U \gets U * 1.19$\\
|
||||
\hspace{1cm} $S \gets S + u$\\
|
||||
Fin Tant que\\\hline
|
||||
\end{tabularx}
|
||||
\end{center}
|
||||
\item \textit{Pas de correction automatisé}
|
||||
\end{enumerate}
|
||||
\end{enumerate}
|
||||
\end{solution}
|
||||
|
||||
\end{document}
|
||||
|
||||
%%% Local Variables:
|
||||
%%% mode: latex
|
||||
%%% TeX-master: "master"
|
||||
%%% End:
|
262
TST/DS/DS_21_04_07/TST1/04_210407_DS8.tex
Normal file
262
TST/DS/DS_21_04_07/TST1/04_210407_DS8.tex
Normal file
@ -0,0 +1,262 @@
|
||||
\documentclass[a4paper,10pt]{article}
|
||||
\usepackage{myXsim}
|
||||
|
||||
% Title Page
|
||||
\title{DS8 \hfill BENHATTAL Chakir}
|
||||
\tribe{TST}
|
||||
\date{\hfillÀ render pour le Mercredi 7 avril}
|
||||
|
||||
\xsimsetup{
|
||||
solution/print = false
|
||||
}
|
||||
|
||||
\begin{document}
|
||||
\maketitle
|
||||
|
||||
\begin{exercise}[subtitle={Automatismes}]
|
||||
\textit{Toutes les questions de cette exercice sont indépendantes et peuvent être répondus séparément}
|
||||
\begin{enumerate}
|
||||
\item De janvier à septembre, une quantité a augmenté de $28\,\%$. Faire un schéma pour représenter la situation puis calculer le taux d'évolution moyen mensuel.
|
||||
\item Une quantité augmente de $28\,\%$ par ans. En 2020, elle est de 111\euro. Quelle était sa valeur en 2019? Faire un schéma pour représenter la situation.
|
||||
\item Déterminer l'équation de la droite \\
|
||||
\begin{tikzpicture}[xscale=0.8, yscale=0.5]
|
||||
\tkzInit[xmin=-5,xmax=5,xstep=1,
|
||||
ymin=-5,ymax=5,ystep=1]
|
||||
\tkzGrid
|
||||
\tkzAxeXY
|
||||
\tkzFct[domain=-5:5,color=red,very thick]%
|
||||
{1.0*\x -1};
|
||||
\end{tikzpicture}
|
||||
\item Résoudre l'équation $4 \times 0.77^x = 25$
|
||||
\end{enumerate}
|
||||
\end{exercise}
|
||||
|
||||
\begin{solution}
|
||||
\begin{enumerate}
|
||||
\item On veut partager cette évolution en 8 évolutions.
|
||||
\[
|
||||
\left(1 + \frac{28}{100}\right)^{\frac{1}{8}} = 1.0313
|
||||
\]
|
||||
Donc le taux d'évolution moyen est
|
||||
\[
|
||||
t_m = 1.0313 - 1 = 0.031300000000000106
|
||||
\]
|
||||
\item Coefficient multiplicateur pour revenir en arrière
|
||||
\[
|
||||
CM = (1 + \frac{28}{100})^{-1} = 0.7812
|
||||
\]
|
||||
On en déduit la quantité en 2019
|
||||
\[
|
||||
111 * 0.7812 = 86.7132
|
||||
\]
|
||||
\item L'équation de la droite est
|
||||
\[
|
||||
y = 1.0 x -1
|
||||
\]
|
||||
\item Il faut penser à faire la division à par $4$ avant d'utiliser le log car sinon, on ne peut pas utiliser la formule $\log(a^n) = n\times \log(a)$.
|
||||
|
||||
\[x = \frac{\log(6.25)}{\log(0.77)}\]
|
||||
\end{enumerate}
|
||||
\end{solution}
|
||||
|
||||
\begin{exercise}[subtitle={Restaurant}]
|
||||
Un \emph{food truck}, ouvert le midi et le soir, propose deux types de formules :
|
||||
|
||||
\setlength\parindent{10mm}
|
||||
\begin{itemize}
|
||||
\item la formule \emph{Burger} ;
|
||||
\item la formule \emph{Wok}.
|
||||
\end{itemize}
|
||||
\setlength\parindent{0mm}
|
||||
|
||||
\medskip
|
||||
|
||||
Le gérant a remarqué que 89\,\% de ses ventes ont lieu le midi. Le quart des ventes du midi correspondent à la formule \emph{Burger}, alors que 43\,\% des ventes du soir correspondent à la formule \emph{Wok}.
|
||||
|
||||
Le gérant se constitue un fichier en notant, pour chaque vente, la formule choisie et le moment de cette vente (midi ou soir).
|
||||
|
||||
On prélève une fiche de façon équiprobable. On définit les quatre évènements suivants:
|
||||
|
||||
\begin{enumerate}
|
||||
\item $M$ : \og la fiche correspond à une vente du midi\fg{} ;
|
||||
\item $S$ : \og la fiche correspond à une vente du soir\fg {};
|
||||
\item $W$ : \og la fiche correspond à une formule \emph{Wok} \fg{} ;
|
||||
\item $B$ : \og la fiche correspond à une formule \emph{Burger} \fg.
|
||||
\end{enumerate}
|
||||
\setlength\parindent{0mm}
|
||||
|
||||
\medskip
|
||||
|
||||
\begin{enumerate}
|
||||
\item Recopier puis compléter l'arbre pondéré
|
||||
|
||||
\begin{center}
|
||||
\begin{tikzpicture}[sloped]
|
||||
\node {.}
|
||||
child {node {$M$}
|
||||
child {node {$W$}
|
||||
edge from parent
|
||||
node[above] {...}
|
||||
}
|
||||
child {node {$B$}
|
||||
edge from parent
|
||||
node[above] {...}
|
||||
}
|
||||
edge from parent
|
||||
node[above] {...}
|
||||
}
|
||||
child[missing] {}
|
||||
child { node {$S$}
|
||||
child {node {$W$}
|
||||
edge from parent
|
||||
node[above] {...}
|
||||
}
|
||||
child {node {$B$}
|
||||
edge from parent
|
||||
node[above] {...}
|
||||
}
|
||||
edge from parent
|
||||
node[above] {...}
|
||||
} ;
|
||||
\end{tikzpicture}
|
||||
\end{center}
|
||||
|
||||
\item Calculer la probabilité de l'évènement $M \cap W$. Interpréter ce résultat dans le contexte de l'exercice.
|
||||
\item Montrer que la probabilité que la fiche choisie corresponde à une formule \emph{Burger} est égale à $0.2852$.
|
||||
\item On a prélevé une fiche correspondant à la formule \emph{Burger}. Quelle est la probabilité, arrondie au millième, que la vente ait eu lieu le soir?
|
||||
\end{enumerate}
|
||||
\end{exercise}
|
||||
|
||||
\begin{solution}
|
||||
\begin{enumerate}
|
||||
\item
|
||||
\begin{center}
|
||||
\begin{tikzpicture}[sloped]
|
||||
\node {.}
|
||||
child {node {$M$}
|
||||
child {node {$W$}
|
||||
edge from parent
|
||||
node[above] {$0.75$}
|
||||
}
|
||||
child {node {$B$}
|
||||
edge from parent
|
||||
node[above] {$0.25$}
|
||||
}
|
||||
edge from parent
|
||||
node[above] {$0.89$}
|
||||
}
|
||||
child[missing] {}
|
||||
child { node {$S$}
|
||||
child {node {$W$}
|
||||
edge from parent
|
||||
node[above] {$0.43$}
|
||||
}
|
||||
child {node {$B$}
|
||||
edge from parent
|
||||
node[above] {$0.57$}
|
||||
}
|
||||
edge from parent
|
||||
node[above] {$0.11$}
|
||||
} ;
|
||||
\end{tikzpicture}
|
||||
\end{center}
|
||||
\item On calcule la probabilité que la vente soit un wok et ait eu lieu à midi
|
||||
\[ P(M\cap W) = P(M) \times P_M(W) = 0.89 \times 0.75 = 0.6675 \]
|
||||
\item Probabilité que la vente soit un burger.
|
||||
\[
|
||||
P(B) = P(M\cap B) + P(S\cap B) = 0.89 \times 0.75 + 0.11 \times 0.43 = 0.2852
|
||||
\]
|
||||
\item On cherche à calculer la quantité $P_B(S)$. Pour cela on utilise la formule de Bayes
|
||||
\[
|
||||
P_B(S) = \frac{P(B\cap S)}{P(B)} = \frac{P_S(B) \times P(S)}{P(B)} = \frac{0.57\times 0.11}{0.2852} = 0.2198457223001402 \approx 0.22
|
||||
\]
|
||||
\end{enumerate}
|
||||
\end{solution}
|
||||
|
||||
\begin{exercise}[subtitle={Continent plastique}]
|
||||
\textit{Les quantités évoqués dans cette exercice sont générés au hasard et sont donc complètement farfelus.}
|
||||
\medskip
|
||||
Le \og continent de plastique\fg{} est la plus grande des plaques de déchets plastiques évoluant sur les océans. Elle occupe actuellement dans l'océan Pacifique une surface dont l'aire est évaluée à plus de $1,6$ million de km$^2$, entre Hawaï et la Californie.
|
||||
|
||||
En 2017, des scientifiques ont estimé qu'il y avait $6$ millions de tonnes de déchets plastiques qui était déversé chaque année dans les océans et que cette quantité augmentait de $13\n\%$ par chaque année.
|
||||
|
||||
On modélise l'évolution de la masse de ces déchets plastiques déversée chaque année, si rien n'est fait pour la réduire, par une suite géométrique $\left(u_n\right)$. L'arrondi au centième du terme $u_n$ représente la masse de ces déchets déversée chaque année, exprimée en million de tonnes, pour l'année $(2017 + n)$.
|
||||
|
||||
\medskip
|
||||
|
||||
\begin{enumerate}
|
||||
\item Expliquer pourquoi la suite $u_n$ est géométrique?
|
||||
\item Calculer $u_1$ et $u_2$.
|
||||
\item Exprimer $u_n$ en fonction de $n$.
|
||||
\item Au début de l'année 2017, il y avait $300$ millions de tonnes de déchets plastique. Calculer la quantité totale de déchets plastiques en 2030.
|
||||
\item On souhaite déterminer en quelle année la masse totale de ces déchets plastiques aura pour la première fois augmenté de $50$\,\% par rapport à sa valeur de 2017.
|
||||
\begin{enumerate}
|
||||
\item Recopier et compléter l'algorithme ci-dessous pour que la variable $N$ contienne la réponse au problème posé.
|
||||
|
||||
\begin{center}
|
||||
\begin{tabularx}{0.4\linewidth}{|X|}\hline
|
||||
$N = 2017$\\
|
||||
$U = 6$ \\
|
||||
$S = 300 + U$ \\
|
||||
while $S < 450$: \\
|
||||
\hspace{1cm} $N = \ldots$\\
|
||||
\hspace{1cm} $U = \ldots$\\
|
||||
\hspace{1cm} $S = \ldots$\\
|
||||
\hline
|
||||
\end{tabularx}
|
||||
\end{center}
|
||||
\item Que contiennent les variables $S$, $U$ et $N$ après exécution de cet algorithme ?
|
||||
|
||||
Interpréter les résultats dans le contexte de l'exercice.
|
||||
\end{enumerate}
|
||||
\end{enumerate}
|
||||
\end{exercise}
|
||||
|
||||
\begin{solution}
|
||||
\begin{enumerate}
|
||||
\item Une augmentation de $13\,\%$ revient à multiplier la quantité par $1.13$. La suite est donc bien géométrique. Son premier terme est $u_0 = 6$ et sa raison est $q = 1.13$
|
||||
\item
|
||||
\[
|
||||
u_1 = u_0 * 1.13 = 6.779999999999999
|
||||
\]
|
||||
\[
|
||||
u_2 = u_0 * 1.13^2 = 7.6614
|
||||
\]
|
||||
\item
|
||||
\[
|
||||
u_n = u_0 \times q^n = 6 \times 1.13^n
|
||||
\]
|
||||
\item On calcule la quantité totale déversée entre 2017 et 2030.
|
||||
\[
|
||||
\sum_{n = 0}^{13} u_n = u_0 \times \frac{1-q^{13}}{1-q} = 6 \times \frac{1 - 1.13^{13}}{1 - 1.13} = 179.91
|
||||
\]
|
||||
On en déduit la quantité totale de déchets en 2030
|
||||
\[
|
||||
300 + 179.91 = 479.90999999999997
|
||||
\]
|
||||
\item
|
||||
\begin{enumerate}
|
||||
\item ~
|
||||
\begin{center}
|
||||
\begin{tabularx}{0.4\linewidth}{|X|}\hline
|
||||
$N \gets 2017$\\
|
||||
$U \gets 6$ \\
|
||||
$S \gets 300 + U$ \\
|
||||
Tant que $S < 450$ \\
|
||||
\hspace{1cm} $N \gets N + 1$\\
|
||||
\hspace{1cm} $U \gets U * 1.13$\\
|
||||
\hspace{1cm} $S \gets S + u$\\
|
||||
Fin Tant que\\\hline
|
||||
\end{tabularx}
|
||||
\end{center}
|
||||
\item \textit{Pas de correction automatisé}
|
||||
\end{enumerate}
|
||||
\end{enumerate}
|
||||
\end{solution}
|
||||
|
||||
\end{document}
|
||||
|
||||
%%% Local Variables:
|
||||
%%% mode: latex
|
||||
%%% TeX-master: "master"
|
||||
%%% End:
|
262
TST/DS/DS_21_04_07/TST1/05_210407_DS8.tex
Normal file
262
TST/DS/DS_21_04_07/TST1/05_210407_DS8.tex
Normal file
@ -0,0 +1,262 @@
|
||||
\documentclass[a4paper,10pt]{article}
|
||||
\usepackage{myXsim}
|
||||
|
||||
% Title Page
|
||||
\title{DS8 \hfill CLAIN Avinash}
|
||||
\tribe{TST}
|
||||
\date{\hfillÀ render pour le Mercredi 7 avril}
|
||||
|
||||
\xsimsetup{
|
||||
solution/print = false
|
||||
}
|
||||
|
||||
\begin{document}
|
||||
\maketitle
|
||||
|
||||
\begin{exercise}[subtitle={Automatismes}]
|
||||
\textit{Toutes les questions de cette exercice sont indépendantes et peuvent être répondus séparément}
|
||||
\begin{enumerate}
|
||||
\item De janvier à septembre, une quantité a augmenté de $15\,\%$. Faire un schéma pour représenter la situation puis calculer le taux d'évolution moyen mensuel.
|
||||
\item Une quantité augmente de $15\,\%$ par ans. En 2020, elle est de 112\euro. Quelle était sa valeur en 2019? Faire un schéma pour représenter la situation.
|
||||
\item Déterminer l'équation de la droite \\
|
||||
\begin{tikzpicture}[xscale=0.8, yscale=0.5]
|
||||
\tkzInit[xmin=-5,xmax=5,xstep=1,
|
||||
ymin=-5,ymax=5,ystep=1]
|
||||
\tkzGrid
|
||||
\tkzAxeXY
|
||||
\tkzFct[domain=-5:5,color=red,very thick]%
|
||||
{4.0*\x -4};
|
||||
\end{tikzpicture}
|
||||
\item Résoudre l'équation $6 \times 0.77^x = 23$
|
||||
\end{enumerate}
|
||||
\end{exercise}
|
||||
|
||||
\begin{solution}
|
||||
\begin{enumerate}
|
||||
\item On veut partager cette évolution en 8 évolutions.
|
||||
\[
|
||||
\left(1 + \frac{15}{100}\right)^{\frac{1}{8}} = 1.0176
|
||||
\]
|
||||
Donc le taux d'évolution moyen est
|
||||
\[
|
||||
t_m = 1.0176 - 1 = 0.01760000000000006
|
||||
\]
|
||||
\item Coefficient multiplicateur pour revenir en arrière
|
||||
\[
|
||||
CM = (1 + \frac{15}{100})^{-1} = 0.8696
|
||||
\]
|
||||
On en déduit la quantité en 2019
|
||||
\[
|
||||
112 * 0.8696 = 97.3952
|
||||
\]
|
||||
\item L'équation de la droite est
|
||||
\[
|
||||
y = 4.0 x -4
|
||||
\]
|
||||
\item Il faut penser à faire la division à par $6$ avant d'utiliser le log car sinon, on ne peut pas utiliser la formule $\log(a^n) = n\times \log(a)$.
|
||||
|
||||
\[x = \frac{\log(3.83)}{\log(0.77)}\]
|
||||
\end{enumerate}
|
||||
\end{solution}
|
||||
|
||||
\begin{exercise}[subtitle={Restaurant}]
|
||||
Un \emph{food truck}, ouvert le midi et le soir, propose deux types de formules :
|
||||
|
||||
\setlength\parindent{10mm}
|
||||
\begin{itemize}
|
||||
\item la formule \emph{Burger} ;
|
||||
\item la formule \emph{Wok}.
|
||||
\end{itemize}
|
||||
\setlength\parindent{0mm}
|
||||
|
||||
\medskip
|
||||
|
||||
Le gérant a remarqué que 92\,\% de ses ventes ont lieu le midi. Le quart des ventes du midi correspondent à la formule \emph{Burger}, alors que 54\,\% des ventes du soir correspondent à la formule \emph{Wok}.
|
||||
|
||||
Le gérant se constitue un fichier en notant, pour chaque vente, la formule choisie et le moment de cette vente (midi ou soir).
|
||||
|
||||
On prélève une fiche de façon équiprobable. On définit les quatre évènements suivants:
|
||||
|
||||
\begin{enumerate}
|
||||
\item $M$ : \og la fiche correspond à une vente du midi\fg{} ;
|
||||
\item $S$ : \og la fiche correspond à une vente du soir\fg {};
|
||||
\item $W$ : \og la fiche correspond à une formule \emph{Wok} \fg{} ;
|
||||
\item $B$ : \og la fiche correspond à une formule \emph{Burger} \fg.
|
||||
\end{enumerate}
|
||||
\setlength\parindent{0mm}
|
||||
|
||||
\medskip
|
||||
|
||||
\begin{enumerate}
|
||||
\item Recopier puis compléter l'arbre pondéré
|
||||
|
||||
\begin{center}
|
||||
\begin{tikzpicture}[sloped]
|
||||
\node {.}
|
||||
child {node {$M$}
|
||||
child {node {$W$}
|
||||
edge from parent
|
||||
node[above] {...}
|
||||
}
|
||||
child {node {$B$}
|
||||
edge from parent
|
||||
node[above] {...}
|
||||
}
|
||||
edge from parent
|
||||
node[above] {...}
|
||||
}
|
||||
child[missing] {}
|
||||
child { node {$S$}
|
||||
child {node {$W$}
|
||||
edge from parent
|
||||
node[above] {...}
|
||||
}
|
||||
child {node {$B$}
|
||||
edge from parent
|
||||
node[above] {...}
|
||||
}
|
||||
edge from parent
|
||||
node[above] {...}
|
||||
} ;
|
||||
\end{tikzpicture}
|
||||
\end{center}
|
||||
|
||||
\item Calculer la probabilité de l'évènement $M \cap W$. Interpréter ce résultat dans le contexte de l'exercice.
|
||||
\item Montrer que la probabilité que la fiche choisie corresponde à une formule \emph{Burger} est égale à $0.2668$.
|
||||
\item On a prélevé une fiche correspondant à la formule \emph{Burger}. Quelle est la probabilité, arrondie au millième, que la vente ait eu lieu le soir?
|
||||
\end{enumerate}
|
||||
\end{exercise}
|
||||
|
||||
\begin{solution}
|
||||
\begin{enumerate}
|
||||
\item
|
||||
\begin{center}
|
||||
\begin{tikzpicture}[sloped]
|
||||
\node {.}
|
||||
child {node {$M$}
|
||||
child {node {$W$}
|
||||
edge from parent
|
||||
node[above] {$0.75$}
|
||||
}
|
||||
child {node {$B$}
|
||||
edge from parent
|
||||
node[above] {$0.25$}
|
||||
}
|
||||
edge from parent
|
||||
node[above] {$0.92$}
|
||||
}
|
||||
child[missing] {}
|
||||
child { node {$S$}
|
||||
child {node {$W$}
|
||||
edge from parent
|
||||
node[above] {$0.54$}
|
||||
}
|
||||
child {node {$B$}
|
||||
edge from parent
|
||||
node[above] {$0.46$}
|
||||
}
|
||||
edge from parent
|
||||
node[above] {$0.08$}
|
||||
} ;
|
||||
\end{tikzpicture}
|
||||
\end{center}
|
||||
\item On calcule la probabilité que la vente soit un wok et ait eu lieu à midi
|
||||
\[ P(M\cap W) = P(M) \times P_M(W) = 0.92 \times 0.75 = 0.69 \]
|
||||
\item Probabilité que la vente soit un burger.
|
||||
\[
|
||||
P(B) = P(M\cap B) + P(S\cap B) = 0.92 \times 0.75 + 0.08 \times 0.54 = 0.2668
|
||||
\]
|
||||
\item On cherche à calculer la quantité $P_B(S)$. Pour cela on utilise la formule de Bayes
|
||||
\[
|
||||
P_B(S) = \frac{P(B\cap S)}{P(B)} = \frac{P_S(B) \times P(S)}{P(B)} = \frac{0.46\times 0.08}{0.2668} = 0.13793103448275862 \approx 0.138
|
||||
\]
|
||||
\end{enumerate}
|
||||
\end{solution}
|
||||
|
||||
\begin{exercise}[subtitle={Continent plastique}]
|
||||
\textit{Les quantités évoqués dans cette exercice sont générés au hasard et sont donc complètement farfelus.}
|
||||
\medskip
|
||||
Le \og continent de plastique\fg{} est la plus grande des plaques de déchets plastiques évoluant sur les océans. Elle occupe actuellement dans l'océan Pacifique une surface dont l'aire est évaluée à plus de $1,6$ million de km$^2$, entre Hawaï et la Californie.
|
||||
|
||||
En 2017, des scientifiques ont estimé qu'il y avait $18$ millions de tonnes de déchets plastiques qui était déversé chaque année dans les océans et que cette quantité augmentait de $21\n\%$ par chaque année.
|
||||
|
||||
On modélise l'évolution de la masse de ces déchets plastiques déversée chaque année, si rien n'est fait pour la réduire, par une suite géométrique $\left(u_n\right)$. L'arrondi au centième du terme $u_n$ représente la masse de ces déchets déversée chaque année, exprimée en million de tonnes, pour l'année $(2017 + n)$.
|
||||
|
||||
\medskip
|
||||
|
||||
\begin{enumerate}
|
||||
\item Expliquer pourquoi la suite $u_n$ est géométrique?
|
||||
\item Calculer $u_1$ et $u_2$.
|
||||
\item Exprimer $u_n$ en fonction de $n$.
|
||||
\item Au début de l'année 2017, il y avait $300$ millions de tonnes de déchets plastique. Calculer la quantité totale de déchets plastiques en 2030.
|
||||
\item On souhaite déterminer en quelle année la masse totale de ces déchets plastiques aura pour la première fois augmenté de $50$\,\% par rapport à sa valeur de 2017.
|
||||
\begin{enumerate}
|
||||
\item Recopier et compléter l'algorithme ci-dessous pour que la variable $N$ contienne la réponse au problème posé.
|
||||
|
||||
\begin{center}
|
||||
\begin{tabularx}{0.4\linewidth}{|X|}\hline
|
||||
$N = 2017$\\
|
||||
$U = 18$ \\
|
||||
$S = 300 + U$ \\
|
||||
while $S < 450$: \\
|
||||
\hspace{1cm} $N = \ldots$\\
|
||||
\hspace{1cm} $U = \ldots$\\
|
||||
\hspace{1cm} $S = \ldots$\\
|
||||
\hline
|
||||
\end{tabularx}
|
||||
\end{center}
|
||||
\item Que contiennent les variables $S$, $U$ et $N$ après exécution de cet algorithme ?
|
||||
|
||||
Interpréter les résultats dans le contexte de l'exercice.
|
||||
\end{enumerate}
|
||||
\end{enumerate}
|
||||
\end{exercise}
|
||||
|
||||
\begin{solution}
|
||||
\begin{enumerate}
|
||||
\item Une augmentation de $21\,\%$ revient à multiplier la quantité par $1.21$. La suite est donc bien géométrique. Son premier terme est $u_0 = 18$ et sa raison est $q = 1.21$
|
||||
\item
|
||||
\[
|
||||
u_1 = u_0 * 1.21 = 21.78
|
||||
\]
|
||||
\[
|
||||
u_2 = u_0 * 1.21^2 = 26.3538
|
||||
\]
|
||||
\item
|
||||
\[
|
||||
u_n = u_0 \times q^n = 18 \times 1.21^n
|
||||
\]
|
||||
\item On calcule la quantité totale déversée entre 2017 et 2030.
|
||||
\[
|
||||
\sum_{n = 0}^{13} u_n = u_0 \times \frac{1-q^{13}}{1-q} = 18 \times \frac{1 - 1.21^{13}}{1 - 1.21} = 935.84
|
||||
\]
|
||||
On en déduit la quantité totale de déchets en 2030
|
||||
\[
|
||||
300 + 935.84 = 1235.8400000000001
|
||||
\]
|
||||
\item
|
||||
\begin{enumerate}
|
||||
\item ~
|
||||
\begin{center}
|
||||
\begin{tabularx}{0.4\linewidth}{|X|}\hline
|
||||
$N \gets 2017$\\
|
||||
$U \gets 18$ \\
|
||||
$S \gets 300 + U$ \\
|
||||
Tant que $S < 450$ \\
|
||||
\hspace{1cm} $N \gets N + 1$\\
|
||||
\hspace{1cm} $U \gets U * 1.21$\\
|
||||
\hspace{1cm} $S \gets S + u$\\
|
||||
Fin Tant que\\\hline
|
||||
\end{tabularx}
|
||||
\end{center}
|
||||
\item \textit{Pas de correction automatisé}
|
||||
\end{enumerate}
|
||||
\end{enumerate}
|
||||
\end{solution}
|
||||
|
||||
\end{document}
|
||||
|
||||
%%% Local Variables:
|
||||
%%% mode: latex
|
||||
%%% TeX-master: "master"
|
||||
%%% End:
|
262
TST/DS/DS_21_04_07/TST1/06_210407_DS8.tex
Normal file
262
TST/DS/DS_21_04_07/TST1/06_210407_DS8.tex
Normal file
@ -0,0 +1,262 @@
|
||||
\documentclass[a4paper,10pt]{article}
|
||||
\usepackage{myXsim}
|
||||
|
||||
% Title Page
|
||||
\title{DS8 \hfill COLASSI Alexis}
|
||||
\tribe{TST}
|
||||
\date{\hfillÀ render pour le Mercredi 7 avril}
|
||||
|
||||
\xsimsetup{
|
||||
solution/print = false
|
||||
}
|
||||
|
||||
\begin{document}
|
||||
\maketitle
|
||||
|
||||
\begin{exercise}[subtitle={Automatismes}]
|
||||
\textit{Toutes les questions de cette exercice sont indépendantes et peuvent être répondus séparément}
|
||||
\begin{enumerate}
|
||||
\item De janvier à septembre, une quantité a augmenté de $27\,\%$. Faire un schéma pour représenter la situation puis calculer le taux d'évolution moyen mensuel.
|
||||
\item Une quantité augmente de $27\,\%$ par ans. En 2020, elle est de 116\euro. Quelle était sa valeur en 2019? Faire un schéma pour représenter la situation.
|
||||
\item Déterminer l'équation de la droite \\
|
||||
\begin{tikzpicture}[xscale=0.8, yscale=0.5]
|
||||
\tkzInit[xmin=-5,xmax=5,xstep=1,
|
||||
ymin=-5,ymax=5,ystep=1]
|
||||
\tkzGrid
|
||||
\tkzAxeXY
|
||||
\tkzFct[domain=-5:5,color=red,very thick]%
|
||||
{2.0*\x -2};
|
||||
\end{tikzpicture}
|
||||
\item Résoudre l'équation $7 \times 0.23^x = 16$
|
||||
\end{enumerate}
|
||||
\end{exercise}
|
||||
|
||||
\begin{solution}
|
||||
\begin{enumerate}
|
||||
\item On veut partager cette évolution en 8 évolutions.
|
||||
\[
|
||||
\left(1 + \frac{27}{100}\right)^{\frac{1}{8}} = 1.0303
|
||||
\]
|
||||
Donc le taux d'évolution moyen est
|
||||
\[
|
||||
t_m = 1.0303 - 1 = 0.030299999999999994
|
||||
\]
|
||||
\item Coefficient multiplicateur pour revenir en arrière
|
||||
\[
|
||||
CM = (1 + \frac{27}{100})^{-1} = 0.7874
|
||||
\]
|
||||
On en déduit la quantité en 2019
|
||||
\[
|
||||
116 * 0.7874 = 91.3384
|
||||
\]
|
||||
\item L'équation de la droite est
|
||||
\[
|
||||
y = 2.0 x -2
|
||||
\]
|
||||
\item Il faut penser à faire la division à par $7$ avant d'utiliser le log car sinon, on ne peut pas utiliser la formule $\log(a^n) = n\times \log(a)$.
|
||||
|
||||
\[x = \frac{\log(2.29)}{\log(0.23)}\]
|
||||
\end{enumerate}
|
||||
\end{solution}
|
||||
|
||||
\begin{exercise}[subtitle={Restaurant}]
|
||||
Un \emph{food truck}, ouvert le midi et le soir, propose deux types de formules :
|
||||
|
||||
\setlength\parindent{10mm}
|
||||
\begin{itemize}
|
||||
\item la formule \emph{Burger} ;
|
||||
\item la formule \emph{Wok}.
|
||||
\end{itemize}
|
||||
\setlength\parindent{0mm}
|
||||
|
||||
\medskip
|
||||
|
||||
Le gérant a remarqué que 53\,\% de ses ventes ont lieu le midi. Le quart des ventes du midi correspondent à la formule \emph{Burger}, alors que 26\,\% des ventes du soir correspondent à la formule \emph{Wok}.
|
||||
|
||||
Le gérant se constitue un fichier en notant, pour chaque vente, la formule choisie et le moment de cette vente (midi ou soir).
|
||||
|
||||
On prélève une fiche de façon équiprobable. On définit les quatre évènements suivants:
|
||||
|
||||
\begin{enumerate}
|
||||
\item $M$ : \og la fiche correspond à une vente du midi\fg{} ;
|
||||
\item $S$ : \og la fiche correspond à une vente du soir\fg {};
|
||||
\item $W$ : \og la fiche correspond à une formule \emph{Wok} \fg{} ;
|
||||
\item $B$ : \og la fiche correspond à une formule \emph{Burger} \fg.
|
||||
\end{enumerate}
|
||||
\setlength\parindent{0mm}
|
||||
|
||||
\medskip
|
||||
|
||||
\begin{enumerate}
|
||||
\item Recopier puis compléter l'arbre pondéré
|
||||
|
||||
\begin{center}
|
||||
\begin{tikzpicture}[sloped]
|
||||
\node {.}
|
||||
child {node {$M$}
|
||||
child {node {$W$}
|
||||
edge from parent
|
||||
node[above] {...}
|
||||
}
|
||||
child {node {$B$}
|
||||
edge from parent
|
||||
node[above] {...}
|
||||
}
|
||||
edge from parent
|
||||
node[above] {...}
|
||||
}
|
||||
child[missing] {}
|
||||
child { node {$S$}
|
||||
child {node {$W$}
|
||||
edge from parent
|
||||
node[above] {...}
|
||||
}
|
||||
child {node {$B$}
|
||||
edge from parent
|
||||
node[above] {...}
|
||||
}
|
||||
edge from parent
|
||||
node[above] {...}
|
||||
} ;
|
||||
\end{tikzpicture}
|
||||
\end{center}
|
||||
|
||||
\item Calculer la probabilité de l'évènement $M \cap W$. Interpréter ce résultat dans le contexte de l'exercice.
|
||||
\item Montrer que la probabilité que la fiche choisie corresponde à une formule \emph{Burger} est égale à $0.4803$.
|
||||
\item On a prélevé une fiche correspondant à la formule \emph{Burger}. Quelle est la probabilité, arrondie au millième, que la vente ait eu lieu le soir?
|
||||
\end{enumerate}
|
||||
\end{exercise}
|
||||
|
||||
\begin{solution}
|
||||
\begin{enumerate}
|
||||
\item
|
||||
\begin{center}
|
||||
\begin{tikzpicture}[sloped]
|
||||
\node {.}
|
||||
child {node {$M$}
|
||||
child {node {$W$}
|
||||
edge from parent
|
||||
node[above] {$0.75$}
|
||||
}
|
||||
child {node {$B$}
|
||||
edge from parent
|
||||
node[above] {$0.25$}
|
||||
}
|
||||
edge from parent
|
||||
node[above] {$0.53$}
|
||||
}
|
||||
child[missing] {}
|
||||
child { node {$S$}
|
||||
child {node {$W$}
|
||||
edge from parent
|
||||
node[above] {$0.26$}
|
||||
}
|
||||
child {node {$B$}
|
||||
edge from parent
|
||||
node[above] {$0.74$}
|
||||
}
|
||||
edge from parent
|
||||
node[above] {$0.47$}
|
||||
} ;
|
||||
\end{tikzpicture}
|
||||
\end{center}
|
||||
\item On calcule la probabilité que la vente soit un wok et ait eu lieu à midi
|
||||
\[ P(M\cap W) = P(M) \times P_M(W) = 0.53 \times 0.75 = 0.3975 \]
|
||||
\item Probabilité que la vente soit un burger.
|
||||
\[
|
||||
P(B) = P(M\cap B) + P(S\cap B) = 0.53 \times 0.75 + 0.47 \times 0.26 = 0.4803
|
||||
\]
|
||||
\item On cherche à calculer la quantité $P_B(S)$. Pour cela on utilise la formule de Bayes
|
||||
\[
|
||||
P_B(S) = \frac{P(B\cap S)}{P(B)} = \frac{P_S(B) \times P(S)}{P(B)} = \frac{0.74\times 0.47}{0.4803} = 0.7241307516135749 \approx 0.724
|
||||
\]
|
||||
\end{enumerate}
|
||||
\end{solution}
|
||||
|
||||
\begin{exercise}[subtitle={Continent plastique}]
|
||||
\textit{Les quantités évoqués dans cette exercice sont générés au hasard et sont donc complètement farfelus.}
|
||||
\medskip
|
||||
Le \og continent de plastique\fg{} est la plus grande des plaques de déchets plastiques évoluant sur les océans. Elle occupe actuellement dans l'océan Pacifique une surface dont l'aire est évaluée à plus de $1,6$ million de km$^2$, entre Hawaï et la Californie.
|
||||
|
||||
En 2017, des scientifiques ont estimé qu'il y avait $3$ millions de tonnes de déchets plastiques qui était déversé chaque année dans les océans et que cette quantité augmentait de $17\n\%$ par chaque année.
|
||||
|
||||
On modélise l'évolution de la masse de ces déchets plastiques déversée chaque année, si rien n'est fait pour la réduire, par une suite géométrique $\left(u_n\right)$. L'arrondi au centième du terme $u_n$ représente la masse de ces déchets déversée chaque année, exprimée en million de tonnes, pour l'année $(2017 + n)$.
|
||||
|
||||
\medskip
|
||||
|
||||
\begin{enumerate}
|
||||
\item Expliquer pourquoi la suite $u_n$ est géométrique?
|
||||
\item Calculer $u_1$ et $u_2$.
|
||||
\item Exprimer $u_n$ en fonction de $n$.
|
||||
\item Au début de l'année 2017, il y avait $300$ millions de tonnes de déchets plastique. Calculer la quantité totale de déchets plastiques en 2030.
|
||||
\item On souhaite déterminer en quelle année la masse totale de ces déchets plastiques aura pour la première fois augmenté de $50$\,\% par rapport à sa valeur de 2017.
|
||||
\begin{enumerate}
|
||||
\item Recopier et compléter l'algorithme ci-dessous pour que la variable $N$ contienne la réponse au problème posé.
|
||||
|
||||
\begin{center}
|
||||
\begin{tabularx}{0.4\linewidth}{|X|}\hline
|
||||
$N = 2017$\\
|
||||
$U = 3$ \\
|
||||
$S = 300 + U$ \\
|
||||
while $S < 450$: \\
|
||||
\hspace{1cm} $N = \ldots$\\
|
||||
\hspace{1cm} $U = \ldots$\\
|
||||
\hspace{1cm} $S = \ldots$\\
|
||||
\hline
|
||||
\end{tabularx}
|
||||
\end{center}
|
||||
\item Que contiennent les variables $S$, $U$ et $N$ après exécution de cet algorithme ?
|
||||
|
||||
Interpréter les résultats dans le contexte de l'exercice.
|
||||
\end{enumerate}
|
||||
\end{enumerate}
|
||||
\end{exercise}
|
||||
|
||||
\begin{solution}
|
||||
\begin{enumerate}
|
||||
\item Une augmentation de $17\,\%$ revient à multiplier la quantité par $1.17$. La suite est donc bien géométrique. Son premier terme est $u_0 = 3$ et sa raison est $q = 1.17$
|
||||
\item
|
||||
\[
|
||||
u_1 = u_0 * 1.17 = 3.51
|
||||
\]
|
||||
\[
|
||||
u_2 = u_0 * 1.17^2 = 4.1067
|
||||
\]
|
||||
\item
|
||||
\[
|
||||
u_n = u_0 \times q^n = 3 \times 1.17^n
|
||||
\]
|
||||
\item On calcule la quantité totale déversée entre 2017 et 2030.
|
||||
\[
|
||||
\sum_{n = 0}^{13} u_n = u_0 \times \frac{1-q^{13}}{1-q} = 3 \times \frac{1 - 1.17^{13}}{1 - 1.17} = 118.21
|
||||
\]
|
||||
On en déduit la quantité totale de déchets en 2030
|
||||
\[
|
||||
300 + 118.21 = 418.21
|
||||
\]
|
||||
\item
|
||||
\begin{enumerate}
|
||||
\item ~
|
||||
\begin{center}
|
||||
\begin{tabularx}{0.4\linewidth}{|X|}\hline
|
||||
$N \gets 2017$\\
|
||||
$U \gets 3$ \\
|
||||
$S \gets 300 + U$ \\
|
||||
Tant que $S < 450$ \\
|
||||
\hspace{1cm} $N \gets N + 1$\\
|
||||
\hspace{1cm} $U \gets U * 1.17$\\
|
||||
\hspace{1cm} $S \gets S + u$\\
|
||||
Fin Tant que\\\hline
|
||||
\end{tabularx}
|
||||
\end{center}
|
||||
\item \textit{Pas de correction automatisé}
|
||||
\end{enumerate}
|
||||
\end{enumerate}
|
||||
\end{solution}
|
||||
|
||||
\end{document}
|
||||
|
||||
%%% Local Variables:
|
||||
%%% mode: latex
|
||||
%%% TeX-master: "master"
|
||||
%%% End:
|
262
TST/DS/DS_21_04_07/TST1/07_210407_DS8.tex
Normal file
262
TST/DS/DS_21_04_07/TST1/07_210407_DS8.tex
Normal file
@ -0,0 +1,262 @@
|
||||
\documentclass[a4paper,10pt]{article}
|
||||
\usepackage{myXsim}
|
||||
|
||||
% Title Page
|
||||
\title{DS8 \hfill COUBAT Alexis}
|
||||
\tribe{TST}
|
||||
\date{\hfillÀ render pour le Mercredi 7 avril}
|
||||
|
||||
\xsimsetup{
|
||||
solution/print = false
|
||||
}
|
||||
|
||||
\begin{document}
|
||||
\maketitle
|
||||
|
||||
\begin{exercise}[subtitle={Automatismes}]
|
||||
\textit{Toutes les questions de cette exercice sont indépendantes et peuvent être répondus séparément}
|
||||
\begin{enumerate}
|
||||
\item De janvier à septembre, une quantité a augmenté de $16\,\%$. Faire un schéma pour représenter la situation puis calculer le taux d'évolution moyen mensuel.
|
||||
\item Une quantité augmente de $16\,\%$ par ans. En 2020, elle est de 120\euro. Quelle était sa valeur en 2019? Faire un schéma pour représenter la situation.
|
||||
\item Déterminer l'équation de la droite \\
|
||||
\begin{tikzpicture}[xscale=0.8, yscale=0.5]
|
||||
\tkzInit[xmin=-5,xmax=5,xstep=1,
|
||||
ymin=-5,ymax=5,ystep=1]
|
||||
\tkzGrid
|
||||
\tkzAxeXY
|
||||
\tkzFct[domain=-5:5,color=red,very thick]%
|
||||
{4.0*\x -4};
|
||||
\end{tikzpicture}
|
||||
\item Résoudre l'équation $10 \times 0.06^x = 34$
|
||||
\end{enumerate}
|
||||
\end{exercise}
|
||||
|
||||
\begin{solution}
|
||||
\begin{enumerate}
|
||||
\item On veut partager cette évolution en 8 évolutions.
|
||||
\[
|
||||
\left(1 + \frac{16}{100}\right)^{\frac{1}{8}} = 1.0187
|
||||
\]
|
||||
Donc le taux d'évolution moyen est
|
||||
\[
|
||||
t_m = 1.0187 - 1 = 0.01869999999999994
|
||||
\]
|
||||
\item Coefficient multiplicateur pour revenir en arrière
|
||||
\[
|
||||
CM = (1 + \frac{16}{100})^{-1} = 0.8621
|
||||
\]
|
||||
On en déduit la quantité en 2019
|
||||
\[
|
||||
120 * 0.8621 = 103.452
|
||||
\]
|
||||
\item L'équation de la droite est
|
||||
\[
|
||||
y = 4.0 x -4
|
||||
\]
|
||||
\item Il faut penser à faire la division à par $10$ avant d'utiliser le log car sinon, on ne peut pas utiliser la formule $\log(a^n) = n\times \log(a)$.
|
||||
|
||||
\[x = \frac{\log(3.4)}{\log(0.06)}\]
|
||||
\end{enumerate}
|
||||
\end{solution}
|
||||
|
||||
\begin{exercise}[subtitle={Restaurant}]
|
||||
Un \emph{food truck}, ouvert le midi et le soir, propose deux types de formules :
|
||||
|
||||
\setlength\parindent{10mm}
|
||||
\begin{itemize}
|
||||
\item la formule \emph{Burger} ;
|
||||
\item la formule \emph{Wok}.
|
||||
\end{itemize}
|
||||
\setlength\parindent{0mm}
|
||||
|
||||
\medskip
|
||||
|
||||
Le gérant a remarqué que 1\,\% de ses ventes ont lieu le midi. Le quart des ventes du midi correspondent à la formule \emph{Burger}, alors que 92\,\% des ventes du soir correspondent à la formule \emph{Wok}.
|
||||
|
||||
Le gérant se constitue un fichier en notant, pour chaque vente, la formule choisie et le moment de cette vente (midi ou soir).
|
||||
|
||||
On prélève une fiche de façon équiprobable. On définit les quatre évènements suivants:
|
||||
|
||||
\begin{enumerate}
|
||||
\item $M$ : \og la fiche correspond à une vente du midi\fg{} ;
|
||||
\item $S$ : \og la fiche correspond à une vente du soir\fg {};
|
||||
\item $W$ : \og la fiche correspond à une formule \emph{Wok} \fg{} ;
|
||||
\item $B$ : \og la fiche correspond à une formule \emph{Burger} \fg.
|
||||
\end{enumerate}
|
||||
\setlength\parindent{0mm}
|
||||
|
||||
\medskip
|
||||
|
||||
\begin{enumerate}
|
||||
\item Recopier puis compléter l'arbre pondéré
|
||||
|
||||
\begin{center}
|
||||
\begin{tikzpicture}[sloped]
|
||||
\node {.}
|
||||
child {node {$M$}
|
||||
child {node {$W$}
|
||||
edge from parent
|
||||
node[above] {...}
|
||||
}
|
||||
child {node {$B$}
|
||||
edge from parent
|
||||
node[above] {...}
|
||||
}
|
||||
edge from parent
|
||||
node[above] {...}
|
||||
}
|
||||
child[missing] {}
|
||||
child { node {$S$}
|
||||
child {node {$W$}
|
||||
edge from parent
|
||||
node[above] {...}
|
||||
}
|
||||
child {node {$B$}
|
||||
edge from parent
|
||||
node[above] {...}
|
||||
}
|
||||
edge from parent
|
||||
node[above] {...}
|
||||
} ;
|
||||
\end{tikzpicture}
|
||||
\end{center}
|
||||
|
||||
\item Calculer la probabilité de l'évènement $M \cap W$. Interpréter ce résultat dans le contexte de l'exercice.
|
||||
\item Montrer que la probabilité que la fiche choisie corresponde à une formule \emph{Burger} est égale à $0.0817$.
|
||||
\item On a prélevé une fiche correspondant à la formule \emph{Burger}. Quelle est la probabilité, arrondie au millième, que la vente ait eu lieu le soir?
|
||||
\end{enumerate}
|
||||
\end{exercise}
|
||||
|
||||
\begin{solution}
|
||||
\begin{enumerate}
|
||||
\item
|
||||
\begin{center}
|
||||
\begin{tikzpicture}[sloped]
|
||||
\node {.}
|
||||
child {node {$M$}
|
||||
child {node {$W$}
|
||||
edge from parent
|
||||
node[above] {$0.75$}
|
||||
}
|
||||
child {node {$B$}
|
||||
edge from parent
|
||||
node[above] {$0.25$}
|
||||
}
|
||||
edge from parent
|
||||
node[above] {$0.01$}
|
||||
}
|
||||
child[missing] {}
|
||||
child { node {$S$}
|
||||
child {node {$W$}
|
||||
edge from parent
|
||||
node[above] {$0.92$}
|
||||
}
|
||||
child {node {$B$}
|
||||
edge from parent
|
||||
node[above] {$0.08$}
|
||||
}
|
||||
edge from parent
|
||||
node[above] {$0.99$}
|
||||
} ;
|
||||
\end{tikzpicture}
|
||||
\end{center}
|
||||
\item On calcule la probabilité que la vente soit un wok et ait eu lieu à midi
|
||||
\[ P(M\cap W) = P(M) \times P_M(W) = 0.01 \times 0.75 = 0.0075 \]
|
||||
\item Probabilité que la vente soit un burger.
|
||||
\[
|
||||
P(B) = P(M\cap B) + P(S\cap B) = 0.01 \times 0.75 + 0.99 \times 0.92 = 0.0817
|
||||
\]
|
||||
\item On cherche à calculer la quantité $P_B(S)$. Pour cela on utilise la formule de Bayes
|
||||
\[
|
||||
P_B(S) = \frac{P(B\cap S)}{P(B)} = \frac{P_S(B) \times P(S)}{P(B)} = \frac{0.08\times 0.99}{0.0817} = 0.9694002447980418 \approx 0.969
|
||||
\]
|
||||
\end{enumerate}
|
||||
\end{solution}
|
||||
|
||||
\begin{exercise}[subtitle={Continent plastique}]
|
||||
\textit{Les quantités évoqués dans cette exercice sont générés au hasard et sont donc complètement farfelus.}
|
||||
\medskip
|
||||
Le \og continent de plastique\fg{} est la plus grande des plaques de déchets plastiques évoluant sur les océans. Elle occupe actuellement dans l'océan Pacifique une surface dont l'aire est évaluée à plus de $1,6$ million de km$^2$, entre Hawaï et la Californie.
|
||||
|
||||
En 2017, des scientifiques ont estimé qu'il y avait $12$ millions de tonnes de déchets plastiques qui était déversé chaque année dans les océans et que cette quantité augmentait de $14\n\%$ par chaque année.
|
||||
|
||||
On modélise l'évolution de la masse de ces déchets plastiques déversée chaque année, si rien n'est fait pour la réduire, par une suite géométrique $\left(u_n\right)$. L'arrondi au centième du terme $u_n$ représente la masse de ces déchets déversée chaque année, exprimée en million de tonnes, pour l'année $(2017 + n)$.
|
||||
|
||||
\medskip
|
||||
|
||||
\begin{enumerate}
|
||||
\item Expliquer pourquoi la suite $u_n$ est géométrique?
|
||||
\item Calculer $u_1$ et $u_2$.
|
||||
\item Exprimer $u_n$ en fonction de $n$.
|
||||
\item Au début de l'année 2017, il y avait $300$ millions de tonnes de déchets plastique. Calculer la quantité totale de déchets plastiques en 2030.
|
||||
\item On souhaite déterminer en quelle année la masse totale de ces déchets plastiques aura pour la première fois augmenté de $50$\,\% par rapport à sa valeur de 2017.
|
||||
\begin{enumerate}
|
||||
\item Recopier et compléter l'algorithme ci-dessous pour que la variable $N$ contienne la réponse au problème posé.
|
||||
|
||||
\begin{center}
|
||||
\begin{tabularx}{0.4\linewidth}{|X|}\hline
|
||||
$N = 2017$\\
|
||||
$U = 12$ \\
|
||||
$S = 300 + U$ \\
|
||||
while $S < 450$: \\
|
||||
\hspace{1cm} $N = \ldots$\\
|
||||
\hspace{1cm} $U = \ldots$\\
|
||||
\hspace{1cm} $S = \ldots$\\
|
||||
\hline
|
||||
\end{tabularx}
|
||||
\end{center}
|
||||
\item Que contiennent les variables $S$, $U$ et $N$ après exécution de cet algorithme ?
|
||||
|
||||
Interpréter les résultats dans le contexte de l'exercice.
|
||||
\end{enumerate}
|
||||
\end{enumerate}
|
||||
\end{exercise}
|
||||
|
||||
\begin{solution}
|
||||
\begin{enumerate}
|
||||
\item Une augmentation de $14\,\%$ revient à multiplier la quantité par $1.1400000000000001$. La suite est donc bien géométrique. Son premier terme est $u_0 = 12$ et sa raison est $q = 1.1400000000000001$
|
||||
\item
|
||||
\[
|
||||
u_1 = u_0 * 1.1400000000000001 = 13.680000000000001
|
||||
\]
|
||||
\[
|
||||
u_2 = u_0 * 1.1400000000000001^2 = 15.5952
|
||||
\]
|
||||
\item
|
||||
\[
|
||||
u_n = u_0 \times q^n = 12 \times 1.1400000000000001^n
|
||||
\]
|
||||
\item On calcule la quantité totale déversée entre 2017 et 2030.
|
||||
\[
|
||||
\sum_{n = 0}^{13} u_n = u_0 \times \frac{1-q^{13}}{1-q} = 12 \times \frac{1 - 1.1400000000000001^{13}}{1 - 1.1400000000000001} = 385.06
|
||||
\]
|
||||
On en déduit la quantité totale de déchets en 2030
|
||||
\[
|
||||
300 + 385.06 = 685.06
|
||||
\]
|
||||
\item
|
||||
\begin{enumerate}
|
||||
\item ~
|
||||
\begin{center}
|
||||
\begin{tabularx}{0.4\linewidth}{|X|}\hline
|
||||
$N \gets 2017$\\
|
||||
$U \gets 12$ \\
|
||||
$S \gets 300 + U$ \\
|
||||
Tant que $S < 450$ \\
|
||||
\hspace{1cm} $N \gets N + 1$\\
|
||||
\hspace{1cm} $U \gets U * 1.1400000000000001$\\
|
||||
\hspace{1cm} $S \gets S + u$\\
|
||||
Fin Tant que\\\hline
|
||||
\end{tabularx}
|
||||
\end{center}
|
||||
\item \textit{Pas de correction automatisé}
|
||||
\end{enumerate}
|
||||
\end{enumerate}
|
||||
\end{solution}
|
||||
|
||||
\end{document}
|
||||
|
||||
%%% Local Variables:
|
||||
%%% mode: latex
|
||||
%%% TeX-master: "master"
|
||||
%%% End:
|
262
TST/DS/DS_21_04_07/TST1/08_210407_DS8.tex
Normal file
262
TST/DS/DS_21_04_07/TST1/08_210407_DS8.tex
Normal file
@ -0,0 +1,262 @@
|
||||
\documentclass[a4paper,10pt]{article}
|
||||
\usepackage{myXsim}
|
||||
|
||||
% Title Page
|
||||
\title{DS8 \hfill COULLON Anis}
|
||||
\tribe{TST}
|
||||
\date{\hfillÀ render pour le Mercredi 7 avril}
|
||||
|
||||
\xsimsetup{
|
||||
solution/print = false
|
||||
}
|
||||
|
||||
\begin{document}
|
||||
\maketitle
|
||||
|
||||
\begin{exercise}[subtitle={Automatismes}]
|
||||
\textit{Toutes les questions de cette exercice sont indépendantes et peuvent être répondus séparément}
|
||||
\begin{enumerate}
|
||||
\item De janvier à septembre, une quantité a augmenté de $14\,\%$. Faire un schéma pour représenter la situation puis calculer le taux d'évolution moyen mensuel.
|
||||
\item Une quantité augmente de $14\,\%$ par ans. En 2020, elle est de 125\euro. Quelle était sa valeur en 2019? Faire un schéma pour représenter la situation.
|
||||
\item Déterminer l'équation de la droite \\
|
||||
\begin{tikzpicture}[xscale=0.8, yscale=0.5]
|
||||
\tkzInit[xmin=-5,xmax=5,xstep=1,
|
||||
ymin=-5,ymax=5,ystep=1]
|
||||
\tkzGrid
|
||||
\tkzAxeXY
|
||||
\tkzFct[domain=-5:5,color=red,very thick]%
|
||||
{0.5*\x -1};
|
||||
\end{tikzpicture}
|
||||
\item Résoudre l'équation $6 \times 0.82^x = 19$
|
||||
\end{enumerate}
|
||||
\end{exercise}
|
||||
|
||||
\begin{solution}
|
||||
\begin{enumerate}
|
||||
\item On veut partager cette évolution en 8 évolutions.
|
||||
\[
|
||||
\left(1 + \frac{14}{100}\right)^{\frac{1}{8}} = 1.0165
|
||||
\]
|
||||
Donc le taux d'évolution moyen est
|
||||
\[
|
||||
t_m = 1.0165 - 1 = 0.01649999999999996
|
||||
\]
|
||||
\item Coefficient multiplicateur pour revenir en arrière
|
||||
\[
|
||||
CM = (1 + \frac{14}{100})^{-1} = 0.8772
|
||||
\]
|
||||
On en déduit la quantité en 2019
|
||||
\[
|
||||
125 * 0.8772 = 109.64999999999999
|
||||
\]
|
||||
\item L'équation de la droite est
|
||||
\[
|
||||
y = 0.5 x -1
|
||||
\]
|
||||
\item Il faut penser à faire la division à par $6$ avant d'utiliser le log car sinon, on ne peut pas utiliser la formule $\log(a^n) = n\times \log(a)$.
|
||||
|
||||
\[x = \frac{\log(3.17)}{\log(0.82)}\]
|
||||
\end{enumerate}
|
||||
\end{solution}
|
||||
|
||||
\begin{exercise}[subtitle={Restaurant}]
|
||||
Un \emph{food truck}, ouvert le midi et le soir, propose deux types de formules :
|
||||
|
||||
\setlength\parindent{10mm}
|
||||
\begin{itemize}
|
||||
\item la formule \emph{Burger} ;
|
||||
\item la formule \emph{Wok}.
|
||||
\end{itemize}
|
||||
\setlength\parindent{0mm}
|
||||
|
||||
\medskip
|
||||
|
||||
Le gérant a remarqué que 30\,\% de ses ventes ont lieu le midi. Le quart des ventes du midi correspondent à la formule \emph{Burger}, alors que 27\,\% des ventes du soir correspondent à la formule \emph{Wok}.
|
||||
|
||||
Le gérant se constitue un fichier en notant, pour chaque vente, la formule choisie et le moment de cette vente (midi ou soir).
|
||||
|
||||
On prélève une fiche de façon équiprobable. On définit les quatre évènements suivants:
|
||||
|
||||
\begin{enumerate}
|
||||
\item $M$ : \og la fiche correspond à une vente du midi\fg{} ;
|
||||
\item $S$ : \og la fiche correspond à une vente du soir\fg {};
|
||||
\item $W$ : \og la fiche correspond à une formule \emph{Wok} \fg{} ;
|
||||
\item $B$ : \og la fiche correspond à une formule \emph{Burger} \fg.
|
||||
\end{enumerate}
|
||||
\setlength\parindent{0mm}
|
||||
|
||||
\medskip
|
||||
|
||||
\begin{enumerate}
|
||||
\item Recopier puis compléter l'arbre pondéré
|
||||
|
||||
\begin{center}
|
||||
\begin{tikzpicture}[sloped]
|
||||
\node {.}
|
||||
child {node {$M$}
|
||||
child {node {$W$}
|
||||
edge from parent
|
||||
node[above] {...}
|
||||
}
|
||||
child {node {$B$}
|
||||
edge from parent
|
||||
node[above] {...}
|
||||
}
|
||||
edge from parent
|
||||
node[above] {...}
|
||||
}
|
||||
child[missing] {}
|
||||
child { node {$S$}
|
||||
child {node {$W$}
|
||||
edge from parent
|
||||
node[above] {...}
|
||||
}
|
||||
child {node {$B$}
|
||||
edge from parent
|
||||
node[above] {...}
|
||||
}
|
||||
edge from parent
|
||||
node[above] {...}
|
||||
} ;
|
||||
\end{tikzpicture}
|
||||
\end{center}
|
||||
|
||||
\item Calculer la probabilité de l'évènement $M \cap W$. Interpréter ce résultat dans le contexte de l'exercice.
|
||||
\item Montrer que la probabilité que la fiche choisie corresponde à une formule \emph{Burger} est égale à $0.586$.
|
||||
\item On a prélevé une fiche correspondant à la formule \emph{Burger}. Quelle est la probabilité, arrondie au millième, que la vente ait eu lieu le soir?
|
||||
\end{enumerate}
|
||||
\end{exercise}
|
||||
|
||||
\begin{solution}
|
||||
\begin{enumerate}
|
||||
\item
|
||||
\begin{center}
|
||||
\begin{tikzpicture}[sloped]
|
||||
\node {.}
|
||||
child {node {$M$}
|
||||
child {node {$W$}
|
||||
edge from parent
|
||||
node[above] {$0.75$}
|
||||
}
|
||||
child {node {$B$}
|
||||
edge from parent
|
||||
node[above] {$0.25$}
|
||||
}
|
||||
edge from parent
|
||||
node[above] {$0.3$}
|
||||
}
|
||||
child[missing] {}
|
||||
child { node {$S$}
|
||||
child {node {$W$}
|
||||
edge from parent
|
||||
node[above] {$0.27$}
|
||||
}
|
||||
child {node {$B$}
|
||||
edge from parent
|
||||
node[above] {$0.73$}
|
||||
}
|
||||
edge from parent
|
||||
node[above] {$0.7$}
|
||||
} ;
|
||||
\end{tikzpicture}
|
||||
\end{center}
|
||||
\item On calcule la probabilité que la vente soit un wok et ait eu lieu à midi
|
||||
\[ P(M\cap W) = P(M) \times P_M(W) = 0.3 \times 0.75 = 0.225 \]
|
||||
\item Probabilité que la vente soit un burger.
|
||||
\[
|
||||
P(B) = P(M\cap B) + P(S\cap B) = 0.3 \times 0.75 + 0.7 \times 0.27 = 0.586
|
||||
\]
|
||||
\item On cherche à calculer la quantité $P_B(S)$. Pour cela on utilise la formule de Bayes
|
||||
\[
|
||||
P_B(S) = \frac{P(B\cap S)}{P(B)} = \frac{P_S(B) \times P(S)}{P(B)} = \frac{0.73\times 0.7}{0.586} = 0.8720136518771332 \approx 0.872
|
||||
\]
|
||||
\end{enumerate}
|
||||
\end{solution}
|
||||
|
||||
\begin{exercise}[subtitle={Continent plastique}]
|
||||
\textit{Les quantités évoqués dans cette exercice sont générés au hasard et sont donc complètement farfelus.}
|
||||
\medskip
|
||||
Le \og continent de plastique\fg{} est la plus grande des plaques de déchets plastiques évoluant sur les océans. Elle occupe actuellement dans l'océan Pacifique une surface dont l'aire est évaluée à plus de $1,6$ million de km$^2$, entre Hawaï et la Californie.
|
||||
|
||||
En 2017, des scientifiques ont estimé qu'il y avait $14$ millions de tonnes de déchets plastiques qui était déversé chaque année dans les océans et que cette quantité augmentait de $25\n\%$ par chaque année.
|
||||
|
||||
On modélise l'évolution de la masse de ces déchets plastiques déversée chaque année, si rien n'est fait pour la réduire, par une suite géométrique $\left(u_n\right)$. L'arrondi au centième du terme $u_n$ représente la masse de ces déchets déversée chaque année, exprimée en million de tonnes, pour l'année $(2017 + n)$.
|
||||
|
||||
\medskip
|
||||
|
||||
\begin{enumerate}
|
||||
\item Expliquer pourquoi la suite $u_n$ est géométrique?
|
||||
\item Calculer $u_1$ et $u_2$.
|
||||
\item Exprimer $u_n$ en fonction de $n$.
|
||||
\item Au début de l'année 2017, il y avait $300$ millions de tonnes de déchets plastique. Calculer la quantité totale de déchets plastiques en 2030.
|
||||
\item On souhaite déterminer en quelle année la masse totale de ces déchets plastiques aura pour la première fois augmenté de $50$\,\% par rapport à sa valeur de 2017.
|
||||
\begin{enumerate}
|
||||
\item Recopier et compléter l'algorithme ci-dessous pour que la variable $N$ contienne la réponse au problème posé.
|
||||
|
||||
\begin{center}
|
||||
\begin{tabularx}{0.4\linewidth}{|X|}\hline
|
||||
$N = 2017$\\
|
||||
$U = 14$ \\
|
||||
$S = 300 + U$ \\
|
||||
while $S < 450$: \\
|
||||
\hspace{1cm} $N = \ldots$\\
|
||||
\hspace{1cm} $U = \ldots$\\
|
||||
\hspace{1cm} $S = \ldots$\\
|
||||
\hline
|
||||
\end{tabularx}
|
||||
\end{center}
|
||||
\item Que contiennent les variables $S$, $U$ et $N$ après exécution de cet algorithme ?
|
||||
|
||||
Interpréter les résultats dans le contexte de l'exercice.
|
||||
\end{enumerate}
|
||||
\end{enumerate}
|
||||
\end{exercise}
|
||||
|
||||
\begin{solution}
|
||||
\begin{enumerate}
|
||||
\item Une augmentation de $25\,\%$ revient à multiplier la quantité par $1.25$. La suite est donc bien géométrique. Son premier terme est $u_0 = 14$ et sa raison est $q = 1.25$
|
||||
\item
|
||||
\[
|
||||
u_1 = u_0 * 1.25 = 17.5
|
||||
\]
|
||||
\[
|
||||
u_2 = u_0 * 1.25^2 = 21.875
|
||||
\]
|
||||
\item
|
||||
\[
|
||||
u_n = u_0 \times q^n = 14 \times 1.25^n
|
||||
\]
|
||||
\item On calcule la quantité totale déversée entre 2017 et 2030.
|
||||
\[
|
||||
\sum_{n = 0}^{13} u_n = u_0 \times \frac{1-q^{13}}{1-q} = 14 \times \frac{1 - 1.25^{13}}{1 - 1.25} = 962.63
|
||||
\]
|
||||
On en déduit la quantité totale de déchets en 2030
|
||||
\[
|
||||
300 + 962.63 = 1262.63
|
||||
\]
|
||||
\item
|
||||
\begin{enumerate}
|
||||
\item ~
|
||||
\begin{center}
|
||||
\begin{tabularx}{0.4\linewidth}{|X|}\hline
|
||||
$N \gets 2017$\\
|
||||
$U \gets 14$ \\
|
||||
$S \gets 300 + U$ \\
|
||||
Tant que $S < 450$ \\
|
||||
\hspace{1cm} $N \gets N + 1$\\
|
||||
\hspace{1cm} $U \gets U * 1.25$\\
|
||||
\hspace{1cm} $S \gets S + u$\\
|
||||
Fin Tant que\\\hline
|
||||
\end{tabularx}
|
||||
\end{center}
|
||||
\item \textit{Pas de correction automatisé}
|
||||
\end{enumerate}
|
||||
\end{enumerate}
|
||||
\end{solution}
|
||||
|
||||
\end{document}
|
||||
|
||||
%%% Local Variables:
|
||||
%%% mode: latex
|
||||
%%% TeX-master: "master"
|
||||
%%% End:
|
262
TST/DS/DS_21_04_07/TST1/09_210407_DS8.tex
Normal file
262
TST/DS/DS_21_04_07/TST1/09_210407_DS8.tex
Normal file
@ -0,0 +1,262 @@
|
||||
\documentclass[a4paper,10pt]{article}
|
||||
\usepackage{myXsim}
|
||||
|
||||
% Title Page
|
||||
\title{DS8 \hfill DINGER Sölen}
|
||||
\tribe{TST}
|
||||
\date{\hfillÀ render pour le Mercredi 7 avril}
|
||||
|
||||
\xsimsetup{
|
||||
solution/print = false
|
||||
}
|
||||
|
||||
\begin{document}
|
||||
\maketitle
|
||||
|
||||
\begin{exercise}[subtitle={Automatismes}]
|
||||
\textit{Toutes les questions de cette exercice sont indépendantes et peuvent être répondus séparément}
|
||||
\begin{enumerate}
|
||||
\item De janvier à septembre, une quantité a augmenté de $30\,\%$. Faire un schéma pour représenter la situation puis calculer le taux d'évolution moyen mensuel.
|
||||
\item Une quantité augmente de $30\,\%$ par ans. En 2020, elle est de 141\euro. Quelle était sa valeur en 2019? Faire un schéma pour représenter la situation.
|
||||
\item Déterminer l'équation de la droite \\
|
||||
\begin{tikzpicture}[xscale=0.8, yscale=0.5]
|
||||
\tkzInit[xmin=-5,xmax=5,xstep=1,
|
||||
ymin=-5,ymax=5,ystep=1]
|
||||
\tkzGrid
|
||||
\tkzAxeXY
|
||||
\tkzFct[domain=-5:5,color=red,very thick]%
|
||||
{2.0*\x -3};
|
||||
\end{tikzpicture}
|
||||
\item Résoudre l'équation $5 \times 0.8^x = 11$
|
||||
\end{enumerate}
|
||||
\end{exercise}
|
||||
|
||||
\begin{solution}
|
||||
\begin{enumerate}
|
||||
\item On veut partager cette évolution en 8 évolutions.
|
||||
\[
|
||||
\left(1 + \frac{30}{100}\right)^{\frac{1}{8}} = 1.0333
|
||||
\]
|
||||
Donc le taux d'évolution moyen est
|
||||
\[
|
||||
t_m = 1.0333 - 1 = 0.03330000000000011
|
||||
\]
|
||||
\item Coefficient multiplicateur pour revenir en arrière
|
||||
\[
|
||||
CM = (1 + \frac{30}{100})^{-1} = 0.7692
|
||||
\]
|
||||
On en déduit la quantité en 2019
|
||||
\[
|
||||
141 * 0.7692 = 108.4572
|
||||
\]
|
||||
\item L'équation de la droite est
|
||||
\[
|
||||
y = 2.0 x -3
|
||||
\]
|
||||
\item Il faut penser à faire la division à par $5$ avant d'utiliser le log car sinon, on ne peut pas utiliser la formule $\log(a^n) = n\times \log(a)$.
|
||||
|
||||
\[x = \frac{\log(2.2)}{\log(0.8)}\]
|
||||
\end{enumerate}
|
||||
\end{solution}
|
||||
|
||||
\begin{exercise}[subtitle={Restaurant}]
|
||||
Un \emph{food truck}, ouvert le midi et le soir, propose deux types de formules :
|
||||
|
||||
\setlength\parindent{10mm}
|
||||
\begin{itemize}
|
||||
\item la formule \emph{Burger} ;
|
||||
\item la formule \emph{Wok}.
|
||||
\end{itemize}
|
||||
\setlength\parindent{0mm}
|
||||
|
||||
\medskip
|
||||
|
||||
Le gérant a remarqué que 57\,\% de ses ventes ont lieu le midi. Le quart des ventes du midi correspondent à la formule \emph{Burger}, alors que 90\,\% des ventes du soir correspondent à la formule \emph{Wok}.
|
||||
|
||||
Le gérant se constitue un fichier en notant, pour chaque vente, la formule choisie et le moment de cette vente (midi ou soir).
|
||||
|
||||
On prélève une fiche de façon équiprobable. On définit les quatre évènements suivants:
|
||||
|
||||
\begin{enumerate}
|
||||
\item $M$ : \og la fiche correspond à une vente du midi\fg{} ;
|
||||
\item $S$ : \og la fiche correspond à une vente du soir\fg {};
|
||||
\item $W$ : \og la fiche correspond à une formule \emph{Wok} \fg{} ;
|
||||
\item $B$ : \og la fiche correspond à une formule \emph{Burger} \fg.
|
||||
\end{enumerate}
|
||||
\setlength\parindent{0mm}
|
||||
|
||||
\medskip
|
||||
|
||||
\begin{enumerate}
|
||||
\item Recopier puis compléter l'arbre pondéré
|
||||
|
||||
\begin{center}
|
||||
\begin{tikzpicture}[sloped]
|
||||
\node {.}
|
||||
child {node {$M$}
|
||||
child {node {$W$}
|
||||
edge from parent
|
||||
node[above] {...}
|
||||
}
|
||||
child {node {$B$}
|
||||
edge from parent
|
||||
node[above] {...}
|
||||
}
|
||||
edge from parent
|
||||
node[above] {...}
|
||||
}
|
||||
child[missing] {}
|
||||
child { node {$S$}
|
||||
child {node {$W$}
|
||||
edge from parent
|
||||
node[above] {...}
|
||||
}
|
||||
child {node {$B$}
|
||||
edge from parent
|
||||
node[above] {...}
|
||||
}
|
||||
edge from parent
|
||||
node[above] {...}
|
||||
} ;
|
||||
\end{tikzpicture}
|
||||
\end{center}
|
||||
|
||||
\item Calculer la probabilité de l'évènement $M \cap W$. Interpréter ce résultat dans le contexte de l'exercice.
|
||||
\item Montrer que la probabilité que la fiche choisie corresponde à une formule \emph{Burger} est égale à $0.187$.
|
||||
\item On a prélevé une fiche correspondant à la formule \emph{Burger}. Quelle est la probabilité, arrondie au millième, que la vente ait eu lieu le soir?
|
||||
\end{enumerate}
|
||||
\end{exercise}
|
||||
|
||||
\begin{solution}
|
||||
\begin{enumerate}
|
||||
\item
|
||||
\begin{center}
|
||||
\begin{tikzpicture}[sloped]
|
||||
\node {.}
|
||||
child {node {$M$}
|
||||
child {node {$W$}
|
||||
edge from parent
|
||||
node[above] {$0.75$}
|
||||
}
|
||||
child {node {$B$}
|
||||
edge from parent
|
||||
node[above] {$0.25$}
|
||||
}
|
||||
edge from parent
|
||||
node[above] {$0.58$}
|
||||
}
|
||||
child[missing] {}
|
||||
child { node {$S$}
|
||||
child {node {$W$}
|
||||
edge from parent
|
||||
node[above] {$0.9$}
|
||||
}
|
||||
child {node {$B$}
|
||||
edge from parent
|
||||
node[above] {$0.1$}
|
||||
}
|
||||
edge from parent
|
||||
node[above] {$0.42$}
|
||||
} ;
|
||||
\end{tikzpicture}
|
||||
\end{center}
|
||||
\item On calcule la probabilité que la vente soit un wok et ait eu lieu à midi
|
||||
\[ P(M\cap W) = P(M) \times P_M(W) = 0.58 \times 0.75 = 0.435 \]
|
||||
\item Probabilité que la vente soit un burger.
|
||||
\[
|
||||
P(B) = P(M\cap B) + P(S\cap B) = 0.58 \times 0.75 + 0.42 \times 0.9 = 0.187
|
||||
\]
|
||||
\item On cherche à calculer la quantité $P_B(S)$. Pour cela on utilise la formule de Bayes
|
||||
\[
|
||||
P_B(S) = \frac{P(B\cap S)}{P(B)} = \frac{P_S(B) \times P(S)}{P(B)} = \frac{0.1\times 0.42}{0.187} = 0.22459893048128343 \approx 0.225
|
||||
\]
|
||||
\end{enumerate}
|
||||
\end{solution}
|
||||
|
||||
\begin{exercise}[subtitle={Continent plastique}]
|
||||
\textit{Les quantités évoqués dans cette exercice sont générés au hasard et sont donc complètement farfelus.}
|
||||
\medskip
|
||||
Le \og continent de plastique\fg{} est la plus grande des plaques de déchets plastiques évoluant sur les océans. Elle occupe actuellement dans l'océan Pacifique une surface dont l'aire est évaluée à plus de $1,6$ million de km$^2$, entre Hawaï et la Californie.
|
||||
|
||||
En 2017, des scientifiques ont estimé qu'il y avait $14$ millions de tonnes de déchets plastiques qui était déversé chaque année dans les océans et que cette quantité augmentait de $19\n\%$ par chaque année.
|
||||
|
||||
On modélise l'évolution de la masse de ces déchets plastiques déversée chaque année, si rien n'est fait pour la réduire, par une suite géométrique $\left(u_n\right)$. L'arrondi au centième du terme $u_n$ représente la masse de ces déchets déversée chaque année, exprimée en million de tonnes, pour l'année $(2017 + n)$.
|
||||
|
||||
\medskip
|
||||
|
||||
\begin{enumerate}
|
||||
\item Expliquer pourquoi la suite $u_n$ est géométrique?
|
||||
\item Calculer $u_1$ et $u_2$.
|
||||
\item Exprimer $u_n$ en fonction de $n$.
|
||||
\item Au début de l'année 2017, il y avait $300$ millions de tonnes de déchets plastique. Calculer la quantité totale de déchets plastiques en 2030.
|
||||
\item On souhaite déterminer en quelle année la masse totale de ces déchets plastiques aura pour la première fois augmenté de $50$\,\% par rapport à sa valeur de 2017.
|
||||
\begin{enumerate}
|
||||
\item Recopier et compléter l'algorithme ci-dessous pour que la variable $N$ contienne la réponse au problème posé.
|
||||
|
||||
\begin{center}
|
||||
\begin{tabularx}{0.4\linewidth}{|X|}\hline
|
||||
$N = 2017$\\
|
||||
$U = 14$ \\
|
||||
$S = 300 + U$ \\
|
||||
while $S < 450$: \\
|
||||
\hspace{1cm} $N = \ldots$\\
|
||||
\hspace{1cm} $U = \ldots$\\
|
||||
\hspace{1cm} $S = \ldots$\\
|
||||
\hline
|
||||
\end{tabularx}
|
||||
\end{center}
|
||||
\item Que contiennent les variables $S$, $U$ et $N$ après exécution de cet algorithme ?
|
||||
|
||||
Interpréter les résultats dans le contexte de l'exercice.
|
||||
\end{enumerate}
|
||||
\end{enumerate}
|
||||
\end{exercise}
|
||||
|
||||
\begin{solution}
|
||||
\begin{enumerate}
|
||||
\item Une augmentation de $19\,\%$ revient à multiplier la quantité par $1.19$. La suite est donc bien géométrique. Son premier terme est $u_0 = 14$ et sa raison est $q = 1.19$
|
||||
\item
|
||||
\[
|
||||
u_1 = u_0 * 1.19 = 16.66
|
||||
\]
|
||||
\[
|
||||
u_2 = u_0 * 1.19^2 = 19.8254
|
||||
\]
|
||||
\item
|
||||
\[
|
||||
u_n = u_0 \times q^n = 14 \times 1.19^n
|
||||
\]
|
||||
\item On calcule la quantité totale déversée entre 2017 et 2030.
|
||||
\[
|
||||
\sum_{n = 0}^{13} u_n = u_0 \times \frac{1-q^{13}}{1-q} = 14 \times \frac{1 - 1.19^{13}}{1 - 1.19} = 633.42
|
||||
\]
|
||||
On en déduit la quantité totale de déchets en 2030
|
||||
\[
|
||||
300 + 633.42 = 933.42
|
||||
\]
|
||||
\item
|
||||
\begin{enumerate}
|
||||
\item ~
|
||||
\begin{center}
|
||||
\begin{tabularx}{0.4\linewidth}{|X|}\hline
|
||||
$N \gets 2017$\\
|
||||
$U \gets 14$ \\
|
||||
$S \gets 300 + U$ \\
|
||||
Tant que $S < 450$ \\
|
||||
\hspace{1cm} $N \gets N + 1$\\
|
||||
\hspace{1cm} $U \gets U * 1.19$\\
|
||||
\hspace{1cm} $S \gets S + u$\\
|
||||
Fin Tant que\\\hline
|
||||
\end{tabularx}
|
||||
\end{center}
|
||||
\item \textit{Pas de correction automatisé}
|
||||
\end{enumerate}
|
||||
\end{enumerate}
|
||||
\end{solution}
|
||||
|
||||
\end{document}
|
||||
|
||||
%%% Local Variables:
|
||||
%%% mode: latex
|
||||
%%% TeX-master: "master"
|
||||
%%% End:
|
262
TST/DS/DS_21_04_07/TST1/10_210407_DS8.tex
Normal file
262
TST/DS/DS_21_04_07/TST1/10_210407_DS8.tex
Normal file
@ -0,0 +1,262 @@
|
||||
\documentclass[a4paper,10pt]{article}
|
||||
\usepackage{myXsim}
|
||||
|
||||
% Title Page
|
||||
\title{DS8 \hfill EYRAUD Cynthia}
|
||||
\tribe{TST}
|
||||
\date{\hfillÀ render pour le Mercredi 7 avril}
|
||||
|
||||
\xsimsetup{
|
||||
solution/print = false
|
||||
}
|
||||
|
||||
\begin{document}
|
||||
\maketitle
|
||||
|
||||
\begin{exercise}[subtitle={Automatismes}]
|
||||
\textit{Toutes les questions de cette exercice sont indépendantes et peuvent être répondus séparément}
|
||||
\begin{enumerate}
|
||||
\item De janvier à septembre, une quantité a augmenté de $29\,\%$. Faire un schéma pour représenter la situation puis calculer le taux d'évolution moyen mensuel.
|
||||
\item Une quantité augmente de $29\,\%$ par ans. En 2020, elle est de 130\euro. Quelle était sa valeur en 2019? Faire un schéma pour représenter la situation.
|
||||
\item Déterminer l'équation de la droite \\
|
||||
\begin{tikzpicture}[xscale=0.8, yscale=0.5]
|
||||
\tkzInit[xmin=-5,xmax=5,xstep=1,
|
||||
ymin=-5,ymax=5,ystep=1]
|
||||
\tkzGrid
|
||||
\tkzAxeXY
|
||||
\tkzFct[domain=-5:5,color=red,very thick]%
|
||||
{4.0*\x -4};
|
||||
\end{tikzpicture}
|
||||
\item Résoudre l'équation $6 \times 0.48^x = 28$
|
||||
\end{enumerate}
|
||||
\end{exercise}
|
||||
|
||||
\begin{solution}
|
||||
\begin{enumerate}
|
||||
\item On veut partager cette évolution en 8 évolutions.
|
||||
\[
|
||||
\left(1 + \frac{29}{100}\right)^{\frac{1}{8}} = 1.0323
|
||||
\]
|
||||
Donc le taux d'évolution moyen est
|
||||
\[
|
||||
t_m = 1.0323 - 1 = 0.032299999999999995
|
||||
\]
|
||||
\item Coefficient multiplicateur pour revenir en arrière
|
||||
\[
|
||||
CM = (1 + \frac{29}{100})^{-1} = 0.7752
|
||||
\]
|
||||
On en déduit la quantité en 2019
|
||||
\[
|
||||
130 * 0.7752 = 100.776
|
||||
\]
|
||||
\item L'équation de la droite est
|
||||
\[
|
||||
y = 4.0 x -4
|
||||
\]
|
||||
\item Il faut penser à faire la division à par $6$ avant d'utiliser le log car sinon, on ne peut pas utiliser la formule $\log(a^n) = n\times \log(a)$.
|
||||
|
||||
\[x = \frac{\log(4.67)}{\log(0.48)}\]
|
||||
\end{enumerate}
|
||||
\end{solution}
|
||||
|
||||
\begin{exercise}[subtitle={Restaurant}]
|
||||
Un \emph{food truck}, ouvert le midi et le soir, propose deux types de formules :
|
||||
|
||||
\setlength\parindent{10mm}
|
||||
\begin{itemize}
|
||||
\item la formule \emph{Burger} ;
|
||||
\item la formule \emph{Wok}.
|
||||
\end{itemize}
|
||||
\setlength\parindent{0mm}
|
||||
|
||||
\medskip
|
||||
|
||||
Le gérant a remarqué que 3\,\% de ses ventes ont lieu le midi. Le quart des ventes du midi correspondent à la formule \emph{Burger}, alors que 33\,\% des ventes du soir correspondent à la formule \emph{Wok}.
|
||||
|
||||
Le gérant se constitue un fichier en notant, pour chaque vente, la formule choisie et le moment de cette vente (midi ou soir).
|
||||
|
||||
On prélève une fiche de façon équiprobable. On définit les quatre évènements suivants:
|
||||
|
||||
\begin{enumerate}
|
||||
\item $M$ : \og la fiche correspond à une vente du midi\fg{} ;
|
||||
\item $S$ : \og la fiche correspond à une vente du soir\fg {};
|
||||
\item $W$ : \og la fiche correspond à une formule \emph{Wok} \fg{} ;
|
||||
\item $B$ : \og la fiche correspond à une formule \emph{Burger} \fg.
|
||||
\end{enumerate}
|
||||
\setlength\parindent{0mm}
|
||||
|
||||
\medskip
|
||||
|
||||
\begin{enumerate}
|
||||
\item Recopier puis compléter l'arbre pondéré
|
||||
|
||||
\begin{center}
|
||||
\begin{tikzpicture}[sloped]
|
||||
\node {.}
|
||||
child {node {$M$}
|
||||
child {node {$W$}
|
||||
edge from parent
|
||||
node[above] {...}
|
||||
}
|
||||
child {node {$B$}
|
||||
edge from parent
|
||||
node[above] {...}
|
||||
}
|
||||
edge from parent
|
||||
node[above] {...}
|
||||
}
|
||||
child[missing] {}
|
||||
child { node {$S$}
|
||||
child {node {$W$}
|
||||
edge from parent
|
||||
node[above] {...}
|
||||
}
|
||||
child {node {$B$}
|
||||
edge from parent
|
||||
node[above] {...}
|
||||
}
|
||||
edge from parent
|
||||
node[above] {...}
|
||||
} ;
|
||||
\end{tikzpicture}
|
||||
\end{center}
|
||||
|
||||
\item Calculer la probabilité de l'évènement $M \cap W$. Interpréter ce résultat dans le contexte de l'exercice.
|
||||
\item Montrer que la probabilité que la fiche choisie corresponde à une formule \emph{Burger} est égale à $0.6574$.
|
||||
\item On a prélevé une fiche correspondant à la formule \emph{Burger}. Quelle est la probabilité, arrondie au millième, que la vente ait eu lieu le soir?
|
||||
\end{enumerate}
|
||||
\end{exercise}
|
||||
|
||||
\begin{solution}
|
||||
\begin{enumerate}
|
||||
\item
|
||||
\begin{center}
|
||||
\begin{tikzpicture}[sloped]
|
||||
\node {.}
|
||||
child {node {$M$}
|
||||
child {node {$W$}
|
||||
edge from parent
|
||||
node[above] {$0.75$}
|
||||
}
|
||||
child {node {$B$}
|
||||
edge from parent
|
||||
node[above] {$0.25$}
|
||||
}
|
||||
edge from parent
|
||||
node[above] {$0.03$}
|
||||
}
|
||||
child[missing] {}
|
||||
child { node {$S$}
|
||||
child {node {$W$}
|
||||
edge from parent
|
||||
node[above] {$0.33$}
|
||||
}
|
||||
child {node {$B$}
|
||||
edge from parent
|
||||
node[above] {$0.67$}
|
||||
}
|
||||
edge from parent
|
||||
node[above] {$0.97$}
|
||||
} ;
|
||||
\end{tikzpicture}
|
||||
\end{center}
|
||||
\item On calcule la probabilité que la vente soit un wok et ait eu lieu à midi
|
||||
\[ P(M\cap W) = P(M) \times P_M(W) = 0.03 \times 0.75 = 0.0225 \]
|
||||
\item Probabilité que la vente soit un burger.
|
||||
\[
|
||||
P(B) = P(M\cap B) + P(S\cap B) = 0.03 \times 0.75 + 0.97 \times 0.33 = 0.6574
|
||||
\]
|
||||
\item On cherche à calculer la quantité $P_B(S)$. Pour cela on utilise la formule de Bayes
|
||||
\[
|
||||
P_B(S) = \frac{P(B\cap S)}{P(B)} = \frac{P_S(B) \times P(S)}{P(B)} = \frac{0.67\times 0.97}{0.6574} = 0.9885914207484029 \approx 0.989
|
||||
\]
|
||||
\end{enumerate}
|
||||
\end{solution}
|
||||
|
||||
\begin{exercise}[subtitle={Continent plastique}]
|
||||
\textit{Les quantités évoqués dans cette exercice sont générés au hasard et sont donc complètement farfelus.}
|
||||
\medskip
|
||||
Le \og continent de plastique\fg{} est la plus grande des plaques de déchets plastiques évoluant sur les océans. Elle occupe actuellement dans l'océan Pacifique une surface dont l'aire est évaluée à plus de $1,6$ million de km$^2$, entre Hawaï et la Californie.
|
||||
|
||||
En 2017, des scientifiques ont estimé qu'il y avait $18$ millions de tonnes de déchets plastiques qui était déversé chaque année dans les océans et que cette quantité augmentait de $25\n\%$ par chaque année.
|
||||
|
||||
On modélise l'évolution de la masse de ces déchets plastiques déversée chaque année, si rien n'est fait pour la réduire, par une suite géométrique $\left(u_n\right)$. L'arrondi au centième du terme $u_n$ représente la masse de ces déchets déversée chaque année, exprimée en million de tonnes, pour l'année $(2017 + n)$.
|
||||
|
||||
\medskip
|
||||
|
||||
\begin{enumerate}
|
||||
\item Expliquer pourquoi la suite $u_n$ est géométrique?
|
||||
\item Calculer $u_1$ et $u_2$.
|
||||
\item Exprimer $u_n$ en fonction de $n$.
|
||||
\item Au début de l'année 2017, il y avait $300$ millions de tonnes de déchets plastique. Calculer la quantité totale de déchets plastiques en 2030.
|
||||
\item On souhaite déterminer en quelle année la masse totale de ces déchets plastiques aura pour la première fois augmenté de $50$\,\% par rapport à sa valeur de 2017.
|
||||
\begin{enumerate}
|
||||
\item Recopier et compléter l'algorithme ci-dessous pour que la variable $N$ contienne la réponse au problème posé.
|
||||
|
||||
\begin{center}
|
||||
\begin{tabularx}{0.4\linewidth}{|X|}\hline
|
||||
$N = 2017$\\
|
||||
$U = 18$ \\
|
||||
$S = 300 + U$ \\
|
||||
while $S < 450$: \\
|
||||
\hspace{1cm} $N = \ldots$\\
|
||||
\hspace{1cm} $U = \ldots$\\
|
||||
\hspace{1cm} $S = \ldots$\\
|
||||
\hline
|
||||
\end{tabularx}
|
||||
\end{center}
|
||||
\item Que contiennent les variables $S$, $U$ et $N$ après exécution de cet algorithme ?
|
||||
|
||||
Interpréter les résultats dans le contexte de l'exercice.
|
||||
\end{enumerate}
|
||||
\end{enumerate}
|
||||
\end{exercise}
|
||||
|
||||
\begin{solution}
|
||||
\begin{enumerate}
|
||||
\item Une augmentation de $25\,\%$ revient à multiplier la quantité par $1.25$. La suite est donc bien géométrique. Son premier terme est $u_0 = 18$ et sa raison est $q = 1.25$
|
||||
\item
|
||||
\[
|
||||
u_1 = u_0 * 1.25 = 22.5
|
||||
\]
|
||||
\[
|
||||
u_2 = u_0 * 1.25^2 = 28.125
|
||||
\]
|
||||
\item
|
||||
\[
|
||||
u_n = u_0 \times q^n = 18 \times 1.25^n
|
||||
\]
|
||||
\item On calcule la quantité totale déversée entre 2017 et 2030.
|
||||
\[
|
||||
\sum_{n = 0}^{13} u_n = u_0 \times \frac{1-q^{13}}{1-q} = 18 \times \frac{1 - 1.25^{13}}{1 - 1.25} = 1237.67
|
||||
\]
|
||||
On en déduit la quantité totale de déchets en 2030
|
||||
\[
|
||||
300 + 1237.67 = 1537.67
|
||||
\]
|
||||
\item
|
||||
\begin{enumerate}
|
||||
\item ~
|
||||
\begin{center}
|
||||
\begin{tabularx}{0.4\linewidth}{|X|}\hline
|
||||
$N \gets 2017$\\
|
||||
$U \gets 18$ \\
|
||||
$S \gets 300 + U$ \\
|
||||
Tant que $S < 450$ \\
|
||||
\hspace{1cm} $N \gets N + 1$\\
|
||||
\hspace{1cm} $U \gets U * 1.25$\\
|
||||
\hspace{1cm} $S \gets S + u$\\
|
||||
Fin Tant que\\\hline
|
||||
\end{tabularx}
|
||||
\end{center}
|
||||
\item \textit{Pas de correction automatisé}
|
||||
\end{enumerate}
|
||||
\end{enumerate}
|
||||
\end{solution}
|
||||
|
||||
\end{document}
|
||||
|
||||
%%% Local Variables:
|
||||
%%% mode: latex
|
||||
%%% TeX-master: "master"
|
||||
%%% End:
|
262
TST/DS/DS_21_04_07/TST1/11_210407_DS8.tex
Normal file
262
TST/DS/DS_21_04_07/TST1/11_210407_DS8.tex
Normal file
@ -0,0 +1,262 @@
|
||||
\documentclass[a4paper,10pt]{article}
|
||||
\usepackage{myXsim}
|
||||
|
||||
% Title Page
|
||||
\title{DS8 \hfill FERREIRA Léo}
|
||||
\tribe{TST}
|
||||
\date{\hfillÀ render pour le Mercredi 7 avril}
|
||||
|
||||
\xsimsetup{
|
||||
solution/print = false
|
||||
}
|
||||
|
||||
\begin{document}
|
||||
\maketitle
|
||||
|
||||
\begin{exercise}[subtitle={Automatismes}]
|
||||
\textit{Toutes les questions de cette exercice sont indépendantes et peuvent être répondus séparément}
|
||||
\begin{enumerate}
|
||||
\item De janvier à septembre, une quantité a augmenté de $10\,\%$. Faire un schéma pour représenter la situation puis calculer le taux d'évolution moyen mensuel.
|
||||
\item Une quantité augmente de $10\,\%$ par ans. En 2020, elle est de 123\euro. Quelle était sa valeur en 2019? Faire un schéma pour représenter la situation.
|
||||
\item Déterminer l'équation de la droite \\
|
||||
\begin{tikzpicture}[xscale=0.8, yscale=0.5]
|
||||
\tkzInit[xmin=-5,xmax=5,xstep=1,
|
||||
ymin=-5,ymax=5,ystep=1]
|
||||
\tkzGrid
|
||||
\tkzAxeXY
|
||||
\tkzFct[domain=-5:5,color=red,very thick]%
|
||||
{1.5*\x -3};
|
||||
\end{tikzpicture}
|
||||
\item Résoudre l'équation $5 \times 0.79^x = 30$
|
||||
\end{enumerate}
|
||||
\end{exercise}
|
||||
|
||||
\begin{solution}
|
||||
\begin{enumerate}
|
||||
\item On veut partager cette évolution en 8 évolutions.
|
||||
\[
|
||||
\left(1 + \frac{10}{100}\right)^{\frac{1}{8}} = 1.012
|
||||
\]
|
||||
Donc le taux d'évolution moyen est
|
||||
\[
|
||||
t_m = 1.012 - 1 = 0.01200000000000001
|
||||
\]
|
||||
\item Coefficient multiplicateur pour revenir en arrière
|
||||
\[
|
||||
CM = (1 + \frac{10}{100})^{-1} = 0.9091
|
||||
\]
|
||||
On en déduit la quantité en 2019
|
||||
\[
|
||||
123 * 0.9091 = 111.8193
|
||||
\]
|
||||
\item L'équation de la droite est
|
||||
\[
|
||||
y = 1.5 x -3
|
||||
\]
|
||||
\item Il faut penser à faire la division à par $5$ avant d'utiliser le log car sinon, on ne peut pas utiliser la formule $\log(a^n) = n\times \log(a)$.
|
||||
|
||||
\[x = \frac{\log(6.0)}{\log(0.79)}\]
|
||||
\end{enumerate}
|
||||
\end{solution}
|
||||
|
||||
\begin{exercise}[subtitle={Restaurant}]
|
||||
Un \emph{food truck}, ouvert le midi et le soir, propose deux types de formules :
|
||||
|
||||
\setlength\parindent{10mm}
|
||||
\begin{itemize}
|
||||
\item la formule \emph{Burger} ;
|
||||
\item la formule \emph{Wok}.
|
||||
\end{itemize}
|
||||
\setlength\parindent{0mm}
|
||||
|
||||
\medskip
|
||||
|
||||
Le gérant a remarqué que 18\,\% de ses ventes ont lieu le midi. Le quart des ventes du midi correspondent à la formule \emph{Burger}, alors que 37\,\% des ventes du soir correspondent à la formule \emph{Wok}.
|
||||
|
||||
Le gérant se constitue un fichier en notant, pour chaque vente, la formule choisie et le moment de cette vente (midi ou soir).
|
||||
|
||||
On prélève une fiche de façon équiprobable. On définit les quatre évènements suivants:
|
||||
|
||||
\begin{enumerate}
|
||||
\item $M$ : \og la fiche correspond à une vente du midi\fg{} ;
|
||||
\item $S$ : \og la fiche correspond à une vente du soir\fg {};
|
||||
\item $W$ : \og la fiche correspond à une formule \emph{Wok} \fg{} ;
|
||||
\item $B$ : \og la fiche correspond à une formule \emph{Burger} \fg.
|
||||
\end{enumerate}
|
||||
\setlength\parindent{0mm}
|
||||
|
||||
\medskip
|
||||
|
||||
\begin{enumerate}
|
||||
\item Recopier puis compléter l'arbre pondéré
|
||||
|
||||
\begin{center}
|
||||
\begin{tikzpicture}[sloped]
|
||||
\node {.}
|
||||
child {node {$M$}
|
||||
child {node {$W$}
|
||||
edge from parent
|
||||
node[above] {...}
|
||||
}
|
||||
child {node {$B$}
|
||||
edge from parent
|
||||
node[above] {...}
|
||||
}
|
||||
edge from parent
|
||||
node[above] {...}
|
||||
}
|
||||
child[missing] {}
|
||||
child { node {$S$}
|
||||
child {node {$W$}
|
||||
edge from parent
|
||||
node[above] {...}
|
||||
}
|
||||
child {node {$B$}
|
||||
edge from parent
|
||||
node[above] {...}
|
||||
}
|
||||
edge from parent
|
||||
node[above] {...}
|
||||
} ;
|
||||
\end{tikzpicture}
|
||||
\end{center}
|
||||
|
||||
\item Calculer la probabilité de l'évènement $M \cap W$. Interpréter ce résultat dans le contexte de l'exercice.
|
||||
\item Montrer que la probabilité que la fiche choisie corresponde à une formule \emph{Burger} est égale à $0.5616$.
|
||||
\item On a prélevé une fiche correspondant à la formule \emph{Burger}. Quelle est la probabilité, arrondie au millième, que la vente ait eu lieu le soir?
|
||||
\end{enumerate}
|
||||
\end{exercise}
|
||||
|
||||
\begin{solution}
|
||||
\begin{enumerate}
|
||||
\item
|
||||
\begin{center}
|
||||
\begin{tikzpicture}[sloped]
|
||||
\node {.}
|
||||
child {node {$M$}
|
||||
child {node {$W$}
|
||||
edge from parent
|
||||
node[above] {$0.75$}
|
||||
}
|
||||
child {node {$B$}
|
||||
edge from parent
|
||||
node[above] {$0.25$}
|
||||
}
|
||||
edge from parent
|
||||
node[above] {$0.18$}
|
||||
}
|
||||
child[missing] {}
|
||||
child { node {$S$}
|
||||
child {node {$W$}
|
||||
edge from parent
|
||||
node[above] {$0.37$}
|
||||
}
|
||||
child {node {$B$}
|
||||
edge from parent
|
||||
node[above] {$0.63$}
|
||||
}
|
||||
edge from parent
|
||||
node[above] {$0.82$}
|
||||
} ;
|
||||
\end{tikzpicture}
|
||||
\end{center}
|
||||
\item On calcule la probabilité que la vente soit un wok et ait eu lieu à midi
|
||||
\[ P(M\cap W) = P(M) \times P_M(W) = 0.18 \times 0.75 = 0.135 \]
|
||||
\item Probabilité que la vente soit un burger.
|
||||
\[
|
||||
P(B) = P(M\cap B) + P(S\cap B) = 0.18 \times 0.75 + 0.82 \times 0.37 = 0.5616
|
||||
\]
|
||||
\item On cherche à calculer la quantité $P_B(S)$. Pour cela on utilise la formule de Bayes
|
||||
\[
|
||||
P_B(S) = \frac{P(B\cap S)}{P(B)} = \frac{P_S(B) \times P(S)}{P(B)} = \frac{0.63\times 0.82}{0.5616} = 0.9198717948717948 \approx 0.92
|
||||
\]
|
||||
\end{enumerate}
|
||||
\end{solution}
|
||||
|
||||
\begin{exercise}[subtitle={Continent plastique}]
|
||||
\textit{Les quantités évoqués dans cette exercice sont générés au hasard et sont donc complètement farfelus.}
|
||||
\medskip
|
||||
Le \og continent de plastique\fg{} est la plus grande des plaques de déchets plastiques évoluant sur les océans. Elle occupe actuellement dans l'océan Pacifique une surface dont l'aire est évaluée à plus de $1,6$ million de km$^2$, entre Hawaï et la Californie.
|
||||
|
||||
En 2017, des scientifiques ont estimé qu'il y avait $3$ millions de tonnes de déchets plastiques qui était déversé chaque année dans les océans et que cette quantité augmentait de $19\n\%$ par chaque année.
|
||||
|
||||
On modélise l'évolution de la masse de ces déchets plastiques déversée chaque année, si rien n'est fait pour la réduire, par une suite géométrique $\left(u_n\right)$. L'arrondi au centième du terme $u_n$ représente la masse de ces déchets déversée chaque année, exprimée en million de tonnes, pour l'année $(2017 + n)$.
|
||||
|
||||
\medskip
|
||||
|
||||
\begin{enumerate}
|
||||
\item Expliquer pourquoi la suite $u_n$ est géométrique?
|
||||
\item Calculer $u_1$ et $u_2$.
|
||||
\item Exprimer $u_n$ en fonction de $n$.
|
||||
\item Au début de l'année 2017, il y avait $300$ millions de tonnes de déchets plastique. Calculer la quantité totale de déchets plastiques en 2030.
|
||||
\item On souhaite déterminer en quelle année la masse totale de ces déchets plastiques aura pour la première fois augmenté de $50$\,\% par rapport à sa valeur de 2017.
|
||||
\begin{enumerate}
|
||||
\item Recopier et compléter l'algorithme ci-dessous pour que la variable $N$ contienne la réponse au problème posé.
|
||||
|
||||
\begin{center}
|
||||
\begin{tabularx}{0.4\linewidth}{|X|}\hline
|
||||
$N = 2017$\\
|
||||
$U = 3$ \\
|
||||
$S = 300 + U$ \\
|
||||
while $S < 450$: \\
|
||||
\hspace{1cm} $N = \ldots$\\
|
||||
\hspace{1cm} $U = \ldots$\\
|
||||
\hspace{1cm} $S = \ldots$\\
|
||||
\hline
|
||||
\end{tabularx}
|
||||
\end{center}
|
||||
\item Que contiennent les variables $S$, $U$ et $N$ après exécution de cet algorithme ?
|
||||
|
||||
Interpréter les résultats dans le contexte de l'exercice.
|
||||
\end{enumerate}
|
||||
\end{enumerate}
|
||||
\end{exercise}
|
||||
|
||||
\begin{solution}
|
||||
\begin{enumerate}
|
||||
\item Une augmentation de $19\,\%$ revient à multiplier la quantité par $1.19$. La suite est donc bien géométrique. Son premier terme est $u_0 = 3$ et sa raison est $q = 1.19$
|
||||
\item
|
||||
\[
|
||||
u_1 = u_0 * 1.19 = 3.57
|
||||
\]
|
||||
\[
|
||||
u_2 = u_0 * 1.19^2 = 4.2483
|
||||
\]
|
||||
\item
|
||||
\[
|
||||
u_n = u_0 \times q^n = 3 \times 1.19^n
|
||||
\]
|
||||
\item On calcule la quantité totale déversée entre 2017 et 2030.
|
||||
\[
|
||||
\sum_{n = 0}^{13} u_n = u_0 \times \frac{1-q^{13}}{1-q} = 3 \times \frac{1 - 1.19^{13}}{1 - 1.19} = 135.73
|
||||
\]
|
||||
On en déduit la quantité totale de déchets en 2030
|
||||
\[
|
||||
300 + 135.73 = 435.73
|
||||
\]
|
||||
\item
|
||||
\begin{enumerate}
|
||||
\item ~
|
||||
\begin{center}
|
||||
\begin{tabularx}{0.4\linewidth}{|X|}\hline
|
||||
$N \gets 2017$\\
|
||||
$U \gets 3$ \\
|
||||
$S \gets 300 + U$ \\
|
||||
Tant que $S < 450$ \\
|
||||
\hspace{1cm} $N \gets N + 1$\\
|
||||
\hspace{1cm} $U \gets U * 1.19$\\
|
||||
\hspace{1cm} $S \gets S + u$\\
|
||||
Fin Tant que\\\hline
|
||||
\end{tabularx}
|
||||
\end{center}
|
||||
\item \textit{Pas de correction automatisé}
|
||||
\end{enumerate}
|
||||
\end{enumerate}
|
||||
\end{solution}
|
||||
|
||||
\end{document}
|
||||
|
||||
%%% Local Variables:
|
||||
%%% mode: latex
|
||||
%%% TeX-master: "master"
|
||||
%%% End:
|
262
TST/DS/DS_21_04_07/TST1/12_210407_DS8.tex
Normal file
262
TST/DS/DS_21_04_07/TST1/12_210407_DS8.tex
Normal file
@ -0,0 +1,262 @@
|
||||
\documentclass[a4paper,10pt]{article}
|
||||
\usepackage{myXsim}
|
||||
|
||||
% Title Page
|
||||
\title{DS8 \hfill FILALI Zakaria}
|
||||
\tribe{TST}
|
||||
\date{\hfillÀ render pour le Mercredi 7 avril}
|
||||
|
||||
\xsimsetup{
|
||||
solution/print = false
|
||||
}
|
||||
|
||||
\begin{document}
|
||||
\maketitle
|
||||
|
||||
\begin{exercise}[subtitle={Automatismes}]
|
||||
\textit{Toutes les questions de cette exercice sont indépendantes et peuvent être répondus séparément}
|
||||
\begin{enumerate}
|
||||
\item De janvier à septembre, une quantité a augmenté de $19\,\%$. Faire un schéma pour représenter la situation puis calculer le taux d'évolution moyen mensuel.
|
||||
\item Une quantité augmente de $19\,\%$ par ans. En 2020, elle est de 122\euro. Quelle était sa valeur en 2019? Faire un schéma pour représenter la situation.
|
||||
\item Déterminer l'équation de la droite \\
|
||||
\begin{tikzpicture}[xscale=0.8, yscale=0.5]
|
||||
\tkzInit[xmin=-5,xmax=5,xstep=1,
|
||||
ymin=-5,ymax=5,ystep=1]
|
||||
\tkzGrid
|
||||
\tkzAxeXY
|
||||
\tkzFct[domain=-5:5,color=red,very thick]%
|
||||
{1.0*\x -2};
|
||||
\end{tikzpicture}
|
||||
\item Résoudre l'équation $8 \times 0.25^x = 25$
|
||||
\end{enumerate}
|
||||
\end{exercise}
|
||||
|
||||
\begin{solution}
|
||||
\begin{enumerate}
|
||||
\item On veut partager cette évolution en 8 évolutions.
|
||||
\[
|
||||
\left(1 + \frac{19}{100}\right)^{\frac{1}{8}} = 1.022
|
||||
\]
|
||||
Donc le taux d'évolution moyen est
|
||||
\[
|
||||
t_m = 1.022 - 1 = 0.02200000000000002
|
||||
\]
|
||||
\item Coefficient multiplicateur pour revenir en arrière
|
||||
\[
|
||||
CM = (1 + \frac{19}{100})^{-1} = 0.8403
|
||||
\]
|
||||
On en déduit la quantité en 2019
|
||||
\[
|
||||
122 * 0.8403 = 102.51660000000001
|
||||
\]
|
||||
\item L'équation de la droite est
|
||||
\[
|
||||
y = 1.0 x -2
|
||||
\]
|
||||
\item Il faut penser à faire la division à par $8$ avant d'utiliser le log car sinon, on ne peut pas utiliser la formule $\log(a^n) = n\times \log(a)$.
|
||||
|
||||
\[x = \frac{\log(3.12)}{\log(0.25)}\]
|
||||
\end{enumerate}
|
||||
\end{solution}
|
||||
|
||||
\begin{exercise}[subtitle={Restaurant}]
|
||||
Un \emph{food truck}, ouvert le midi et le soir, propose deux types de formules :
|
||||
|
||||
\setlength\parindent{10mm}
|
||||
\begin{itemize}
|
||||
\item la formule \emph{Burger} ;
|
||||
\item la formule \emph{Wok}.
|
||||
\end{itemize}
|
||||
\setlength\parindent{0mm}
|
||||
|
||||
\medskip
|
||||
|
||||
Le gérant a remarqué que 47\,\% de ses ventes ont lieu le midi. Le quart des ventes du midi correspondent à la formule \emph{Burger}, alors que 69\,\% des ventes du soir correspondent à la formule \emph{Wok}.
|
||||
|
||||
Le gérant se constitue un fichier en notant, pour chaque vente, la formule choisie et le moment de cette vente (midi ou soir).
|
||||
|
||||
On prélève une fiche de façon équiprobable. On définit les quatre évènements suivants:
|
||||
|
||||
\begin{enumerate}
|
||||
\item $M$ : \og la fiche correspond à une vente du midi\fg{} ;
|
||||
\item $S$ : \og la fiche correspond à une vente du soir\fg {};
|
||||
\item $W$ : \og la fiche correspond à une formule \emph{Wok} \fg{} ;
|
||||
\item $B$ : \og la fiche correspond à une formule \emph{Burger} \fg.
|
||||
\end{enumerate}
|
||||
\setlength\parindent{0mm}
|
||||
|
||||
\medskip
|
||||
|
||||
\begin{enumerate}
|
||||
\item Recopier puis compléter l'arbre pondéré
|
||||
|
||||
\begin{center}
|
||||
\begin{tikzpicture}[sloped]
|
||||
\node {.}
|
||||
child {node {$M$}
|
||||
child {node {$W$}
|
||||
edge from parent
|
||||
node[above] {...}
|
||||
}
|
||||
child {node {$B$}
|
||||
edge from parent
|
||||
node[above] {...}
|
||||
}
|
||||
edge from parent
|
||||
node[above] {...}
|
||||
}
|
||||
child[missing] {}
|
||||
child { node {$S$}
|
||||
child {node {$W$}
|
||||
edge from parent
|
||||
node[above] {...}
|
||||
}
|
||||
child {node {$B$}
|
||||
edge from parent
|
||||
node[above] {...}
|
||||
}
|
||||
edge from parent
|
||||
node[above] {...}
|
||||
} ;
|
||||
\end{tikzpicture}
|
||||
\end{center}
|
||||
|
||||
\item Calculer la probabilité de l'évènement $M \cap W$. Interpréter ce résultat dans le contexte de l'exercice.
|
||||
\item Montrer que la probabilité que la fiche choisie corresponde à une formule \emph{Burger} est égale à $0.2818$.
|
||||
\item On a prélevé une fiche correspondant à la formule \emph{Burger}. Quelle est la probabilité, arrondie au millième, que la vente ait eu lieu le soir?
|
||||
\end{enumerate}
|
||||
\end{exercise}
|
||||
|
||||
\begin{solution}
|
||||
\begin{enumerate}
|
||||
\item
|
||||
\begin{center}
|
||||
\begin{tikzpicture}[sloped]
|
||||
\node {.}
|
||||
child {node {$M$}
|
||||
child {node {$W$}
|
||||
edge from parent
|
||||
node[above] {$0.75$}
|
||||
}
|
||||
child {node {$B$}
|
||||
edge from parent
|
||||
node[above] {$0.25$}
|
||||
}
|
||||
edge from parent
|
||||
node[above] {$0.47$}
|
||||
}
|
||||
child[missing] {}
|
||||
child { node {$S$}
|
||||
child {node {$W$}
|
||||
edge from parent
|
||||
node[above] {$0.69$}
|
||||
}
|
||||
child {node {$B$}
|
||||
edge from parent
|
||||
node[above] {$0.31$}
|
||||
}
|
||||
edge from parent
|
||||
node[above] {$0.53$}
|
||||
} ;
|
||||
\end{tikzpicture}
|
||||
\end{center}
|
||||
\item On calcule la probabilité que la vente soit un wok et ait eu lieu à midi
|
||||
\[ P(M\cap W) = P(M) \times P_M(W) = 0.47 \times 0.75 = 0.3525 \]
|
||||
\item Probabilité que la vente soit un burger.
|
||||
\[
|
||||
P(B) = P(M\cap B) + P(S\cap B) = 0.47 \times 0.75 + 0.53 \times 0.69 = 0.2818
|
||||
\]
|
||||
\item On cherche à calculer la quantité $P_B(S)$. Pour cela on utilise la formule de Bayes
|
||||
\[
|
||||
P_B(S) = \frac{P(B\cap S)}{P(B)} = \frac{P_S(B) \times P(S)}{P(B)} = \frac{0.31\times 0.53}{0.2818} = 0.5830376153300213 \approx 0.583
|
||||
\]
|
||||
\end{enumerate}
|
||||
\end{solution}
|
||||
|
||||
\begin{exercise}[subtitle={Continent plastique}]
|
||||
\textit{Les quantités évoqués dans cette exercice sont générés au hasard et sont donc complètement farfelus.}
|
||||
\medskip
|
||||
Le \og continent de plastique\fg{} est la plus grande des plaques de déchets plastiques évoluant sur les océans. Elle occupe actuellement dans l'océan Pacifique une surface dont l'aire est évaluée à plus de $1,6$ million de km$^2$, entre Hawaï et la Californie.
|
||||
|
||||
En 2017, des scientifiques ont estimé qu'il y avait $11$ millions de tonnes de déchets plastiques qui était déversé chaque année dans les océans et que cette quantité augmentait de $16\n\%$ par chaque année.
|
||||
|
||||
On modélise l'évolution de la masse de ces déchets plastiques déversée chaque année, si rien n'est fait pour la réduire, par une suite géométrique $\left(u_n\right)$. L'arrondi au centième du terme $u_n$ représente la masse de ces déchets déversée chaque année, exprimée en million de tonnes, pour l'année $(2017 + n)$.
|
||||
|
||||
\medskip
|
||||
|
||||
\begin{enumerate}
|
||||
\item Expliquer pourquoi la suite $u_n$ est géométrique?
|
||||
\item Calculer $u_1$ et $u_2$.
|
||||
\item Exprimer $u_n$ en fonction de $n$.
|
||||
\item Au début de l'année 2017, il y avait $300$ millions de tonnes de déchets plastique. Calculer la quantité totale de déchets plastiques en 2030.
|
||||
\item On souhaite déterminer en quelle année la masse totale de ces déchets plastiques aura pour la première fois augmenté de $50$\,\% par rapport à sa valeur de 2017.
|
||||
\begin{enumerate}
|
||||
\item Recopier et compléter l'algorithme ci-dessous pour que la variable $N$ contienne la réponse au problème posé.
|
||||
|
||||
\begin{center}
|
||||
\begin{tabularx}{0.4\linewidth}{|X|}\hline
|
||||
$N = 2017$\\
|
||||
$U = 11$ \\
|
||||
$S = 300 + U$ \\
|
||||
while $S < 450$: \\
|
||||
\hspace{1cm} $N = \ldots$\\
|
||||
\hspace{1cm} $U = \ldots$\\
|
||||
\hspace{1cm} $S = \ldots$\\
|
||||
\hline
|
||||
\end{tabularx}
|
||||
\end{center}
|
||||
\item Que contiennent les variables $S$, $U$ et $N$ après exécution de cet algorithme ?
|
||||
|
||||
Interpréter les résultats dans le contexte de l'exercice.
|
||||
\end{enumerate}
|
||||
\end{enumerate}
|
||||
\end{exercise}
|
||||
|
||||
\begin{solution}
|
||||
\begin{enumerate}
|
||||
\item Une augmentation de $16\,\%$ revient à multiplier la quantité par $1.16$. La suite est donc bien géométrique. Son premier terme est $u_0 = 11$ et sa raison est $q = 1.16$
|
||||
\item
|
||||
\[
|
||||
u_1 = u_0 * 1.16 = 12.76
|
||||
\]
|
||||
\[
|
||||
u_2 = u_0 * 1.16^2 = 14.8016
|
||||
\]
|
||||
\item
|
||||
\[
|
||||
u_n = u_0 \times q^n = 11 \times 1.16^n
|
||||
\]
|
||||
\item On calcule la quantité totale déversée entre 2017 et 2030.
|
||||
\[
|
||||
\sum_{n = 0}^{13} u_n = u_0 \times \frac{1-q^{13}}{1-q} = 11 \times \frac{1 - 1.16^{13}}{1 - 1.16} = 404.65
|
||||
\]
|
||||
On en déduit la quantité totale de déchets en 2030
|
||||
\[
|
||||
300 + 404.65 = 704.65
|
||||
\]
|
||||
\item
|
||||
\begin{enumerate}
|
||||
\item ~
|
||||
\begin{center}
|
||||
\begin{tabularx}{0.4\linewidth}{|X|}\hline
|
||||
$N \gets 2017$\\
|
||||
$U \gets 11$ \\
|
||||
$S \gets 300 + U$ \\
|
||||
Tant que $S < 450$ \\
|
||||
\hspace{1cm} $N \gets N + 1$\\
|
||||
\hspace{1cm} $U \gets U * 1.16$\\
|
||||
\hspace{1cm} $S \gets S + u$\\
|
||||
Fin Tant que\\\hline
|
||||
\end{tabularx}
|
||||
\end{center}
|
||||
\item \textit{Pas de correction automatisé}
|
||||
\end{enumerate}
|
||||
\end{enumerate}
|
||||
\end{solution}
|
||||
|
||||
\end{document}
|
||||
|
||||
%%% Local Variables:
|
||||
%%% mode: latex
|
||||
%%% TeX-master: "master"
|
||||
%%% End:
|
262
TST/DS/DS_21_04_07/TST1/13_210407_DS8.tex
Normal file
262
TST/DS/DS_21_04_07/TST1/13_210407_DS8.tex
Normal file
@ -0,0 +1,262 @@
|
||||
\documentclass[a4paper,10pt]{article}
|
||||
\usepackage{myXsim}
|
||||
|
||||
% Title Page
|
||||
\title{DS8 \hfill FOIGNY Romain}
|
||||
\tribe{TST}
|
||||
\date{\hfillÀ render pour le Mercredi 7 avril}
|
||||
|
||||
\xsimsetup{
|
||||
solution/print = false
|
||||
}
|
||||
|
||||
\begin{document}
|
||||
\maketitle
|
||||
|
||||
\begin{exercise}[subtitle={Automatismes}]
|
||||
\textit{Toutes les questions de cette exercice sont indépendantes et peuvent être répondus séparément}
|
||||
\begin{enumerate}
|
||||
\item De janvier à septembre, une quantité a augmenté de $22\,\%$. Faire un schéma pour représenter la situation puis calculer le taux d'évolution moyen mensuel.
|
||||
\item Une quantité augmente de $22\,\%$ par ans. En 2020, elle est de 125\euro. Quelle était sa valeur en 2019? Faire un schéma pour représenter la situation.
|
||||
\item Déterminer l'équation de la droite \\
|
||||
\begin{tikzpicture}[xscale=0.8, yscale=0.5]
|
||||
\tkzInit[xmin=-5,xmax=5,xstep=1,
|
||||
ymin=-5,ymax=5,ystep=1]
|
||||
\tkzGrid
|
||||
\tkzAxeXY
|
||||
\tkzFct[domain=-5:5,color=red,very thick]%
|
||||
{2.6666666666666665*\x -4};
|
||||
\end{tikzpicture}
|
||||
\item Résoudre l'équation $9 \times 0.35^x = 44$
|
||||
\end{enumerate}
|
||||
\end{exercise}
|
||||
|
||||
\begin{solution}
|
||||
\begin{enumerate}
|
||||
\item On veut partager cette évolution en 8 évolutions.
|
||||
\[
|
||||
\left(1 + \frac{22}{100}\right)^{\frac{1}{8}} = 1.0252
|
||||
\]
|
||||
Donc le taux d'évolution moyen est
|
||||
\[
|
||||
t_m = 1.0252 - 1 = 0.02519999999999989
|
||||
\]
|
||||
\item Coefficient multiplicateur pour revenir en arrière
|
||||
\[
|
||||
CM = (1 + \frac{22}{100})^{-1} = 0.8197
|
||||
\]
|
||||
On en déduit la quantité en 2019
|
||||
\[
|
||||
125 * 0.8197 = 102.46249999999999
|
||||
\]
|
||||
\item L'équation de la droite est
|
||||
\[
|
||||
y = 2.6666666666666665 x -4
|
||||
\]
|
||||
\item Il faut penser à faire la division à par $9$ avant d'utiliser le log car sinon, on ne peut pas utiliser la formule $\log(a^n) = n\times \log(a)$.
|
||||
|
||||
\[x = \frac{\log(4.89)}{\log(0.35)}\]
|
||||
\end{enumerate}
|
||||
\end{solution}
|
||||
|
||||
\begin{exercise}[subtitle={Restaurant}]
|
||||
Un \emph{food truck}, ouvert le midi et le soir, propose deux types de formules :
|
||||
|
||||
\setlength\parindent{10mm}
|
||||
\begin{itemize}
|
||||
\item la formule \emph{Burger} ;
|
||||
\item la formule \emph{Wok}.
|
||||
\end{itemize}
|
||||
\setlength\parindent{0mm}
|
||||
|
||||
\medskip
|
||||
|
||||
Le gérant a remarqué que 14\,\% de ses ventes ont lieu le midi. Le quart des ventes du midi correspondent à la formule \emph{Burger}, alors que 17\,\% des ventes du soir correspondent à la formule \emph{Wok}.
|
||||
|
||||
Le gérant se constitue un fichier en notant, pour chaque vente, la formule choisie et le moment de cette vente (midi ou soir).
|
||||
|
||||
On prélève une fiche de façon équiprobable. On définit les quatre évènements suivants:
|
||||
|
||||
\begin{enumerate}
|
||||
\item $M$ : \og la fiche correspond à une vente du midi\fg{} ;
|
||||
\item $S$ : \og la fiche correspond à une vente du soir\fg {};
|
||||
\item $W$ : \og la fiche correspond à une formule \emph{Wok} \fg{} ;
|
||||
\item $B$ : \og la fiche correspond à une formule \emph{Burger} \fg.
|
||||
\end{enumerate}
|
||||
\setlength\parindent{0mm}
|
||||
|
||||
\medskip
|
||||
|
||||
\begin{enumerate}
|
||||
\item Recopier puis compléter l'arbre pondéré
|
||||
|
||||
\begin{center}
|
||||
\begin{tikzpicture}[sloped]
|
||||
\node {.}
|
||||
child {node {$M$}
|
||||
child {node {$W$}
|
||||
edge from parent
|
||||
node[above] {...}
|
||||
}
|
||||
child {node {$B$}
|
||||
edge from parent
|
||||
node[above] {...}
|
||||
}
|
||||
edge from parent
|
||||
node[above] {...}
|
||||
}
|
||||
child[missing] {}
|
||||
child { node {$S$}
|
||||
child {node {$W$}
|
||||
edge from parent
|
||||
node[above] {...}
|
||||
}
|
||||
child {node {$B$}
|
||||
edge from parent
|
||||
node[above] {...}
|
||||
}
|
||||
edge from parent
|
||||
node[above] {...}
|
||||
} ;
|
||||
\end{tikzpicture}
|
||||
\end{center}
|
||||
|
||||
\item Calculer la probabilité de l'évènement $M \cap W$. Interpréter ce résultat dans le contexte de l'exercice.
|
||||
\item Montrer que la probabilité que la fiche choisie corresponde à une formule \emph{Burger} est égale à $0.7488$.
|
||||
\item On a prélevé une fiche correspondant à la formule \emph{Burger}. Quelle est la probabilité, arrondie au millième, que la vente ait eu lieu le soir?
|
||||
\end{enumerate}
|
||||
\end{exercise}
|
||||
|
||||
\begin{solution}
|
||||
\begin{enumerate}
|
||||
\item
|
||||
\begin{center}
|
||||
\begin{tikzpicture}[sloped]
|
||||
\node {.}
|
||||
child {node {$M$}
|
||||
child {node {$W$}
|
||||
edge from parent
|
||||
node[above] {$0.75$}
|
||||
}
|
||||
child {node {$B$}
|
||||
edge from parent
|
||||
node[above] {$0.25$}
|
||||
}
|
||||
edge from parent
|
||||
node[above] {$0.14$}
|
||||
}
|
||||
child[missing] {}
|
||||
child { node {$S$}
|
||||
child {node {$W$}
|
||||
edge from parent
|
||||
node[above] {$0.17$}
|
||||
}
|
||||
child {node {$B$}
|
||||
edge from parent
|
||||
node[above] {$0.83$}
|
||||
}
|
||||
edge from parent
|
||||
node[above] {$0.86$}
|
||||
} ;
|
||||
\end{tikzpicture}
|
||||
\end{center}
|
||||
\item On calcule la probabilité que la vente soit un wok et ait eu lieu à midi
|
||||
\[ P(M\cap W) = P(M) \times P_M(W) = 0.14 \times 0.75 = 0.105 \]
|
||||
\item Probabilité que la vente soit un burger.
|
||||
\[
|
||||
P(B) = P(M\cap B) + P(S\cap B) = 0.14 \times 0.75 + 0.86 \times 0.17 = 0.7488
|
||||
\]
|
||||
\item On cherche à calculer la quantité $P_B(S)$. Pour cela on utilise la formule de Bayes
|
||||
\[
|
||||
P_B(S) = \frac{P(B\cap S)}{P(B)} = \frac{P_S(B) \times P(S)}{P(B)} = \frac{0.83\times 0.86}{0.7488} = 0.953258547008547 \approx 0.953
|
||||
\]
|
||||
\end{enumerate}
|
||||
\end{solution}
|
||||
|
||||
\begin{exercise}[subtitle={Continent plastique}]
|
||||
\textit{Les quantités évoqués dans cette exercice sont générés au hasard et sont donc complètement farfelus.}
|
||||
\medskip
|
||||
Le \og continent de plastique\fg{} est la plus grande des plaques de déchets plastiques évoluant sur les océans. Elle occupe actuellement dans l'océan Pacifique une surface dont l'aire est évaluée à plus de $1,6$ million de km$^2$, entre Hawaï et la Californie.
|
||||
|
||||
En 2017, des scientifiques ont estimé qu'il y avait $11$ millions de tonnes de déchets plastiques qui était déversé chaque année dans les océans et que cette quantité augmentait de $12\n\%$ par chaque année.
|
||||
|
||||
On modélise l'évolution de la masse de ces déchets plastiques déversée chaque année, si rien n'est fait pour la réduire, par une suite géométrique $\left(u_n\right)$. L'arrondi au centième du terme $u_n$ représente la masse de ces déchets déversée chaque année, exprimée en million de tonnes, pour l'année $(2017 + n)$.
|
||||
|
||||
\medskip
|
||||
|
||||
\begin{enumerate}
|
||||
\item Expliquer pourquoi la suite $u_n$ est géométrique?
|
||||
\item Calculer $u_1$ et $u_2$.
|
||||
\item Exprimer $u_n$ en fonction de $n$.
|
||||
\item Au début de l'année 2017, il y avait $300$ millions de tonnes de déchets plastique. Calculer la quantité totale de déchets plastiques en 2030.
|
||||
\item On souhaite déterminer en quelle année la masse totale de ces déchets plastiques aura pour la première fois augmenté de $50$\,\% par rapport à sa valeur de 2017.
|
||||
\begin{enumerate}
|
||||
\item Recopier et compléter l'algorithme ci-dessous pour que la variable $N$ contienne la réponse au problème posé.
|
||||
|
||||
\begin{center}
|
||||
\begin{tabularx}{0.4\linewidth}{|X|}\hline
|
||||
$N = 2017$\\
|
||||
$U = 11$ \\
|
||||
$S = 300 + U$ \\
|
||||
while $S < 450$: \\
|
||||
\hspace{1cm} $N = \ldots$\\
|
||||
\hspace{1cm} $U = \ldots$\\
|
||||
\hspace{1cm} $S = \ldots$\\
|
||||
\hline
|
||||
\end{tabularx}
|
||||
\end{center}
|
||||
\item Que contiennent les variables $S$, $U$ et $N$ après exécution de cet algorithme ?
|
||||
|
||||
Interpréter les résultats dans le contexte de l'exercice.
|
||||
\end{enumerate}
|
||||
\end{enumerate}
|
||||
\end{exercise}
|
||||
|
||||
\begin{solution}
|
||||
\begin{enumerate}
|
||||
\item Une augmentation de $12\,\%$ revient à multiplier la quantité par $1.12$. La suite est donc bien géométrique. Son premier terme est $u_0 = 11$ et sa raison est $q = 1.12$
|
||||
\item
|
||||
\[
|
||||
u_1 = u_0 * 1.12 = 12.32
|
||||
\]
|
||||
\[
|
||||
u_2 = u_0 * 1.12^2 = 13.7984
|
||||
\]
|
||||
\item
|
||||
\[
|
||||
u_n = u_0 \times q^n = 11 \times 1.12^n
|
||||
\]
|
||||
\item On calcule la quantité totale déversée entre 2017 et 2030.
|
||||
\[
|
||||
\sum_{n = 0}^{13} u_n = u_0 \times \frac{1-q^{13}}{1-q} = 11 \times \frac{1 - 1.12^{13}}{1 - 1.12} = 308.32
|
||||
\]
|
||||
On en déduit la quantité totale de déchets en 2030
|
||||
\[
|
||||
300 + 308.32 = 608.3199999999999
|
||||
\]
|
||||
\item
|
||||
\begin{enumerate}
|
||||
\item ~
|
||||
\begin{center}
|
||||
\begin{tabularx}{0.4\linewidth}{|X|}\hline
|
||||
$N \gets 2017$\\
|
||||
$U \gets 11$ \\
|
||||
$S \gets 300 + U$ \\
|
||||
Tant que $S < 450$ \\
|
||||
\hspace{1cm} $N \gets N + 1$\\
|
||||
\hspace{1cm} $U \gets U * 1.12$\\
|
||||
\hspace{1cm} $S \gets S + u$\\
|
||||
Fin Tant que\\\hline
|
||||
\end{tabularx}
|
||||
\end{center}
|
||||
\item \textit{Pas de correction automatisé}
|
||||
\end{enumerate}
|
||||
\end{enumerate}
|
||||
\end{solution}
|
||||
|
||||
\end{document}
|
||||
|
||||
%%% Local Variables:
|
||||
%%% mode: latex
|
||||
%%% TeX-master: "master"
|
||||
%%% End:
|
262
TST/DS/DS_21_04_07/TST1/14_210407_DS8.tex
Normal file
262
TST/DS/DS_21_04_07/TST1/14_210407_DS8.tex
Normal file
@ -0,0 +1,262 @@
|
||||
\documentclass[a4paper,10pt]{article}
|
||||
\usepackage{myXsim}
|
||||
|
||||
% Title Page
|
||||
\title{DS8 \hfill HIPOLITO DA SILVA Andréa}
|
||||
\tribe{TST}
|
||||
\date{\hfillÀ render pour le Mercredi 7 avril}
|
||||
|
||||
\xsimsetup{
|
||||
solution/print = false
|
||||
}
|
||||
|
||||
\begin{document}
|
||||
\maketitle
|
||||
|
||||
\begin{exercise}[subtitle={Automatismes}]
|
||||
\textit{Toutes les questions de cette exercice sont indépendantes et peuvent être répondus séparément}
|
||||
\begin{enumerate}
|
||||
\item De janvier à septembre, une quantité a augmenté de $23\,\%$. Faire un schéma pour représenter la situation puis calculer le taux d'évolution moyen mensuel.
|
||||
\item Une quantité augmente de $23\,\%$ par ans. En 2020, elle est de 149\euro. Quelle était sa valeur en 2019? Faire un schéma pour représenter la situation.
|
||||
\item Déterminer l'équation de la droite \\
|
||||
\begin{tikzpicture}[xscale=0.8, yscale=0.5]
|
||||
\tkzInit[xmin=-5,xmax=5,xstep=1,
|
||||
ymin=-5,ymax=5,ystep=1]
|
||||
\tkzGrid
|
||||
\tkzAxeXY
|
||||
\tkzFct[domain=-5:5,color=red,very thick]%
|
||||
{0.6666666666666666*\x -1};
|
||||
\end{tikzpicture}
|
||||
\item Résoudre l'équation $7 \times 0.38^x = 21$
|
||||
\end{enumerate}
|
||||
\end{exercise}
|
||||
|
||||
\begin{solution}
|
||||
\begin{enumerate}
|
||||
\item On veut partager cette évolution en 8 évolutions.
|
||||
\[
|
||||
\left(1 + \frac{23}{100}\right)^{\frac{1}{8}} = 1.0262
|
||||
\]
|
||||
Donc le taux d'évolution moyen est
|
||||
\[
|
||||
t_m = 1.0262 - 1 = 0.0262
|
||||
\]
|
||||
\item Coefficient multiplicateur pour revenir en arrière
|
||||
\[
|
||||
CM = (1 + \frac{23}{100})^{-1} = 0.813
|
||||
\]
|
||||
On en déduit la quantité en 2019
|
||||
\[
|
||||
149 * 0.813 = 121.13699999999999
|
||||
\]
|
||||
\item L'équation de la droite est
|
||||
\[
|
||||
y = 0.6666666666666666 x -1
|
||||
\]
|
||||
\item Il faut penser à faire la division à par $7$ avant d'utiliser le log car sinon, on ne peut pas utiliser la formule $\log(a^n) = n\times \log(a)$.
|
||||
|
||||
\[x = \frac{\log(3.0)}{\log(0.38)}\]
|
||||
\end{enumerate}
|
||||
\end{solution}
|
||||
|
||||
\begin{exercise}[subtitle={Restaurant}]
|
||||
Un \emph{food truck}, ouvert le midi et le soir, propose deux types de formules :
|
||||
|
||||
\setlength\parindent{10mm}
|
||||
\begin{itemize}
|
||||
\item la formule \emph{Burger} ;
|
||||
\item la formule \emph{Wok}.
|
||||
\end{itemize}
|
||||
\setlength\parindent{0mm}
|
||||
|
||||
\medskip
|
||||
|
||||
Le gérant a remarqué que 35\,\% de ses ventes ont lieu le midi. Le quart des ventes du midi correspondent à la formule \emph{Burger}, alors que 13\,\% des ventes du soir correspondent à la formule \emph{Wok}.
|
||||
|
||||
Le gérant se constitue un fichier en notant, pour chaque vente, la formule choisie et le moment de cette vente (midi ou soir).
|
||||
|
||||
On prélève une fiche de façon équiprobable. On définit les quatre évènements suivants:
|
||||
|
||||
\begin{enumerate}
|
||||
\item $M$ : \og la fiche correspond à une vente du midi\fg{} ;
|
||||
\item $S$ : \og la fiche correspond à une vente du soir\fg {};
|
||||
\item $W$ : \og la fiche correspond à une formule \emph{Wok} \fg{} ;
|
||||
\item $B$ : \og la fiche correspond à une formule \emph{Burger} \fg.
|
||||
\end{enumerate}
|
||||
\setlength\parindent{0mm}
|
||||
|
||||
\medskip
|
||||
|
||||
\begin{enumerate}
|
||||
\item Recopier puis compléter l'arbre pondéré
|
||||
|
||||
\begin{center}
|
||||
\begin{tikzpicture}[sloped]
|
||||
\node {.}
|
||||
child {node {$M$}
|
||||
child {node {$W$}
|
||||
edge from parent
|
||||
node[above] {...}
|
||||
}
|
||||
child {node {$B$}
|
||||
edge from parent
|
||||
node[above] {...}
|
||||
}
|
||||
edge from parent
|
||||
node[above] {...}
|
||||
}
|
||||
child[missing] {}
|
||||
child { node {$S$}
|
||||
child {node {$W$}
|
||||
edge from parent
|
||||
node[above] {...}
|
||||
}
|
||||
child {node {$B$}
|
||||
edge from parent
|
||||
node[above] {...}
|
||||
}
|
||||
edge from parent
|
||||
node[above] {...}
|
||||
} ;
|
||||
\end{tikzpicture}
|
||||
\end{center}
|
||||
|
||||
\item Calculer la probabilité de l'évènement $M \cap W$. Interpréter ce résultat dans le contexte de l'exercice.
|
||||
\item Montrer que la probabilité que la fiche choisie corresponde à une formule \emph{Burger} est égale à $0.653$.
|
||||
\item On a prélevé une fiche correspondant à la formule \emph{Burger}. Quelle est la probabilité, arrondie au millième, que la vente ait eu lieu le soir?
|
||||
\end{enumerate}
|
||||
\end{exercise}
|
||||
|
||||
\begin{solution}
|
||||
\begin{enumerate}
|
||||
\item
|
||||
\begin{center}
|
||||
\begin{tikzpicture}[sloped]
|
||||
\node {.}
|
||||
child {node {$M$}
|
||||
child {node {$W$}
|
||||
edge from parent
|
||||
node[above] {$0.75$}
|
||||
}
|
||||
child {node {$B$}
|
||||
edge from parent
|
||||
node[above] {$0.25$}
|
||||
}
|
||||
edge from parent
|
||||
node[above] {$0.35$}
|
||||
}
|
||||
child[missing] {}
|
||||
child { node {$S$}
|
||||
child {node {$W$}
|
||||
edge from parent
|
||||
node[above] {$0.13$}
|
||||
}
|
||||
child {node {$B$}
|
||||
edge from parent
|
||||
node[above] {$0.87$}
|
||||
}
|
||||
edge from parent
|
||||
node[above] {$0.65$}
|
||||
} ;
|
||||
\end{tikzpicture}
|
||||
\end{center}
|
||||
\item On calcule la probabilité que la vente soit un wok et ait eu lieu à midi
|
||||
\[ P(M\cap W) = P(M) \times P_M(W) = 0.35 \times 0.75 = 0.2625 \]
|
||||
\item Probabilité que la vente soit un burger.
|
||||
\[
|
||||
P(B) = P(M\cap B) + P(S\cap B) = 0.35 \times 0.75 + 0.65 \times 0.13 = 0.653
|
||||
\]
|
||||
\item On cherche à calculer la quantité $P_B(S)$. Pour cela on utilise la formule de Bayes
|
||||
\[
|
||||
P_B(S) = \frac{P(B\cap S)}{P(B)} = \frac{P_S(B) \times P(S)}{P(B)} = \frac{0.87\times 0.65}{0.653} = 0.8660030627871362 \approx 0.866
|
||||
\]
|
||||
\end{enumerate}
|
||||
\end{solution}
|
||||
|
||||
\begin{exercise}[subtitle={Continent plastique}]
|
||||
\textit{Les quantités évoqués dans cette exercice sont générés au hasard et sont donc complètement farfelus.}
|
||||
\medskip
|
||||
Le \og continent de plastique\fg{} est la plus grande des plaques de déchets plastiques évoluant sur les océans. Elle occupe actuellement dans l'océan Pacifique une surface dont l'aire est évaluée à plus de $1,6$ million de km$^2$, entre Hawaï et la Californie.
|
||||
|
||||
En 2017, des scientifiques ont estimé qu'il y avait $4$ millions de tonnes de déchets plastiques qui était déversé chaque année dans les océans et que cette quantité augmentait de $23\n\%$ par chaque année.
|
||||
|
||||
On modélise l'évolution de la masse de ces déchets plastiques déversée chaque année, si rien n'est fait pour la réduire, par une suite géométrique $\left(u_n\right)$. L'arrondi au centième du terme $u_n$ représente la masse de ces déchets déversée chaque année, exprimée en million de tonnes, pour l'année $(2017 + n)$.
|
||||
|
||||
\medskip
|
||||
|
||||
\begin{enumerate}
|
||||
\item Expliquer pourquoi la suite $u_n$ est géométrique?
|
||||
\item Calculer $u_1$ et $u_2$.
|
||||
\item Exprimer $u_n$ en fonction de $n$.
|
||||
\item Au début de l'année 2017, il y avait $300$ millions de tonnes de déchets plastique. Calculer la quantité totale de déchets plastiques en 2030.
|
||||
\item On souhaite déterminer en quelle année la masse totale de ces déchets plastiques aura pour la première fois augmenté de $50$\,\% par rapport à sa valeur de 2017.
|
||||
\begin{enumerate}
|
||||
\item Recopier et compléter l'algorithme ci-dessous pour que la variable $N$ contienne la réponse au problème posé.
|
||||
|
||||
\begin{center}
|
||||
\begin{tabularx}{0.4\linewidth}{|X|}\hline
|
||||
$N = 2017$\\
|
||||
$U = 4$ \\
|
||||
$S = 300 + U$ \\
|
||||
while $S < 450$: \\
|
||||
\hspace{1cm} $N = \ldots$\\
|
||||
\hspace{1cm} $U = \ldots$\\
|
||||
\hspace{1cm} $S = \ldots$\\
|
||||
\hline
|
||||
\end{tabularx}
|
||||
\end{center}
|
||||
\item Que contiennent les variables $S$, $U$ et $N$ après exécution de cet algorithme ?
|
||||
|
||||
Interpréter les résultats dans le contexte de l'exercice.
|
||||
\end{enumerate}
|
||||
\end{enumerate}
|
||||
\end{exercise}
|
||||
|
||||
\begin{solution}
|
||||
\begin{enumerate}
|
||||
\item Une augmentation de $23\,\%$ revient à multiplier la quantité par $1.23$. La suite est donc bien géométrique. Son premier terme est $u_0 = 4$ et sa raison est $q = 1.23$
|
||||
\item
|
||||
\[
|
||||
u_1 = u_0 * 1.23 = 4.92
|
||||
\]
|
||||
\[
|
||||
u_2 = u_0 * 1.23^2 = 6.0516
|
||||
\]
|
||||
\item
|
||||
\[
|
||||
u_n = u_0 \times q^n = 4 \times 1.23^n
|
||||
\]
|
||||
\item On calcule la quantité totale déversée entre 2017 et 2030.
|
||||
\[
|
||||
\sum_{n = 0}^{13} u_n = u_0 \times \frac{1-q^{13}}{1-q} = 4 \times \frac{1 - 1.23^{13}}{1 - 1.23} = 239.12
|
||||
\]
|
||||
On en déduit la quantité totale de déchets en 2030
|
||||
\[
|
||||
300 + 239.12 = 539.12
|
||||
\]
|
||||
\item
|
||||
\begin{enumerate}
|
||||
\item ~
|
||||
\begin{center}
|
||||
\begin{tabularx}{0.4\linewidth}{|X|}\hline
|
||||
$N \gets 2017$\\
|
||||
$U \gets 4$ \\
|
||||
$S \gets 300 + U$ \\
|
||||
Tant que $S < 450$ \\
|
||||
\hspace{1cm} $N \gets N + 1$\\
|
||||
\hspace{1cm} $U \gets U * 1.23$\\
|
||||
\hspace{1cm} $S \gets S + u$\\
|
||||
Fin Tant que\\\hline
|
||||
\end{tabularx}
|
||||
\end{center}
|
||||
\item \textit{Pas de correction automatisé}
|
||||
\end{enumerate}
|
||||
\end{enumerate}
|
||||
\end{solution}
|
||||
|
||||
\end{document}
|
||||
|
||||
%%% Local Variables:
|
||||
%%% mode: latex
|
||||
%%% TeX-master: "master"
|
||||
%%% End:
|
262
TST/DS/DS_21_04_07/TST1/15_210407_DS8.tex
Normal file
262
TST/DS/DS_21_04_07/TST1/15_210407_DS8.tex
Normal file
@ -0,0 +1,262 @@
|
||||
\documentclass[a4paper,10pt]{article}
|
||||
\usepackage{myXsim}
|
||||
|
||||
% Title Page
|
||||
\title{DS8 \hfill HUMBERT Rayan}
|
||||
\tribe{TST}
|
||||
\date{\hfillÀ render pour le Mercredi 7 avril}
|
||||
|
||||
\xsimsetup{
|
||||
solution/print = false
|
||||
}
|
||||
|
||||
\begin{document}
|
||||
\maketitle
|
||||
|
||||
\begin{exercise}[subtitle={Automatismes}]
|
||||
\textit{Toutes les questions de cette exercice sont indépendantes et peuvent être répondus séparément}
|
||||
\begin{enumerate}
|
||||
\item De janvier à septembre, une quantité a augmenté de $17\,\%$. Faire un schéma pour représenter la situation puis calculer le taux d'évolution moyen mensuel.
|
||||
\item Une quantité augmente de $17\,\%$ par ans. En 2020, elle est de 149\euro. Quelle était sa valeur en 2019? Faire un schéma pour représenter la situation.
|
||||
\item Déterminer l'équation de la droite \\
|
||||
\begin{tikzpicture}[xscale=0.8, yscale=0.5]
|
||||
\tkzInit[xmin=-5,xmax=5,xstep=1,
|
||||
ymin=-5,ymax=5,ystep=1]
|
||||
\tkzGrid
|
||||
\tkzAxeXY
|
||||
\tkzFct[domain=-5:5,color=red,very thick]%
|
||||
{0.5*\x -1};
|
||||
\end{tikzpicture}
|
||||
\item Résoudre l'équation $5 \times 0.02^x = 46$
|
||||
\end{enumerate}
|
||||
\end{exercise}
|
||||
|
||||
\begin{solution}
|
||||
\begin{enumerate}
|
||||
\item On veut partager cette évolution en 8 évolutions.
|
||||
\[
|
||||
\left(1 + \frac{17}{100}\right)^{\frac{1}{8}} = 1.0198
|
||||
\]
|
||||
Donc le taux d'évolution moyen est
|
||||
\[
|
||||
t_m = 1.0198 - 1 = 0.01980000000000004
|
||||
\]
|
||||
\item Coefficient multiplicateur pour revenir en arrière
|
||||
\[
|
||||
CM = (1 + \frac{17}{100})^{-1} = 0.8547
|
||||
\]
|
||||
On en déduit la quantité en 2019
|
||||
\[
|
||||
149 * 0.8547 = 127.3503
|
||||
\]
|
||||
\item L'équation de la droite est
|
||||
\[
|
||||
y = 0.5 x -1
|
||||
\]
|
||||
\item Il faut penser à faire la division à par $5$ avant d'utiliser le log car sinon, on ne peut pas utiliser la formule $\log(a^n) = n\times \log(a)$.
|
||||
|
||||
\[x = \frac{\log(9.2)}{\log(0.02)}\]
|
||||
\end{enumerate}
|
||||
\end{solution}
|
||||
|
||||
\begin{exercise}[subtitle={Restaurant}]
|
||||
Un \emph{food truck}, ouvert le midi et le soir, propose deux types de formules :
|
||||
|
||||
\setlength\parindent{10mm}
|
||||
\begin{itemize}
|
||||
\item la formule \emph{Burger} ;
|
||||
\item la formule \emph{Wok}.
|
||||
\end{itemize}
|
||||
\setlength\parindent{0mm}
|
||||
|
||||
\medskip
|
||||
|
||||
Le gérant a remarqué que 65\,\% de ses ventes ont lieu le midi. Le quart des ventes du midi correspondent à la formule \emph{Burger}, alors que 93\,\% des ventes du soir correspondent à la formule \emph{Wok}.
|
||||
|
||||
Le gérant se constitue un fichier en notant, pour chaque vente, la formule choisie et le moment de cette vente (midi ou soir).
|
||||
|
||||
On prélève une fiche de façon équiprobable. On définit les quatre évènements suivants:
|
||||
|
||||
\begin{enumerate}
|
||||
\item $M$ : \og la fiche correspond à une vente du midi\fg{} ;
|
||||
\item $S$ : \og la fiche correspond à une vente du soir\fg {};
|
||||
\item $W$ : \og la fiche correspond à une formule \emph{Wok} \fg{} ;
|
||||
\item $B$ : \og la fiche correspond à une formule \emph{Burger} \fg.
|
||||
\end{enumerate}
|
||||
\setlength\parindent{0mm}
|
||||
|
||||
\medskip
|
||||
|
||||
\begin{enumerate}
|
||||
\item Recopier puis compléter l'arbre pondéré
|
||||
|
||||
\begin{center}
|
||||
\begin{tikzpicture}[sloped]
|
||||
\node {.}
|
||||
child {node {$M$}
|
||||
child {node {$W$}
|
||||
edge from parent
|
||||
node[above] {...}
|
||||
}
|
||||
child {node {$B$}
|
||||
edge from parent
|
||||
node[above] {...}
|
||||
}
|
||||
edge from parent
|
||||
node[above] {...}
|
||||
}
|
||||
child[missing] {}
|
||||
child { node {$S$}
|
||||
child {node {$W$}
|
||||
edge from parent
|
||||
node[above] {...}
|
||||
}
|
||||
child {node {$B$}
|
||||
edge from parent
|
||||
node[above] {...}
|
||||
}
|
||||
edge from parent
|
||||
node[above] {...}
|
||||
} ;
|
||||
\end{tikzpicture}
|
||||
\end{center}
|
||||
|
||||
\item Calculer la probabilité de l'évènement $M \cap W$. Interpréter ce résultat dans le contexte de l'exercice.
|
||||
\item Montrer que la probabilité que la fiche choisie corresponde à une formule \emph{Burger} est égale à $0.187$.
|
||||
\item On a prélevé une fiche correspondant à la formule \emph{Burger}. Quelle est la probabilité, arrondie au millième, que la vente ait eu lieu le soir?
|
||||
\end{enumerate}
|
||||
\end{exercise}
|
||||
|
||||
\begin{solution}
|
||||
\begin{enumerate}
|
||||
\item
|
||||
\begin{center}
|
||||
\begin{tikzpicture}[sloped]
|
||||
\node {.}
|
||||
child {node {$M$}
|
||||
child {node {$W$}
|
||||
edge from parent
|
||||
node[above] {$0.75$}
|
||||
}
|
||||
child {node {$B$}
|
||||
edge from parent
|
||||
node[above] {$0.25$}
|
||||
}
|
||||
edge from parent
|
||||
node[above] {$0.65$}
|
||||
}
|
||||
child[missing] {}
|
||||
child { node {$S$}
|
||||
child {node {$W$}
|
||||
edge from parent
|
||||
node[above] {$0.93$}
|
||||
}
|
||||
child {node {$B$}
|
||||
edge from parent
|
||||
node[above] {$0.07$}
|
||||
}
|
||||
edge from parent
|
||||
node[above] {$0.35$}
|
||||
} ;
|
||||
\end{tikzpicture}
|
||||
\end{center}
|
||||
\item On calcule la probabilité que la vente soit un wok et ait eu lieu à midi
|
||||
\[ P(M\cap W) = P(M) \times P_M(W) = 0.65 \times 0.75 = 0.4875 \]
|
||||
\item Probabilité que la vente soit un burger.
|
||||
\[
|
||||
P(B) = P(M\cap B) + P(S\cap B) = 0.65 \times 0.75 + 0.35 \times 0.93 = 0.187
|
||||
\]
|
||||
\item On cherche à calculer la quantité $P_B(S)$. Pour cela on utilise la formule de Bayes
|
||||
\[
|
||||
P_B(S) = \frac{P(B\cap S)}{P(B)} = \frac{P_S(B) \times P(S)}{P(B)} = \frac{0.07\times 0.35}{0.187} = 0.13101604278074866 \approx 0.131
|
||||
\]
|
||||
\end{enumerate}
|
||||
\end{solution}
|
||||
|
||||
\begin{exercise}[subtitle={Continent plastique}]
|
||||
\textit{Les quantités évoqués dans cette exercice sont générés au hasard et sont donc complètement farfelus.}
|
||||
\medskip
|
||||
Le \og continent de plastique\fg{} est la plus grande des plaques de déchets plastiques évoluant sur les océans. Elle occupe actuellement dans l'océan Pacifique une surface dont l'aire est évaluée à plus de $1,6$ million de km$^2$, entre Hawaï et la Californie.
|
||||
|
||||
En 2017, des scientifiques ont estimé qu'il y avait $15$ millions de tonnes de déchets plastiques qui était déversé chaque année dans les océans et que cette quantité augmentait de $12\n\%$ par chaque année.
|
||||
|
||||
On modélise l'évolution de la masse de ces déchets plastiques déversée chaque année, si rien n'est fait pour la réduire, par une suite géométrique $\left(u_n\right)$. L'arrondi au centième du terme $u_n$ représente la masse de ces déchets déversée chaque année, exprimée en million de tonnes, pour l'année $(2017 + n)$.
|
||||
|
||||
\medskip
|
||||
|
||||
\begin{enumerate}
|
||||
\item Expliquer pourquoi la suite $u_n$ est géométrique?
|
||||
\item Calculer $u_1$ et $u_2$.
|
||||
\item Exprimer $u_n$ en fonction de $n$.
|
||||
\item Au début de l'année 2017, il y avait $300$ millions de tonnes de déchets plastique. Calculer la quantité totale de déchets plastiques en 2030.
|
||||
\item On souhaite déterminer en quelle année la masse totale de ces déchets plastiques aura pour la première fois augmenté de $50$\,\% par rapport à sa valeur de 2017.
|
||||
\begin{enumerate}
|
||||
\item Recopier et compléter l'algorithme ci-dessous pour que la variable $N$ contienne la réponse au problème posé.
|
||||
|
||||
\begin{center}
|
||||
\begin{tabularx}{0.4\linewidth}{|X|}\hline
|
||||
$N = 2017$\\
|
||||
$U = 15$ \\
|
||||
$S = 300 + U$ \\
|
||||
while $S < 450$: \\
|
||||
\hspace{1cm} $N = \ldots$\\
|
||||
\hspace{1cm} $U = \ldots$\\
|
||||
\hspace{1cm} $S = \ldots$\\
|
||||
\hline
|
||||
\end{tabularx}
|
||||
\end{center}
|
||||
\item Que contiennent les variables $S$, $U$ et $N$ après exécution de cet algorithme ?
|
||||
|
||||
Interpréter les résultats dans le contexte de l'exercice.
|
||||
\end{enumerate}
|
||||
\end{enumerate}
|
||||
\end{exercise}
|
||||
|
||||
\begin{solution}
|
||||
\begin{enumerate}
|
||||
\item Une augmentation de $12\,\%$ revient à multiplier la quantité par $1.12$. La suite est donc bien géométrique. Son premier terme est $u_0 = 15$ et sa raison est $q = 1.12$
|
||||
\item
|
||||
\[
|
||||
u_1 = u_0 * 1.12 = 16.8
|
||||
\]
|
||||
\[
|
||||
u_2 = u_0 * 1.12^2 = 18.816
|
||||
\]
|
||||
\item
|
||||
\[
|
||||
u_n = u_0 \times q^n = 15 \times 1.12^n
|
||||
\]
|
||||
\item On calcule la quantité totale déversée entre 2017 et 2030.
|
||||
\[
|
||||
\sum_{n = 0}^{13} u_n = u_0 \times \frac{1-q^{13}}{1-q} = 15 \times \frac{1 - 1.12^{13}}{1 - 1.12} = 420.44
|
||||
\]
|
||||
On en déduit la quantité totale de déchets en 2030
|
||||
\[
|
||||
300 + 420.44 = 720.44
|
||||
\]
|
||||
\item
|
||||
\begin{enumerate}
|
||||
\item ~
|
||||
\begin{center}
|
||||
\begin{tabularx}{0.4\linewidth}{|X|}\hline
|
||||
$N \gets 2017$\\
|
||||
$U \gets 15$ \\
|
||||
$S \gets 300 + U$ \\
|
||||
Tant que $S < 450$ \\
|
||||
\hspace{1cm} $N \gets N + 1$\\
|
||||
\hspace{1cm} $U \gets U * 1.12$\\
|
||||
\hspace{1cm} $S \gets S + u$\\
|
||||
Fin Tant que\\\hline
|
||||
\end{tabularx}
|
||||
\end{center}
|
||||
\item \textit{Pas de correction automatisé}
|
||||
\end{enumerate}
|
||||
\end{enumerate}
|
||||
\end{solution}
|
||||
|
||||
\end{document}
|
||||
|
||||
%%% Local Variables:
|
||||
%%% mode: latex
|
||||
%%% TeX-master: "master"
|
||||
%%% End:
|
262
TST/DS/DS_21_04_07/TST1/16_210407_DS8.tex
Normal file
262
TST/DS/DS_21_04_07/TST1/16_210407_DS8.tex
Normal file
@ -0,0 +1,262 @@
|
||||
\documentclass[a4paper,10pt]{article}
|
||||
\usepackage{myXsim}
|
||||
|
||||
% Title Page
|
||||
\title{DS8 \hfill MASSON Grace}
|
||||
\tribe{TST}
|
||||
\date{\hfillÀ render pour le Mercredi 7 avril}
|
||||
|
||||
\xsimsetup{
|
||||
solution/print = false
|
||||
}
|
||||
|
||||
\begin{document}
|
||||
\maketitle
|
||||
|
||||
\begin{exercise}[subtitle={Automatismes}]
|
||||
\textit{Toutes les questions de cette exercice sont indépendantes et peuvent être répondus séparément}
|
||||
\begin{enumerate}
|
||||
\item De janvier à septembre, une quantité a augmenté de $24\,\%$. Faire un schéma pour représenter la situation puis calculer le taux d'évolution moyen mensuel.
|
||||
\item Une quantité augmente de $24\,\%$ par ans. En 2020, elle est de 145\euro. Quelle était sa valeur en 2019? Faire un schéma pour représenter la situation.
|
||||
\item Déterminer l'équation de la droite \\
|
||||
\begin{tikzpicture}[xscale=0.8, yscale=0.5]
|
||||
\tkzInit[xmin=-5,xmax=5,xstep=1,
|
||||
ymin=-5,ymax=5,ystep=1]
|
||||
\tkzGrid
|
||||
\tkzAxeXY
|
||||
\tkzFct[domain=-5:5,color=red,very thick]%
|
||||
{3.0*\x -3};
|
||||
\end{tikzpicture}
|
||||
\item Résoudre l'équation $5 \times 0.53^x = 18$
|
||||
\end{enumerate}
|
||||
\end{exercise}
|
||||
|
||||
\begin{solution}
|
||||
\begin{enumerate}
|
||||
\item On veut partager cette évolution en 8 évolutions.
|
||||
\[
|
||||
\left(1 + \frac{24}{100}\right)^{\frac{1}{8}} = 1.0273
|
||||
\]
|
||||
Donc le taux d'évolution moyen est
|
||||
\[
|
||||
t_m = 1.0273 - 1 = 0.027300000000000102
|
||||
\]
|
||||
\item Coefficient multiplicateur pour revenir en arrière
|
||||
\[
|
||||
CM = (1 + \frac{24}{100})^{-1} = 0.8065
|
||||
\]
|
||||
On en déduit la quantité en 2019
|
||||
\[
|
||||
145 * 0.8065 = 116.9425
|
||||
\]
|
||||
\item L'équation de la droite est
|
||||
\[
|
||||
y = 3.0 x -3
|
||||
\]
|
||||
\item Il faut penser à faire la division à par $5$ avant d'utiliser le log car sinon, on ne peut pas utiliser la formule $\log(a^n) = n\times \log(a)$.
|
||||
|
||||
\[x = \frac{\log(3.6)}{\log(0.53)}\]
|
||||
\end{enumerate}
|
||||
\end{solution}
|
||||
|
||||
\begin{exercise}[subtitle={Restaurant}]
|
||||
Un \emph{food truck}, ouvert le midi et le soir, propose deux types de formules :
|
||||
|
||||
\setlength\parindent{10mm}
|
||||
\begin{itemize}
|
||||
\item la formule \emph{Burger} ;
|
||||
\item la formule \emph{Wok}.
|
||||
\end{itemize}
|
||||
\setlength\parindent{0mm}
|
||||
|
||||
\medskip
|
||||
|
||||
Le gérant a remarqué que 73\,\% de ses ventes ont lieu le midi. Le quart des ventes du midi correspondent à la formule \emph{Burger}, alors que 56\,\% des ventes du soir correspondent à la formule \emph{Wok}.
|
||||
|
||||
Le gérant se constitue un fichier en notant, pour chaque vente, la formule choisie et le moment de cette vente (midi ou soir).
|
||||
|
||||
On prélève une fiche de façon équiprobable. On définit les quatre évènements suivants:
|
||||
|
||||
\begin{enumerate}
|
||||
\item $M$ : \og la fiche correspond à une vente du midi\fg{} ;
|
||||
\item $S$ : \og la fiche correspond à une vente du soir\fg {};
|
||||
\item $W$ : \og la fiche correspond à une formule \emph{Wok} \fg{} ;
|
||||
\item $B$ : \og la fiche correspond à une formule \emph{Burger} \fg.
|
||||
\end{enumerate}
|
||||
\setlength\parindent{0mm}
|
||||
|
||||
\medskip
|
||||
|
||||
\begin{enumerate}
|
||||
\item Recopier puis compléter l'arbre pondéré
|
||||
|
||||
\begin{center}
|
||||
\begin{tikzpicture}[sloped]
|
||||
\node {.}
|
||||
child {node {$M$}
|
||||
child {node {$W$}
|
||||
edge from parent
|
||||
node[above] {...}
|
||||
}
|
||||
child {node {$B$}
|
||||
edge from parent
|
||||
node[above] {...}
|
||||
}
|
||||
edge from parent
|
||||
node[above] {...}
|
||||
}
|
||||
child[missing] {}
|
||||
child { node {$S$}
|
||||
child {node {$W$}
|
||||
edge from parent
|
||||
node[above] {...}
|
||||
}
|
||||
child {node {$B$}
|
||||
edge from parent
|
||||
node[above] {...}
|
||||
}
|
||||
edge from parent
|
||||
node[above] {...}
|
||||
} ;
|
||||
\end{tikzpicture}
|
||||
\end{center}
|
||||
|
||||
\item Calculer la probabilité de l'évènement $M \cap W$. Interpréter ce résultat dans le contexte de l'exercice.
|
||||
\item Montrer que la probabilité que la fiche choisie corresponde à une formule \emph{Burger} est égale à $0.2986$.
|
||||
\item On a prélevé une fiche correspondant à la formule \emph{Burger}. Quelle est la probabilité, arrondie au millième, que la vente ait eu lieu le soir?
|
||||
\end{enumerate}
|
||||
\end{exercise}
|
||||
|
||||
\begin{solution}
|
||||
\begin{enumerate}
|
||||
\item
|
||||
\begin{center}
|
||||
\begin{tikzpicture}[sloped]
|
||||
\node {.}
|
||||
child {node {$M$}
|
||||
child {node {$W$}
|
||||
edge from parent
|
||||
node[above] {$0.75$}
|
||||
}
|
||||
child {node {$B$}
|
||||
edge from parent
|
||||
node[above] {$0.25$}
|
||||
}
|
||||
edge from parent
|
||||
node[above] {$0.73$}
|
||||
}
|
||||
child[missing] {}
|
||||
child { node {$S$}
|
||||
child {node {$W$}
|
||||
edge from parent
|
||||
node[above] {$0.57$}
|
||||
}
|
||||
child {node {$B$}
|
||||
edge from parent
|
||||
node[above] {$0.43$}
|
||||
}
|
||||
edge from parent
|
||||
node[above] {$0.27$}
|
||||
} ;
|
||||
\end{tikzpicture}
|
||||
\end{center}
|
||||
\item On calcule la probabilité que la vente soit un wok et ait eu lieu à midi
|
||||
\[ P(M\cap W) = P(M) \times P_M(W) = 0.73 \times 0.75 = 0.5475 \]
|
||||
\item Probabilité que la vente soit un burger.
|
||||
\[
|
||||
P(B) = P(M\cap B) + P(S\cap B) = 0.73 \times 0.75 + 0.27 \times 0.57 = 0.2986
|
||||
\]
|
||||
\item On cherche à calculer la quantité $P_B(S)$. Pour cela on utilise la formule de Bayes
|
||||
\[
|
||||
P_B(S) = \frac{P(B\cap S)}{P(B)} = \frac{P_S(B) \times P(S)}{P(B)} = \frac{0.43\times 0.27}{0.2986} = 0.38881446751507037 \approx 0.389
|
||||
\]
|
||||
\end{enumerate}
|
||||
\end{solution}
|
||||
|
||||
\begin{exercise}[subtitle={Continent plastique}]
|
||||
\textit{Les quantités évoqués dans cette exercice sont générés au hasard et sont donc complètement farfelus.}
|
||||
\medskip
|
||||
Le \og continent de plastique\fg{} est la plus grande des plaques de déchets plastiques évoluant sur les océans. Elle occupe actuellement dans l'océan Pacifique une surface dont l'aire est évaluée à plus de $1,6$ million de km$^2$, entre Hawaï et la Californie.
|
||||
|
||||
En 2017, des scientifiques ont estimé qu'il y avait $4$ millions de tonnes de déchets plastiques qui était déversé chaque année dans les océans et que cette quantité augmentait de $19\n\%$ par chaque année.
|
||||
|
||||
On modélise l'évolution de la masse de ces déchets plastiques déversée chaque année, si rien n'est fait pour la réduire, par une suite géométrique $\left(u_n\right)$. L'arrondi au centième du terme $u_n$ représente la masse de ces déchets déversée chaque année, exprimée en million de tonnes, pour l'année $(2017 + n)$.
|
||||
|
||||
\medskip
|
||||
|
||||
\begin{enumerate}
|
||||
\item Expliquer pourquoi la suite $u_n$ est géométrique?
|
||||
\item Calculer $u_1$ et $u_2$.
|
||||
\item Exprimer $u_n$ en fonction de $n$.
|
||||
\item Au début de l'année 2017, il y avait $300$ millions de tonnes de déchets plastique. Calculer la quantité totale de déchets plastiques en 2030.
|
||||
\item On souhaite déterminer en quelle année la masse totale de ces déchets plastiques aura pour la première fois augmenté de $50$\,\% par rapport à sa valeur de 2017.
|
||||
\begin{enumerate}
|
||||
\item Recopier et compléter l'algorithme ci-dessous pour que la variable $N$ contienne la réponse au problème posé.
|
||||
|
||||
\begin{center}
|
||||
\begin{tabularx}{0.4\linewidth}{|X|}\hline
|
||||
$N = 2017$\\
|
||||
$U = 4$ \\
|
||||
$S = 300 + U$ \\
|
||||
while $S < 450$: \\
|
||||
\hspace{1cm} $N = \ldots$\\
|
||||
\hspace{1cm} $U = \ldots$\\
|
||||
\hspace{1cm} $S = \ldots$\\
|
||||
\hline
|
||||
\end{tabularx}
|
||||
\end{center}
|
||||
\item Que contiennent les variables $S$, $U$ et $N$ après exécution de cet algorithme ?
|
||||
|
||||
Interpréter les résultats dans le contexte de l'exercice.
|
||||
\end{enumerate}
|
||||
\end{enumerate}
|
||||
\end{exercise}
|
||||
|
||||
\begin{solution}
|
||||
\begin{enumerate}
|
||||
\item Une augmentation de $19\,\%$ revient à multiplier la quantité par $1.19$. La suite est donc bien géométrique. Son premier terme est $u_0 = 4$ et sa raison est $q = 1.19$
|
||||
\item
|
||||
\[
|
||||
u_1 = u_0 * 1.19 = 4.76
|
||||
\]
|
||||
\[
|
||||
u_2 = u_0 * 1.19^2 = 5.6644
|
||||
\]
|
||||
\item
|
||||
\[
|
||||
u_n = u_0 \times q^n = 4 \times 1.19^n
|
||||
\]
|
||||
\item On calcule la quantité totale déversée entre 2017 et 2030.
|
||||
\[
|
||||
\sum_{n = 0}^{13} u_n = u_0 \times \frac{1-q^{13}}{1-q} = 4 \times \frac{1 - 1.19^{13}}{1 - 1.19} = 180.98
|
||||
\]
|
||||
On en déduit la quantité totale de déchets en 2030
|
||||
\[
|
||||
300 + 180.98 = 480.98
|
||||
\]
|
||||
\item
|
||||
\begin{enumerate}
|
||||
\item ~
|
||||
\begin{center}
|
||||
\begin{tabularx}{0.4\linewidth}{|X|}\hline
|
||||
$N \gets 2017$\\
|
||||
$U \gets 4$ \\
|
||||
$S \gets 300 + U$ \\
|
||||
Tant que $S < 450$ \\
|
||||
\hspace{1cm} $N \gets N + 1$\\
|
||||
\hspace{1cm} $U \gets U * 1.19$\\
|
||||
\hspace{1cm} $S \gets S + u$\\
|
||||
Fin Tant que\\\hline
|
||||
\end{tabularx}
|
||||
\end{center}
|
||||
\item \textit{Pas de correction automatisé}
|
||||
\end{enumerate}
|
||||
\end{enumerate}
|
||||
\end{solution}
|
||||
|
||||
\end{document}
|
||||
|
||||
%%% Local Variables:
|
||||
%%% mode: latex
|
||||
%%% TeX-master: "master"
|
||||
%%% End:
|
262
TST/DS/DS_21_04_07/TST1/17_210407_DS8.tex
Normal file
262
TST/DS/DS_21_04_07/TST1/17_210407_DS8.tex
Normal file
@ -0,0 +1,262 @@
|
||||
\documentclass[a4paper,10pt]{article}
|
||||
\usepackage{myXsim}
|
||||
|
||||
% Title Page
|
||||
\title{DS8 \hfill MOKHTARI Nissrine}
|
||||
\tribe{TST}
|
||||
\date{\hfillÀ render pour le Mercredi 7 avril}
|
||||
|
||||
\xsimsetup{
|
||||
solution/print = false
|
||||
}
|
||||
|
||||
\begin{document}
|
||||
\maketitle
|
||||
|
||||
\begin{exercise}[subtitle={Automatismes}]
|
||||
\textit{Toutes les questions de cette exercice sont indépendantes et peuvent être répondus séparément}
|
||||
\begin{enumerate}
|
||||
\item De janvier à septembre, une quantité a augmenté de $11\,\%$. Faire un schéma pour représenter la situation puis calculer le taux d'évolution moyen mensuel.
|
||||
\item Une quantité augmente de $11\,\%$ par ans. En 2020, elle est de 138\euro. Quelle était sa valeur en 2019? Faire un schéma pour représenter la situation.
|
||||
\item Déterminer l'équation de la droite \\
|
||||
\begin{tikzpicture}[xscale=0.8, yscale=0.5]
|
||||
\tkzInit[xmin=-5,xmax=5,xstep=1,
|
||||
ymin=-5,ymax=5,ystep=1]
|
||||
\tkzGrid
|
||||
\tkzAxeXY
|
||||
\tkzFct[domain=-5:5,color=red,very thick]%
|
||||
{2.0*\x -3};
|
||||
\end{tikzpicture}
|
||||
\item Résoudre l'équation $8 \times 0.17^x = 35$
|
||||
\end{enumerate}
|
||||
\end{exercise}
|
||||
|
||||
\begin{solution}
|
||||
\begin{enumerate}
|
||||
\item On veut partager cette évolution en 8 évolutions.
|
||||
\[
|
||||
\left(1 + \frac{11}{100}\right)^{\frac{1}{8}} = 1.0131
|
||||
\]
|
||||
Donc le taux d'évolution moyen est
|
||||
\[
|
||||
t_m = 1.0131 - 1 = 0.01309999999999989
|
||||
\]
|
||||
\item Coefficient multiplicateur pour revenir en arrière
|
||||
\[
|
||||
CM = (1 + \frac{11}{100})^{-1} = 0.9009
|
||||
\]
|
||||
On en déduit la quantité en 2019
|
||||
\[
|
||||
138 * 0.9009 = 124.3242
|
||||
\]
|
||||
\item L'équation de la droite est
|
||||
\[
|
||||
y = 2.0 x -3
|
||||
\]
|
||||
\item Il faut penser à faire la division à par $8$ avant d'utiliser le log car sinon, on ne peut pas utiliser la formule $\log(a^n) = n\times \log(a)$.
|
||||
|
||||
\[x = \frac{\log(4.38)}{\log(0.17)}\]
|
||||
\end{enumerate}
|
||||
\end{solution}
|
||||
|
||||
\begin{exercise}[subtitle={Restaurant}]
|
||||
Un \emph{food truck}, ouvert le midi et le soir, propose deux types de formules :
|
||||
|
||||
\setlength\parindent{10mm}
|
||||
\begin{itemize}
|
||||
\item la formule \emph{Burger} ;
|
||||
\item la formule \emph{Wok}.
|
||||
\end{itemize}
|
||||
\setlength\parindent{0mm}
|
||||
|
||||
\medskip
|
||||
|
||||
Le gérant a remarqué que 62\,\% de ses ventes ont lieu le midi. Le quart des ventes du midi correspondent à la formule \emph{Burger}, alors que 28\,\% des ventes du soir correspondent à la formule \emph{Wok}.
|
||||
|
||||
Le gérant se constitue un fichier en notant, pour chaque vente, la formule choisie et le moment de cette vente (midi ou soir).
|
||||
|
||||
On prélève une fiche de façon équiprobable. On définit les quatre évènements suivants:
|
||||
|
||||
\begin{enumerate}
|
||||
\item $M$ : \og la fiche correspond à une vente du midi\fg{} ;
|
||||
\item $S$ : \og la fiche correspond à une vente du soir\fg {};
|
||||
\item $W$ : \og la fiche correspond à une formule \emph{Wok} \fg{} ;
|
||||
\item $B$ : \og la fiche correspond à une formule \emph{Burger} \fg.
|
||||
\end{enumerate}
|
||||
\setlength\parindent{0mm}
|
||||
|
||||
\medskip
|
||||
|
||||
\begin{enumerate}
|
||||
\item Recopier puis compléter l'arbre pondéré
|
||||
|
||||
\begin{center}
|
||||
\begin{tikzpicture}[sloped]
|
||||
\node {.}
|
||||
child {node {$M$}
|
||||
child {node {$W$}
|
||||
edge from parent
|
||||
node[above] {...}
|
||||
}
|
||||
child {node {$B$}
|
||||
edge from parent
|
||||
node[above] {...}
|
||||
}
|
||||
edge from parent
|
||||
node[above] {...}
|
||||
}
|
||||
child[missing] {}
|
||||
child { node {$S$}
|
||||
child {node {$W$}
|
||||
edge from parent
|
||||
node[above] {...}
|
||||
}
|
||||
child {node {$B$}
|
||||
edge from parent
|
||||
node[above] {...}
|
||||
}
|
||||
edge from parent
|
||||
node[above] {...}
|
||||
} ;
|
||||
\end{tikzpicture}
|
||||
\end{center}
|
||||
|
||||
\item Calculer la probabilité de l'évènement $M \cap W$. Interpréter ce résultat dans le contexte de l'exercice.
|
||||
\item Montrer que la probabilité que la fiche choisie corresponde à une formule \emph{Burger} est égale à $0.4248$.
|
||||
\item On a prélevé une fiche correspondant à la formule \emph{Burger}. Quelle est la probabilité, arrondie au millième, que la vente ait eu lieu le soir?
|
||||
\end{enumerate}
|
||||
\end{exercise}
|
||||
|
||||
\begin{solution}
|
||||
\begin{enumerate}
|
||||
\item
|
||||
\begin{center}
|
||||
\begin{tikzpicture}[sloped]
|
||||
\node {.}
|
||||
child {node {$M$}
|
||||
child {node {$W$}
|
||||
edge from parent
|
||||
node[above] {$0.75$}
|
||||
}
|
||||
child {node {$B$}
|
||||
edge from parent
|
||||
node[above] {$0.25$}
|
||||
}
|
||||
edge from parent
|
||||
node[above] {$0.62$}
|
||||
}
|
||||
child[missing] {}
|
||||
child { node {$S$}
|
||||
child {node {$W$}
|
||||
edge from parent
|
||||
node[above] {$0.29$}
|
||||
}
|
||||
child {node {$B$}
|
||||
edge from parent
|
||||
node[above] {$0.71$}
|
||||
}
|
||||
edge from parent
|
||||
node[above] {$0.38$}
|
||||
} ;
|
||||
\end{tikzpicture}
|
||||
\end{center}
|
||||
\item On calcule la probabilité que la vente soit un wok et ait eu lieu à midi
|
||||
\[ P(M\cap W) = P(M) \times P_M(W) = 0.62 \times 0.75 = 0.465 \]
|
||||
\item Probabilité que la vente soit un burger.
|
||||
\[
|
||||
P(B) = P(M\cap B) + P(S\cap B) = 0.62 \times 0.75 + 0.38 \times 0.29 = 0.4248
|
||||
\]
|
||||
\item On cherche à calculer la quantité $P_B(S)$. Pour cela on utilise la formule de Bayes
|
||||
\[
|
||||
P_B(S) = \frac{P(B\cap S)}{P(B)} = \frac{P_S(B) \times P(S)}{P(B)} = \frac{0.71\times 0.38}{0.4248} = 0.6351224105461393 \approx 0.635
|
||||
\]
|
||||
\end{enumerate}
|
||||
\end{solution}
|
||||
|
||||
\begin{exercise}[subtitle={Continent plastique}]
|
||||
\textit{Les quantités évoqués dans cette exercice sont générés au hasard et sont donc complètement farfelus.}
|
||||
\medskip
|
||||
Le \og continent de plastique\fg{} est la plus grande des plaques de déchets plastiques évoluant sur les océans. Elle occupe actuellement dans l'océan Pacifique une surface dont l'aire est évaluée à plus de $1,6$ million de km$^2$, entre Hawaï et la Californie.
|
||||
|
||||
En 2017, des scientifiques ont estimé qu'il y avait $4$ millions de tonnes de déchets plastiques qui était déversé chaque année dans les océans et que cette quantité augmentait de $30\n\%$ par chaque année.
|
||||
|
||||
On modélise l'évolution de la masse de ces déchets plastiques déversée chaque année, si rien n'est fait pour la réduire, par une suite géométrique $\left(u_n\right)$. L'arrondi au centième du terme $u_n$ représente la masse de ces déchets déversée chaque année, exprimée en million de tonnes, pour l'année $(2017 + n)$.
|
||||
|
||||
\medskip
|
||||
|
||||
\begin{enumerate}
|
||||
\item Expliquer pourquoi la suite $u_n$ est géométrique?
|
||||
\item Calculer $u_1$ et $u_2$.
|
||||
\item Exprimer $u_n$ en fonction de $n$.
|
||||
\item Au début de l'année 2017, il y avait $300$ millions de tonnes de déchets plastique. Calculer la quantité totale de déchets plastiques en 2030.
|
||||
\item On souhaite déterminer en quelle année la masse totale de ces déchets plastiques aura pour la première fois augmenté de $50$\,\% par rapport à sa valeur de 2017.
|
||||
\begin{enumerate}
|
||||
\item Recopier et compléter l'algorithme ci-dessous pour que la variable $N$ contienne la réponse au problème posé.
|
||||
|
||||
\begin{center}
|
||||
\begin{tabularx}{0.4\linewidth}{|X|}\hline
|
||||
$N = 2017$\\
|
||||
$U = 4$ \\
|
||||
$S = 300 + U$ \\
|
||||
while $S < 450$: \\
|
||||
\hspace{1cm} $N = \ldots$\\
|
||||
\hspace{1cm} $U = \ldots$\\
|
||||
\hspace{1cm} $S = \ldots$\\
|
||||
\hline
|
||||
\end{tabularx}
|
||||
\end{center}
|
||||
\item Que contiennent les variables $S$, $U$ et $N$ après exécution de cet algorithme ?
|
||||
|
||||
Interpréter les résultats dans le contexte de l'exercice.
|
||||
\end{enumerate}
|
||||
\end{enumerate}
|
||||
\end{exercise}
|
||||
|
||||
\begin{solution}
|
||||
\begin{enumerate}
|
||||
\item Une augmentation de $30\,\%$ revient à multiplier la quantité par $1.3$. La suite est donc bien géométrique. Son premier terme est $u_0 = 4$ et sa raison est $q = 1.3$
|
||||
\item
|
||||
\[
|
||||
u_1 = u_0 * 1.3 = 5.2
|
||||
\]
|
||||
\[
|
||||
u_2 = u_0 * 1.3^2 = 6.76
|
||||
\]
|
||||
\item
|
||||
\[
|
||||
u_n = u_0 \times q^n = 4 \times 1.3^n
|
||||
\]
|
||||
\item On calcule la quantité totale déversée entre 2017 et 2030.
|
||||
\[
|
||||
\sum_{n = 0}^{13} u_n = u_0 \times \frac{1-q^{13}}{1-q} = 4 \times \frac{1 - 1.3^{13}}{1 - 1.3} = 390.5
|
||||
\]
|
||||
On en déduit la quantité totale de déchets en 2030
|
||||
\[
|
||||
300 + 390.5 = 690.5
|
||||
\]
|
||||
\item
|
||||
\begin{enumerate}
|
||||
\item ~
|
||||
\begin{center}
|
||||
\begin{tabularx}{0.4\linewidth}{|X|}\hline
|
||||
$N \gets 2017$\\
|
||||
$U \gets 4$ \\
|
||||
$S \gets 300 + U$ \\
|
||||
Tant que $S < 450$ \\
|
||||
\hspace{1cm} $N \gets N + 1$\\
|
||||
\hspace{1cm} $U \gets U * 1.3$\\
|
||||
\hspace{1cm} $S \gets S + u$\\
|
||||
Fin Tant que\\\hline
|
||||
\end{tabularx}
|
||||
\end{center}
|
||||
\item \textit{Pas de correction automatisé}
|
||||
\end{enumerate}
|
||||
\end{enumerate}
|
||||
\end{solution}
|
||||
|
||||
\end{document}
|
||||
|
||||
%%% Local Variables:
|
||||
%%% mode: latex
|
||||
%%% TeX-master: "master"
|
||||
%%% End:
|
262
TST/DS/DS_21_04_07/TST1/18_210407_DS8.tex
Normal file
262
TST/DS/DS_21_04_07/TST1/18_210407_DS8.tex
Normal file
@ -0,0 +1,262 @@
|
||||
\documentclass[a4paper,10pt]{article}
|
||||
\usepackage{myXsim}
|
||||
|
||||
% Title Page
|
||||
\title{DS8 \hfill MOUFAQ Amine}
|
||||
\tribe{TST}
|
||||
\date{\hfillÀ render pour le Mercredi 7 avril}
|
||||
|
||||
\xsimsetup{
|
||||
solution/print = false
|
||||
}
|
||||
|
||||
\begin{document}
|
||||
\maketitle
|
||||
|
||||
\begin{exercise}[subtitle={Automatismes}]
|
||||
\textit{Toutes les questions de cette exercice sont indépendantes et peuvent être répondus séparément}
|
||||
\begin{enumerate}
|
||||
\item De janvier à septembre, une quantité a augmenté de $16\,\%$. Faire un schéma pour représenter la situation puis calculer le taux d'évolution moyen mensuel.
|
||||
\item Une quantité augmente de $16\,\%$ par ans. En 2020, elle est de 115\euro. Quelle était sa valeur en 2019? Faire un schéma pour représenter la situation.
|
||||
\item Déterminer l'équation de la droite \\
|
||||
\begin{tikzpicture}[xscale=0.8, yscale=0.5]
|
||||
\tkzInit[xmin=-5,xmax=5,xstep=1,
|
||||
ymin=-5,ymax=5,ystep=1]
|
||||
\tkzGrid
|
||||
\tkzAxeXY
|
||||
\tkzFct[domain=-5:5,color=red,very thick]%
|
||||
{0.5*\x -1};
|
||||
\end{tikzpicture}
|
||||
\item Résoudre l'équation $6 \times 0.24^x = 41$
|
||||
\end{enumerate}
|
||||
\end{exercise}
|
||||
|
||||
\begin{solution}
|
||||
\begin{enumerate}
|
||||
\item On veut partager cette évolution en 8 évolutions.
|
||||
\[
|
||||
\left(1 + \frac{16}{100}\right)^{\frac{1}{8}} = 1.0187
|
||||
\]
|
||||
Donc le taux d'évolution moyen est
|
||||
\[
|
||||
t_m = 1.0187 - 1 = 0.01869999999999994
|
||||
\]
|
||||
\item Coefficient multiplicateur pour revenir en arrière
|
||||
\[
|
||||
CM = (1 + \frac{16}{100})^{-1} = 0.8621
|
||||
\]
|
||||
On en déduit la quantité en 2019
|
||||
\[
|
||||
115 * 0.8621 = 99.1415
|
||||
\]
|
||||
\item L'équation de la droite est
|
||||
\[
|
||||
y = 0.5 x -1
|
||||
\]
|
||||
\item Il faut penser à faire la division à par $6$ avant d'utiliser le log car sinon, on ne peut pas utiliser la formule $\log(a^n) = n\times \log(a)$.
|
||||
|
||||
\[x = \frac{\log(6.83)}{\log(0.24)}\]
|
||||
\end{enumerate}
|
||||
\end{solution}
|
||||
|
||||
\begin{exercise}[subtitle={Restaurant}]
|
||||
Un \emph{food truck}, ouvert le midi et le soir, propose deux types de formules :
|
||||
|
||||
\setlength\parindent{10mm}
|
||||
\begin{itemize}
|
||||
\item la formule \emph{Burger} ;
|
||||
\item la formule \emph{Wok}.
|
||||
\end{itemize}
|
||||
\setlength\parindent{0mm}
|
||||
|
||||
\medskip
|
||||
|
||||
Le gérant a remarqué que 78\,\% de ses ventes ont lieu le midi. Le quart des ventes du midi correspondent à la formule \emph{Burger}, alors que 37\,\% des ventes du soir correspondent à la formule \emph{Wok}.
|
||||
|
||||
Le gérant se constitue un fichier en notant, pour chaque vente, la formule choisie et le moment de cette vente (midi ou soir).
|
||||
|
||||
On prélève une fiche de façon équiprobable. On définit les quatre évènements suivants:
|
||||
|
||||
\begin{enumerate}
|
||||
\item $M$ : \og la fiche correspond à une vente du midi\fg{} ;
|
||||
\item $S$ : \og la fiche correspond à une vente du soir\fg {};
|
||||
\item $W$ : \og la fiche correspond à une formule \emph{Wok} \fg{} ;
|
||||
\item $B$ : \og la fiche correspond à une formule \emph{Burger} \fg.
|
||||
\end{enumerate}
|
||||
\setlength\parindent{0mm}
|
||||
|
||||
\medskip
|
||||
|
||||
\begin{enumerate}
|
||||
\item Recopier puis compléter l'arbre pondéré
|
||||
|
||||
\begin{center}
|
||||
\begin{tikzpicture}[sloped]
|
||||
\node {.}
|
||||
child {node {$M$}
|
||||
child {node {$W$}
|
||||
edge from parent
|
||||
node[above] {...}
|
||||
}
|
||||
child {node {$B$}
|
||||
edge from parent
|
||||
node[above] {...}
|
||||
}
|
||||
edge from parent
|
||||
node[above] {...}
|
||||
}
|
||||
child[missing] {}
|
||||
child { node {$S$}
|
||||
child {node {$W$}
|
||||
edge from parent
|
||||
node[above] {...}
|
||||
}
|
||||
child {node {$B$}
|
||||
edge from parent
|
||||
node[above] {...}
|
||||
}
|
||||
edge from parent
|
||||
node[above] {...}
|
||||
} ;
|
||||
\end{tikzpicture}
|
||||
\end{center}
|
||||
|
||||
\item Calculer la probabilité de l'évènement $M \cap W$. Interpréter ce résultat dans le contexte de l'exercice.
|
||||
\item Montrer que la probabilité que la fiche choisie corresponde à une formule \emph{Burger} est égale à $0.3336$.
|
||||
\item On a prélevé une fiche correspondant à la formule \emph{Burger}. Quelle est la probabilité, arrondie au millième, que la vente ait eu lieu le soir?
|
||||
\end{enumerate}
|
||||
\end{exercise}
|
||||
|
||||
\begin{solution}
|
||||
\begin{enumerate}
|
||||
\item
|
||||
\begin{center}
|
||||
\begin{tikzpicture}[sloped]
|
||||
\node {.}
|
||||
child {node {$M$}
|
||||
child {node {$W$}
|
||||
edge from parent
|
||||
node[above] {$0.75$}
|
||||
}
|
||||
child {node {$B$}
|
||||
edge from parent
|
||||
node[above] {$0.25$}
|
||||
}
|
||||
edge from parent
|
||||
node[above] {$0.78$}
|
||||
}
|
||||
child[missing] {}
|
||||
child { node {$S$}
|
||||
child {node {$W$}
|
||||
edge from parent
|
||||
node[above] {$0.37$}
|
||||
}
|
||||
child {node {$B$}
|
||||
edge from parent
|
||||
node[above] {$0.63$}
|
||||
}
|
||||
edge from parent
|
||||
node[above] {$0.22$}
|
||||
} ;
|
||||
\end{tikzpicture}
|
||||
\end{center}
|
||||
\item On calcule la probabilité que la vente soit un wok et ait eu lieu à midi
|
||||
\[ P(M\cap W) = P(M) \times P_M(W) = 0.78 \times 0.75 = 0.585 \]
|
||||
\item Probabilité que la vente soit un burger.
|
||||
\[
|
||||
P(B) = P(M\cap B) + P(S\cap B) = 0.78 \times 0.75 + 0.22 \times 0.37 = 0.3336
|
||||
\]
|
||||
\item On cherche à calculer la quantité $P_B(S)$. Pour cela on utilise la formule de Bayes
|
||||
\[
|
||||
P_B(S) = \frac{P(B\cap S)}{P(B)} = \frac{P_S(B) \times P(S)}{P(B)} = \frac{0.63\times 0.22}{0.3336} = 0.41546762589928055 \approx 0.415
|
||||
\]
|
||||
\end{enumerate}
|
||||
\end{solution}
|
||||
|
||||
\begin{exercise}[subtitle={Continent plastique}]
|
||||
\textit{Les quantités évoqués dans cette exercice sont générés au hasard et sont donc complètement farfelus.}
|
||||
\medskip
|
||||
Le \og continent de plastique\fg{} est la plus grande des plaques de déchets plastiques évoluant sur les océans. Elle occupe actuellement dans l'océan Pacifique une surface dont l'aire est évaluée à plus de $1,6$ million de km$^2$, entre Hawaï et la Californie.
|
||||
|
||||
En 2017, des scientifiques ont estimé qu'il y avait $10$ millions de tonnes de déchets plastiques qui était déversé chaque année dans les océans et que cette quantité augmentait de $24\n\%$ par chaque année.
|
||||
|
||||
On modélise l'évolution de la masse de ces déchets plastiques déversée chaque année, si rien n'est fait pour la réduire, par une suite géométrique $\left(u_n\right)$. L'arrondi au centième du terme $u_n$ représente la masse de ces déchets déversée chaque année, exprimée en million de tonnes, pour l'année $(2017 + n)$.
|
||||
|
||||
\medskip
|
||||
|
||||
\begin{enumerate}
|
||||
\item Expliquer pourquoi la suite $u_n$ est géométrique?
|
||||
\item Calculer $u_1$ et $u_2$.
|
||||
\item Exprimer $u_n$ en fonction de $n$.
|
||||
\item Au début de l'année 2017, il y avait $300$ millions de tonnes de déchets plastique. Calculer la quantité totale de déchets plastiques en 2030.
|
||||
\item On souhaite déterminer en quelle année la masse totale de ces déchets plastiques aura pour la première fois augmenté de $50$\,\% par rapport à sa valeur de 2017.
|
||||
\begin{enumerate}
|
||||
\item Recopier et compléter l'algorithme ci-dessous pour que la variable $N$ contienne la réponse au problème posé.
|
||||
|
||||
\begin{center}
|
||||
\begin{tabularx}{0.4\linewidth}{|X|}\hline
|
||||
$N = 2017$\\
|
||||
$U = 10$ \\
|
||||
$S = 300 + U$ \\
|
||||
while $S < 450$: \\
|
||||
\hspace{1cm} $N = \ldots$\\
|
||||
\hspace{1cm} $U = \ldots$\\
|
||||
\hspace{1cm} $S = \ldots$\\
|
||||
\hline
|
||||
\end{tabularx}
|
||||
\end{center}
|
||||
\item Que contiennent les variables $S$, $U$ et $N$ après exécution de cet algorithme ?
|
||||
|
||||
Interpréter les résultats dans le contexte de l'exercice.
|
||||
\end{enumerate}
|
||||
\end{enumerate}
|
||||
\end{exercise}
|
||||
|
||||
\begin{solution}
|
||||
\begin{enumerate}
|
||||
\item Une augmentation de $24\,\%$ revient à multiplier la quantité par $1.24$. La suite est donc bien géométrique. Son premier terme est $u_0 = 10$ et sa raison est $q = 1.24$
|
||||
\item
|
||||
\[
|
||||
u_1 = u_0 * 1.24 = 12.4
|
||||
\]
|
||||
\[
|
||||
u_2 = u_0 * 1.24^2 = 15.376
|
||||
\]
|
||||
\item
|
||||
\[
|
||||
u_n = u_0 \times q^n = 10 \times 1.24^n
|
||||
\]
|
||||
\item On calcule la quantité totale déversée entre 2017 et 2030.
|
||||
\[
|
||||
\sum_{n = 0}^{13} u_n = u_0 \times \frac{1-q^{13}}{1-q} = 10 \times \frac{1 - 1.24^{13}}{1 - 1.24} = 641.1
|
||||
\]
|
||||
On en déduit la quantité totale de déchets en 2030
|
||||
\[
|
||||
300 + 641.1 = 941.1
|
||||
\]
|
||||
\item
|
||||
\begin{enumerate}
|
||||
\item ~
|
||||
\begin{center}
|
||||
\begin{tabularx}{0.4\linewidth}{|X|}\hline
|
||||
$N \gets 2017$\\
|
||||
$U \gets 10$ \\
|
||||
$S \gets 300 + U$ \\
|
||||
Tant que $S < 450$ \\
|
||||
\hspace{1cm} $N \gets N + 1$\\
|
||||
\hspace{1cm} $U \gets U * 1.24$\\
|
||||
\hspace{1cm} $S \gets S + u$\\
|
||||
Fin Tant que\\\hline
|
||||
\end{tabularx}
|
||||
\end{center}
|
||||
\item \textit{Pas de correction automatisé}
|
||||
\end{enumerate}
|
||||
\end{enumerate}
|
||||
\end{solution}
|
||||
|
||||
\end{document}
|
||||
|
||||
%%% Local Variables:
|
||||
%%% mode: latex
|
||||
%%% TeX-master: "master"
|
||||
%%% End:
|
262
TST/DS/DS_21_04_07/TST1/19_210407_DS8.tex
Normal file
262
TST/DS/DS_21_04_07/TST1/19_210407_DS8.tex
Normal file
@ -0,0 +1,262 @@
|
||||
\documentclass[a4paper,10pt]{article}
|
||||
\usepackage{myXsim}
|
||||
|
||||
% Title Page
|
||||
\title{DS8 \hfill ONAL Yakub}
|
||||
\tribe{TST}
|
||||
\date{\hfillÀ render pour le Mercredi 7 avril}
|
||||
|
||||
\xsimsetup{
|
||||
solution/print = false
|
||||
}
|
||||
|
||||
\begin{document}
|
||||
\maketitle
|
||||
|
||||
\begin{exercise}[subtitle={Automatismes}]
|
||||
\textit{Toutes les questions de cette exercice sont indépendantes et peuvent être répondus séparément}
|
||||
\begin{enumerate}
|
||||
\item De janvier à septembre, une quantité a augmenté de $27\,\%$. Faire un schéma pour représenter la situation puis calculer le taux d'évolution moyen mensuel.
|
||||
\item Une quantité augmente de $27\,\%$ par ans. En 2020, elle est de 132\euro. Quelle était sa valeur en 2019? Faire un schéma pour représenter la situation.
|
||||
\item Déterminer l'équation de la droite \\
|
||||
\begin{tikzpicture}[xscale=0.8, yscale=0.5]
|
||||
\tkzInit[xmin=-5,xmax=5,xstep=1,
|
||||
ymin=-5,ymax=5,ystep=1]
|
||||
\tkzGrid
|
||||
\tkzAxeXY
|
||||
\tkzFct[domain=-5:5,color=red,very thick]%
|
||||
{1.0*\x -1};
|
||||
\end{tikzpicture}
|
||||
\item Résoudre l'équation $10 \times 0.18^x = 7$
|
||||
\end{enumerate}
|
||||
\end{exercise}
|
||||
|
||||
\begin{solution}
|
||||
\begin{enumerate}
|
||||
\item On veut partager cette évolution en 8 évolutions.
|
||||
\[
|
||||
\left(1 + \frac{27}{100}\right)^{\frac{1}{8}} = 1.0303
|
||||
\]
|
||||
Donc le taux d'évolution moyen est
|
||||
\[
|
||||
t_m = 1.0303 - 1 = 0.030299999999999994
|
||||
\]
|
||||
\item Coefficient multiplicateur pour revenir en arrière
|
||||
\[
|
||||
CM = (1 + \frac{27}{100})^{-1} = 0.7874
|
||||
\]
|
||||
On en déduit la quantité en 2019
|
||||
\[
|
||||
132 * 0.7874 = 103.9368
|
||||
\]
|
||||
\item L'équation de la droite est
|
||||
\[
|
||||
y = 1.0 x -1
|
||||
\]
|
||||
\item Il faut penser à faire la division à par $10$ avant d'utiliser le log car sinon, on ne peut pas utiliser la formule $\log(a^n) = n\times \log(a)$.
|
||||
|
||||
\[x = \frac{\log(0.7)}{\log(0.18)}\]
|
||||
\end{enumerate}
|
||||
\end{solution}
|
||||
|
||||
\begin{exercise}[subtitle={Restaurant}]
|
||||
Un \emph{food truck}, ouvert le midi et le soir, propose deux types de formules :
|
||||
|
||||
\setlength\parindent{10mm}
|
||||
\begin{itemize}
|
||||
\item la formule \emph{Burger} ;
|
||||
\item la formule \emph{Wok}.
|
||||
\end{itemize}
|
||||
\setlength\parindent{0mm}
|
||||
|
||||
\medskip
|
||||
|
||||
Le gérant a remarqué que 78\,\% de ses ventes ont lieu le midi. Le quart des ventes du midi correspondent à la formule \emph{Burger}, alors que 28\,\% des ventes du soir correspondent à la formule \emph{Wok}.
|
||||
|
||||
Le gérant se constitue un fichier en notant, pour chaque vente, la formule choisie et le moment de cette vente (midi ou soir).
|
||||
|
||||
On prélève une fiche de façon équiprobable. On définit les quatre évènements suivants:
|
||||
|
||||
\begin{enumerate}
|
||||
\item $M$ : \og la fiche correspond à une vente du midi\fg{} ;
|
||||
\item $S$ : \og la fiche correspond à une vente du soir\fg {};
|
||||
\item $W$ : \og la fiche correspond à une formule \emph{Wok} \fg{} ;
|
||||
\item $B$ : \og la fiche correspond à une formule \emph{Burger} \fg.
|
||||
\end{enumerate}
|
||||
\setlength\parindent{0mm}
|
||||
|
||||
\medskip
|
||||
|
||||
\begin{enumerate}
|
||||
\item Recopier puis compléter l'arbre pondéré
|
||||
|
||||
\begin{center}
|
||||
\begin{tikzpicture}[sloped]
|
||||
\node {.}
|
||||
child {node {$M$}
|
||||
child {node {$W$}
|
||||
edge from parent
|
||||
node[above] {...}
|
||||
}
|
||||
child {node {$B$}
|
||||
edge from parent
|
||||
node[above] {...}
|
||||
}
|
||||
edge from parent
|
||||
node[above] {...}
|
||||
}
|
||||
child[missing] {}
|
||||
child { node {$S$}
|
||||
child {node {$W$}
|
||||
edge from parent
|
||||
node[above] {...}
|
||||
}
|
||||
child {node {$B$}
|
||||
edge from parent
|
||||
node[above] {...}
|
||||
}
|
||||
edge from parent
|
||||
node[above] {...}
|
||||
} ;
|
||||
\end{tikzpicture}
|
||||
\end{center}
|
||||
|
||||
\item Calculer la probabilité de l'évènement $M \cap W$. Interpréter ce résultat dans le contexte de l'exercice.
|
||||
\item Montrer que la probabilité que la fiche choisie corresponde à une formule \emph{Burger} est égale à $0.3534$.
|
||||
\item On a prélevé une fiche correspondant à la formule \emph{Burger}. Quelle est la probabilité, arrondie au millième, que la vente ait eu lieu le soir?
|
||||
\end{enumerate}
|
||||
\end{exercise}
|
||||
|
||||
\begin{solution}
|
||||
\begin{enumerate}
|
||||
\item
|
||||
\begin{center}
|
||||
\begin{tikzpicture}[sloped]
|
||||
\node {.}
|
||||
child {node {$M$}
|
||||
child {node {$W$}
|
||||
edge from parent
|
||||
node[above] {$0.75$}
|
||||
}
|
||||
child {node {$B$}
|
||||
edge from parent
|
||||
node[above] {$0.25$}
|
||||
}
|
||||
edge from parent
|
||||
node[above] {$0.78$}
|
||||
}
|
||||
child[missing] {}
|
||||
child { node {$S$}
|
||||
child {node {$W$}
|
||||
edge from parent
|
||||
node[above] {$0.28$}
|
||||
}
|
||||
child {node {$B$}
|
||||
edge from parent
|
||||
node[above] {$0.72$}
|
||||
}
|
||||
edge from parent
|
||||
node[above] {$0.22$}
|
||||
} ;
|
||||
\end{tikzpicture}
|
||||
\end{center}
|
||||
\item On calcule la probabilité que la vente soit un wok et ait eu lieu à midi
|
||||
\[ P(M\cap W) = P(M) \times P_M(W) = 0.78 \times 0.75 = 0.585 \]
|
||||
\item Probabilité que la vente soit un burger.
|
||||
\[
|
||||
P(B) = P(M\cap B) + P(S\cap B) = 0.78 \times 0.75 + 0.22 \times 0.28 = 0.3534
|
||||
\]
|
||||
\item On cherche à calculer la quantité $P_B(S)$. Pour cela on utilise la formule de Bayes
|
||||
\[
|
||||
P_B(S) = \frac{P(B\cap S)}{P(B)} = \frac{P_S(B) \times P(S)}{P(B)} = \frac{0.72\times 0.22}{0.3534} = 0.4482173174872665 \approx 0.448
|
||||
\]
|
||||
\end{enumerate}
|
||||
\end{solution}
|
||||
|
||||
\begin{exercise}[subtitle={Continent plastique}]
|
||||
\textit{Les quantités évoqués dans cette exercice sont générés au hasard et sont donc complètement farfelus.}
|
||||
\medskip
|
||||
Le \og continent de plastique\fg{} est la plus grande des plaques de déchets plastiques évoluant sur les océans. Elle occupe actuellement dans l'océan Pacifique une surface dont l'aire est évaluée à plus de $1,6$ million de km$^2$, entre Hawaï et la Californie.
|
||||
|
||||
En 2017, des scientifiques ont estimé qu'il y avait $3$ millions de tonnes de déchets plastiques qui était déversé chaque année dans les océans et que cette quantité augmentait de $21\n\%$ par chaque année.
|
||||
|
||||
On modélise l'évolution de la masse de ces déchets plastiques déversée chaque année, si rien n'est fait pour la réduire, par une suite géométrique $\left(u_n\right)$. L'arrondi au centième du terme $u_n$ représente la masse de ces déchets déversée chaque année, exprimée en million de tonnes, pour l'année $(2017 + n)$.
|
||||
|
||||
\medskip
|
||||
|
||||
\begin{enumerate}
|
||||
\item Expliquer pourquoi la suite $u_n$ est géométrique?
|
||||
\item Calculer $u_1$ et $u_2$.
|
||||
\item Exprimer $u_n$ en fonction de $n$.
|
||||
\item Au début de l'année 2017, il y avait $300$ millions de tonnes de déchets plastique. Calculer la quantité totale de déchets plastiques en 2030.
|
||||
\item On souhaite déterminer en quelle année la masse totale de ces déchets plastiques aura pour la première fois augmenté de $50$\,\% par rapport à sa valeur de 2017.
|
||||
\begin{enumerate}
|
||||
\item Recopier et compléter l'algorithme ci-dessous pour que la variable $N$ contienne la réponse au problème posé.
|
||||
|
||||
\begin{center}
|
||||
\begin{tabularx}{0.4\linewidth}{|X|}\hline
|
||||
$N = 2017$\\
|
||||
$U = 3$ \\
|
||||
$S = 300 + U$ \\
|
||||
while $S < 450$: \\
|
||||
\hspace{1cm} $N = \ldots$\\
|
||||
\hspace{1cm} $U = \ldots$\\
|
||||
\hspace{1cm} $S = \ldots$\\
|
||||
\hline
|
||||
\end{tabularx}
|
||||
\end{center}
|
||||
\item Que contiennent les variables $S$, $U$ et $N$ après exécution de cet algorithme ?
|
||||
|
||||
Interpréter les résultats dans le contexte de l'exercice.
|
||||
\end{enumerate}
|
||||
\end{enumerate}
|
||||
\end{exercise}
|
||||
|
||||
\begin{solution}
|
||||
\begin{enumerate}
|
||||
\item Une augmentation de $21\,\%$ revient à multiplier la quantité par $1.21$. La suite est donc bien géométrique. Son premier terme est $u_0 = 3$ et sa raison est $q = 1.21$
|
||||
\item
|
||||
\[
|
||||
u_1 = u_0 * 1.21 = 3.63
|
||||
\]
|
||||
\[
|
||||
u_2 = u_0 * 1.21^2 = 4.3923
|
||||
\]
|
||||
\item
|
||||
\[
|
||||
u_n = u_0 \times q^n = 3 \times 1.21^n
|
||||
\]
|
||||
\item On calcule la quantité totale déversée entre 2017 et 2030.
|
||||
\[
|
||||
\sum_{n = 0}^{13} u_n = u_0 \times \frac{1-q^{13}}{1-q} = 3 \times \frac{1 - 1.21^{13}}{1 - 1.21} = 155.97
|
||||
\]
|
||||
On en déduit la quantité totale de déchets en 2030
|
||||
\[
|
||||
300 + 155.97 = 455.97
|
||||
\]
|
||||
\item
|
||||
\begin{enumerate}
|
||||
\item ~
|
||||
\begin{center}
|
||||
\begin{tabularx}{0.4\linewidth}{|X|}\hline
|
||||
$N \gets 2017$\\
|
||||
$U \gets 3$ \\
|
||||
$S \gets 300 + U$ \\
|
||||
Tant que $S < 450$ \\
|
||||
\hspace{1cm} $N \gets N + 1$\\
|
||||
\hspace{1cm} $U \gets U * 1.21$\\
|
||||
\hspace{1cm} $S \gets S + u$\\
|
||||
Fin Tant que\\\hline
|
||||
\end{tabularx}
|
||||
\end{center}
|
||||
\item \textit{Pas de correction automatisé}
|
||||
\end{enumerate}
|
||||
\end{enumerate}
|
||||
\end{solution}
|
||||
|
||||
\end{document}
|
||||
|
||||
%%% Local Variables:
|
||||
%%% mode: latex
|
||||
%%% TeX-master: "master"
|
||||
%%% End:
|
262
TST/DS/DS_21_04_07/TST1/20_210407_DS8.tex
Normal file
262
TST/DS/DS_21_04_07/TST1/20_210407_DS8.tex
Normal file
@ -0,0 +1,262 @@
|
||||
\documentclass[a4paper,10pt]{article}
|
||||
\usepackage{myXsim}
|
||||
|
||||
% Title Page
|
||||
\title{DS8 \hfill SORIANO Laura}
|
||||
\tribe{TST}
|
||||
\date{\hfillÀ render pour le Mercredi 7 avril}
|
||||
|
||||
\xsimsetup{
|
||||
solution/print = false
|
||||
}
|
||||
|
||||
\begin{document}
|
||||
\maketitle
|
||||
|
||||
\begin{exercise}[subtitle={Automatismes}]
|
||||
\textit{Toutes les questions de cette exercice sont indépendantes et peuvent être répondus séparément}
|
||||
\begin{enumerate}
|
||||
\item De janvier à septembre, une quantité a augmenté de $26\,\%$. Faire un schéma pour représenter la situation puis calculer le taux d'évolution moyen mensuel.
|
||||
\item Une quantité augmente de $26\,\%$ par ans. En 2020, elle est de 122\euro. Quelle était sa valeur en 2019? Faire un schéma pour représenter la situation.
|
||||
\item Déterminer l'équation de la droite \\
|
||||
\begin{tikzpicture}[xscale=0.8, yscale=0.5]
|
||||
\tkzInit[xmin=-5,xmax=5,xstep=1,
|
||||
ymin=-5,ymax=5,ystep=1]
|
||||
\tkzGrid
|
||||
\tkzAxeXY
|
||||
\tkzFct[domain=-5:5,color=red,very thick]%
|
||||
{3.0*\x -3};
|
||||
\end{tikzpicture}
|
||||
\item Résoudre l'équation $2 \times 0.02^x = 10$
|
||||
\end{enumerate}
|
||||
\end{exercise}
|
||||
|
||||
\begin{solution}
|
||||
\begin{enumerate}
|
||||
\item On veut partager cette évolution en 8 évolutions.
|
||||
\[
|
||||
\left(1 + \frac{26}{100}\right)^{\frac{1}{8}} = 1.0293
|
||||
\]
|
||||
Donc le taux d'évolution moyen est
|
||||
\[
|
||||
t_m = 1.0293 - 1 = 0.029300000000000104
|
||||
\]
|
||||
\item Coefficient multiplicateur pour revenir en arrière
|
||||
\[
|
||||
CM = (1 + \frac{26}{100})^{-1} = 0.7937
|
||||
\]
|
||||
On en déduit la quantité en 2019
|
||||
\[
|
||||
122 * 0.7937 = 96.8314
|
||||
\]
|
||||
\item L'équation de la droite est
|
||||
\[
|
||||
y = 3.0 x -3
|
||||
\]
|
||||
\item Il faut penser à faire la division à par $2$ avant d'utiliser le log car sinon, on ne peut pas utiliser la formule $\log(a^n) = n\times \log(a)$.
|
||||
|
||||
\[x = \frac{\log(5.0)}{\log(0.02)}\]
|
||||
\end{enumerate}
|
||||
\end{solution}
|
||||
|
||||
\begin{exercise}[subtitle={Restaurant}]
|
||||
Un \emph{food truck}, ouvert le midi et le soir, propose deux types de formules :
|
||||
|
||||
\setlength\parindent{10mm}
|
||||
\begin{itemize}
|
||||
\item la formule \emph{Burger} ;
|
||||
\item la formule \emph{Wok}.
|
||||
\end{itemize}
|
||||
\setlength\parindent{0mm}
|
||||
|
||||
\medskip
|
||||
|
||||
Le gérant a remarqué que 0\,\% de ses ventes ont lieu le midi. Le quart des ventes du midi correspondent à la formule \emph{Burger}, alors que 21\,\% des ventes du soir correspondent à la formule \emph{Wok}.
|
||||
|
||||
Le gérant se constitue un fichier en notant, pour chaque vente, la formule choisie et le moment de cette vente (midi ou soir).
|
||||
|
||||
On prélève une fiche de façon équiprobable. On définit les quatre évènements suivants:
|
||||
|
||||
\begin{enumerate}
|
||||
\item $M$ : \og la fiche correspond à une vente du midi\fg{} ;
|
||||
\item $S$ : \og la fiche correspond à une vente du soir\fg {};
|
||||
\item $W$ : \og la fiche correspond à une formule \emph{Wok} \fg{} ;
|
||||
\item $B$ : \og la fiche correspond à une formule \emph{Burger} \fg.
|
||||
\end{enumerate}
|
||||
\setlength\parindent{0mm}
|
||||
|
||||
\medskip
|
||||
|
||||
\begin{enumerate}
|
||||
\item Recopier puis compléter l'arbre pondéré
|
||||
|
||||
\begin{center}
|
||||
\begin{tikzpicture}[sloped]
|
||||
\node {.}
|
||||
child {node {$M$}
|
||||
child {node {$W$}
|
||||
edge from parent
|
||||
node[above] {...}
|
||||
}
|
||||
child {node {$B$}
|
||||
edge from parent
|
||||
node[above] {...}
|
||||
}
|
||||
edge from parent
|
||||
node[above] {...}
|
||||
}
|
||||
child[missing] {}
|
||||
child { node {$S$}
|
||||
child {node {$W$}
|
||||
edge from parent
|
||||
node[above] {...}
|
||||
}
|
||||
child {node {$B$}
|
||||
edge from parent
|
||||
node[above] {...}
|
||||
}
|
||||
edge from parent
|
||||
node[above] {...}
|
||||
} ;
|
||||
\end{tikzpicture}
|
||||
\end{center}
|
||||
|
||||
\item Calculer la probabilité de l'évènement $M \cap W$. Interpréter ce résultat dans le contexte de l'exercice.
|
||||
\item Montrer que la probabilité que la fiche choisie corresponde à une formule \emph{Burger} est égale à $0.79$.
|
||||
\item On a prélevé une fiche correspondant à la formule \emph{Burger}. Quelle est la probabilité, arrondie au millième, que la vente ait eu lieu le soir?
|
||||
\end{enumerate}
|
||||
\end{exercise}
|
||||
|
||||
\begin{solution}
|
||||
\begin{enumerate}
|
||||
\item
|
||||
\begin{center}
|
||||
\begin{tikzpicture}[sloped]
|
||||
\node {.}
|
||||
child {node {$M$}
|
||||
child {node {$W$}
|
||||
edge from parent
|
||||
node[above] {$0.75$}
|
||||
}
|
||||
child {node {$B$}
|
||||
edge from parent
|
||||
node[above] {$0.25$}
|
||||
}
|
||||
edge from parent
|
||||
node[above] {$0.0$}
|
||||
}
|
||||
child[missing] {}
|
||||
child { node {$S$}
|
||||
child {node {$W$}
|
||||
edge from parent
|
||||
node[above] {$0.21$}
|
||||
}
|
||||
child {node {$B$}
|
||||
edge from parent
|
||||
node[above] {$0.79$}
|
||||
}
|
||||
edge from parent
|
||||
node[above] {$1.0$}
|
||||
} ;
|
||||
\end{tikzpicture}
|
||||
\end{center}
|
||||
\item On calcule la probabilité que la vente soit un wok et ait eu lieu à midi
|
||||
\[ P(M\cap W) = P(M) \times P_M(W) = 0.0 \times 0.75 = 0.0 \]
|
||||
\item Probabilité que la vente soit un burger.
|
||||
\[
|
||||
P(B) = P(M\cap B) + P(S\cap B) = 0.0 \times 0.75 + 1.0 \times 0.21 = 0.79
|
||||
\]
|
||||
\item On cherche à calculer la quantité $P_B(S)$. Pour cela on utilise la formule de Bayes
|
||||
\[
|
||||
P_B(S) = \frac{P(B\cap S)}{P(B)} = \frac{P_S(B) \times P(S)}{P(B)} = \frac{0.79\times 1.0}{0.79} = 1.0 \approx 1.0
|
||||
\]
|
||||
\end{enumerate}
|
||||
\end{solution}
|
||||
|
||||
\begin{exercise}[subtitle={Continent plastique}]
|
||||
\textit{Les quantités évoqués dans cette exercice sont générés au hasard et sont donc complètement farfelus.}
|
||||
\medskip
|
||||
Le \og continent de plastique\fg{} est la plus grande des plaques de déchets plastiques évoluant sur les océans. Elle occupe actuellement dans l'océan Pacifique une surface dont l'aire est évaluée à plus de $1,6$ million de km$^2$, entre Hawaï et la Californie.
|
||||
|
||||
En 2017, des scientifiques ont estimé qu'il y avait $11$ millions de tonnes de déchets plastiques qui était déversé chaque année dans les océans et que cette quantité augmentait de $16\n\%$ par chaque année.
|
||||
|
||||
On modélise l'évolution de la masse de ces déchets plastiques déversée chaque année, si rien n'est fait pour la réduire, par une suite géométrique $\left(u_n\right)$. L'arrondi au centième du terme $u_n$ représente la masse de ces déchets déversée chaque année, exprimée en million de tonnes, pour l'année $(2017 + n)$.
|
||||
|
||||
\medskip
|
||||
|
||||
\begin{enumerate}
|
||||
\item Expliquer pourquoi la suite $u_n$ est géométrique?
|
||||
\item Calculer $u_1$ et $u_2$.
|
||||
\item Exprimer $u_n$ en fonction de $n$.
|
||||
\item Au début de l'année 2017, il y avait $300$ millions de tonnes de déchets plastique. Calculer la quantité totale de déchets plastiques en 2030.
|
||||
\item On souhaite déterminer en quelle année la masse totale de ces déchets plastiques aura pour la première fois augmenté de $50$\,\% par rapport à sa valeur de 2017.
|
||||
\begin{enumerate}
|
||||
\item Recopier et compléter l'algorithme ci-dessous pour que la variable $N$ contienne la réponse au problème posé.
|
||||
|
||||
\begin{center}
|
||||
\begin{tabularx}{0.4\linewidth}{|X|}\hline
|
||||
$N = 2017$\\
|
||||
$U = 11$ \\
|
||||
$S = 300 + U$ \\
|
||||
while $S < 450$: \\
|
||||
\hspace{1cm} $N = \ldots$\\
|
||||
\hspace{1cm} $U = \ldots$\\
|
||||
\hspace{1cm} $S = \ldots$\\
|
||||
\hline
|
||||
\end{tabularx}
|
||||
\end{center}
|
||||
\item Que contiennent les variables $S$, $U$ et $N$ après exécution de cet algorithme ?
|
||||
|
||||
Interpréter les résultats dans le contexte de l'exercice.
|
||||
\end{enumerate}
|
||||
\end{enumerate}
|
||||
\end{exercise}
|
||||
|
||||
\begin{solution}
|
||||
\begin{enumerate}
|
||||
\item Une augmentation de $16\,\%$ revient à multiplier la quantité par $1.16$. La suite est donc bien géométrique. Son premier terme est $u_0 = 11$ et sa raison est $q = 1.16$
|
||||
\item
|
||||
\[
|
||||
u_1 = u_0 * 1.16 = 12.76
|
||||
\]
|
||||
\[
|
||||
u_2 = u_0 * 1.16^2 = 14.8016
|
||||
\]
|
||||
\item
|
||||
\[
|
||||
u_n = u_0 \times q^n = 11 \times 1.16^n
|
||||
\]
|
||||
\item On calcule la quantité totale déversée entre 2017 et 2030.
|
||||
\[
|
||||
\sum_{n = 0}^{13} u_n = u_0 \times \frac{1-q^{13}}{1-q} = 11 \times \frac{1 - 1.16^{13}}{1 - 1.16} = 404.65
|
||||
\]
|
||||
On en déduit la quantité totale de déchets en 2030
|
||||
\[
|
||||
300 + 404.65 = 704.65
|
||||
\]
|
||||
\item
|
||||
\begin{enumerate}
|
||||
\item ~
|
||||
\begin{center}
|
||||
\begin{tabularx}{0.4\linewidth}{|X|}\hline
|
||||
$N \gets 2017$\\
|
||||
$U \gets 11$ \\
|
||||
$S \gets 300 + U$ \\
|
||||
Tant que $S < 450$ \\
|
||||
\hspace{1cm} $N \gets N + 1$\\
|
||||
\hspace{1cm} $U \gets U * 1.16$\\
|
||||
\hspace{1cm} $S \gets S + u$\\
|
||||
Fin Tant que\\\hline
|
||||
\end{tabularx}
|
||||
\end{center}
|
||||
\item \textit{Pas de correction automatisé}
|
||||
\end{enumerate}
|
||||
\end{enumerate}
|
||||
\end{solution}
|
||||
|
||||
\end{document}
|
||||
|
||||
%%% Local Variables:
|
||||
%%% mode: latex
|
||||
%%% TeX-master: "master"
|
||||
%%% End:
|
262
TST/DS/DS_21_04_07/TST1/21_210407_DS8.tex
Normal file
262
TST/DS/DS_21_04_07/TST1/21_210407_DS8.tex
Normal file
@ -0,0 +1,262 @@
|
||||
\documentclass[a4paper,10pt]{article}
|
||||
\usepackage{myXsim}
|
||||
|
||||
% Title Page
|
||||
\title{DS8 \hfill VECCHIO Léa}
|
||||
\tribe{TST}
|
||||
\date{\hfillÀ render pour le Mercredi 7 avril}
|
||||
|
||||
\xsimsetup{
|
||||
solution/print = false
|
||||
}
|
||||
|
||||
\begin{document}
|
||||
\maketitle
|
||||
|
||||
\begin{exercise}[subtitle={Automatismes}]
|
||||
\textit{Toutes les questions de cette exercice sont indépendantes et peuvent être répondus séparément}
|
||||
\begin{enumerate}
|
||||
\item De janvier à septembre, une quantité a augmenté de $28\,\%$. Faire un schéma pour représenter la situation puis calculer le taux d'évolution moyen mensuel.
|
||||
\item Une quantité augmente de $28\,\%$ par ans. En 2020, elle est de 113\euro. Quelle était sa valeur en 2019? Faire un schéma pour représenter la situation.
|
||||
\item Déterminer l'équation de la droite \\
|
||||
\begin{tikzpicture}[xscale=0.8, yscale=0.5]
|
||||
\tkzInit[xmin=-5,xmax=5,xstep=1,
|
||||
ymin=-5,ymax=5,ystep=1]
|
||||
\tkzGrid
|
||||
\tkzAxeXY
|
||||
\tkzFct[domain=-5:5,color=red,very thick]%
|
||||
{2.0*\x -2};
|
||||
\end{tikzpicture}
|
||||
\item Résoudre l'équation $8 \times 0.84^x = 26$
|
||||
\end{enumerate}
|
||||
\end{exercise}
|
||||
|
||||
\begin{solution}
|
||||
\begin{enumerate}
|
||||
\item On veut partager cette évolution en 8 évolutions.
|
||||
\[
|
||||
\left(1 + \frac{28}{100}\right)^{\frac{1}{8}} = 1.0313
|
||||
\]
|
||||
Donc le taux d'évolution moyen est
|
||||
\[
|
||||
t_m = 1.0313 - 1 = 0.031300000000000106
|
||||
\]
|
||||
\item Coefficient multiplicateur pour revenir en arrière
|
||||
\[
|
||||
CM = (1 + \frac{28}{100})^{-1} = 0.7812
|
||||
\]
|
||||
On en déduit la quantité en 2019
|
||||
\[
|
||||
113 * 0.7812 = 88.2756
|
||||
\]
|
||||
\item L'équation de la droite est
|
||||
\[
|
||||
y = 2.0 x -2
|
||||
\]
|
||||
\item Il faut penser à faire la division à par $8$ avant d'utiliser le log car sinon, on ne peut pas utiliser la formule $\log(a^n) = n\times \log(a)$.
|
||||
|
||||
\[x = \frac{\log(3.25)}{\log(0.84)}\]
|
||||
\end{enumerate}
|
||||
\end{solution}
|
||||
|
||||
\begin{exercise}[subtitle={Restaurant}]
|
||||
Un \emph{food truck}, ouvert le midi et le soir, propose deux types de formules :
|
||||
|
||||
\setlength\parindent{10mm}
|
||||
\begin{itemize}
|
||||
\item la formule \emph{Burger} ;
|
||||
\item la formule \emph{Wok}.
|
||||
\end{itemize}
|
||||
\setlength\parindent{0mm}
|
||||
|
||||
\medskip
|
||||
|
||||
Le gérant a remarqué que 18\,\% de ses ventes ont lieu le midi. Le quart des ventes du midi correspondent à la formule \emph{Burger}, alors que 16\,\% des ventes du soir correspondent à la formule \emph{Wok}.
|
||||
|
||||
Le gérant se constitue un fichier en notant, pour chaque vente, la formule choisie et le moment de cette vente (midi ou soir).
|
||||
|
||||
On prélève une fiche de façon équiprobable. On définit les quatre évènements suivants:
|
||||
|
||||
\begin{enumerate}
|
||||
\item $M$ : \og la fiche correspond à une vente du midi\fg{} ;
|
||||
\item $S$ : \og la fiche correspond à une vente du soir\fg {};
|
||||
\item $W$ : \og la fiche correspond à une formule \emph{Wok} \fg{} ;
|
||||
\item $B$ : \og la fiche correspond à une formule \emph{Burger} \fg.
|
||||
\end{enumerate}
|
||||
\setlength\parindent{0mm}
|
||||
|
||||
\medskip
|
||||
|
||||
\begin{enumerate}
|
||||
\item Recopier puis compléter l'arbre pondéré
|
||||
|
||||
\begin{center}
|
||||
\begin{tikzpicture}[sloped]
|
||||
\node {.}
|
||||
child {node {$M$}
|
||||
child {node {$W$}
|
||||
edge from parent
|
||||
node[above] {...}
|
||||
}
|
||||
child {node {$B$}
|
||||
edge from parent
|
||||
node[above] {...}
|
||||
}
|
||||
edge from parent
|
||||
node[above] {...}
|
||||
}
|
||||
child[missing] {}
|
||||
child { node {$S$}
|
||||
child {node {$W$}
|
||||
edge from parent
|
||||
node[above] {...}
|
||||
}
|
||||
child {node {$B$}
|
||||
edge from parent
|
||||
node[above] {...}
|
||||
}
|
||||
edge from parent
|
||||
node[above] {...}
|
||||
} ;
|
||||
\end{tikzpicture}
|
||||
\end{center}
|
||||
|
||||
\item Calculer la probabilité de l'évènement $M \cap W$. Interpréter ce résultat dans le contexte de l'exercice.
|
||||
\item Montrer que la probabilité que la fiche choisie corresponde à une formule \emph{Burger} est égale à $0.7338$.
|
||||
\item On a prélevé une fiche correspondant à la formule \emph{Burger}. Quelle est la probabilité, arrondie au millième, que la vente ait eu lieu le soir?
|
||||
\end{enumerate}
|
||||
\end{exercise}
|
||||
|
||||
\begin{solution}
|
||||
\begin{enumerate}
|
||||
\item
|
||||
\begin{center}
|
||||
\begin{tikzpicture}[sloped]
|
||||
\node {.}
|
||||
child {node {$M$}
|
||||
child {node {$W$}
|
||||
edge from parent
|
||||
node[above] {$0.75$}
|
||||
}
|
||||
child {node {$B$}
|
||||
edge from parent
|
||||
node[above] {$0.25$}
|
||||
}
|
||||
edge from parent
|
||||
node[above] {$0.18$}
|
||||
}
|
||||
child[missing] {}
|
||||
child { node {$S$}
|
||||
child {node {$W$}
|
||||
edge from parent
|
||||
node[above] {$0.16$}
|
||||
}
|
||||
child {node {$B$}
|
||||
edge from parent
|
||||
node[above] {$0.84$}
|
||||
}
|
||||
edge from parent
|
||||
node[above] {$0.82$}
|
||||
} ;
|
||||
\end{tikzpicture}
|
||||
\end{center}
|
||||
\item On calcule la probabilité que la vente soit un wok et ait eu lieu à midi
|
||||
\[ P(M\cap W) = P(M) \times P_M(W) = 0.18 \times 0.75 = 0.135 \]
|
||||
\item Probabilité que la vente soit un burger.
|
||||
\[
|
||||
P(B) = P(M\cap B) + P(S\cap B) = 0.18 \times 0.75 + 0.82 \times 0.16 = 0.7338
|
||||
\]
|
||||
\item On cherche à calculer la quantité $P_B(S)$. Pour cela on utilise la formule de Bayes
|
||||
\[
|
||||
P_B(S) = \frac{P(B\cap S)}{P(B)} = \frac{P_S(B) \times P(S)}{P(B)} = \frac{0.84\times 0.82}{0.7338} = 0.9386753883892068 \approx 0.939
|
||||
\]
|
||||
\end{enumerate}
|
||||
\end{solution}
|
||||
|
||||
\begin{exercise}[subtitle={Continent plastique}]
|
||||
\textit{Les quantités évoqués dans cette exercice sont générés au hasard et sont donc complètement farfelus.}
|
||||
\medskip
|
||||
Le \og continent de plastique\fg{} est la plus grande des plaques de déchets plastiques évoluant sur les océans. Elle occupe actuellement dans l'océan Pacifique une surface dont l'aire est évaluée à plus de $1,6$ million de km$^2$, entre Hawaï et la Californie.
|
||||
|
||||
En 2017, des scientifiques ont estimé qu'il y avait $2$ millions de tonnes de déchets plastiques qui était déversé chaque année dans les océans et que cette quantité augmentait de $24\n\%$ par chaque année.
|
||||
|
||||
On modélise l'évolution de la masse de ces déchets plastiques déversée chaque année, si rien n'est fait pour la réduire, par une suite géométrique $\left(u_n\right)$. L'arrondi au centième du terme $u_n$ représente la masse de ces déchets déversée chaque année, exprimée en million de tonnes, pour l'année $(2017 + n)$.
|
||||
|
||||
\medskip
|
||||
|
||||
\begin{enumerate}
|
||||
\item Expliquer pourquoi la suite $u_n$ est géométrique?
|
||||
\item Calculer $u_1$ et $u_2$.
|
||||
\item Exprimer $u_n$ en fonction de $n$.
|
||||
\item Au début de l'année 2017, il y avait $300$ millions de tonnes de déchets plastique. Calculer la quantité totale de déchets plastiques en 2030.
|
||||
\item On souhaite déterminer en quelle année la masse totale de ces déchets plastiques aura pour la première fois augmenté de $50$\,\% par rapport à sa valeur de 2017.
|
||||
\begin{enumerate}
|
||||
\item Recopier et compléter l'algorithme ci-dessous pour que la variable $N$ contienne la réponse au problème posé.
|
||||
|
||||
\begin{center}
|
||||
\begin{tabularx}{0.4\linewidth}{|X|}\hline
|
||||
$N = 2017$\\
|
||||
$U = 2$ \\
|
||||
$S = 300 + U$ \\
|
||||
while $S < 450$: \\
|
||||
\hspace{1cm} $N = \ldots$\\
|
||||
\hspace{1cm} $U = \ldots$\\
|
||||
\hspace{1cm} $S = \ldots$\\
|
||||
\hline
|
||||
\end{tabularx}
|
||||
\end{center}
|
||||
\item Que contiennent les variables $S$, $U$ et $N$ après exécution de cet algorithme ?
|
||||
|
||||
Interpréter les résultats dans le contexte de l'exercice.
|
||||
\end{enumerate}
|
||||
\end{enumerate}
|
||||
\end{exercise}
|
||||
|
||||
\begin{solution}
|
||||
\begin{enumerate}
|
||||
\item Une augmentation de $24\,\%$ revient à multiplier la quantité par $1.24$. La suite est donc bien géométrique. Son premier terme est $u_0 = 2$ et sa raison est $q = 1.24$
|
||||
\item
|
||||
\[
|
||||
u_1 = u_0 * 1.24 = 2.48
|
||||
\]
|
||||
\[
|
||||
u_2 = u_0 * 1.24^2 = 3.0752
|
||||
\]
|
||||
\item
|
||||
\[
|
||||
u_n = u_0 \times q^n = 2 \times 1.24^n
|
||||
\]
|
||||
\item On calcule la quantité totale déversée entre 2017 et 2030.
|
||||
\[
|
||||
\sum_{n = 0}^{13} u_n = u_0 \times \frac{1-q^{13}}{1-q} = 2 \times \frac{1 - 1.24^{13}}{1 - 1.24} = 128.22
|
||||
\]
|
||||
On en déduit la quantité totale de déchets en 2030
|
||||
\[
|
||||
300 + 128.22 = 428.22
|
||||
\]
|
||||
\item
|
||||
\begin{enumerate}
|
||||
\item ~
|
||||
\begin{center}
|
||||
\begin{tabularx}{0.4\linewidth}{|X|}\hline
|
||||
$N \gets 2017$\\
|
||||
$U \gets 2$ \\
|
||||
$S \gets 300 + U$ \\
|
||||
Tant que $S < 450$ \\
|
||||
\hspace{1cm} $N \gets N + 1$\\
|
||||
\hspace{1cm} $U \gets U * 1.24$\\
|
||||
\hspace{1cm} $S \gets S + u$\\
|
||||
Fin Tant que\\\hline
|
||||
\end{tabularx}
|
||||
\end{center}
|
||||
\item \textit{Pas de correction automatisé}
|
||||
\end{enumerate}
|
||||
\end{enumerate}
|
||||
\end{solution}
|
||||
|
||||
\end{document}
|
||||
|
||||
%%% Local Variables:
|
||||
%%% mode: latex
|
||||
%%% TeX-master: "master"
|
||||
%%% End:
|
BIN
TST/DS/DS_21_04_07/TST1/all_210407_DS8.pdf
Normal file
BIN
TST/DS/DS_21_04_07/TST1/all_210407_DS8.pdf
Normal file
Binary file not shown.
262
TST/DS/DS_21_04_07/TST1/corr_01_210407_DS8.tex
Normal file
262
TST/DS/DS_21_04_07/TST1/corr_01_210407_DS8.tex
Normal file
@ -0,0 +1,262 @@
|
||||
\documentclass[a4paper,10pt]{article}
|
||||
\usepackage{myXsim}
|
||||
|
||||
% Title Page
|
||||
\title{DS8 \hfill AIOUAZ Ahmed}
|
||||
\tribe{TST}
|
||||
\date{\hfillÀ render pour le Mercredi 7 avril}
|
||||
|
||||
\xsimsetup{
|
||||
solution/print = true
|
||||
}
|
||||
|
||||
\begin{document}
|
||||
\maketitle
|
||||
|
||||
\begin{exercise}[subtitle={Automatismes}]
|
||||
\textit{Toutes les questions de cette exercice sont indépendantes et peuvent être répondus séparément}
|
||||
\begin{enumerate}
|
||||
\item De janvier à septembre, une quantité a augmenté de $20\,\%$. Faire un schéma pour représenter la situation puis calculer le taux d'évolution moyen mensuel.
|
||||
\item Une quantité augmente de $20\,\%$ par ans. En 2020, elle est de 110\euro. Quelle était sa valeur en 2019? Faire un schéma pour représenter la situation.
|
||||
\item Déterminer l'équation de la droite \\
|
||||
\begin{tikzpicture}[xscale=0.8, yscale=0.5]
|
||||
\tkzInit[xmin=-5,xmax=5,xstep=1,
|
||||
ymin=-5,ymax=5,ystep=1]
|
||||
\tkzGrid
|
||||
\tkzAxeXY
|
||||
\tkzFct[domain=-5:5,color=red,very thick]%
|
||||
{3.0*\x -3};
|
||||
\end{tikzpicture}
|
||||
\item Résoudre l'équation $2 \times 0.07^x = 20$
|
||||
\end{enumerate}
|
||||
\end{exercise}
|
||||
|
||||
\begin{solution}
|
||||
\begin{enumerate}
|
||||
\item On veut partager cette évolution en 8 évolutions.
|
||||
\[
|
||||
\left(1 + \frac{20}{100}\right)^{\frac{1}{8}} = 1.0231
|
||||
\]
|
||||
Donc le taux d'évolution moyen est
|
||||
\[
|
||||
t_m = 1.0231 - 1 = 0.0230999999999999
|
||||
\]
|
||||
\item Coefficient multiplicateur pour revenir en arrière
|
||||
\[
|
||||
CM = (1 + \frac{20}{100})^{-1} = 0.8333
|
||||
\]
|
||||
On en déduit la quantité en 2019
|
||||
\[
|
||||
110 * 0.8333 = 91.66300000000001
|
||||
\]
|
||||
\item L'équation de la droite est
|
||||
\[
|
||||
y = 3.0 x -3
|
||||
\]
|
||||
\item Il faut penser à faire la division à par $2$ avant d'utiliser le log car sinon, on ne peut pas utiliser la formule $\log(a^n) = n\times \log(a)$.
|
||||
|
||||
\[x = \frac{\log(10.0)}{\log(0.07)}\]
|
||||
\end{enumerate}
|
||||
\end{solution}
|
||||
|
||||
\begin{exercise}[subtitle={Restaurant}]
|
||||
Un \emph{food truck}, ouvert le midi et le soir, propose deux types de formules :
|
||||
|
||||
\setlength\parindent{10mm}
|
||||
\begin{itemize}
|
||||
\item la formule \emph{Burger} ;
|
||||
\item la formule \emph{Wok}.
|
||||
\end{itemize}
|
||||
\setlength\parindent{0mm}
|
||||
|
||||
\medskip
|
||||
|
||||
Le gérant a remarqué que 9\,\% de ses ventes ont lieu le midi. Le quart des ventes du midi correspondent à la formule \emph{Burger}, alors que 3\,\% des ventes du soir correspondent à la formule \emph{Wok}.
|
||||
|
||||
Le gérant se constitue un fichier en notant, pour chaque vente, la formule choisie et le moment de cette vente (midi ou soir).
|
||||
|
||||
On prélève une fiche de façon équiprobable. On définit les quatre évènements suivants:
|
||||
|
||||
\begin{enumerate}
|
||||
\item $M$ : \og la fiche correspond à une vente du midi\fg{} ;
|
||||
\item $S$ : \og la fiche correspond à une vente du soir\fg {};
|
||||
\item $W$ : \og la fiche correspond à une formule \emph{Wok} \fg{} ;
|
||||
\item $B$ : \og la fiche correspond à une formule \emph{Burger} \fg.
|
||||
\end{enumerate}
|
||||
\setlength\parindent{0mm}
|
||||
|
||||
\medskip
|
||||
|
||||
\begin{enumerate}
|
||||
\item Recopier puis compléter l'arbre pondéré
|
||||
|
||||
\begin{center}
|
||||
\begin{tikzpicture}[sloped]
|
||||
\node {.}
|
||||
child {node {$M$}
|
||||
child {node {$W$}
|
||||
edge from parent
|
||||
node[above] {...}
|
||||
}
|
||||
child {node {$B$}
|
||||
edge from parent
|
||||
node[above] {...}
|
||||
}
|
||||
edge from parent
|
||||
node[above] {...}
|
||||
}
|
||||
child[missing] {}
|
||||
child { node {$S$}
|
||||
child {node {$W$}
|
||||
edge from parent
|
||||
node[above] {...}
|
||||
}
|
||||
child {node {$B$}
|
||||
edge from parent
|
||||
node[above] {...}
|
||||
}
|
||||
edge from parent
|
||||
node[above] {...}
|
||||
} ;
|
||||
\end{tikzpicture}
|
||||
\end{center}
|
||||
|
||||
\item Calculer la probabilité de l'évènement $M \cap W$. Interpréter ce résultat dans le contexte de l'exercice.
|
||||
\item Montrer que la probabilité que la fiche choisie corresponde à une formule \emph{Burger} est égale à $0.9052$.
|
||||
\item On a prélevé une fiche correspondant à la formule \emph{Burger}. Quelle est la probabilité, arrondie au millième, que la vente ait eu lieu le soir?
|
||||
\end{enumerate}
|
||||
\end{exercise}
|
||||
|
||||
\begin{solution}
|
||||
\begin{enumerate}
|
||||
\item
|
||||
\begin{center}
|
||||
\begin{tikzpicture}[sloped]
|
||||
\node {.}
|
||||
child {node {$M$}
|
||||
child {node {$W$}
|
||||
edge from parent
|
||||
node[above] {$0.75$}
|
||||
}
|
||||
child {node {$B$}
|
||||
edge from parent
|
||||
node[above] {$0.25$}
|
||||
}
|
||||
edge from parent
|
||||
node[above] {$0.09$}
|
||||
}
|
||||
child[missing] {}
|
||||
child { node {$S$}
|
||||
child {node {$W$}
|
||||
edge from parent
|
||||
node[above] {$0.03$}
|
||||
}
|
||||
child {node {$B$}
|
||||
edge from parent
|
||||
node[above] {$0.97$}
|
||||
}
|
||||
edge from parent
|
||||
node[above] {$0.91$}
|
||||
} ;
|
||||
\end{tikzpicture}
|
||||
\end{center}
|
||||
\item On calcule la probabilité que la vente soit un wok et ait eu lieu à midi
|
||||
\[ P(M\cap W) = P(M) \times P_M(W) = 0.09 \times 0.75 = 0.0675 \]
|
||||
\item Probabilité que la vente soit un burger.
|
||||
\[
|
||||
P(B) = P(M\cap B) + P(S\cap B) = 0.09 \times 0.75 + 0.91 \times 0.03 = 0.9052
|
||||
\]
|
||||
\item On cherche à calculer la quantité $P_B(S)$. Pour cela on utilise la formule de Bayes
|
||||
\[
|
||||
P_B(S) = \frac{P(B\cap S)}{P(B)} = \frac{P_S(B) \times P(S)}{P(B)} = \frac{0.97\times 0.91}{0.9052} = 0.975143614670791 \approx 0.975
|
||||
\]
|
||||
\end{enumerate}
|
||||
\end{solution}
|
||||
|
||||
\begin{exercise}[subtitle={Continent plastique}]
|
||||
\textit{Les quantités évoqués dans cette exercice sont générés au hasard et sont donc complètement farfelus.}
|
||||
\medskip
|
||||
Le \og continent de plastique\fg{} est la plus grande des plaques de déchets plastiques évoluant sur les océans. Elle occupe actuellement dans l'océan Pacifique une surface dont l'aire est évaluée à plus de $1,6$ million de km$^2$, entre Hawaï et la Californie.
|
||||
|
||||
En 2017, des scientifiques ont estimé qu'il y avait $14$ millions de tonnes de déchets plastiques qui était déversé chaque année dans les océans et que cette quantité augmentait de $28\n\%$ par chaque année.
|
||||
|
||||
On modélise l'évolution de la masse de ces déchets plastiques déversée chaque année, si rien n'est fait pour la réduire, par une suite géométrique $\left(u_n\right)$. L'arrondi au centième du terme $u_n$ représente la masse de ces déchets déversée chaque année, exprimée en million de tonnes, pour l'année $(2017 + n)$.
|
||||
|
||||
\medskip
|
||||
|
||||
\begin{enumerate}
|
||||
\item Expliquer pourquoi la suite $u_n$ est géométrique?
|
||||
\item Calculer $u_1$ et $u_2$.
|
||||
\item Exprimer $u_n$ en fonction de $n$.
|
||||
\item Au début de l'année 2017, il y avait $300$ millions de tonnes de déchets plastique. Calculer la quantité totale de déchets plastiques en 2030.
|
||||
\item On souhaite déterminer en quelle année la masse totale de ces déchets plastiques aura pour la première fois augmenté de $50$\,\% par rapport à sa valeur de 2017.
|
||||
\begin{enumerate}
|
||||
\item Recopier et compléter l'algorithme ci-dessous pour que la variable $N$ contienne la réponse au problème posé.
|
||||
|
||||
\begin{center}
|
||||
\begin{tabularx}{0.4\linewidth}{|X|}\hline
|
||||
$N = 2017$\\
|
||||
$U = 14$ \\
|
||||
$S = 300 + U$ \\
|
||||
while $S < 450$: \\
|
||||
\hspace{1cm} $N = \ldots$\\
|
||||
\hspace{1cm} $U = \ldots$\\
|
||||
\hspace{1cm} $S = \ldots$\\
|
||||
\hline
|
||||
\end{tabularx}
|
||||
\end{center}
|
||||
\item Que contiennent les variables $S$, $U$ et $N$ après exécution de cet algorithme ?
|
||||
|
||||
Interpréter les résultats dans le contexte de l'exercice.
|
||||
\end{enumerate}
|
||||
\end{enumerate}
|
||||
\end{exercise}
|
||||
|
||||
\begin{solution}
|
||||
\begin{enumerate}
|
||||
\item Une augmentation de $28\,\%$ revient à multiplier la quantité par $1.28$. La suite est donc bien géométrique. Son premier terme est $u_0 = 14$ et sa raison est $q = 1.28$
|
||||
\item
|
||||
\[
|
||||
u_1 = u_0 * 1.28 = 17.92
|
||||
\]
|
||||
\[
|
||||
u_2 = u_0 * 1.28^2 = 22.9376
|
||||
\]
|
||||
\item
|
||||
\[
|
||||
u_n = u_0 \times q^n = 14 \times 1.28^n
|
||||
\]
|
||||
\item On calcule la quantité totale déversée entre 2017 et 2030.
|
||||
\[
|
||||
\sum_{n = 0}^{13} u_n = u_0 \times \frac{1-q^{13}}{1-q} = 14 \times \frac{1 - 1.28^{13}}{1 - 1.28} = 1187.94
|
||||
\]
|
||||
On en déduit la quantité totale de déchets en 2030
|
||||
\[
|
||||
300 + 1187.94 = 1487.94
|
||||
\]
|
||||
\item
|
||||
\begin{enumerate}
|
||||
\item ~
|
||||
\begin{center}
|
||||
\begin{tabularx}{0.4\linewidth}{|X|}\hline
|
||||
$N \gets 2017$\\
|
||||
$U \gets 14$ \\
|
||||
$S \gets 300 + U$ \\
|
||||
Tant que $S < 450$ \\
|
||||
\hspace{1cm} $N \gets N + 1$\\
|
||||
\hspace{1cm} $U \gets U * 1.28$\\
|
||||
\hspace{1cm} $S \gets S + u$\\
|
||||
Fin Tant que\\\hline
|
||||
\end{tabularx}
|
||||
\end{center}
|
||||
\item \textit{Pas de correction automatisé}
|
||||
\end{enumerate}
|
||||
\end{enumerate}
|
||||
\end{solution}
|
||||
|
||||
\end{document}
|
||||
|
||||
%%% Local Variables:
|
||||
%%% mode: latex
|
||||
%%% TeX-master: "master"
|
||||
%%% End:
|
262
TST/DS/DS_21_04_07/TST1/corr_02_210407_DS8.tex
Normal file
262
TST/DS/DS_21_04_07/TST1/corr_02_210407_DS8.tex
Normal file
@ -0,0 +1,262 @@
|
||||
\documentclass[a4paper,10pt]{article}
|
||||
\usepackage{myXsim}
|
||||
|
||||
% Title Page
|
||||
\title{DS8 \hfill BAHBAH Zakaria}
|
||||
\tribe{TST}
|
||||
\date{\hfillÀ render pour le Mercredi 7 avril}
|
||||
|
||||
\xsimsetup{
|
||||
solution/print = true
|
||||
}
|
||||
|
||||
\begin{document}
|
||||
\maketitle
|
||||
|
||||
\begin{exercise}[subtitle={Automatismes}]
|
||||
\textit{Toutes les questions de cette exercice sont indépendantes et peuvent être répondus séparément}
|
||||
\begin{enumerate}
|
||||
\item De janvier à septembre, une quantité a augmenté de $28\,\%$. Faire un schéma pour représenter la situation puis calculer le taux d'évolution moyen mensuel.
|
||||
\item Une quantité augmente de $28\,\%$ par ans. En 2020, elle est de 138\euro. Quelle était sa valeur en 2019? Faire un schéma pour représenter la situation.
|
||||
\item Déterminer l'équation de la droite \\
|
||||
\begin{tikzpicture}[xscale=0.8, yscale=0.5]
|
||||
\tkzInit[xmin=-5,xmax=5,xstep=1,
|
||||
ymin=-5,ymax=5,ystep=1]
|
||||
\tkzGrid
|
||||
\tkzAxeXY
|
||||
\tkzFct[domain=-5:5,color=red,very thick]%
|
||||
{1.0*\x -2};
|
||||
\end{tikzpicture}
|
||||
\item Résoudre l'équation $7 \times 0.54^x = 18$
|
||||
\end{enumerate}
|
||||
\end{exercise}
|
||||
|
||||
\begin{solution}
|
||||
\begin{enumerate}
|
||||
\item On veut partager cette évolution en 8 évolutions.
|
||||
\[
|
||||
\left(1 + \frac{28}{100}\right)^{\frac{1}{8}} = 1.0313
|
||||
\]
|
||||
Donc le taux d'évolution moyen est
|
||||
\[
|
||||
t_m = 1.0313 - 1 = 0.031300000000000106
|
||||
\]
|
||||
\item Coefficient multiplicateur pour revenir en arrière
|
||||
\[
|
||||
CM = (1 + \frac{28}{100})^{-1} = 0.7812
|
||||
\]
|
||||
On en déduit la quantité en 2019
|
||||
\[
|
||||
138 * 0.7812 = 107.8056
|
||||
\]
|
||||
\item L'équation de la droite est
|
||||
\[
|
||||
y = 1.0 x -2
|
||||
\]
|
||||
\item Il faut penser à faire la division à par $7$ avant d'utiliser le log car sinon, on ne peut pas utiliser la formule $\log(a^n) = n\times \log(a)$.
|
||||
|
||||
\[x = \frac{\log(2.57)}{\log(0.54)}\]
|
||||
\end{enumerate}
|
||||
\end{solution}
|
||||
|
||||
\begin{exercise}[subtitle={Restaurant}]
|
||||
Un \emph{food truck}, ouvert le midi et le soir, propose deux types de formules :
|
||||
|
||||
\setlength\parindent{10mm}
|
||||
\begin{itemize}
|
||||
\item la formule \emph{Burger} ;
|
||||
\item la formule \emph{Wok}.
|
||||
\end{itemize}
|
||||
\setlength\parindent{0mm}
|
||||
|
||||
\medskip
|
||||
|
||||
Le gérant a remarqué que 57\,\% de ses ventes ont lieu le midi. Le quart des ventes du midi correspondent à la formule \emph{Burger}, alors que 49\,\% des ventes du soir correspondent à la formule \emph{Wok}.
|
||||
|
||||
Le gérant se constitue un fichier en notant, pour chaque vente, la formule choisie et le moment de cette vente (midi ou soir).
|
||||
|
||||
On prélève une fiche de façon équiprobable. On définit les quatre évènements suivants:
|
||||
|
||||
\begin{enumerate}
|
||||
\item $M$ : \og la fiche correspond à une vente du midi\fg{} ;
|
||||
\item $S$ : \og la fiche correspond à une vente du soir\fg {};
|
||||
\item $W$ : \og la fiche correspond à une formule \emph{Wok} \fg{} ;
|
||||
\item $B$ : \og la fiche correspond à une formule \emph{Burger} \fg.
|
||||
\end{enumerate}
|
||||
\setlength\parindent{0mm}
|
||||
|
||||
\medskip
|
||||
|
||||
\begin{enumerate}
|
||||
\item Recopier puis compléter l'arbre pondéré
|
||||
|
||||
\begin{center}
|
||||
\begin{tikzpicture}[sloped]
|
||||
\node {.}
|
||||
child {node {$M$}
|
||||
child {node {$W$}
|
||||
edge from parent
|
||||
node[above] {...}
|
||||
}
|
||||
child {node {$B$}
|
||||
edge from parent
|
||||
node[above] {...}
|
||||
}
|
||||
edge from parent
|
||||
node[above] {...}
|
||||
}
|
||||
child[missing] {}
|
||||
child { node {$S$}
|
||||
child {node {$W$}
|
||||
edge from parent
|
||||
node[above] {...}
|
||||
}
|
||||
child {node {$B$}
|
||||
edge from parent
|
||||
node[above] {...}
|
||||
}
|
||||
edge from parent
|
||||
node[above] {...}
|
||||
} ;
|
||||
\end{tikzpicture}
|
||||
\end{center}
|
||||
|
||||
\item Calculer la probabilité de l'évènement $M \cap W$. Interpréter ce résultat dans le contexte de l'exercice.
|
||||
\item Montrer que la probabilité que la fiche choisie corresponde à une formule \emph{Burger} est égale à $0.3592$.
|
||||
\item On a prélevé une fiche correspondant à la formule \emph{Burger}. Quelle est la probabilité, arrondie au millième, que la vente ait eu lieu le soir?
|
||||
\end{enumerate}
|
||||
\end{exercise}
|
||||
|
||||
\begin{solution}
|
||||
\begin{enumerate}
|
||||
\item
|
||||
\begin{center}
|
||||
\begin{tikzpicture}[sloped]
|
||||
\node {.}
|
||||
child {node {$M$}
|
||||
child {node {$W$}
|
||||
edge from parent
|
||||
node[above] {$0.75$}
|
||||
}
|
||||
child {node {$B$}
|
||||
edge from parent
|
||||
node[above] {$0.25$}
|
||||
}
|
||||
edge from parent
|
||||
node[above] {$0.58$}
|
||||
}
|
||||
child[missing] {}
|
||||
child { node {$S$}
|
||||
child {node {$W$}
|
||||
edge from parent
|
||||
node[above] {$0.49$}
|
||||
}
|
||||
child {node {$B$}
|
||||
edge from parent
|
||||
node[above] {$0.51$}
|
||||
}
|
||||
edge from parent
|
||||
node[above] {$0.42$}
|
||||
} ;
|
||||
\end{tikzpicture}
|
||||
\end{center}
|
||||
\item On calcule la probabilité que la vente soit un wok et ait eu lieu à midi
|
||||
\[ P(M\cap W) = P(M) \times P_M(W) = 0.58 \times 0.75 = 0.435 \]
|
||||
\item Probabilité que la vente soit un burger.
|
||||
\[
|
||||
P(B) = P(M\cap B) + P(S\cap B) = 0.58 \times 0.75 + 0.42 \times 0.49 = 0.3592
|
||||
\]
|
||||
\item On cherche à calculer la quantité $P_B(S)$. Pour cela on utilise la formule de Bayes
|
||||
\[
|
||||
P_B(S) = \frac{P(B\cap S)}{P(B)} = \frac{P_S(B) \times P(S)}{P(B)} = \frac{0.51\times 0.42}{0.3592} = 0.5963251670378619 \approx 0.596
|
||||
\]
|
||||
\end{enumerate}
|
||||
\end{solution}
|
||||
|
||||
\begin{exercise}[subtitle={Continent plastique}]
|
||||
\textit{Les quantités évoqués dans cette exercice sont générés au hasard et sont donc complètement farfelus.}
|
||||
\medskip
|
||||
Le \og continent de plastique\fg{} est la plus grande des plaques de déchets plastiques évoluant sur les océans. Elle occupe actuellement dans l'océan Pacifique une surface dont l'aire est évaluée à plus de $1,6$ million de km$^2$, entre Hawaï et la Californie.
|
||||
|
||||
En 2017, des scientifiques ont estimé qu'il y avait $9$ millions de tonnes de déchets plastiques qui était déversé chaque année dans les océans et que cette quantité augmentait de $26\n\%$ par chaque année.
|
||||
|
||||
On modélise l'évolution de la masse de ces déchets plastiques déversée chaque année, si rien n'est fait pour la réduire, par une suite géométrique $\left(u_n\right)$. L'arrondi au centième du terme $u_n$ représente la masse de ces déchets déversée chaque année, exprimée en million de tonnes, pour l'année $(2017 + n)$.
|
||||
|
||||
\medskip
|
||||
|
||||
\begin{enumerate}
|
||||
\item Expliquer pourquoi la suite $u_n$ est géométrique?
|
||||
\item Calculer $u_1$ et $u_2$.
|
||||
\item Exprimer $u_n$ en fonction de $n$.
|
||||
\item Au début de l'année 2017, il y avait $300$ millions de tonnes de déchets plastique. Calculer la quantité totale de déchets plastiques en 2030.
|
||||
\item On souhaite déterminer en quelle année la masse totale de ces déchets plastiques aura pour la première fois augmenté de $50$\,\% par rapport à sa valeur de 2017.
|
||||
\begin{enumerate}
|
||||
\item Recopier et compléter l'algorithme ci-dessous pour que la variable $N$ contienne la réponse au problème posé.
|
||||
|
||||
\begin{center}
|
||||
\begin{tabularx}{0.4\linewidth}{|X|}\hline
|
||||
$N = 2017$\\
|
||||
$U = 9$ \\
|
||||
$S = 300 + U$ \\
|
||||
while $S < 450$: \\
|
||||
\hspace{1cm} $N = \ldots$\\
|
||||
\hspace{1cm} $U = \ldots$\\
|
||||
\hspace{1cm} $S = \ldots$\\
|
||||
\hline
|
||||
\end{tabularx}
|
||||
\end{center}
|
||||
\item Que contiennent les variables $S$, $U$ et $N$ après exécution de cet algorithme ?
|
||||
|
||||
Interpréter les résultats dans le contexte de l'exercice.
|
||||
\end{enumerate}
|
||||
\end{enumerate}
|
||||
\end{exercise}
|
||||
|
||||
\begin{solution}
|
||||
\begin{enumerate}
|
||||
\item Une augmentation de $26\,\%$ revient à multiplier la quantité par $1.26$. La suite est donc bien géométrique. Son premier terme est $u_0 = 9$ et sa raison est $q = 1.26$
|
||||
\item
|
||||
\[
|
||||
u_1 = u_0 * 1.26 = 11.34
|
||||
\]
|
||||
\[
|
||||
u_2 = u_0 * 1.26^2 = 14.2884
|
||||
\]
|
||||
\item
|
||||
\[
|
||||
u_n = u_0 \times q^n = 9 \times 1.26^n
|
||||
\]
|
||||
\item On calcule la quantité totale déversée entre 2017 et 2030.
|
||||
\[
|
||||
\sum_{n = 0}^{13} u_n = u_0 \times \frac{1-q^{13}}{1-q} = 9 \times \frac{1 - 1.26^{13}}{1 - 1.26} = 663.76
|
||||
\]
|
||||
On en déduit la quantité totale de déchets en 2030
|
||||
\[
|
||||
300 + 663.76 = 963.76
|
||||
\]
|
||||
\item
|
||||
\begin{enumerate}
|
||||
\item ~
|
||||
\begin{center}
|
||||
\begin{tabularx}{0.4\linewidth}{|X|}\hline
|
||||
$N \gets 2017$\\
|
||||
$U \gets 9$ \\
|
||||
$S \gets 300 + U$ \\
|
||||
Tant que $S < 450$ \\
|
||||
\hspace{1cm} $N \gets N + 1$\\
|
||||
\hspace{1cm} $U \gets U * 1.26$\\
|
||||
\hspace{1cm} $S \gets S + u$\\
|
||||
Fin Tant que\\\hline
|
||||
\end{tabularx}
|
||||
\end{center}
|
||||
\item \textit{Pas de correction automatisé}
|
||||
\end{enumerate}
|
||||
\end{enumerate}
|
||||
\end{solution}
|
||||
|
||||
\end{document}
|
||||
|
||||
%%% Local Variables:
|
||||
%%% mode: latex
|
||||
%%% TeX-master: "master"
|
||||
%%% End:
|
262
TST/DS/DS_21_04_07/TST1/corr_03_210407_DS8.tex
Normal file
262
TST/DS/DS_21_04_07/TST1/corr_03_210407_DS8.tex
Normal file
@ -0,0 +1,262 @@
|
||||
\documentclass[a4paper,10pt]{article}
|
||||
\usepackage{myXsim}
|
||||
|
||||
% Title Page
|
||||
\title{DS8 \hfill BALLOFFET Kenza}
|
||||
\tribe{TST}
|
||||
\date{\hfillÀ render pour le Mercredi 7 avril}
|
||||
|
||||
\xsimsetup{
|
||||
solution/print = true
|
||||
}
|
||||
|
||||
\begin{document}
|
||||
\maketitle
|
||||
|
||||
\begin{exercise}[subtitle={Automatismes}]
|
||||
\textit{Toutes les questions de cette exercice sont indépendantes et peuvent être répondus séparément}
|
||||
\begin{enumerate}
|
||||
\item De janvier à septembre, une quantité a augmenté de $12\,\%$. Faire un schéma pour représenter la situation puis calculer le taux d'évolution moyen mensuel.
|
||||
\item Une quantité augmente de $12\,\%$ par ans. En 2020, elle est de 126\euro. Quelle était sa valeur en 2019? Faire un schéma pour représenter la situation.
|
||||
\item Déterminer l'équation de la droite \\
|
||||
\begin{tikzpicture}[xscale=0.8, yscale=0.5]
|
||||
\tkzInit[xmin=-5,xmax=5,xstep=1,
|
||||
ymin=-5,ymax=5,ystep=1]
|
||||
\tkzGrid
|
||||
\tkzAxeXY
|
||||
\tkzFct[domain=-5:5,color=red,very thick]%
|
||||
{2.6666666666666665*\x -4};
|
||||
\end{tikzpicture}
|
||||
\item Résoudre l'équation $8 \times 0.54^x = 18$
|
||||
\end{enumerate}
|
||||
\end{exercise}
|
||||
|
||||
\begin{solution}
|
||||
\begin{enumerate}
|
||||
\item On veut partager cette évolution en 8 évolutions.
|
||||
\[
|
||||
\left(1 + \frac{12}{100}\right)^{\frac{1}{8}} = 1.0143
|
||||
\]
|
||||
Donc le taux d'évolution moyen est
|
||||
\[
|
||||
t_m = 1.0143 - 1 = 0.01429999999999998
|
||||
\]
|
||||
\item Coefficient multiplicateur pour revenir en arrière
|
||||
\[
|
||||
CM = (1 + \frac{12}{100})^{-1} = 0.8929
|
||||
\]
|
||||
On en déduit la quantité en 2019
|
||||
\[
|
||||
126 * 0.8929 = 112.50540000000001
|
||||
\]
|
||||
\item L'équation de la droite est
|
||||
\[
|
||||
y = 2.6666666666666665 x -4
|
||||
\]
|
||||
\item Il faut penser à faire la division à par $8$ avant d'utiliser le log car sinon, on ne peut pas utiliser la formule $\log(a^n) = n\times \log(a)$.
|
||||
|
||||
\[x = \frac{\log(2.25)}{\log(0.54)}\]
|
||||
\end{enumerate}
|
||||
\end{solution}
|
||||
|
||||
\begin{exercise}[subtitle={Restaurant}]
|
||||
Un \emph{food truck}, ouvert le midi et le soir, propose deux types de formules :
|
||||
|
||||
\setlength\parindent{10mm}
|
||||
\begin{itemize}
|
||||
\item la formule \emph{Burger} ;
|
||||
\item la formule \emph{Wok}.
|
||||
\end{itemize}
|
||||
\setlength\parindent{0mm}
|
||||
|
||||
\medskip
|
||||
|
||||
Le gérant a remarqué que 94\,\% de ses ventes ont lieu le midi. Le quart des ventes du midi correspondent à la formule \emph{Burger}, alors que 37\,\% des ventes du soir correspondent à la formule \emph{Wok}.
|
||||
|
||||
Le gérant se constitue un fichier en notant, pour chaque vente, la formule choisie et le moment de cette vente (midi ou soir).
|
||||
|
||||
On prélève une fiche de façon équiprobable. On définit les quatre évènements suivants:
|
||||
|
||||
\begin{enumerate}
|
||||
\item $M$ : \og la fiche correspond à une vente du midi\fg{} ;
|
||||
\item $S$ : \og la fiche correspond à une vente du soir\fg {};
|
||||
\item $W$ : \og la fiche correspond à une formule \emph{Wok} \fg{} ;
|
||||
\item $B$ : \og la fiche correspond à une formule \emph{Burger} \fg.
|
||||
\end{enumerate}
|
||||
\setlength\parindent{0mm}
|
||||
|
||||
\medskip
|
||||
|
||||
\begin{enumerate}
|
||||
\item Recopier puis compléter l'arbre pondéré
|
||||
|
||||
\begin{center}
|
||||
\begin{tikzpicture}[sloped]
|
||||
\node {.}
|
||||
child {node {$M$}
|
||||
child {node {$W$}
|
||||
edge from parent
|
||||
node[above] {...}
|
||||
}
|
||||
child {node {$B$}
|
||||
edge from parent
|
||||
node[above] {...}
|
||||
}
|
||||
edge from parent
|
||||
node[above] {...}
|
||||
}
|
||||
child[missing] {}
|
||||
child { node {$S$}
|
||||
child {node {$W$}
|
||||
edge from parent
|
||||
node[above] {...}
|
||||
}
|
||||
child {node {$B$}
|
||||
edge from parent
|
||||
node[above] {...}
|
||||
}
|
||||
edge from parent
|
||||
node[above] {...}
|
||||
} ;
|
||||
\end{tikzpicture}
|
||||
\end{center}
|
||||
|
||||
\item Calculer la probabilité de l'évènement $M \cap W$. Interpréter ce résultat dans le contexte de l'exercice.
|
||||
\item Montrer que la probabilité que la fiche choisie corresponde à une formule \emph{Burger} est égale à $0.2728$.
|
||||
\item On a prélevé une fiche correspondant à la formule \emph{Burger}. Quelle est la probabilité, arrondie au millième, que la vente ait eu lieu le soir?
|
||||
\end{enumerate}
|
||||
\end{exercise}
|
||||
|
||||
\begin{solution}
|
||||
\begin{enumerate}
|
||||
\item
|
||||
\begin{center}
|
||||
\begin{tikzpicture}[sloped]
|
||||
\node {.}
|
||||
child {node {$M$}
|
||||
child {node {$W$}
|
||||
edge from parent
|
||||
node[above] {$0.75$}
|
||||
}
|
||||
child {node {$B$}
|
||||
edge from parent
|
||||
node[above] {$0.25$}
|
||||
}
|
||||
edge from parent
|
||||
node[above] {$0.94$}
|
||||
}
|
||||
child[missing] {}
|
||||
child { node {$S$}
|
||||
child {node {$W$}
|
||||
edge from parent
|
||||
node[above] {$0.37$}
|
||||
}
|
||||
child {node {$B$}
|
||||
edge from parent
|
||||
node[above] {$0.63$}
|
||||
}
|
||||
edge from parent
|
||||
node[above] {$0.06$}
|
||||
} ;
|
||||
\end{tikzpicture}
|
||||
\end{center}
|
||||
\item On calcule la probabilité que la vente soit un wok et ait eu lieu à midi
|
||||
\[ P(M\cap W) = P(M) \times P_M(W) = 0.94 \times 0.75 = 0.705 \]
|
||||
\item Probabilité que la vente soit un burger.
|
||||
\[
|
||||
P(B) = P(M\cap B) + P(S\cap B) = 0.94 \times 0.75 + 0.06 \times 0.37 = 0.2728
|
||||
\]
|
||||
\item On cherche à calculer la quantité $P_B(S)$. Pour cela on utilise la formule de Bayes
|
||||
\[
|
||||
P_B(S) = \frac{P(B\cap S)}{P(B)} = \frac{P_S(B) \times P(S)}{P(B)} = \frac{0.63\times 0.06}{0.2728} = 0.13856304985337245 \approx 0.139
|
||||
\]
|
||||
\end{enumerate}
|
||||
\end{solution}
|
||||
|
||||
\begin{exercise}[subtitle={Continent plastique}]
|
||||
\textit{Les quantités évoqués dans cette exercice sont générés au hasard et sont donc complètement farfelus.}
|
||||
\medskip
|
||||
Le \og continent de plastique\fg{} est la plus grande des plaques de déchets plastiques évoluant sur les océans. Elle occupe actuellement dans l'océan Pacifique une surface dont l'aire est évaluée à plus de $1,6$ million de km$^2$, entre Hawaï et la Californie.
|
||||
|
||||
En 2017, des scientifiques ont estimé qu'il y avait $15$ millions de tonnes de déchets plastiques qui était déversé chaque année dans les océans et que cette quantité augmentait de $19\n\%$ par chaque année.
|
||||
|
||||
On modélise l'évolution de la masse de ces déchets plastiques déversée chaque année, si rien n'est fait pour la réduire, par une suite géométrique $\left(u_n\right)$. L'arrondi au centième du terme $u_n$ représente la masse de ces déchets déversée chaque année, exprimée en million de tonnes, pour l'année $(2017 + n)$.
|
||||
|
||||
\medskip
|
||||
|
||||
\begin{enumerate}
|
||||
\item Expliquer pourquoi la suite $u_n$ est géométrique?
|
||||
\item Calculer $u_1$ et $u_2$.
|
||||
\item Exprimer $u_n$ en fonction de $n$.
|
||||
\item Au début de l'année 2017, il y avait $300$ millions de tonnes de déchets plastique. Calculer la quantité totale de déchets plastiques en 2030.
|
||||
\item On souhaite déterminer en quelle année la masse totale de ces déchets plastiques aura pour la première fois augmenté de $50$\,\% par rapport à sa valeur de 2017.
|
||||
\begin{enumerate}
|
||||
\item Recopier et compléter l'algorithme ci-dessous pour que la variable $N$ contienne la réponse au problème posé.
|
||||
|
||||
\begin{center}
|
||||
\begin{tabularx}{0.4\linewidth}{|X|}\hline
|
||||
$N = 2017$\\
|
||||
$U = 15$ \\
|
||||
$S = 300 + U$ \\
|
||||
while $S < 450$: \\
|
||||
\hspace{1cm} $N = \ldots$\\
|
||||
\hspace{1cm} $U = \ldots$\\
|
||||
\hspace{1cm} $S = \ldots$\\
|
||||
\hline
|
||||
\end{tabularx}
|
||||
\end{center}
|
||||
\item Que contiennent les variables $S$, $U$ et $N$ après exécution de cet algorithme ?
|
||||
|
||||
Interpréter les résultats dans le contexte de l'exercice.
|
||||
\end{enumerate}
|
||||
\end{enumerate}
|
||||
\end{exercise}
|
||||
|
||||
\begin{solution}
|
||||
\begin{enumerate}
|
||||
\item Une augmentation de $19\,\%$ revient à multiplier la quantité par $1.19$. La suite est donc bien géométrique. Son premier terme est $u_0 = 15$ et sa raison est $q = 1.19$
|
||||
\item
|
||||
\[
|
||||
u_1 = u_0 * 1.19 = 17.849999999999998
|
||||
\]
|
||||
\[
|
||||
u_2 = u_0 * 1.19^2 = 21.2415
|
||||
\]
|
||||
\item
|
||||
\[
|
||||
u_n = u_0 \times q^n = 15 \times 1.19^n
|
||||
\]
|
||||
\item On calcule la quantité totale déversée entre 2017 et 2030.
|
||||
\[
|
||||
\sum_{n = 0}^{13} u_n = u_0 \times \frac{1-q^{13}}{1-q} = 15 \times \frac{1 - 1.19^{13}}{1 - 1.19} = 678.67
|
||||
\]
|
||||
On en déduit la quantité totale de déchets en 2030
|
||||
\[
|
||||
300 + 678.67 = 978.67
|
||||
\]
|
||||
\item
|
||||
\begin{enumerate}
|
||||
\item ~
|
||||
\begin{center}
|
||||
\begin{tabularx}{0.4\linewidth}{|X|}\hline
|
||||
$N \gets 2017$\\
|
||||
$U \gets 15$ \\
|
||||
$S \gets 300 + U$ \\
|
||||
Tant que $S < 450$ \\
|
||||
\hspace{1cm} $N \gets N + 1$\\
|
||||
\hspace{1cm} $U \gets U * 1.19$\\
|
||||
\hspace{1cm} $S \gets S + u$\\
|
||||
Fin Tant que\\\hline
|
||||
\end{tabularx}
|
||||
\end{center}
|
||||
\item \textit{Pas de correction automatisé}
|
||||
\end{enumerate}
|
||||
\end{enumerate}
|
||||
\end{solution}
|
||||
|
||||
\end{document}
|
||||
|
||||
%%% Local Variables:
|
||||
%%% mode: latex
|
||||
%%% TeX-master: "master"
|
||||
%%% End:
|
262
TST/DS/DS_21_04_07/TST1/corr_04_210407_DS8.tex
Normal file
262
TST/DS/DS_21_04_07/TST1/corr_04_210407_DS8.tex
Normal file
@ -0,0 +1,262 @@
|
||||
\documentclass[a4paper,10pt]{article}
|
||||
\usepackage{myXsim}
|
||||
|
||||
% Title Page
|
||||
\title{DS8 \hfill BENHATTAL Chakir}
|
||||
\tribe{TST}
|
||||
\date{\hfillÀ render pour le Mercredi 7 avril}
|
||||
|
||||
\xsimsetup{
|
||||
solution/print = true
|
||||
}
|
||||
|
||||
\begin{document}
|
||||
\maketitle
|
||||
|
||||
\begin{exercise}[subtitle={Automatismes}]
|
||||
\textit{Toutes les questions de cette exercice sont indépendantes et peuvent être répondus séparément}
|
||||
\begin{enumerate}
|
||||
\item De janvier à septembre, une quantité a augmenté de $28\,\%$. Faire un schéma pour représenter la situation puis calculer le taux d'évolution moyen mensuel.
|
||||
\item Une quantité augmente de $28\,\%$ par ans. En 2020, elle est de 111\euro. Quelle était sa valeur en 2019? Faire un schéma pour représenter la situation.
|
||||
\item Déterminer l'équation de la droite \\
|
||||
\begin{tikzpicture}[xscale=0.8, yscale=0.5]
|
||||
\tkzInit[xmin=-5,xmax=5,xstep=1,
|
||||
ymin=-5,ymax=5,ystep=1]
|
||||
\tkzGrid
|
||||
\tkzAxeXY
|
||||
\tkzFct[domain=-5:5,color=red,very thick]%
|
||||
{1.0*\x -1};
|
||||
\end{tikzpicture}
|
||||
\item Résoudre l'équation $4 \times 0.77^x = 25$
|
||||
\end{enumerate}
|
||||
\end{exercise}
|
||||
|
||||
\begin{solution}
|
||||
\begin{enumerate}
|
||||
\item On veut partager cette évolution en 8 évolutions.
|
||||
\[
|
||||
\left(1 + \frac{28}{100}\right)^{\frac{1}{8}} = 1.0313
|
||||
\]
|
||||
Donc le taux d'évolution moyen est
|
||||
\[
|
||||
t_m = 1.0313 - 1 = 0.031300000000000106
|
||||
\]
|
||||
\item Coefficient multiplicateur pour revenir en arrière
|
||||
\[
|
||||
CM = (1 + \frac{28}{100})^{-1} = 0.7812
|
||||
\]
|
||||
On en déduit la quantité en 2019
|
||||
\[
|
||||
111 * 0.7812 = 86.7132
|
||||
\]
|
||||
\item L'équation de la droite est
|
||||
\[
|
||||
y = 1.0 x -1
|
||||
\]
|
||||
\item Il faut penser à faire la division à par $4$ avant d'utiliser le log car sinon, on ne peut pas utiliser la formule $\log(a^n) = n\times \log(a)$.
|
||||
|
||||
\[x = \frac{\log(6.25)}{\log(0.77)}\]
|
||||
\end{enumerate}
|
||||
\end{solution}
|
||||
|
||||
\begin{exercise}[subtitle={Restaurant}]
|
||||
Un \emph{food truck}, ouvert le midi et le soir, propose deux types de formules :
|
||||
|
||||
\setlength\parindent{10mm}
|
||||
\begin{itemize}
|
||||
\item la formule \emph{Burger} ;
|
||||
\item la formule \emph{Wok}.
|
||||
\end{itemize}
|
||||
\setlength\parindent{0mm}
|
||||
|
||||
\medskip
|
||||
|
||||
Le gérant a remarqué que 89\,\% de ses ventes ont lieu le midi. Le quart des ventes du midi correspondent à la formule \emph{Burger}, alors que 43\,\% des ventes du soir correspondent à la formule \emph{Wok}.
|
||||
|
||||
Le gérant se constitue un fichier en notant, pour chaque vente, la formule choisie et le moment de cette vente (midi ou soir).
|
||||
|
||||
On prélève une fiche de façon équiprobable. On définit les quatre évènements suivants:
|
||||
|
||||
\begin{enumerate}
|
||||
\item $M$ : \og la fiche correspond à une vente du midi\fg{} ;
|
||||
\item $S$ : \og la fiche correspond à une vente du soir\fg {};
|
||||
\item $W$ : \og la fiche correspond à une formule \emph{Wok} \fg{} ;
|
||||
\item $B$ : \og la fiche correspond à une formule \emph{Burger} \fg.
|
||||
\end{enumerate}
|
||||
\setlength\parindent{0mm}
|
||||
|
||||
\medskip
|
||||
|
||||
\begin{enumerate}
|
||||
\item Recopier puis compléter l'arbre pondéré
|
||||
|
||||
\begin{center}
|
||||
\begin{tikzpicture}[sloped]
|
||||
\node {.}
|
||||
child {node {$M$}
|
||||
child {node {$W$}
|
||||
edge from parent
|
||||
node[above] {...}
|
||||
}
|
||||
child {node {$B$}
|
||||
edge from parent
|
||||
node[above] {...}
|
||||
}
|
||||
edge from parent
|
||||
node[above] {...}
|
||||
}
|
||||
child[missing] {}
|
||||
child { node {$S$}
|
||||
child {node {$W$}
|
||||
edge from parent
|
||||
node[above] {...}
|
||||
}
|
||||
child {node {$B$}
|
||||
edge from parent
|
||||
node[above] {...}
|
||||
}
|
||||
edge from parent
|
||||
node[above] {...}
|
||||
} ;
|
||||
\end{tikzpicture}
|
||||
\end{center}
|
||||
|
||||
\item Calculer la probabilité de l'évènement $M \cap W$. Interpréter ce résultat dans le contexte de l'exercice.
|
||||
\item Montrer que la probabilité que la fiche choisie corresponde à une formule \emph{Burger} est égale à $0.2852$.
|
||||
\item On a prélevé une fiche correspondant à la formule \emph{Burger}. Quelle est la probabilité, arrondie au millième, que la vente ait eu lieu le soir?
|
||||
\end{enumerate}
|
||||
\end{exercise}
|
||||
|
||||
\begin{solution}
|
||||
\begin{enumerate}
|
||||
\item
|
||||
\begin{center}
|
||||
\begin{tikzpicture}[sloped]
|
||||
\node {.}
|
||||
child {node {$M$}
|
||||
child {node {$W$}
|
||||
edge from parent
|
||||
node[above] {$0.75$}
|
||||
}
|
||||
child {node {$B$}
|
||||
edge from parent
|
||||
node[above] {$0.25$}
|
||||
}
|
||||
edge from parent
|
||||
node[above] {$0.89$}
|
||||
}
|
||||
child[missing] {}
|
||||
child { node {$S$}
|
||||
child {node {$W$}
|
||||
edge from parent
|
||||
node[above] {$0.43$}
|
||||
}
|
||||
child {node {$B$}
|
||||
edge from parent
|
||||
node[above] {$0.57$}
|
||||
}
|
||||
edge from parent
|
||||
node[above] {$0.11$}
|
||||
} ;
|
||||
\end{tikzpicture}
|
||||
\end{center}
|
||||
\item On calcule la probabilité que la vente soit un wok et ait eu lieu à midi
|
||||
\[ P(M\cap W) = P(M) \times P_M(W) = 0.89 \times 0.75 = 0.6675 \]
|
||||
\item Probabilité que la vente soit un burger.
|
||||
\[
|
||||
P(B) = P(M\cap B) + P(S\cap B) = 0.89 \times 0.75 + 0.11 \times 0.43 = 0.2852
|
||||
\]
|
||||
\item On cherche à calculer la quantité $P_B(S)$. Pour cela on utilise la formule de Bayes
|
||||
\[
|
||||
P_B(S) = \frac{P(B\cap S)}{P(B)} = \frac{P_S(B) \times P(S)}{P(B)} = \frac{0.57\times 0.11}{0.2852} = 0.2198457223001402 \approx 0.22
|
||||
\]
|
||||
\end{enumerate}
|
||||
\end{solution}
|
||||
|
||||
\begin{exercise}[subtitle={Continent plastique}]
|
||||
\textit{Les quantités évoqués dans cette exercice sont générés au hasard et sont donc complètement farfelus.}
|
||||
\medskip
|
||||
Le \og continent de plastique\fg{} est la plus grande des plaques de déchets plastiques évoluant sur les océans. Elle occupe actuellement dans l'océan Pacifique une surface dont l'aire est évaluée à plus de $1,6$ million de km$^2$, entre Hawaï et la Californie.
|
||||
|
||||
En 2017, des scientifiques ont estimé qu'il y avait $6$ millions de tonnes de déchets plastiques qui était déversé chaque année dans les océans et que cette quantité augmentait de $13\n\%$ par chaque année.
|
||||
|
||||
On modélise l'évolution de la masse de ces déchets plastiques déversée chaque année, si rien n'est fait pour la réduire, par une suite géométrique $\left(u_n\right)$. L'arrondi au centième du terme $u_n$ représente la masse de ces déchets déversée chaque année, exprimée en million de tonnes, pour l'année $(2017 + n)$.
|
||||
|
||||
\medskip
|
||||
|
||||
\begin{enumerate}
|
||||
\item Expliquer pourquoi la suite $u_n$ est géométrique?
|
||||
\item Calculer $u_1$ et $u_2$.
|
||||
\item Exprimer $u_n$ en fonction de $n$.
|
||||
\item Au début de l'année 2017, il y avait $300$ millions de tonnes de déchets plastique. Calculer la quantité totale de déchets plastiques en 2030.
|
||||
\item On souhaite déterminer en quelle année la masse totale de ces déchets plastiques aura pour la première fois augmenté de $50$\,\% par rapport à sa valeur de 2017.
|
||||
\begin{enumerate}
|
||||
\item Recopier et compléter l'algorithme ci-dessous pour que la variable $N$ contienne la réponse au problème posé.
|
||||
|
||||
\begin{center}
|
||||
\begin{tabularx}{0.4\linewidth}{|X|}\hline
|
||||
$N = 2017$\\
|
||||
$U = 6$ \\
|
||||
$S = 300 + U$ \\
|
||||
while $S < 450$: \\
|
||||
\hspace{1cm} $N = \ldots$\\
|
||||
\hspace{1cm} $U = \ldots$\\
|
||||
\hspace{1cm} $S = \ldots$\\
|
||||
\hline
|
||||
\end{tabularx}
|
||||
\end{center}
|
||||
\item Que contiennent les variables $S$, $U$ et $N$ après exécution de cet algorithme ?
|
||||
|
||||
Interpréter les résultats dans le contexte de l'exercice.
|
||||
\end{enumerate}
|
||||
\end{enumerate}
|
||||
\end{exercise}
|
||||
|
||||
\begin{solution}
|
||||
\begin{enumerate}
|
||||
\item Une augmentation de $13\,\%$ revient à multiplier la quantité par $1.13$. La suite est donc bien géométrique. Son premier terme est $u_0 = 6$ et sa raison est $q = 1.13$
|
||||
\item
|
||||
\[
|
||||
u_1 = u_0 * 1.13 = 6.779999999999999
|
||||
\]
|
||||
\[
|
||||
u_2 = u_0 * 1.13^2 = 7.6614
|
||||
\]
|
||||
\item
|
||||
\[
|
||||
u_n = u_0 \times q^n = 6 \times 1.13^n
|
||||
\]
|
||||
\item On calcule la quantité totale déversée entre 2017 et 2030.
|
||||
\[
|
||||
\sum_{n = 0}^{13} u_n = u_0 \times \frac{1-q^{13}}{1-q} = 6 \times \frac{1 - 1.13^{13}}{1 - 1.13} = 179.91
|
||||
\]
|
||||
On en déduit la quantité totale de déchets en 2030
|
||||
\[
|
||||
300 + 179.91 = 479.90999999999997
|
||||
\]
|
||||
\item
|
||||
\begin{enumerate}
|
||||
\item ~
|
||||
\begin{center}
|
||||
\begin{tabularx}{0.4\linewidth}{|X|}\hline
|
||||
$N \gets 2017$\\
|
||||
$U \gets 6$ \\
|
||||
$S \gets 300 + U$ \\
|
||||
Tant que $S < 450$ \\
|
||||
\hspace{1cm} $N \gets N + 1$\\
|
||||
\hspace{1cm} $U \gets U * 1.13$\\
|
||||
\hspace{1cm} $S \gets S + u$\\
|
||||
Fin Tant que\\\hline
|
||||
\end{tabularx}
|
||||
\end{center}
|
||||
\item \textit{Pas de correction automatisé}
|
||||
\end{enumerate}
|
||||
\end{enumerate}
|
||||
\end{solution}
|
||||
|
||||
\end{document}
|
||||
|
||||
%%% Local Variables:
|
||||
%%% mode: latex
|
||||
%%% TeX-master: "master"
|
||||
%%% End:
|
262
TST/DS/DS_21_04_07/TST1/corr_05_210407_DS8.tex
Normal file
262
TST/DS/DS_21_04_07/TST1/corr_05_210407_DS8.tex
Normal file
@ -0,0 +1,262 @@
|
||||
\documentclass[a4paper,10pt]{article}
|
||||
\usepackage{myXsim}
|
||||
|
||||
% Title Page
|
||||
\title{DS8 \hfill CLAIN Avinash}
|
||||
\tribe{TST}
|
||||
\date{\hfillÀ render pour le Mercredi 7 avril}
|
||||
|
||||
\xsimsetup{
|
||||
solution/print = true
|
||||
}
|
||||
|
||||
\begin{document}
|
||||
\maketitle
|
||||
|
||||
\begin{exercise}[subtitle={Automatismes}]
|
||||
\textit{Toutes les questions de cette exercice sont indépendantes et peuvent être répondus séparément}
|
||||
\begin{enumerate}
|
||||
\item De janvier à septembre, une quantité a augmenté de $15\,\%$. Faire un schéma pour représenter la situation puis calculer le taux d'évolution moyen mensuel.
|
||||
\item Une quantité augmente de $15\,\%$ par ans. En 2020, elle est de 112\euro. Quelle était sa valeur en 2019? Faire un schéma pour représenter la situation.
|
||||
\item Déterminer l'équation de la droite \\
|
||||
\begin{tikzpicture}[xscale=0.8, yscale=0.5]
|
||||
\tkzInit[xmin=-5,xmax=5,xstep=1,
|
||||
ymin=-5,ymax=5,ystep=1]
|
||||
\tkzGrid
|
||||
\tkzAxeXY
|
||||
\tkzFct[domain=-5:5,color=red,very thick]%
|
||||
{4.0*\x -4};
|
||||
\end{tikzpicture}
|
||||
\item Résoudre l'équation $6 \times 0.77^x = 23$
|
||||
\end{enumerate}
|
||||
\end{exercise}
|
||||
|
||||
\begin{solution}
|
||||
\begin{enumerate}
|
||||
\item On veut partager cette évolution en 8 évolutions.
|
||||
\[
|
||||
\left(1 + \frac{15}{100}\right)^{\frac{1}{8}} = 1.0176
|
||||
\]
|
||||
Donc le taux d'évolution moyen est
|
||||
\[
|
||||
t_m = 1.0176 - 1 = 0.01760000000000006
|
||||
\]
|
||||
\item Coefficient multiplicateur pour revenir en arrière
|
||||
\[
|
||||
CM = (1 + \frac{15}{100})^{-1} = 0.8696
|
||||
\]
|
||||
On en déduit la quantité en 2019
|
||||
\[
|
||||
112 * 0.8696 = 97.3952
|
||||
\]
|
||||
\item L'équation de la droite est
|
||||
\[
|
||||
y = 4.0 x -4
|
||||
\]
|
||||
\item Il faut penser à faire la division à par $6$ avant d'utiliser le log car sinon, on ne peut pas utiliser la formule $\log(a^n) = n\times \log(a)$.
|
||||
|
||||
\[x = \frac{\log(3.83)}{\log(0.77)}\]
|
||||
\end{enumerate}
|
||||
\end{solution}
|
||||
|
||||
\begin{exercise}[subtitle={Restaurant}]
|
||||
Un \emph{food truck}, ouvert le midi et le soir, propose deux types de formules :
|
||||
|
||||
\setlength\parindent{10mm}
|
||||
\begin{itemize}
|
||||
\item la formule \emph{Burger} ;
|
||||
\item la formule \emph{Wok}.
|
||||
\end{itemize}
|
||||
\setlength\parindent{0mm}
|
||||
|
||||
\medskip
|
||||
|
||||
Le gérant a remarqué que 92\,\% de ses ventes ont lieu le midi. Le quart des ventes du midi correspondent à la formule \emph{Burger}, alors que 54\,\% des ventes du soir correspondent à la formule \emph{Wok}.
|
||||
|
||||
Le gérant se constitue un fichier en notant, pour chaque vente, la formule choisie et le moment de cette vente (midi ou soir).
|
||||
|
||||
On prélève une fiche de façon équiprobable. On définit les quatre évènements suivants:
|
||||
|
||||
\begin{enumerate}
|
||||
\item $M$ : \og la fiche correspond à une vente du midi\fg{} ;
|
||||
\item $S$ : \og la fiche correspond à une vente du soir\fg {};
|
||||
\item $W$ : \og la fiche correspond à une formule \emph{Wok} \fg{} ;
|
||||
\item $B$ : \og la fiche correspond à une formule \emph{Burger} \fg.
|
||||
\end{enumerate}
|
||||
\setlength\parindent{0mm}
|
||||
|
||||
\medskip
|
||||
|
||||
\begin{enumerate}
|
||||
\item Recopier puis compléter l'arbre pondéré
|
||||
|
||||
\begin{center}
|
||||
\begin{tikzpicture}[sloped]
|
||||
\node {.}
|
||||
child {node {$M$}
|
||||
child {node {$W$}
|
||||
edge from parent
|
||||
node[above] {...}
|
||||
}
|
||||
child {node {$B$}
|
||||
edge from parent
|
||||
node[above] {...}
|
||||
}
|
||||
edge from parent
|
||||
node[above] {...}
|
||||
}
|
||||
child[missing] {}
|
||||
child { node {$S$}
|
||||
child {node {$W$}
|
||||
edge from parent
|
||||
node[above] {...}
|
||||
}
|
||||
child {node {$B$}
|
||||
edge from parent
|
||||
node[above] {...}
|
||||
}
|
||||
edge from parent
|
||||
node[above] {...}
|
||||
} ;
|
||||
\end{tikzpicture}
|
||||
\end{center}
|
||||
|
||||
\item Calculer la probabilité de l'évènement $M \cap W$. Interpréter ce résultat dans le contexte de l'exercice.
|
||||
\item Montrer que la probabilité que la fiche choisie corresponde à une formule \emph{Burger} est égale à $0.2668$.
|
||||
\item On a prélevé une fiche correspondant à la formule \emph{Burger}. Quelle est la probabilité, arrondie au millième, que la vente ait eu lieu le soir?
|
||||
\end{enumerate}
|
||||
\end{exercise}
|
||||
|
||||
\begin{solution}
|
||||
\begin{enumerate}
|
||||
\item
|
||||
\begin{center}
|
||||
\begin{tikzpicture}[sloped]
|
||||
\node {.}
|
||||
child {node {$M$}
|
||||
child {node {$W$}
|
||||
edge from parent
|
||||
node[above] {$0.75$}
|
||||
}
|
||||
child {node {$B$}
|
||||
edge from parent
|
||||
node[above] {$0.25$}
|
||||
}
|
||||
edge from parent
|
||||
node[above] {$0.92$}
|
||||
}
|
||||
child[missing] {}
|
||||
child { node {$S$}
|
||||
child {node {$W$}
|
||||
edge from parent
|
||||
node[above] {$0.54$}
|
||||
}
|
||||
child {node {$B$}
|
||||
edge from parent
|
||||
node[above] {$0.46$}
|
||||
}
|
||||
edge from parent
|
||||
node[above] {$0.08$}
|
||||
} ;
|
||||
\end{tikzpicture}
|
||||
\end{center}
|
||||
\item On calcule la probabilité que la vente soit un wok et ait eu lieu à midi
|
||||
\[ P(M\cap W) = P(M) \times P_M(W) = 0.92 \times 0.75 = 0.69 \]
|
||||
\item Probabilité que la vente soit un burger.
|
||||
\[
|
||||
P(B) = P(M\cap B) + P(S\cap B) = 0.92 \times 0.75 + 0.08 \times 0.54 = 0.2668
|
||||
\]
|
||||
\item On cherche à calculer la quantité $P_B(S)$. Pour cela on utilise la formule de Bayes
|
||||
\[
|
||||
P_B(S) = \frac{P(B\cap S)}{P(B)} = \frac{P_S(B) \times P(S)}{P(B)} = \frac{0.46\times 0.08}{0.2668} = 0.13793103448275862 \approx 0.138
|
||||
\]
|
||||
\end{enumerate}
|
||||
\end{solution}
|
||||
|
||||
\begin{exercise}[subtitle={Continent plastique}]
|
||||
\textit{Les quantités évoqués dans cette exercice sont générés au hasard et sont donc complètement farfelus.}
|
||||
\medskip
|
||||
Le \og continent de plastique\fg{} est la plus grande des plaques de déchets plastiques évoluant sur les océans. Elle occupe actuellement dans l'océan Pacifique une surface dont l'aire est évaluée à plus de $1,6$ million de km$^2$, entre Hawaï et la Californie.
|
||||
|
||||
En 2017, des scientifiques ont estimé qu'il y avait $18$ millions de tonnes de déchets plastiques qui était déversé chaque année dans les océans et que cette quantité augmentait de $21\n\%$ par chaque année.
|
||||
|
||||
On modélise l'évolution de la masse de ces déchets plastiques déversée chaque année, si rien n'est fait pour la réduire, par une suite géométrique $\left(u_n\right)$. L'arrondi au centième du terme $u_n$ représente la masse de ces déchets déversée chaque année, exprimée en million de tonnes, pour l'année $(2017 + n)$.
|
||||
|
||||
\medskip
|
||||
|
||||
\begin{enumerate}
|
||||
\item Expliquer pourquoi la suite $u_n$ est géométrique?
|
||||
\item Calculer $u_1$ et $u_2$.
|
||||
\item Exprimer $u_n$ en fonction de $n$.
|
||||
\item Au début de l'année 2017, il y avait $300$ millions de tonnes de déchets plastique. Calculer la quantité totale de déchets plastiques en 2030.
|
||||
\item On souhaite déterminer en quelle année la masse totale de ces déchets plastiques aura pour la première fois augmenté de $50$\,\% par rapport à sa valeur de 2017.
|
||||
\begin{enumerate}
|
||||
\item Recopier et compléter l'algorithme ci-dessous pour que la variable $N$ contienne la réponse au problème posé.
|
||||
|
||||
\begin{center}
|
||||
\begin{tabularx}{0.4\linewidth}{|X|}\hline
|
||||
$N = 2017$\\
|
||||
$U = 18$ \\
|
||||
$S = 300 + U$ \\
|
||||
while $S < 450$: \\
|
||||
\hspace{1cm} $N = \ldots$\\
|
||||
\hspace{1cm} $U = \ldots$\\
|
||||
\hspace{1cm} $S = \ldots$\\
|
||||
\hline
|
||||
\end{tabularx}
|
||||
\end{center}
|
||||
\item Que contiennent les variables $S$, $U$ et $N$ après exécution de cet algorithme ?
|
||||
|
||||
Interpréter les résultats dans le contexte de l'exercice.
|
||||
\end{enumerate}
|
||||
\end{enumerate}
|
||||
\end{exercise}
|
||||
|
||||
\begin{solution}
|
||||
\begin{enumerate}
|
||||
\item Une augmentation de $21\,\%$ revient à multiplier la quantité par $1.21$. La suite est donc bien géométrique. Son premier terme est $u_0 = 18$ et sa raison est $q = 1.21$
|
||||
\item
|
||||
\[
|
||||
u_1 = u_0 * 1.21 = 21.78
|
||||
\]
|
||||
\[
|
||||
u_2 = u_0 * 1.21^2 = 26.3538
|
||||
\]
|
||||
\item
|
||||
\[
|
||||
u_n = u_0 \times q^n = 18 \times 1.21^n
|
||||
\]
|
||||
\item On calcule la quantité totale déversée entre 2017 et 2030.
|
||||
\[
|
||||
\sum_{n = 0}^{13} u_n = u_0 \times \frac{1-q^{13}}{1-q} = 18 \times \frac{1 - 1.21^{13}}{1 - 1.21} = 935.84
|
||||
\]
|
||||
On en déduit la quantité totale de déchets en 2030
|
||||
\[
|
||||
300 + 935.84 = 1235.8400000000001
|
||||
\]
|
||||
\item
|
||||
\begin{enumerate}
|
||||
\item ~
|
||||
\begin{center}
|
||||
\begin{tabularx}{0.4\linewidth}{|X|}\hline
|
||||
$N \gets 2017$\\
|
||||
$U \gets 18$ \\
|
||||
$S \gets 300 + U$ \\
|
||||
Tant que $S < 450$ \\
|
||||
\hspace{1cm} $N \gets N + 1$\\
|
||||
\hspace{1cm} $U \gets U * 1.21$\\
|
||||
\hspace{1cm} $S \gets S + u$\\
|
||||
Fin Tant que\\\hline
|
||||
\end{tabularx}
|
||||
\end{center}
|
||||
\item \textit{Pas de correction automatisé}
|
||||
\end{enumerate}
|
||||
\end{enumerate}
|
||||
\end{solution}
|
||||
|
||||
\end{document}
|
||||
|
||||
%%% Local Variables:
|
||||
%%% mode: latex
|
||||
%%% TeX-master: "master"
|
||||
%%% End:
|
262
TST/DS/DS_21_04_07/TST1/corr_06_210407_DS8.tex
Normal file
262
TST/DS/DS_21_04_07/TST1/corr_06_210407_DS8.tex
Normal file
@ -0,0 +1,262 @@
|
||||
\documentclass[a4paper,10pt]{article}
|
||||
\usepackage{myXsim}
|
||||
|
||||
% Title Page
|
||||
\title{DS8 \hfill COLASSI Alexis}
|
||||
\tribe{TST}
|
||||
\date{\hfillÀ render pour le Mercredi 7 avril}
|
||||
|
||||
\xsimsetup{
|
||||
solution/print = true
|
||||
}
|
||||
|
||||
\begin{document}
|
||||
\maketitle
|
||||
|
||||
\begin{exercise}[subtitle={Automatismes}]
|
||||
\textit{Toutes les questions de cette exercice sont indépendantes et peuvent être répondus séparément}
|
||||
\begin{enumerate}
|
||||
\item De janvier à septembre, une quantité a augmenté de $27\,\%$. Faire un schéma pour représenter la situation puis calculer le taux d'évolution moyen mensuel.
|
||||
\item Une quantité augmente de $27\,\%$ par ans. En 2020, elle est de 116\euro. Quelle était sa valeur en 2019? Faire un schéma pour représenter la situation.
|
||||
\item Déterminer l'équation de la droite \\
|
||||
\begin{tikzpicture}[xscale=0.8, yscale=0.5]
|
||||
\tkzInit[xmin=-5,xmax=5,xstep=1,
|
||||
ymin=-5,ymax=5,ystep=1]
|
||||
\tkzGrid
|
||||
\tkzAxeXY
|
||||
\tkzFct[domain=-5:5,color=red,very thick]%
|
||||
{2.0*\x -2};
|
||||
\end{tikzpicture}
|
||||
\item Résoudre l'équation $7 \times 0.23^x = 16$
|
||||
\end{enumerate}
|
||||
\end{exercise}
|
||||
|
||||
\begin{solution}
|
||||
\begin{enumerate}
|
||||
\item On veut partager cette évolution en 8 évolutions.
|
||||
\[
|
||||
\left(1 + \frac{27}{100}\right)^{\frac{1}{8}} = 1.0303
|
||||
\]
|
||||
Donc le taux d'évolution moyen est
|
||||
\[
|
||||
t_m = 1.0303 - 1 = 0.030299999999999994
|
||||
\]
|
||||
\item Coefficient multiplicateur pour revenir en arrière
|
||||
\[
|
||||
CM = (1 + \frac{27}{100})^{-1} = 0.7874
|
||||
\]
|
||||
On en déduit la quantité en 2019
|
||||
\[
|
||||
116 * 0.7874 = 91.3384
|
||||
\]
|
||||
\item L'équation de la droite est
|
||||
\[
|
||||
y = 2.0 x -2
|
||||
\]
|
||||
\item Il faut penser à faire la division à par $7$ avant d'utiliser le log car sinon, on ne peut pas utiliser la formule $\log(a^n) = n\times \log(a)$.
|
||||
|
||||
\[x = \frac{\log(2.29)}{\log(0.23)}\]
|
||||
\end{enumerate}
|
||||
\end{solution}
|
||||
|
||||
\begin{exercise}[subtitle={Restaurant}]
|
||||
Un \emph{food truck}, ouvert le midi et le soir, propose deux types de formules :
|
||||
|
||||
\setlength\parindent{10mm}
|
||||
\begin{itemize}
|
||||
\item la formule \emph{Burger} ;
|
||||
\item la formule \emph{Wok}.
|
||||
\end{itemize}
|
||||
\setlength\parindent{0mm}
|
||||
|
||||
\medskip
|
||||
|
||||
Le gérant a remarqué que 53\,\% de ses ventes ont lieu le midi. Le quart des ventes du midi correspondent à la formule \emph{Burger}, alors que 26\,\% des ventes du soir correspondent à la formule \emph{Wok}.
|
||||
|
||||
Le gérant se constitue un fichier en notant, pour chaque vente, la formule choisie et le moment de cette vente (midi ou soir).
|
||||
|
||||
On prélève une fiche de façon équiprobable. On définit les quatre évènements suivants:
|
||||
|
||||
\begin{enumerate}
|
||||
\item $M$ : \og la fiche correspond à une vente du midi\fg{} ;
|
||||
\item $S$ : \og la fiche correspond à une vente du soir\fg {};
|
||||
\item $W$ : \og la fiche correspond à une formule \emph{Wok} \fg{} ;
|
||||
\item $B$ : \og la fiche correspond à une formule \emph{Burger} \fg.
|
||||
\end{enumerate}
|
||||
\setlength\parindent{0mm}
|
||||
|
||||
\medskip
|
||||
|
||||
\begin{enumerate}
|
||||
\item Recopier puis compléter l'arbre pondéré
|
||||
|
||||
\begin{center}
|
||||
\begin{tikzpicture}[sloped]
|
||||
\node {.}
|
||||
child {node {$M$}
|
||||
child {node {$W$}
|
||||
edge from parent
|
||||
node[above] {...}
|
||||
}
|
||||
child {node {$B$}
|
||||
edge from parent
|
||||
node[above] {...}
|
||||
}
|
||||
edge from parent
|
||||
node[above] {...}
|
||||
}
|
||||
child[missing] {}
|
||||
child { node {$S$}
|
||||
child {node {$W$}
|
||||
edge from parent
|
||||
node[above] {...}
|
||||
}
|
||||
child {node {$B$}
|
||||
edge from parent
|
||||
node[above] {...}
|
||||
}
|
||||
edge from parent
|
||||
node[above] {...}
|
||||
} ;
|
||||
\end{tikzpicture}
|
||||
\end{center}
|
||||
|
||||
\item Calculer la probabilité de l'évènement $M \cap W$. Interpréter ce résultat dans le contexte de l'exercice.
|
||||
\item Montrer que la probabilité que la fiche choisie corresponde à une formule \emph{Burger} est égale à $0.4803$.
|
||||
\item On a prélevé une fiche correspondant à la formule \emph{Burger}. Quelle est la probabilité, arrondie au millième, que la vente ait eu lieu le soir?
|
||||
\end{enumerate}
|
||||
\end{exercise}
|
||||
|
||||
\begin{solution}
|
||||
\begin{enumerate}
|
||||
\item
|
||||
\begin{center}
|
||||
\begin{tikzpicture}[sloped]
|
||||
\node {.}
|
||||
child {node {$M$}
|
||||
child {node {$W$}
|
||||
edge from parent
|
||||
node[above] {$0.75$}
|
||||
}
|
||||
child {node {$B$}
|
||||
edge from parent
|
||||
node[above] {$0.25$}
|
||||
}
|
||||
edge from parent
|
||||
node[above] {$0.53$}
|
||||
}
|
||||
child[missing] {}
|
||||
child { node {$S$}
|
||||
child {node {$W$}
|
||||
edge from parent
|
||||
node[above] {$0.26$}
|
||||
}
|
||||
child {node {$B$}
|
||||
edge from parent
|
||||
node[above] {$0.74$}
|
||||
}
|
||||
edge from parent
|
||||
node[above] {$0.47$}
|
||||
} ;
|
||||
\end{tikzpicture}
|
||||
\end{center}
|
||||
\item On calcule la probabilité que la vente soit un wok et ait eu lieu à midi
|
||||
\[ P(M\cap W) = P(M) \times P_M(W) = 0.53 \times 0.75 = 0.3975 \]
|
||||
\item Probabilité que la vente soit un burger.
|
||||
\[
|
||||
P(B) = P(M\cap B) + P(S\cap B) = 0.53 \times 0.75 + 0.47 \times 0.26 = 0.4803
|
||||
\]
|
||||
\item On cherche à calculer la quantité $P_B(S)$. Pour cela on utilise la formule de Bayes
|
||||
\[
|
||||
P_B(S) = \frac{P(B\cap S)}{P(B)} = \frac{P_S(B) \times P(S)}{P(B)} = \frac{0.74\times 0.47}{0.4803} = 0.7241307516135749 \approx 0.724
|
||||
\]
|
||||
\end{enumerate}
|
||||
\end{solution}
|
||||
|
||||
\begin{exercise}[subtitle={Continent plastique}]
|
||||
\textit{Les quantités évoqués dans cette exercice sont générés au hasard et sont donc complètement farfelus.}
|
||||
\medskip
|
||||
Le \og continent de plastique\fg{} est la plus grande des plaques de déchets plastiques évoluant sur les océans. Elle occupe actuellement dans l'océan Pacifique une surface dont l'aire est évaluée à plus de $1,6$ million de km$^2$, entre Hawaï et la Californie.
|
||||
|
||||
En 2017, des scientifiques ont estimé qu'il y avait $3$ millions de tonnes de déchets plastiques qui était déversé chaque année dans les océans et que cette quantité augmentait de $17\n\%$ par chaque année.
|
||||
|
||||
On modélise l'évolution de la masse de ces déchets plastiques déversée chaque année, si rien n'est fait pour la réduire, par une suite géométrique $\left(u_n\right)$. L'arrondi au centième du terme $u_n$ représente la masse de ces déchets déversée chaque année, exprimée en million de tonnes, pour l'année $(2017 + n)$.
|
||||
|
||||
\medskip
|
||||
|
||||
\begin{enumerate}
|
||||
\item Expliquer pourquoi la suite $u_n$ est géométrique?
|
||||
\item Calculer $u_1$ et $u_2$.
|
||||
\item Exprimer $u_n$ en fonction de $n$.
|
||||
\item Au début de l'année 2017, il y avait $300$ millions de tonnes de déchets plastique. Calculer la quantité totale de déchets plastiques en 2030.
|
||||
\item On souhaite déterminer en quelle année la masse totale de ces déchets plastiques aura pour la première fois augmenté de $50$\,\% par rapport à sa valeur de 2017.
|
||||
\begin{enumerate}
|
||||
\item Recopier et compléter l'algorithme ci-dessous pour que la variable $N$ contienne la réponse au problème posé.
|
||||
|
||||
\begin{center}
|
||||
\begin{tabularx}{0.4\linewidth}{|X|}\hline
|
||||
$N = 2017$\\
|
||||
$U = 3$ \\
|
||||
$S = 300 + U$ \\
|
||||
while $S < 450$: \\
|
||||
\hspace{1cm} $N = \ldots$\\
|
||||
\hspace{1cm} $U = \ldots$\\
|
||||
\hspace{1cm} $S = \ldots$\\
|
||||
\hline
|
||||
\end{tabularx}
|
||||
\end{center}
|
||||
\item Que contiennent les variables $S$, $U$ et $N$ après exécution de cet algorithme ?
|
||||
|
||||
Interpréter les résultats dans le contexte de l'exercice.
|
||||
\end{enumerate}
|
||||
\end{enumerate}
|
||||
\end{exercise}
|
||||
|
||||
\begin{solution}
|
||||
\begin{enumerate}
|
||||
\item Une augmentation de $17\,\%$ revient à multiplier la quantité par $1.17$. La suite est donc bien géométrique. Son premier terme est $u_0 = 3$ et sa raison est $q = 1.17$
|
||||
\item
|
||||
\[
|
||||
u_1 = u_0 * 1.17 = 3.51
|
||||
\]
|
||||
\[
|
||||
u_2 = u_0 * 1.17^2 = 4.1067
|
||||
\]
|
||||
\item
|
||||
\[
|
||||
u_n = u_0 \times q^n = 3 \times 1.17^n
|
||||
\]
|
||||
\item On calcule la quantité totale déversée entre 2017 et 2030.
|
||||
\[
|
||||
\sum_{n = 0}^{13} u_n = u_0 \times \frac{1-q^{13}}{1-q} = 3 \times \frac{1 - 1.17^{13}}{1 - 1.17} = 118.21
|
||||
\]
|
||||
On en déduit la quantité totale de déchets en 2030
|
||||
\[
|
||||
300 + 118.21 = 418.21
|
||||
\]
|
||||
\item
|
||||
\begin{enumerate}
|
||||
\item ~
|
||||
\begin{center}
|
||||
\begin{tabularx}{0.4\linewidth}{|X|}\hline
|
||||
$N \gets 2017$\\
|
||||
$U \gets 3$ \\
|
||||
$S \gets 300 + U$ \\
|
||||
Tant que $S < 450$ \\
|
||||
\hspace{1cm} $N \gets N + 1$\\
|
||||
\hspace{1cm} $U \gets U * 1.17$\\
|
||||
\hspace{1cm} $S \gets S + u$\\
|
||||
Fin Tant que\\\hline
|
||||
\end{tabularx}
|
||||
\end{center}
|
||||
\item \textit{Pas de correction automatisé}
|
||||
\end{enumerate}
|
||||
\end{enumerate}
|
||||
\end{solution}
|
||||
|
||||
\end{document}
|
||||
|
||||
%%% Local Variables:
|
||||
%%% mode: latex
|
||||
%%% TeX-master: "master"
|
||||
%%% End:
|
262
TST/DS/DS_21_04_07/TST1/corr_07_210407_DS8.tex
Normal file
262
TST/DS/DS_21_04_07/TST1/corr_07_210407_DS8.tex
Normal file
@ -0,0 +1,262 @@
|
||||
\documentclass[a4paper,10pt]{article}
|
||||
\usepackage{myXsim}
|
||||
|
||||
% Title Page
|
||||
\title{DS8 \hfill COUBAT Alexis}
|
||||
\tribe{TST}
|
||||
\date{\hfillÀ render pour le Mercredi 7 avril}
|
||||
|
||||
\xsimsetup{
|
||||
solution/print = true
|
||||
}
|
||||
|
||||
\begin{document}
|
||||
\maketitle
|
||||
|
||||
\begin{exercise}[subtitle={Automatismes}]
|
||||
\textit{Toutes les questions de cette exercice sont indépendantes et peuvent être répondus séparément}
|
||||
\begin{enumerate}
|
||||
\item De janvier à septembre, une quantité a augmenté de $16\,\%$. Faire un schéma pour représenter la situation puis calculer le taux d'évolution moyen mensuel.
|
||||
\item Une quantité augmente de $16\,\%$ par ans. En 2020, elle est de 120\euro. Quelle était sa valeur en 2019? Faire un schéma pour représenter la situation.
|
||||
\item Déterminer l'équation de la droite \\
|
||||
\begin{tikzpicture}[xscale=0.8, yscale=0.5]
|
||||
\tkzInit[xmin=-5,xmax=5,xstep=1,
|
||||
ymin=-5,ymax=5,ystep=1]
|
||||
\tkzGrid
|
||||
\tkzAxeXY
|
||||
\tkzFct[domain=-5:5,color=red,very thick]%
|
||||
{4.0*\x -4};
|
||||
\end{tikzpicture}
|
||||
\item Résoudre l'équation $10 \times 0.06^x = 34$
|
||||
\end{enumerate}
|
||||
\end{exercise}
|
||||
|
||||
\begin{solution}
|
||||
\begin{enumerate}
|
||||
\item On veut partager cette évolution en 8 évolutions.
|
||||
\[
|
||||
\left(1 + \frac{16}{100}\right)^{\frac{1}{8}} = 1.0187
|
||||
\]
|
||||
Donc le taux d'évolution moyen est
|
||||
\[
|
||||
t_m = 1.0187 - 1 = 0.01869999999999994
|
||||
\]
|
||||
\item Coefficient multiplicateur pour revenir en arrière
|
||||
\[
|
||||
CM = (1 + \frac{16}{100})^{-1} = 0.8621
|
||||
\]
|
||||
On en déduit la quantité en 2019
|
||||
\[
|
||||
120 * 0.8621 = 103.452
|
||||
\]
|
||||
\item L'équation de la droite est
|
||||
\[
|
||||
y = 4.0 x -4
|
||||
\]
|
||||
\item Il faut penser à faire la division à par $10$ avant d'utiliser le log car sinon, on ne peut pas utiliser la formule $\log(a^n) = n\times \log(a)$.
|
||||
|
||||
\[x = \frac{\log(3.4)}{\log(0.06)}\]
|
||||
\end{enumerate}
|
||||
\end{solution}
|
||||
|
||||
\begin{exercise}[subtitle={Restaurant}]
|
||||
Un \emph{food truck}, ouvert le midi et le soir, propose deux types de formules :
|
||||
|
||||
\setlength\parindent{10mm}
|
||||
\begin{itemize}
|
||||
\item la formule \emph{Burger} ;
|
||||
\item la formule \emph{Wok}.
|
||||
\end{itemize}
|
||||
\setlength\parindent{0mm}
|
||||
|
||||
\medskip
|
||||
|
||||
Le gérant a remarqué que 1\,\% de ses ventes ont lieu le midi. Le quart des ventes du midi correspondent à la formule \emph{Burger}, alors que 92\,\% des ventes du soir correspondent à la formule \emph{Wok}.
|
||||
|
||||
Le gérant se constitue un fichier en notant, pour chaque vente, la formule choisie et le moment de cette vente (midi ou soir).
|
||||
|
||||
On prélève une fiche de façon équiprobable. On définit les quatre évènements suivants:
|
||||
|
||||
\begin{enumerate}
|
||||
\item $M$ : \og la fiche correspond à une vente du midi\fg{} ;
|
||||
\item $S$ : \og la fiche correspond à une vente du soir\fg {};
|
||||
\item $W$ : \og la fiche correspond à une formule \emph{Wok} \fg{} ;
|
||||
\item $B$ : \og la fiche correspond à une formule \emph{Burger} \fg.
|
||||
\end{enumerate}
|
||||
\setlength\parindent{0mm}
|
||||
|
||||
\medskip
|
||||
|
||||
\begin{enumerate}
|
||||
\item Recopier puis compléter l'arbre pondéré
|
||||
|
||||
\begin{center}
|
||||
\begin{tikzpicture}[sloped]
|
||||
\node {.}
|
||||
child {node {$M$}
|
||||
child {node {$W$}
|
||||
edge from parent
|
||||
node[above] {...}
|
||||
}
|
||||
child {node {$B$}
|
||||
edge from parent
|
||||
node[above] {...}
|
||||
}
|
||||
edge from parent
|
||||
node[above] {...}
|
||||
}
|
||||
child[missing] {}
|
||||
child { node {$S$}
|
||||
child {node {$W$}
|
||||
edge from parent
|
||||
node[above] {...}
|
||||
}
|
||||
child {node {$B$}
|
||||
edge from parent
|
||||
node[above] {...}
|
||||
}
|
||||
edge from parent
|
||||
node[above] {...}
|
||||
} ;
|
||||
\end{tikzpicture}
|
||||
\end{center}
|
||||
|
||||
\item Calculer la probabilité de l'évènement $M \cap W$. Interpréter ce résultat dans le contexte de l'exercice.
|
||||
\item Montrer que la probabilité que la fiche choisie corresponde à une formule \emph{Burger} est égale à $0.0817$.
|
||||
\item On a prélevé une fiche correspondant à la formule \emph{Burger}. Quelle est la probabilité, arrondie au millième, que la vente ait eu lieu le soir?
|
||||
\end{enumerate}
|
||||
\end{exercise}
|
||||
|
||||
\begin{solution}
|
||||
\begin{enumerate}
|
||||
\item
|
||||
\begin{center}
|
||||
\begin{tikzpicture}[sloped]
|
||||
\node {.}
|
||||
child {node {$M$}
|
||||
child {node {$W$}
|
||||
edge from parent
|
||||
node[above] {$0.75$}
|
||||
}
|
||||
child {node {$B$}
|
||||
edge from parent
|
||||
node[above] {$0.25$}
|
||||
}
|
||||
edge from parent
|
||||
node[above] {$0.01$}
|
||||
}
|
||||
child[missing] {}
|
||||
child { node {$S$}
|
||||
child {node {$W$}
|
||||
edge from parent
|
||||
node[above] {$0.92$}
|
||||
}
|
||||
child {node {$B$}
|
||||
edge from parent
|
||||
node[above] {$0.08$}
|
||||
}
|
||||
edge from parent
|
||||
node[above] {$0.99$}
|
||||
} ;
|
||||
\end{tikzpicture}
|
||||
\end{center}
|
||||
\item On calcule la probabilité que la vente soit un wok et ait eu lieu à midi
|
||||
\[ P(M\cap W) = P(M) \times P_M(W) = 0.01 \times 0.75 = 0.0075 \]
|
||||
\item Probabilité que la vente soit un burger.
|
||||
\[
|
||||
P(B) = P(M\cap B) + P(S\cap B) = 0.01 \times 0.75 + 0.99 \times 0.92 = 0.0817
|
||||
\]
|
||||
\item On cherche à calculer la quantité $P_B(S)$. Pour cela on utilise la formule de Bayes
|
||||
\[
|
||||
P_B(S) = \frac{P(B\cap S)}{P(B)} = \frac{P_S(B) \times P(S)}{P(B)} = \frac{0.08\times 0.99}{0.0817} = 0.9694002447980418 \approx 0.969
|
||||
\]
|
||||
\end{enumerate}
|
||||
\end{solution}
|
||||
|
||||
\begin{exercise}[subtitle={Continent plastique}]
|
||||
\textit{Les quantités évoqués dans cette exercice sont générés au hasard et sont donc complètement farfelus.}
|
||||
\medskip
|
||||
Le \og continent de plastique\fg{} est la plus grande des plaques de déchets plastiques évoluant sur les océans. Elle occupe actuellement dans l'océan Pacifique une surface dont l'aire est évaluée à plus de $1,6$ million de km$^2$, entre Hawaï et la Californie.
|
||||
|
||||
En 2017, des scientifiques ont estimé qu'il y avait $12$ millions de tonnes de déchets plastiques qui était déversé chaque année dans les océans et que cette quantité augmentait de $14\n\%$ par chaque année.
|
||||
|
||||
On modélise l'évolution de la masse de ces déchets plastiques déversée chaque année, si rien n'est fait pour la réduire, par une suite géométrique $\left(u_n\right)$. L'arrondi au centième du terme $u_n$ représente la masse de ces déchets déversée chaque année, exprimée en million de tonnes, pour l'année $(2017 + n)$.
|
||||
|
||||
\medskip
|
||||
|
||||
\begin{enumerate}
|
||||
\item Expliquer pourquoi la suite $u_n$ est géométrique?
|
||||
\item Calculer $u_1$ et $u_2$.
|
||||
\item Exprimer $u_n$ en fonction de $n$.
|
||||
\item Au début de l'année 2017, il y avait $300$ millions de tonnes de déchets plastique. Calculer la quantité totale de déchets plastiques en 2030.
|
||||
\item On souhaite déterminer en quelle année la masse totale de ces déchets plastiques aura pour la première fois augmenté de $50$\,\% par rapport à sa valeur de 2017.
|
||||
\begin{enumerate}
|
||||
\item Recopier et compléter l'algorithme ci-dessous pour que la variable $N$ contienne la réponse au problème posé.
|
||||
|
||||
\begin{center}
|
||||
\begin{tabularx}{0.4\linewidth}{|X|}\hline
|
||||
$N = 2017$\\
|
||||
$U = 12$ \\
|
||||
$S = 300 + U$ \\
|
||||
while $S < 450$: \\
|
||||
\hspace{1cm} $N = \ldots$\\
|
||||
\hspace{1cm} $U = \ldots$\\
|
||||
\hspace{1cm} $S = \ldots$\\
|
||||
\hline
|
||||
\end{tabularx}
|
||||
\end{center}
|
||||
\item Que contiennent les variables $S$, $U$ et $N$ après exécution de cet algorithme ?
|
||||
|
||||
Interpréter les résultats dans le contexte de l'exercice.
|
||||
\end{enumerate}
|
||||
\end{enumerate}
|
||||
\end{exercise}
|
||||
|
||||
\begin{solution}
|
||||
\begin{enumerate}
|
||||
\item Une augmentation de $14\,\%$ revient à multiplier la quantité par $1.1400000000000001$. La suite est donc bien géométrique. Son premier terme est $u_0 = 12$ et sa raison est $q = 1.1400000000000001$
|
||||
\item
|
||||
\[
|
||||
u_1 = u_0 * 1.1400000000000001 = 13.680000000000001
|
||||
\]
|
||||
\[
|
||||
u_2 = u_0 * 1.1400000000000001^2 = 15.5952
|
||||
\]
|
||||
\item
|
||||
\[
|
||||
u_n = u_0 \times q^n = 12 \times 1.1400000000000001^n
|
||||
\]
|
||||
\item On calcule la quantité totale déversée entre 2017 et 2030.
|
||||
\[
|
||||
\sum_{n = 0}^{13} u_n = u_0 \times \frac{1-q^{13}}{1-q} = 12 \times \frac{1 - 1.1400000000000001^{13}}{1 - 1.1400000000000001} = 385.06
|
||||
\]
|
||||
On en déduit la quantité totale de déchets en 2030
|
||||
\[
|
||||
300 + 385.06 = 685.06
|
||||
\]
|
||||
\item
|
||||
\begin{enumerate}
|
||||
\item ~
|
||||
\begin{center}
|
||||
\begin{tabularx}{0.4\linewidth}{|X|}\hline
|
||||
$N \gets 2017$\\
|
||||
$U \gets 12$ \\
|
||||
$S \gets 300 + U$ \\
|
||||
Tant que $S < 450$ \\
|
||||
\hspace{1cm} $N \gets N + 1$\\
|
||||
\hspace{1cm} $U \gets U * 1.1400000000000001$\\
|
||||
\hspace{1cm} $S \gets S + u$\\
|
||||
Fin Tant que\\\hline
|
||||
\end{tabularx}
|
||||
\end{center}
|
||||
\item \textit{Pas de correction automatisé}
|
||||
\end{enumerate}
|
||||
\end{enumerate}
|
||||
\end{solution}
|
||||
|
||||
\end{document}
|
||||
|
||||
%%% Local Variables:
|
||||
%%% mode: latex
|
||||
%%% TeX-master: "master"
|
||||
%%% End:
|
262
TST/DS/DS_21_04_07/TST1/corr_08_210407_DS8.tex
Normal file
262
TST/DS/DS_21_04_07/TST1/corr_08_210407_DS8.tex
Normal file
@ -0,0 +1,262 @@
|
||||
\documentclass[a4paper,10pt]{article}
|
||||
\usepackage{myXsim}
|
||||
|
||||
% Title Page
|
||||
\title{DS8 \hfill COULLON Anis}
|
||||
\tribe{TST}
|
||||
\date{\hfillÀ render pour le Mercredi 7 avril}
|
||||
|
||||
\xsimsetup{
|
||||
solution/print = true
|
||||
}
|
||||
|
||||
\begin{document}
|
||||
\maketitle
|
||||
|
||||
\begin{exercise}[subtitle={Automatismes}]
|
||||
\textit{Toutes les questions de cette exercice sont indépendantes et peuvent être répondus séparément}
|
||||
\begin{enumerate}
|
||||
\item De janvier à septembre, une quantité a augmenté de $14\,\%$. Faire un schéma pour représenter la situation puis calculer le taux d'évolution moyen mensuel.
|
||||
\item Une quantité augmente de $14\,\%$ par ans. En 2020, elle est de 125\euro. Quelle était sa valeur en 2019? Faire un schéma pour représenter la situation.
|
||||
\item Déterminer l'équation de la droite \\
|
||||
\begin{tikzpicture}[xscale=0.8, yscale=0.5]
|
||||
\tkzInit[xmin=-5,xmax=5,xstep=1,
|
||||
ymin=-5,ymax=5,ystep=1]
|
||||
\tkzGrid
|
||||
\tkzAxeXY
|
||||
\tkzFct[domain=-5:5,color=red,very thick]%
|
||||
{0.5*\x -1};
|
||||
\end{tikzpicture}
|
||||
\item Résoudre l'équation $6 \times 0.82^x = 19$
|
||||
\end{enumerate}
|
||||
\end{exercise}
|
||||
|
||||
\begin{solution}
|
||||
\begin{enumerate}
|
||||
\item On veut partager cette évolution en 8 évolutions.
|
||||
\[
|
||||
\left(1 + \frac{14}{100}\right)^{\frac{1}{8}} = 1.0165
|
||||
\]
|
||||
Donc le taux d'évolution moyen est
|
||||
\[
|
||||
t_m = 1.0165 - 1 = 0.01649999999999996
|
||||
\]
|
||||
\item Coefficient multiplicateur pour revenir en arrière
|
||||
\[
|
||||
CM = (1 + \frac{14}{100})^{-1} = 0.8772
|
||||
\]
|
||||
On en déduit la quantité en 2019
|
||||
\[
|
||||
125 * 0.8772 = 109.64999999999999
|
||||
\]
|
||||
\item L'équation de la droite est
|
||||
\[
|
||||
y = 0.5 x -1
|
||||
\]
|
||||
\item Il faut penser à faire la division à par $6$ avant d'utiliser le log car sinon, on ne peut pas utiliser la formule $\log(a^n) = n\times \log(a)$.
|
||||
|
||||
\[x = \frac{\log(3.17)}{\log(0.82)}\]
|
||||
\end{enumerate}
|
||||
\end{solution}
|
||||
|
||||
\begin{exercise}[subtitle={Restaurant}]
|
||||
Un \emph{food truck}, ouvert le midi et le soir, propose deux types de formules :
|
||||
|
||||
\setlength\parindent{10mm}
|
||||
\begin{itemize}
|
||||
\item la formule \emph{Burger} ;
|
||||
\item la formule \emph{Wok}.
|
||||
\end{itemize}
|
||||
\setlength\parindent{0mm}
|
||||
|
||||
\medskip
|
||||
|
||||
Le gérant a remarqué que 30\,\% de ses ventes ont lieu le midi. Le quart des ventes du midi correspondent à la formule \emph{Burger}, alors que 27\,\% des ventes du soir correspondent à la formule \emph{Wok}.
|
||||
|
||||
Le gérant se constitue un fichier en notant, pour chaque vente, la formule choisie et le moment de cette vente (midi ou soir).
|
||||
|
||||
On prélève une fiche de façon équiprobable. On définit les quatre évènements suivants:
|
||||
|
||||
\begin{enumerate}
|
||||
\item $M$ : \og la fiche correspond à une vente du midi\fg{} ;
|
||||
\item $S$ : \og la fiche correspond à une vente du soir\fg {};
|
||||
\item $W$ : \og la fiche correspond à une formule \emph{Wok} \fg{} ;
|
||||
\item $B$ : \og la fiche correspond à une formule \emph{Burger} \fg.
|
||||
\end{enumerate}
|
||||
\setlength\parindent{0mm}
|
||||
|
||||
\medskip
|
||||
|
||||
\begin{enumerate}
|
||||
\item Recopier puis compléter l'arbre pondéré
|
||||
|
||||
\begin{center}
|
||||
\begin{tikzpicture}[sloped]
|
||||
\node {.}
|
||||
child {node {$M$}
|
||||
child {node {$W$}
|
||||
edge from parent
|
||||
node[above] {...}
|
||||
}
|
||||
child {node {$B$}
|
||||
edge from parent
|
||||
node[above] {...}
|
||||
}
|
||||
edge from parent
|
||||
node[above] {...}
|
||||
}
|
||||
child[missing] {}
|
||||
child { node {$S$}
|
||||
child {node {$W$}
|
||||
edge from parent
|
||||
node[above] {...}
|
||||
}
|
||||
child {node {$B$}
|
||||
edge from parent
|
||||
node[above] {...}
|
||||
}
|
||||
edge from parent
|
||||
node[above] {...}
|
||||
} ;
|
||||
\end{tikzpicture}
|
||||
\end{center}
|
||||
|
||||
\item Calculer la probabilité de l'évènement $M \cap W$. Interpréter ce résultat dans le contexte de l'exercice.
|
||||
\item Montrer que la probabilité que la fiche choisie corresponde à une formule \emph{Burger} est égale à $0.586$.
|
||||
\item On a prélevé une fiche correspondant à la formule \emph{Burger}. Quelle est la probabilité, arrondie au millième, que la vente ait eu lieu le soir?
|
||||
\end{enumerate}
|
||||
\end{exercise}
|
||||
|
||||
\begin{solution}
|
||||
\begin{enumerate}
|
||||
\item
|
||||
\begin{center}
|
||||
\begin{tikzpicture}[sloped]
|
||||
\node {.}
|
||||
child {node {$M$}
|
||||
child {node {$W$}
|
||||
edge from parent
|
||||
node[above] {$0.75$}
|
||||
}
|
||||
child {node {$B$}
|
||||
edge from parent
|
||||
node[above] {$0.25$}
|
||||
}
|
||||
edge from parent
|
||||
node[above] {$0.3$}
|
||||
}
|
||||
child[missing] {}
|
||||
child { node {$S$}
|
||||
child {node {$W$}
|
||||
edge from parent
|
||||
node[above] {$0.27$}
|
||||
}
|
||||
child {node {$B$}
|
||||
edge from parent
|
||||
node[above] {$0.73$}
|
||||
}
|
||||
edge from parent
|
||||
node[above] {$0.7$}
|
||||
} ;
|
||||
\end{tikzpicture}
|
||||
\end{center}
|
||||
\item On calcule la probabilité que la vente soit un wok et ait eu lieu à midi
|
||||
\[ P(M\cap W) = P(M) \times P_M(W) = 0.3 \times 0.75 = 0.225 \]
|
||||
\item Probabilité que la vente soit un burger.
|
||||
\[
|
||||
P(B) = P(M\cap B) + P(S\cap B) = 0.3 \times 0.75 + 0.7 \times 0.27 = 0.586
|
||||
\]
|
||||
\item On cherche à calculer la quantité $P_B(S)$. Pour cela on utilise la formule de Bayes
|
||||
\[
|
||||
P_B(S) = \frac{P(B\cap S)}{P(B)} = \frac{P_S(B) \times P(S)}{P(B)} = \frac{0.73\times 0.7}{0.586} = 0.8720136518771332 \approx 0.872
|
||||
\]
|
||||
\end{enumerate}
|
||||
\end{solution}
|
||||
|
||||
\begin{exercise}[subtitle={Continent plastique}]
|
||||
\textit{Les quantités évoqués dans cette exercice sont générés au hasard et sont donc complètement farfelus.}
|
||||
\medskip
|
||||
Le \og continent de plastique\fg{} est la plus grande des plaques de déchets plastiques évoluant sur les océans. Elle occupe actuellement dans l'océan Pacifique une surface dont l'aire est évaluée à plus de $1,6$ million de km$^2$, entre Hawaï et la Californie.
|
||||
|
||||
En 2017, des scientifiques ont estimé qu'il y avait $14$ millions de tonnes de déchets plastiques qui était déversé chaque année dans les océans et que cette quantité augmentait de $25\n\%$ par chaque année.
|
||||
|
||||
On modélise l'évolution de la masse de ces déchets plastiques déversée chaque année, si rien n'est fait pour la réduire, par une suite géométrique $\left(u_n\right)$. L'arrondi au centième du terme $u_n$ représente la masse de ces déchets déversée chaque année, exprimée en million de tonnes, pour l'année $(2017 + n)$.
|
||||
|
||||
\medskip
|
||||
|
||||
\begin{enumerate}
|
||||
\item Expliquer pourquoi la suite $u_n$ est géométrique?
|
||||
\item Calculer $u_1$ et $u_2$.
|
||||
\item Exprimer $u_n$ en fonction de $n$.
|
||||
\item Au début de l'année 2017, il y avait $300$ millions de tonnes de déchets plastique. Calculer la quantité totale de déchets plastiques en 2030.
|
||||
\item On souhaite déterminer en quelle année la masse totale de ces déchets plastiques aura pour la première fois augmenté de $50$\,\% par rapport à sa valeur de 2017.
|
||||
\begin{enumerate}
|
||||
\item Recopier et compléter l'algorithme ci-dessous pour que la variable $N$ contienne la réponse au problème posé.
|
||||
|
||||
\begin{center}
|
||||
\begin{tabularx}{0.4\linewidth}{|X|}\hline
|
||||
$N = 2017$\\
|
||||
$U = 14$ \\
|
||||
$S = 300 + U$ \\
|
||||
while $S < 450$: \\
|
||||
\hspace{1cm} $N = \ldots$\\
|
||||
\hspace{1cm} $U = \ldots$\\
|
||||
\hspace{1cm} $S = \ldots$\\
|
||||
\hline
|
||||
\end{tabularx}
|
||||
\end{center}
|
||||
\item Que contiennent les variables $S$, $U$ et $N$ après exécution de cet algorithme ?
|
||||
|
||||
Interpréter les résultats dans le contexte de l'exercice.
|
||||
\end{enumerate}
|
||||
\end{enumerate}
|
||||
\end{exercise}
|
||||
|
||||
\begin{solution}
|
||||
\begin{enumerate}
|
||||
\item Une augmentation de $25\,\%$ revient à multiplier la quantité par $1.25$. La suite est donc bien géométrique. Son premier terme est $u_0 = 14$ et sa raison est $q = 1.25$
|
||||
\item
|
||||
\[
|
||||
u_1 = u_0 * 1.25 = 17.5
|
||||
\]
|
||||
\[
|
||||
u_2 = u_0 * 1.25^2 = 21.875
|
||||
\]
|
||||
\item
|
||||
\[
|
||||
u_n = u_0 \times q^n = 14 \times 1.25^n
|
||||
\]
|
||||
\item On calcule la quantité totale déversée entre 2017 et 2030.
|
||||
\[
|
||||
\sum_{n = 0}^{13} u_n = u_0 \times \frac{1-q^{13}}{1-q} = 14 \times \frac{1 - 1.25^{13}}{1 - 1.25} = 962.63
|
||||
\]
|
||||
On en déduit la quantité totale de déchets en 2030
|
||||
\[
|
||||
300 + 962.63 = 1262.63
|
||||
\]
|
||||
\item
|
||||
\begin{enumerate}
|
||||
\item ~
|
||||
\begin{center}
|
||||
\begin{tabularx}{0.4\linewidth}{|X|}\hline
|
||||
$N \gets 2017$\\
|
||||
$U \gets 14$ \\
|
||||
$S \gets 300 + U$ \\
|
||||
Tant que $S < 450$ \\
|
||||
\hspace{1cm} $N \gets N + 1$\\
|
||||
\hspace{1cm} $U \gets U * 1.25$\\
|
||||
\hspace{1cm} $S \gets S + u$\\
|
||||
Fin Tant que\\\hline
|
||||
\end{tabularx}
|
||||
\end{center}
|
||||
\item \textit{Pas de correction automatisé}
|
||||
\end{enumerate}
|
||||
\end{enumerate}
|
||||
\end{solution}
|
||||
|
||||
\end{document}
|
||||
|
||||
%%% Local Variables:
|
||||
%%% mode: latex
|
||||
%%% TeX-master: "master"
|
||||
%%% End:
|
262
TST/DS/DS_21_04_07/TST1/corr_09_210407_DS8.tex
Normal file
262
TST/DS/DS_21_04_07/TST1/corr_09_210407_DS8.tex
Normal file
@ -0,0 +1,262 @@
|
||||
\documentclass[a4paper,10pt]{article}
|
||||
\usepackage{myXsim}
|
||||
|
||||
% Title Page
|
||||
\title{DS8 \hfill DINGER Sölen}
|
||||
\tribe{TST}
|
||||
\date{\hfillÀ render pour le Mercredi 7 avril}
|
||||
|
||||
\xsimsetup{
|
||||
solution/print = true
|
||||
}
|
||||
|
||||
\begin{document}
|
||||
\maketitle
|
||||
|
||||
\begin{exercise}[subtitle={Automatismes}]
|
||||
\textit{Toutes les questions de cette exercice sont indépendantes et peuvent être répondus séparément}
|
||||
\begin{enumerate}
|
||||
\item De janvier à septembre, une quantité a augmenté de $30\,\%$. Faire un schéma pour représenter la situation puis calculer le taux d'évolution moyen mensuel.
|
||||
\item Une quantité augmente de $30\,\%$ par ans. En 2020, elle est de 141\euro. Quelle était sa valeur en 2019? Faire un schéma pour représenter la situation.
|
||||
\item Déterminer l'équation de la droite \\
|
||||
\begin{tikzpicture}[xscale=0.8, yscale=0.5]
|
||||
\tkzInit[xmin=-5,xmax=5,xstep=1,
|
||||
ymin=-5,ymax=5,ystep=1]
|
||||
\tkzGrid
|
||||
\tkzAxeXY
|
||||
\tkzFct[domain=-5:5,color=red,very thick]%
|
||||
{2.0*\x -3};
|
||||
\end{tikzpicture}
|
||||
\item Résoudre l'équation $5 \times 0.8^x = 11$
|
||||
\end{enumerate}
|
||||
\end{exercise}
|
||||
|
||||
\begin{solution}
|
||||
\begin{enumerate}
|
||||
\item On veut partager cette évolution en 8 évolutions.
|
||||
\[
|
||||
\left(1 + \frac{30}{100}\right)^{\frac{1}{8}} = 1.0333
|
||||
\]
|
||||
Donc le taux d'évolution moyen est
|
||||
\[
|
||||
t_m = 1.0333 - 1 = 0.03330000000000011
|
||||
\]
|
||||
\item Coefficient multiplicateur pour revenir en arrière
|
||||
\[
|
||||
CM = (1 + \frac{30}{100})^{-1} = 0.7692
|
||||
\]
|
||||
On en déduit la quantité en 2019
|
||||
\[
|
||||
141 * 0.7692 = 108.4572
|
||||
\]
|
||||
\item L'équation de la droite est
|
||||
\[
|
||||
y = 2.0 x -3
|
||||
\]
|
||||
\item Il faut penser à faire la division à par $5$ avant d'utiliser le log car sinon, on ne peut pas utiliser la formule $\log(a^n) = n\times \log(a)$.
|
||||
|
||||
\[x = \frac{\log(2.2)}{\log(0.8)}\]
|
||||
\end{enumerate}
|
||||
\end{solution}
|
||||
|
||||
\begin{exercise}[subtitle={Restaurant}]
|
||||
Un \emph{food truck}, ouvert le midi et le soir, propose deux types de formules :
|
||||
|
||||
\setlength\parindent{10mm}
|
||||
\begin{itemize}
|
||||
\item la formule \emph{Burger} ;
|
||||
\item la formule \emph{Wok}.
|
||||
\end{itemize}
|
||||
\setlength\parindent{0mm}
|
||||
|
||||
\medskip
|
||||
|
||||
Le gérant a remarqué que 57\,\% de ses ventes ont lieu le midi. Le quart des ventes du midi correspondent à la formule \emph{Burger}, alors que 90\,\% des ventes du soir correspondent à la formule \emph{Wok}.
|
||||
|
||||
Le gérant se constitue un fichier en notant, pour chaque vente, la formule choisie et le moment de cette vente (midi ou soir).
|
||||
|
||||
On prélève une fiche de façon équiprobable. On définit les quatre évènements suivants:
|
||||
|
||||
\begin{enumerate}
|
||||
\item $M$ : \og la fiche correspond à une vente du midi\fg{} ;
|
||||
\item $S$ : \og la fiche correspond à une vente du soir\fg {};
|
||||
\item $W$ : \og la fiche correspond à une formule \emph{Wok} \fg{} ;
|
||||
\item $B$ : \og la fiche correspond à une formule \emph{Burger} \fg.
|
||||
\end{enumerate}
|
||||
\setlength\parindent{0mm}
|
||||
|
||||
\medskip
|
||||
|
||||
\begin{enumerate}
|
||||
\item Recopier puis compléter l'arbre pondéré
|
||||
|
||||
\begin{center}
|
||||
\begin{tikzpicture}[sloped]
|
||||
\node {.}
|
||||
child {node {$M$}
|
||||
child {node {$W$}
|
||||
edge from parent
|
||||
node[above] {...}
|
||||
}
|
||||
child {node {$B$}
|
||||
edge from parent
|
||||
node[above] {...}
|
||||
}
|
||||
edge from parent
|
||||
node[above] {...}
|
||||
}
|
||||
child[missing] {}
|
||||
child { node {$S$}
|
||||
child {node {$W$}
|
||||
edge from parent
|
||||
node[above] {...}
|
||||
}
|
||||
child {node {$B$}
|
||||
edge from parent
|
||||
node[above] {...}
|
||||
}
|
||||
edge from parent
|
||||
node[above] {...}
|
||||
} ;
|
||||
\end{tikzpicture}
|
||||
\end{center}
|
||||
|
||||
\item Calculer la probabilité de l'évènement $M \cap W$. Interpréter ce résultat dans le contexte de l'exercice.
|
||||
\item Montrer que la probabilité que la fiche choisie corresponde à une formule \emph{Burger} est égale à $0.187$.
|
||||
\item On a prélevé une fiche correspondant à la formule \emph{Burger}. Quelle est la probabilité, arrondie au millième, que la vente ait eu lieu le soir?
|
||||
\end{enumerate}
|
||||
\end{exercise}
|
||||
|
||||
\begin{solution}
|
||||
\begin{enumerate}
|
||||
\item
|
||||
\begin{center}
|
||||
\begin{tikzpicture}[sloped]
|
||||
\node {.}
|
||||
child {node {$M$}
|
||||
child {node {$W$}
|
||||
edge from parent
|
||||
node[above] {$0.75$}
|
||||
}
|
||||
child {node {$B$}
|
||||
edge from parent
|
||||
node[above] {$0.25$}
|
||||
}
|
||||
edge from parent
|
||||
node[above] {$0.58$}
|
||||
}
|
||||
child[missing] {}
|
||||
child { node {$S$}
|
||||
child {node {$W$}
|
||||
edge from parent
|
||||
node[above] {$0.9$}
|
||||
}
|
||||
child {node {$B$}
|
||||
edge from parent
|
||||
node[above] {$0.1$}
|
||||
}
|
||||
edge from parent
|
||||
node[above] {$0.42$}
|
||||
} ;
|
||||
\end{tikzpicture}
|
||||
\end{center}
|
||||
\item On calcule la probabilité que la vente soit un wok et ait eu lieu à midi
|
||||
\[ P(M\cap W) = P(M) \times P_M(W) = 0.58 \times 0.75 = 0.435 \]
|
||||
\item Probabilité que la vente soit un burger.
|
||||
\[
|
||||
P(B) = P(M\cap B) + P(S\cap B) = 0.58 \times 0.75 + 0.42 \times 0.9 = 0.187
|
||||
\]
|
||||
\item On cherche à calculer la quantité $P_B(S)$. Pour cela on utilise la formule de Bayes
|
||||
\[
|
||||
P_B(S) = \frac{P(B\cap S)}{P(B)} = \frac{P_S(B) \times P(S)}{P(B)} = \frac{0.1\times 0.42}{0.187} = 0.22459893048128343 \approx 0.225
|
||||
\]
|
||||
\end{enumerate}
|
||||
\end{solution}
|
||||
|
||||
\begin{exercise}[subtitle={Continent plastique}]
|
||||
\textit{Les quantités évoqués dans cette exercice sont générés au hasard et sont donc complètement farfelus.}
|
||||
\medskip
|
||||
Le \og continent de plastique\fg{} est la plus grande des plaques de déchets plastiques évoluant sur les océans. Elle occupe actuellement dans l'océan Pacifique une surface dont l'aire est évaluée à plus de $1,6$ million de km$^2$, entre Hawaï et la Californie.
|
||||
|
||||
En 2017, des scientifiques ont estimé qu'il y avait $14$ millions de tonnes de déchets plastiques qui était déversé chaque année dans les océans et que cette quantité augmentait de $19\n\%$ par chaque année.
|
||||
|
||||
On modélise l'évolution de la masse de ces déchets plastiques déversée chaque année, si rien n'est fait pour la réduire, par une suite géométrique $\left(u_n\right)$. L'arrondi au centième du terme $u_n$ représente la masse de ces déchets déversée chaque année, exprimée en million de tonnes, pour l'année $(2017 + n)$.
|
||||
|
||||
\medskip
|
||||
|
||||
\begin{enumerate}
|
||||
\item Expliquer pourquoi la suite $u_n$ est géométrique?
|
||||
\item Calculer $u_1$ et $u_2$.
|
||||
\item Exprimer $u_n$ en fonction de $n$.
|
||||
\item Au début de l'année 2017, il y avait $300$ millions de tonnes de déchets plastique. Calculer la quantité totale de déchets plastiques en 2030.
|
||||
\item On souhaite déterminer en quelle année la masse totale de ces déchets plastiques aura pour la première fois augmenté de $50$\,\% par rapport à sa valeur de 2017.
|
||||
\begin{enumerate}
|
||||
\item Recopier et compléter l'algorithme ci-dessous pour que la variable $N$ contienne la réponse au problème posé.
|
||||
|
||||
\begin{center}
|
||||
\begin{tabularx}{0.4\linewidth}{|X|}\hline
|
||||
$N = 2017$\\
|
||||
$U = 14$ \\
|
||||
$S = 300 + U$ \\
|
||||
while $S < 450$: \\
|
||||
\hspace{1cm} $N = \ldots$\\
|
||||
\hspace{1cm} $U = \ldots$\\
|
||||
\hspace{1cm} $S = \ldots$\\
|
||||
\hline
|
||||
\end{tabularx}
|
||||
\end{center}
|
||||
\item Que contiennent les variables $S$, $U$ et $N$ après exécution de cet algorithme ?
|
||||
|
||||
Interpréter les résultats dans le contexte de l'exercice.
|
||||
\end{enumerate}
|
||||
\end{enumerate}
|
||||
\end{exercise}
|
||||
|
||||
\begin{solution}
|
||||
\begin{enumerate}
|
||||
\item Une augmentation de $19\,\%$ revient à multiplier la quantité par $1.19$. La suite est donc bien géométrique. Son premier terme est $u_0 = 14$ et sa raison est $q = 1.19$
|
||||
\item
|
||||
\[
|
||||
u_1 = u_0 * 1.19 = 16.66
|
||||
\]
|
||||
\[
|
||||
u_2 = u_0 * 1.19^2 = 19.8254
|
||||
\]
|
||||
\item
|
||||
\[
|
||||
u_n = u_0 \times q^n = 14 \times 1.19^n
|
||||
\]
|
||||
\item On calcule la quantité totale déversée entre 2017 et 2030.
|
||||
\[
|
||||
\sum_{n = 0}^{13} u_n = u_0 \times \frac{1-q^{13}}{1-q} = 14 \times \frac{1 - 1.19^{13}}{1 - 1.19} = 633.42
|
||||
\]
|
||||
On en déduit la quantité totale de déchets en 2030
|
||||
\[
|
||||
300 + 633.42 = 933.42
|
||||
\]
|
||||
\item
|
||||
\begin{enumerate}
|
||||
\item ~
|
||||
\begin{center}
|
||||
\begin{tabularx}{0.4\linewidth}{|X|}\hline
|
||||
$N \gets 2017$\\
|
||||
$U \gets 14$ \\
|
||||
$S \gets 300 + U$ \\
|
||||
Tant que $S < 450$ \\
|
||||
\hspace{1cm} $N \gets N + 1$\\
|
||||
\hspace{1cm} $U \gets U * 1.19$\\
|
||||
\hspace{1cm} $S \gets S + u$\\
|
||||
Fin Tant que\\\hline
|
||||
\end{tabularx}
|
||||
\end{center}
|
||||
\item \textit{Pas de correction automatisé}
|
||||
\end{enumerate}
|
||||
\end{enumerate}
|
||||
\end{solution}
|
||||
|
||||
\end{document}
|
||||
|
||||
%%% Local Variables:
|
||||
%%% mode: latex
|
||||
%%% TeX-master: "master"
|
||||
%%% End:
|
262
TST/DS/DS_21_04_07/TST1/corr_10_210407_DS8.tex
Normal file
262
TST/DS/DS_21_04_07/TST1/corr_10_210407_DS8.tex
Normal file
@ -0,0 +1,262 @@
|
||||
\documentclass[a4paper,10pt]{article}
|
||||
\usepackage{myXsim}
|
||||
|
||||
% Title Page
|
||||
\title{DS8 \hfill EYRAUD Cynthia}
|
||||
\tribe{TST}
|
||||
\date{\hfillÀ render pour le Mercredi 7 avril}
|
||||
|
||||
\xsimsetup{
|
||||
solution/print = true
|
||||
}
|
||||
|
||||
\begin{document}
|
||||
\maketitle
|
||||
|
||||
\begin{exercise}[subtitle={Automatismes}]
|
||||
\textit{Toutes les questions de cette exercice sont indépendantes et peuvent être répondus séparément}
|
||||
\begin{enumerate}
|
||||
\item De janvier à septembre, une quantité a augmenté de $29\,\%$. Faire un schéma pour représenter la situation puis calculer le taux d'évolution moyen mensuel.
|
||||
\item Une quantité augmente de $29\,\%$ par ans. En 2020, elle est de 130\euro. Quelle était sa valeur en 2019? Faire un schéma pour représenter la situation.
|
||||
\item Déterminer l'équation de la droite \\
|
||||
\begin{tikzpicture}[xscale=0.8, yscale=0.5]
|
||||
\tkzInit[xmin=-5,xmax=5,xstep=1,
|
||||
ymin=-5,ymax=5,ystep=1]
|
||||
\tkzGrid
|
||||
\tkzAxeXY
|
||||
\tkzFct[domain=-5:5,color=red,very thick]%
|
||||
{4.0*\x -4};
|
||||
\end{tikzpicture}
|
||||
\item Résoudre l'équation $6 \times 0.48^x = 28$
|
||||
\end{enumerate}
|
||||
\end{exercise}
|
||||
|
||||
\begin{solution}
|
||||
\begin{enumerate}
|
||||
\item On veut partager cette évolution en 8 évolutions.
|
||||
\[
|
||||
\left(1 + \frac{29}{100}\right)^{\frac{1}{8}} = 1.0323
|
||||
\]
|
||||
Donc le taux d'évolution moyen est
|
||||
\[
|
||||
t_m = 1.0323 - 1 = 0.032299999999999995
|
||||
\]
|
||||
\item Coefficient multiplicateur pour revenir en arrière
|
||||
\[
|
||||
CM = (1 + \frac{29}{100})^{-1} = 0.7752
|
||||
\]
|
||||
On en déduit la quantité en 2019
|
||||
\[
|
||||
130 * 0.7752 = 100.776
|
||||
\]
|
||||
\item L'équation de la droite est
|
||||
\[
|
||||
y = 4.0 x -4
|
||||
\]
|
||||
\item Il faut penser à faire la division à par $6$ avant d'utiliser le log car sinon, on ne peut pas utiliser la formule $\log(a^n) = n\times \log(a)$.
|
||||
|
||||
\[x = \frac{\log(4.67)}{\log(0.48)}\]
|
||||
\end{enumerate}
|
||||
\end{solution}
|
||||
|
||||
\begin{exercise}[subtitle={Restaurant}]
|
||||
Un \emph{food truck}, ouvert le midi et le soir, propose deux types de formules :
|
||||
|
||||
\setlength\parindent{10mm}
|
||||
\begin{itemize}
|
||||
\item la formule \emph{Burger} ;
|
||||
\item la formule \emph{Wok}.
|
||||
\end{itemize}
|
||||
\setlength\parindent{0mm}
|
||||
|
||||
\medskip
|
||||
|
||||
Le gérant a remarqué que 3\,\% de ses ventes ont lieu le midi. Le quart des ventes du midi correspondent à la formule \emph{Burger}, alors que 33\,\% des ventes du soir correspondent à la formule \emph{Wok}.
|
||||
|
||||
Le gérant se constitue un fichier en notant, pour chaque vente, la formule choisie et le moment de cette vente (midi ou soir).
|
||||
|
||||
On prélève une fiche de façon équiprobable. On définit les quatre évènements suivants:
|
||||
|
||||
\begin{enumerate}
|
||||
\item $M$ : \og la fiche correspond à une vente du midi\fg{} ;
|
||||
\item $S$ : \og la fiche correspond à une vente du soir\fg {};
|
||||
\item $W$ : \og la fiche correspond à une formule \emph{Wok} \fg{} ;
|
||||
\item $B$ : \og la fiche correspond à une formule \emph{Burger} \fg.
|
||||
\end{enumerate}
|
||||
\setlength\parindent{0mm}
|
||||
|
||||
\medskip
|
||||
|
||||
\begin{enumerate}
|
||||
\item Recopier puis compléter l'arbre pondéré
|
||||
|
||||
\begin{center}
|
||||
\begin{tikzpicture}[sloped]
|
||||
\node {.}
|
||||
child {node {$M$}
|
||||
child {node {$W$}
|
||||
edge from parent
|
||||
node[above] {...}
|
||||
}
|
||||
child {node {$B$}
|
||||
edge from parent
|
||||
node[above] {...}
|
||||
}
|
||||
edge from parent
|
||||
node[above] {...}
|
||||
}
|
||||
child[missing] {}
|
||||
child { node {$S$}
|
||||
child {node {$W$}
|
||||
edge from parent
|
||||
node[above] {...}
|
||||
}
|
||||
child {node {$B$}
|
||||
edge from parent
|
||||
node[above] {...}
|
||||
}
|
||||
edge from parent
|
||||
node[above] {...}
|
||||
} ;
|
||||
\end{tikzpicture}
|
||||
\end{center}
|
||||
|
||||
\item Calculer la probabilité de l'évènement $M \cap W$. Interpréter ce résultat dans le contexte de l'exercice.
|
||||
\item Montrer que la probabilité que la fiche choisie corresponde à une formule \emph{Burger} est égale à $0.6574$.
|
||||
\item On a prélevé une fiche correspondant à la formule \emph{Burger}. Quelle est la probabilité, arrondie au millième, que la vente ait eu lieu le soir?
|
||||
\end{enumerate}
|
||||
\end{exercise}
|
||||
|
||||
\begin{solution}
|
||||
\begin{enumerate}
|
||||
\item
|
||||
\begin{center}
|
||||
\begin{tikzpicture}[sloped]
|
||||
\node {.}
|
||||
child {node {$M$}
|
||||
child {node {$W$}
|
||||
edge from parent
|
||||
node[above] {$0.75$}
|
||||
}
|
||||
child {node {$B$}
|
||||
edge from parent
|
||||
node[above] {$0.25$}
|
||||
}
|
||||
edge from parent
|
||||
node[above] {$0.03$}
|
||||
}
|
||||
child[missing] {}
|
||||
child { node {$S$}
|
||||
child {node {$W$}
|
||||
edge from parent
|
||||
node[above] {$0.33$}
|
||||
}
|
||||
child {node {$B$}
|
||||
edge from parent
|
||||
node[above] {$0.67$}
|
||||
}
|
||||
edge from parent
|
||||
node[above] {$0.97$}
|
||||
} ;
|
||||
\end{tikzpicture}
|
||||
\end{center}
|
||||
\item On calcule la probabilité que la vente soit un wok et ait eu lieu à midi
|
||||
\[ P(M\cap W) = P(M) \times P_M(W) = 0.03 \times 0.75 = 0.0225 \]
|
||||
\item Probabilité que la vente soit un burger.
|
||||
\[
|
||||
P(B) = P(M\cap B) + P(S\cap B) = 0.03 \times 0.75 + 0.97 \times 0.33 = 0.6574
|
||||
\]
|
||||
\item On cherche à calculer la quantité $P_B(S)$. Pour cela on utilise la formule de Bayes
|
||||
\[
|
||||
P_B(S) = \frac{P(B\cap S)}{P(B)} = \frac{P_S(B) \times P(S)}{P(B)} = \frac{0.67\times 0.97}{0.6574} = 0.9885914207484029 \approx 0.989
|
||||
\]
|
||||
\end{enumerate}
|
||||
\end{solution}
|
||||
|
||||
\begin{exercise}[subtitle={Continent plastique}]
|
||||
\textit{Les quantités évoqués dans cette exercice sont générés au hasard et sont donc complètement farfelus.}
|
||||
\medskip
|
||||
Le \og continent de plastique\fg{} est la plus grande des plaques de déchets plastiques évoluant sur les océans. Elle occupe actuellement dans l'océan Pacifique une surface dont l'aire est évaluée à plus de $1,6$ million de km$^2$, entre Hawaï et la Californie.
|
||||
|
||||
En 2017, des scientifiques ont estimé qu'il y avait $18$ millions de tonnes de déchets plastiques qui était déversé chaque année dans les océans et que cette quantité augmentait de $25\n\%$ par chaque année.
|
||||
|
||||
On modélise l'évolution de la masse de ces déchets plastiques déversée chaque année, si rien n'est fait pour la réduire, par une suite géométrique $\left(u_n\right)$. L'arrondi au centième du terme $u_n$ représente la masse de ces déchets déversée chaque année, exprimée en million de tonnes, pour l'année $(2017 + n)$.
|
||||
|
||||
\medskip
|
||||
|
||||
\begin{enumerate}
|
||||
\item Expliquer pourquoi la suite $u_n$ est géométrique?
|
||||
\item Calculer $u_1$ et $u_2$.
|
||||
\item Exprimer $u_n$ en fonction de $n$.
|
||||
\item Au début de l'année 2017, il y avait $300$ millions de tonnes de déchets plastique. Calculer la quantité totale de déchets plastiques en 2030.
|
||||
\item On souhaite déterminer en quelle année la masse totale de ces déchets plastiques aura pour la première fois augmenté de $50$\,\% par rapport à sa valeur de 2017.
|
||||
\begin{enumerate}
|
||||
\item Recopier et compléter l'algorithme ci-dessous pour que la variable $N$ contienne la réponse au problème posé.
|
||||
|
||||
\begin{center}
|
||||
\begin{tabularx}{0.4\linewidth}{|X|}\hline
|
||||
$N = 2017$\\
|
||||
$U = 18$ \\
|
||||
$S = 300 + U$ \\
|
||||
while $S < 450$: \\
|
||||
\hspace{1cm} $N = \ldots$\\
|
||||
\hspace{1cm} $U = \ldots$\\
|
||||
\hspace{1cm} $S = \ldots$\\
|
||||
\hline
|
||||
\end{tabularx}
|
||||
\end{center}
|
||||
\item Que contiennent les variables $S$, $U$ et $N$ après exécution de cet algorithme ?
|
||||
|
||||
Interpréter les résultats dans le contexte de l'exercice.
|
||||
\end{enumerate}
|
||||
\end{enumerate}
|
||||
\end{exercise}
|
||||
|
||||
\begin{solution}
|
||||
\begin{enumerate}
|
||||
\item Une augmentation de $25\,\%$ revient à multiplier la quantité par $1.25$. La suite est donc bien géométrique. Son premier terme est $u_0 = 18$ et sa raison est $q = 1.25$
|
||||
\item
|
||||
\[
|
||||
u_1 = u_0 * 1.25 = 22.5
|
||||
\]
|
||||
\[
|
||||
u_2 = u_0 * 1.25^2 = 28.125
|
||||
\]
|
||||
\item
|
||||
\[
|
||||
u_n = u_0 \times q^n = 18 \times 1.25^n
|
||||
\]
|
||||
\item On calcule la quantité totale déversée entre 2017 et 2030.
|
||||
\[
|
||||
\sum_{n = 0}^{13} u_n = u_0 \times \frac{1-q^{13}}{1-q} = 18 \times \frac{1 - 1.25^{13}}{1 - 1.25} = 1237.67
|
||||
\]
|
||||
On en déduit la quantité totale de déchets en 2030
|
||||
\[
|
||||
300 + 1237.67 = 1537.67
|
||||
\]
|
||||
\item
|
||||
\begin{enumerate}
|
||||
\item ~
|
||||
\begin{center}
|
||||
\begin{tabularx}{0.4\linewidth}{|X|}\hline
|
||||
$N \gets 2017$\\
|
||||
$U \gets 18$ \\
|
||||
$S \gets 300 + U$ \\
|
||||
Tant que $S < 450$ \\
|
||||
\hspace{1cm} $N \gets N + 1$\\
|
||||
\hspace{1cm} $U \gets U * 1.25$\\
|
||||
\hspace{1cm} $S \gets S + u$\\
|
||||
Fin Tant que\\\hline
|
||||
\end{tabularx}
|
||||
\end{center}
|
||||
\item \textit{Pas de correction automatisé}
|
||||
\end{enumerate}
|
||||
\end{enumerate}
|
||||
\end{solution}
|
||||
|
||||
\end{document}
|
||||
|
||||
%%% Local Variables:
|
||||
%%% mode: latex
|
||||
%%% TeX-master: "master"
|
||||
%%% End:
|
262
TST/DS/DS_21_04_07/TST1/corr_11_210407_DS8.tex
Normal file
262
TST/DS/DS_21_04_07/TST1/corr_11_210407_DS8.tex
Normal file
@ -0,0 +1,262 @@
|
||||
\documentclass[a4paper,10pt]{article}
|
||||
\usepackage{myXsim}
|
||||
|
||||
% Title Page
|
||||
\title{DS8 \hfill FERREIRA Léo}
|
||||
\tribe{TST}
|
||||
\date{\hfillÀ render pour le Mercredi 7 avril}
|
||||
|
||||
\xsimsetup{
|
||||
solution/print = true
|
||||
}
|
||||
|
||||
\begin{document}
|
||||
\maketitle
|
||||
|
||||
\begin{exercise}[subtitle={Automatismes}]
|
||||
\textit{Toutes les questions de cette exercice sont indépendantes et peuvent être répondus séparément}
|
||||
\begin{enumerate}
|
||||
\item De janvier à septembre, une quantité a augmenté de $10\,\%$. Faire un schéma pour représenter la situation puis calculer le taux d'évolution moyen mensuel.
|
||||
\item Une quantité augmente de $10\,\%$ par ans. En 2020, elle est de 123\euro. Quelle était sa valeur en 2019? Faire un schéma pour représenter la situation.
|
||||
\item Déterminer l'équation de la droite \\
|
||||
\begin{tikzpicture}[xscale=0.8, yscale=0.5]
|
||||
\tkzInit[xmin=-5,xmax=5,xstep=1,
|
||||
ymin=-5,ymax=5,ystep=1]
|
||||
\tkzGrid
|
||||
\tkzAxeXY
|
||||
\tkzFct[domain=-5:5,color=red,very thick]%
|
||||
{1.5*\x -3};
|
||||
\end{tikzpicture}
|
||||
\item Résoudre l'équation $5 \times 0.79^x = 30$
|
||||
\end{enumerate}
|
||||
\end{exercise}
|
||||
|
||||
\begin{solution}
|
||||
\begin{enumerate}
|
||||
\item On veut partager cette évolution en 8 évolutions.
|
||||
\[
|
||||
\left(1 + \frac{10}{100}\right)^{\frac{1}{8}} = 1.012
|
||||
\]
|
||||
Donc le taux d'évolution moyen est
|
||||
\[
|
||||
t_m = 1.012 - 1 = 0.01200000000000001
|
||||
\]
|
||||
\item Coefficient multiplicateur pour revenir en arrière
|
||||
\[
|
||||
CM = (1 + \frac{10}{100})^{-1} = 0.9091
|
||||
\]
|
||||
On en déduit la quantité en 2019
|
||||
\[
|
||||
123 * 0.9091 = 111.8193
|
||||
\]
|
||||
\item L'équation de la droite est
|
||||
\[
|
||||
y = 1.5 x -3
|
||||
\]
|
||||
\item Il faut penser à faire la division à par $5$ avant d'utiliser le log car sinon, on ne peut pas utiliser la formule $\log(a^n) = n\times \log(a)$.
|
||||
|
||||
\[x = \frac{\log(6.0)}{\log(0.79)}\]
|
||||
\end{enumerate}
|
||||
\end{solution}
|
||||
|
||||
\begin{exercise}[subtitle={Restaurant}]
|
||||
Un \emph{food truck}, ouvert le midi et le soir, propose deux types de formules :
|
||||
|
||||
\setlength\parindent{10mm}
|
||||
\begin{itemize}
|
||||
\item la formule \emph{Burger} ;
|
||||
\item la formule \emph{Wok}.
|
||||
\end{itemize}
|
||||
\setlength\parindent{0mm}
|
||||
|
||||
\medskip
|
||||
|
||||
Le gérant a remarqué que 18\,\% de ses ventes ont lieu le midi. Le quart des ventes du midi correspondent à la formule \emph{Burger}, alors que 37\,\% des ventes du soir correspondent à la formule \emph{Wok}.
|
||||
|
||||
Le gérant se constitue un fichier en notant, pour chaque vente, la formule choisie et le moment de cette vente (midi ou soir).
|
||||
|
||||
On prélève une fiche de façon équiprobable. On définit les quatre évènements suivants:
|
||||
|
||||
\begin{enumerate}
|
||||
\item $M$ : \og la fiche correspond à une vente du midi\fg{} ;
|
||||
\item $S$ : \og la fiche correspond à une vente du soir\fg {};
|
||||
\item $W$ : \og la fiche correspond à une formule \emph{Wok} \fg{} ;
|
||||
\item $B$ : \og la fiche correspond à une formule \emph{Burger} \fg.
|
||||
\end{enumerate}
|
||||
\setlength\parindent{0mm}
|
||||
|
||||
\medskip
|
||||
|
||||
\begin{enumerate}
|
||||
\item Recopier puis compléter l'arbre pondéré
|
||||
|
||||
\begin{center}
|
||||
\begin{tikzpicture}[sloped]
|
||||
\node {.}
|
||||
child {node {$M$}
|
||||
child {node {$W$}
|
||||
edge from parent
|
||||
node[above] {...}
|
||||
}
|
||||
child {node {$B$}
|
||||
edge from parent
|
||||
node[above] {...}
|
||||
}
|
||||
edge from parent
|
||||
node[above] {...}
|
||||
}
|
||||
child[missing] {}
|
||||
child { node {$S$}
|
||||
child {node {$W$}
|
||||
edge from parent
|
||||
node[above] {...}
|
||||
}
|
||||
child {node {$B$}
|
||||
edge from parent
|
||||
node[above] {...}
|
||||
}
|
||||
edge from parent
|
||||
node[above] {...}
|
||||
} ;
|
||||
\end{tikzpicture}
|
||||
\end{center}
|
||||
|
||||
\item Calculer la probabilité de l'évènement $M \cap W$. Interpréter ce résultat dans le contexte de l'exercice.
|
||||
\item Montrer que la probabilité que la fiche choisie corresponde à une formule \emph{Burger} est égale à $0.5616$.
|
||||
\item On a prélevé une fiche correspondant à la formule \emph{Burger}. Quelle est la probabilité, arrondie au millième, que la vente ait eu lieu le soir?
|
||||
\end{enumerate}
|
||||
\end{exercise}
|
||||
|
||||
\begin{solution}
|
||||
\begin{enumerate}
|
||||
\item
|
||||
\begin{center}
|
||||
\begin{tikzpicture}[sloped]
|
||||
\node {.}
|
||||
child {node {$M$}
|
||||
child {node {$W$}
|
||||
edge from parent
|
||||
node[above] {$0.75$}
|
||||
}
|
||||
child {node {$B$}
|
||||
edge from parent
|
||||
node[above] {$0.25$}
|
||||
}
|
||||
edge from parent
|
||||
node[above] {$0.18$}
|
||||
}
|
||||
child[missing] {}
|
||||
child { node {$S$}
|
||||
child {node {$W$}
|
||||
edge from parent
|
||||
node[above] {$0.37$}
|
||||
}
|
||||
child {node {$B$}
|
||||
edge from parent
|
||||
node[above] {$0.63$}
|
||||
}
|
||||
edge from parent
|
||||
node[above] {$0.82$}
|
||||
} ;
|
||||
\end{tikzpicture}
|
||||
\end{center}
|
||||
\item On calcule la probabilité que la vente soit un wok et ait eu lieu à midi
|
||||
\[ P(M\cap W) = P(M) \times P_M(W) = 0.18 \times 0.75 = 0.135 \]
|
||||
\item Probabilité que la vente soit un burger.
|
||||
\[
|
||||
P(B) = P(M\cap B) + P(S\cap B) = 0.18 \times 0.75 + 0.82 \times 0.37 = 0.5616
|
||||
\]
|
||||
\item On cherche à calculer la quantité $P_B(S)$. Pour cela on utilise la formule de Bayes
|
||||
\[
|
||||
P_B(S) = \frac{P(B\cap S)}{P(B)} = \frac{P_S(B) \times P(S)}{P(B)} = \frac{0.63\times 0.82}{0.5616} = 0.9198717948717948 \approx 0.92
|
||||
\]
|
||||
\end{enumerate}
|
||||
\end{solution}
|
||||
|
||||
\begin{exercise}[subtitle={Continent plastique}]
|
||||
\textit{Les quantités évoqués dans cette exercice sont générés au hasard et sont donc complètement farfelus.}
|
||||
\medskip
|
||||
Le \og continent de plastique\fg{} est la plus grande des plaques de déchets plastiques évoluant sur les océans. Elle occupe actuellement dans l'océan Pacifique une surface dont l'aire est évaluée à plus de $1,6$ million de km$^2$, entre Hawaï et la Californie.
|
||||
|
||||
En 2017, des scientifiques ont estimé qu'il y avait $3$ millions de tonnes de déchets plastiques qui était déversé chaque année dans les océans et que cette quantité augmentait de $19\n\%$ par chaque année.
|
||||
|
||||
On modélise l'évolution de la masse de ces déchets plastiques déversée chaque année, si rien n'est fait pour la réduire, par une suite géométrique $\left(u_n\right)$. L'arrondi au centième du terme $u_n$ représente la masse de ces déchets déversée chaque année, exprimée en million de tonnes, pour l'année $(2017 + n)$.
|
||||
|
||||
\medskip
|
||||
|
||||
\begin{enumerate}
|
||||
\item Expliquer pourquoi la suite $u_n$ est géométrique?
|
||||
\item Calculer $u_1$ et $u_2$.
|
||||
\item Exprimer $u_n$ en fonction de $n$.
|
||||
\item Au début de l'année 2017, il y avait $300$ millions de tonnes de déchets plastique. Calculer la quantité totale de déchets plastiques en 2030.
|
||||
\item On souhaite déterminer en quelle année la masse totale de ces déchets plastiques aura pour la première fois augmenté de $50$\,\% par rapport à sa valeur de 2017.
|
||||
\begin{enumerate}
|
||||
\item Recopier et compléter l'algorithme ci-dessous pour que la variable $N$ contienne la réponse au problème posé.
|
||||
|
||||
\begin{center}
|
||||
\begin{tabularx}{0.4\linewidth}{|X|}\hline
|
||||
$N = 2017$\\
|
||||
$U = 3$ \\
|
||||
$S = 300 + U$ \\
|
||||
while $S < 450$: \\
|
||||
\hspace{1cm} $N = \ldots$\\
|
||||
\hspace{1cm} $U = \ldots$\\
|
||||
\hspace{1cm} $S = \ldots$\\
|
||||
\hline
|
||||
\end{tabularx}
|
||||
\end{center}
|
||||
\item Que contiennent les variables $S$, $U$ et $N$ après exécution de cet algorithme ?
|
||||
|
||||
Interpréter les résultats dans le contexte de l'exercice.
|
||||
\end{enumerate}
|
||||
\end{enumerate}
|
||||
\end{exercise}
|
||||
|
||||
\begin{solution}
|
||||
\begin{enumerate}
|
||||
\item Une augmentation de $19\,\%$ revient à multiplier la quantité par $1.19$. La suite est donc bien géométrique. Son premier terme est $u_0 = 3$ et sa raison est $q = 1.19$
|
||||
\item
|
||||
\[
|
||||
u_1 = u_0 * 1.19 = 3.57
|
||||
\]
|
||||
\[
|
||||
u_2 = u_0 * 1.19^2 = 4.2483
|
||||
\]
|
||||
\item
|
||||
\[
|
||||
u_n = u_0 \times q^n = 3 \times 1.19^n
|
||||
\]
|
||||
\item On calcule la quantité totale déversée entre 2017 et 2030.
|
||||
\[
|
||||
\sum_{n = 0}^{13} u_n = u_0 \times \frac{1-q^{13}}{1-q} = 3 \times \frac{1 - 1.19^{13}}{1 - 1.19} = 135.73
|
||||
\]
|
||||
On en déduit la quantité totale de déchets en 2030
|
||||
\[
|
||||
300 + 135.73 = 435.73
|
||||
\]
|
||||
\item
|
||||
\begin{enumerate}
|
||||
\item ~
|
||||
\begin{center}
|
||||
\begin{tabularx}{0.4\linewidth}{|X|}\hline
|
||||
$N \gets 2017$\\
|
||||
$U \gets 3$ \\
|
||||
$S \gets 300 + U$ \\
|
||||
Tant que $S < 450$ \\
|
||||
\hspace{1cm} $N \gets N + 1$\\
|
||||
\hspace{1cm} $U \gets U * 1.19$\\
|
||||
\hspace{1cm} $S \gets S + u$\\
|
||||
Fin Tant que\\\hline
|
||||
\end{tabularx}
|
||||
\end{center}
|
||||
\item \textit{Pas de correction automatisé}
|
||||
\end{enumerate}
|
||||
\end{enumerate}
|
||||
\end{solution}
|
||||
|
||||
\end{document}
|
||||
|
||||
%%% Local Variables:
|
||||
%%% mode: latex
|
||||
%%% TeX-master: "master"
|
||||
%%% End:
|
262
TST/DS/DS_21_04_07/TST1/corr_12_210407_DS8.tex
Normal file
262
TST/DS/DS_21_04_07/TST1/corr_12_210407_DS8.tex
Normal file
@ -0,0 +1,262 @@
|
||||
\documentclass[a4paper,10pt]{article}
|
||||
\usepackage{myXsim}
|
||||
|
||||
% Title Page
|
||||
\title{DS8 \hfill FILALI Zakaria}
|
||||
\tribe{TST}
|
||||
\date{\hfillÀ render pour le Mercredi 7 avril}
|
||||
|
||||
\xsimsetup{
|
||||
solution/print = true
|
||||
}
|
||||
|
||||
\begin{document}
|
||||
\maketitle
|
||||
|
||||
\begin{exercise}[subtitle={Automatismes}]
|
||||
\textit{Toutes les questions de cette exercice sont indépendantes et peuvent être répondus séparément}
|
||||
\begin{enumerate}
|
||||
\item De janvier à septembre, une quantité a augmenté de $19\,\%$. Faire un schéma pour représenter la situation puis calculer le taux d'évolution moyen mensuel.
|
||||
\item Une quantité augmente de $19\,\%$ par ans. En 2020, elle est de 122\euro. Quelle était sa valeur en 2019? Faire un schéma pour représenter la situation.
|
||||
\item Déterminer l'équation de la droite \\
|
||||
\begin{tikzpicture}[xscale=0.8, yscale=0.5]
|
||||
\tkzInit[xmin=-5,xmax=5,xstep=1,
|
||||
ymin=-5,ymax=5,ystep=1]
|
||||
\tkzGrid
|
||||
\tkzAxeXY
|
||||
\tkzFct[domain=-5:5,color=red,very thick]%
|
||||
{1.0*\x -2};
|
||||
\end{tikzpicture}
|
||||
\item Résoudre l'équation $8 \times 0.25^x = 25$
|
||||
\end{enumerate}
|
||||
\end{exercise}
|
||||
|
||||
\begin{solution}
|
||||
\begin{enumerate}
|
||||
\item On veut partager cette évolution en 8 évolutions.
|
||||
\[
|
||||
\left(1 + \frac{19}{100}\right)^{\frac{1}{8}} = 1.022
|
||||
\]
|
||||
Donc le taux d'évolution moyen est
|
||||
\[
|
||||
t_m = 1.022 - 1 = 0.02200000000000002
|
||||
\]
|
||||
\item Coefficient multiplicateur pour revenir en arrière
|
||||
\[
|
||||
CM = (1 + \frac{19}{100})^{-1} = 0.8403
|
||||
\]
|
||||
On en déduit la quantité en 2019
|
||||
\[
|
||||
122 * 0.8403 = 102.51660000000001
|
||||
\]
|
||||
\item L'équation de la droite est
|
||||
\[
|
||||
y = 1.0 x -2
|
||||
\]
|
||||
\item Il faut penser à faire la division à par $8$ avant d'utiliser le log car sinon, on ne peut pas utiliser la formule $\log(a^n) = n\times \log(a)$.
|
||||
|
||||
\[x = \frac{\log(3.12)}{\log(0.25)}\]
|
||||
\end{enumerate}
|
||||
\end{solution}
|
||||
|
||||
\begin{exercise}[subtitle={Restaurant}]
|
||||
Un \emph{food truck}, ouvert le midi et le soir, propose deux types de formules :
|
||||
|
||||
\setlength\parindent{10mm}
|
||||
\begin{itemize}
|
||||
\item la formule \emph{Burger} ;
|
||||
\item la formule \emph{Wok}.
|
||||
\end{itemize}
|
||||
\setlength\parindent{0mm}
|
||||
|
||||
\medskip
|
||||
|
||||
Le gérant a remarqué que 47\,\% de ses ventes ont lieu le midi. Le quart des ventes du midi correspondent à la formule \emph{Burger}, alors que 69\,\% des ventes du soir correspondent à la formule \emph{Wok}.
|
||||
|
||||
Le gérant se constitue un fichier en notant, pour chaque vente, la formule choisie et le moment de cette vente (midi ou soir).
|
||||
|
||||
On prélève une fiche de façon équiprobable. On définit les quatre évènements suivants:
|
||||
|
||||
\begin{enumerate}
|
||||
\item $M$ : \og la fiche correspond à une vente du midi\fg{} ;
|
||||
\item $S$ : \og la fiche correspond à une vente du soir\fg {};
|
||||
\item $W$ : \og la fiche correspond à une formule \emph{Wok} \fg{} ;
|
||||
\item $B$ : \og la fiche correspond à une formule \emph{Burger} \fg.
|
||||
\end{enumerate}
|
||||
\setlength\parindent{0mm}
|
||||
|
||||
\medskip
|
||||
|
||||
\begin{enumerate}
|
||||
\item Recopier puis compléter l'arbre pondéré
|
||||
|
||||
\begin{center}
|
||||
\begin{tikzpicture}[sloped]
|
||||
\node {.}
|
||||
child {node {$M$}
|
||||
child {node {$W$}
|
||||
edge from parent
|
||||
node[above] {...}
|
||||
}
|
||||
child {node {$B$}
|
||||
edge from parent
|
||||
node[above] {...}
|
||||
}
|
||||
edge from parent
|
||||
node[above] {...}
|
||||
}
|
||||
child[missing] {}
|
||||
child { node {$S$}
|
||||
child {node {$W$}
|
||||
edge from parent
|
||||
node[above] {...}
|
||||
}
|
||||
child {node {$B$}
|
||||
edge from parent
|
||||
node[above] {...}
|
||||
}
|
||||
edge from parent
|
||||
node[above] {...}
|
||||
} ;
|
||||
\end{tikzpicture}
|
||||
\end{center}
|
||||
|
||||
\item Calculer la probabilité de l'évènement $M \cap W$. Interpréter ce résultat dans le contexte de l'exercice.
|
||||
\item Montrer que la probabilité que la fiche choisie corresponde à une formule \emph{Burger} est égale à $0.2818$.
|
||||
\item On a prélevé une fiche correspondant à la formule \emph{Burger}. Quelle est la probabilité, arrondie au millième, que la vente ait eu lieu le soir?
|
||||
\end{enumerate}
|
||||
\end{exercise}
|
||||
|
||||
\begin{solution}
|
||||
\begin{enumerate}
|
||||
\item
|
||||
\begin{center}
|
||||
\begin{tikzpicture}[sloped]
|
||||
\node {.}
|
||||
child {node {$M$}
|
||||
child {node {$W$}
|
||||
edge from parent
|
||||
node[above] {$0.75$}
|
||||
}
|
||||
child {node {$B$}
|
||||
edge from parent
|
||||
node[above] {$0.25$}
|
||||
}
|
||||
edge from parent
|
||||
node[above] {$0.47$}
|
||||
}
|
||||
child[missing] {}
|
||||
child { node {$S$}
|
||||
child {node {$W$}
|
||||
edge from parent
|
||||
node[above] {$0.69$}
|
||||
}
|
||||
child {node {$B$}
|
||||
edge from parent
|
||||
node[above] {$0.31$}
|
||||
}
|
||||
edge from parent
|
||||
node[above] {$0.53$}
|
||||
} ;
|
||||
\end{tikzpicture}
|
||||
\end{center}
|
||||
\item On calcule la probabilité que la vente soit un wok et ait eu lieu à midi
|
||||
\[ P(M\cap W) = P(M) \times P_M(W) = 0.47 \times 0.75 = 0.3525 \]
|
||||
\item Probabilité que la vente soit un burger.
|
||||
\[
|
||||
P(B) = P(M\cap B) + P(S\cap B) = 0.47 \times 0.75 + 0.53 \times 0.69 = 0.2818
|
||||
\]
|
||||
\item On cherche à calculer la quantité $P_B(S)$. Pour cela on utilise la formule de Bayes
|
||||
\[
|
||||
P_B(S) = \frac{P(B\cap S)}{P(B)} = \frac{P_S(B) \times P(S)}{P(B)} = \frac{0.31\times 0.53}{0.2818} = 0.5830376153300213 \approx 0.583
|
||||
\]
|
||||
\end{enumerate}
|
||||
\end{solution}
|
||||
|
||||
\begin{exercise}[subtitle={Continent plastique}]
|
||||
\textit{Les quantités évoqués dans cette exercice sont générés au hasard et sont donc complètement farfelus.}
|
||||
\medskip
|
||||
Le \og continent de plastique\fg{} est la plus grande des plaques de déchets plastiques évoluant sur les océans. Elle occupe actuellement dans l'océan Pacifique une surface dont l'aire est évaluée à plus de $1,6$ million de km$^2$, entre Hawaï et la Californie.
|
||||
|
||||
En 2017, des scientifiques ont estimé qu'il y avait $11$ millions de tonnes de déchets plastiques qui était déversé chaque année dans les océans et que cette quantité augmentait de $16\n\%$ par chaque année.
|
||||
|
||||
On modélise l'évolution de la masse de ces déchets plastiques déversée chaque année, si rien n'est fait pour la réduire, par une suite géométrique $\left(u_n\right)$. L'arrondi au centième du terme $u_n$ représente la masse de ces déchets déversée chaque année, exprimée en million de tonnes, pour l'année $(2017 + n)$.
|
||||
|
||||
\medskip
|
||||
|
||||
\begin{enumerate}
|
||||
\item Expliquer pourquoi la suite $u_n$ est géométrique?
|
||||
\item Calculer $u_1$ et $u_2$.
|
||||
\item Exprimer $u_n$ en fonction de $n$.
|
||||
\item Au début de l'année 2017, il y avait $300$ millions de tonnes de déchets plastique. Calculer la quantité totale de déchets plastiques en 2030.
|
||||
\item On souhaite déterminer en quelle année la masse totale de ces déchets plastiques aura pour la première fois augmenté de $50$\,\% par rapport à sa valeur de 2017.
|
||||
\begin{enumerate}
|
||||
\item Recopier et compléter l'algorithme ci-dessous pour que la variable $N$ contienne la réponse au problème posé.
|
||||
|
||||
\begin{center}
|
||||
\begin{tabularx}{0.4\linewidth}{|X|}\hline
|
||||
$N = 2017$\\
|
||||
$U = 11$ \\
|
||||
$S = 300 + U$ \\
|
||||
while $S < 450$: \\
|
||||
\hspace{1cm} $N = \ldots$\\
|
||||
\hspace{1cm} $U = \ldots$\\
|
||||
\hspace{1cm} $S = \ldots$\\
|
||||
\hline
|
||||
\end{tabularx}
|
||||
\end{center}
|
||||
\item Que contiennent les variables $S$, $U$ et $N$ après exécution de cet algorithme ?
|
||||
|
||||
Interpréter les résultats dans le contexte de l'exercice.
|
||||
\end{enumerate}
|
||||
\end{enumerate}
|
||||
\end{exercise}
|
||||
|
||||
\begin{solution}
|
||||
\begin{enumerate}
|
||||
\item Une augmentation de $16\,\%$ revient à multiplier la quantité par $1.16$. La suite est donc bien géométrique. Son premier terme est $u_0 = 11$ et sa raison est $q = 1.16$
|
||||
\item
|
||||
\[
|
||||
u_1 = u_0 * 1.16 = 12.76
|
||||
\]
|
||||
\[
|
||||
u_2 = u_0 * 1.16^2 = 14.8016
|
||||
\]
|
||||
\item
|
||||
\[
|
||||
u_n = u_0 \times q^n = 11 \times 1.16^n
|
||||
\]
|
||||
\item On calcule la quantité totale déversée entre 2017 et 2030.
|
||||
\[
|
||||
\sum_{n = 0}^{13} u_n = u_0 \times \frac{1-q^{13}}{1-q} = 11 \times \frac{1 - 1.16^{13}}{1 - 1.16} = 404.65
|
||||
\]
|
||||
On en déduit la quantité totale de déchets en 2030
|
||||
\[
|
||||
300 + 404.65 = 704.65
|
||||
\]
|
||||
\item
|
||||
\begin{enumerate}
|
||||
\item ~
|
||||
\begin{center}
|
||||
\begin{tabularx}{0.4\linewidth}{|X|}\hline
|
||||
$N \gets 2017$\\
|
||||
$U \gets 11$ \\
|
||||
$S \gets 300 + U$ \\
|
||||
Tant que $S < 450$ \\
|
||||
\hspace{1cm} $N \gets N + 1$\\
|
||||
\hspace{1cm} $U \gets U * 1.16$\\
|
||||
\hspace{1cm} $S \gets S + u$\\
|
||||
Fin Tant que\\\hline
|
||||
\end{tabularx}
|
||||
\end{center}
|
||||
\item \textit{Pas de correction automatisé}
|
||||
\end{enumerate}
|
||||
\end{enumerate}
|
||||
\end{solution}
|
||||
|
||||
\end{document}
|
||||
|
||||
%%% Local Variables:
|
||||
%%% mode: latex
|
||||
%%% TeX-master: "master"
|
||||
%%% End:
|
262
TST/DS/DS_21_04_07/TST1/corr_13_210407_DS8.tex
Normal file
262
TST/DS/DS_21_04_07/TST1/corr_13_210407_DS8.tex
Normal file
@ -0,0 +1,262 @@
|
||||
\documentclass[a4paper,10pt]{article}
|
||||
\usepackage{myXsim}
|
||||
|
||||
% Title Page
|
||||
\title{DS8 \hfill FOIGNY Romain}
|
||||
\tribe{TST}
|
||||
\date{\hfillÀ render pour le Mercredi 7 avril}
|
||||
|
||||
\xsimsetup{
|
||||
solution/print = true
|
||||
}
|
||||
|
||||
\begin{document}
|
||||
\maketitle
|
||||
|
||||
\begin{exercise}[subtitle={Automatismes}]
|
||||
\textit{Toutes les questions de cette exercice sont indépendantes et peuvent être répondus séparément}
|
||||
\begin{enumerate}
|
||||
\item De janvier à septembre, une quantité a augmenté de $22\,\%$. Faire un schéma pour représenter la situation puis calculer le taux d'évolution moyen mensuel.
|
||||
\item Une quantité augmente de $22\,\%$ par ans. En 2020, elle est de 125\euro. Quelle était sa valeur en 2019? Faire un schéma pour représenter la situation.
|
||||
\item Déterminer l'équation de la droite \\
|
||||
\begin{tikzpicture}[xscale=0.8, yscale=0.5]
|
||||
\tkzInit[xmin=-5,xmax=5,xstep=1,
|
||||
ymin=-5,ymax=5,ystep=1]
|
||||
\tkzGrid
|
||||
\tkzAxeXY
|
||||
\tkzFct[domain=-5:5,color=red,very thick]%
|
||||
{2.6666666666666665*\x -4};
|
||||
\end{tikzpicture}
|
||||
\item Résoudre l'équation $9 \times 0.35^x = 44$
|
||||
\end{enumerate}
|
||||
\end{exercise}
|
||||
|
||||
\begin{solution}
|
||||
\begin{enumerate}
|
||||
\item On veut partager cette évolution en 8 évolutions.
|
||||
\[
|
||||
\left(1 + \frac{22}{100}\right)^{\frac{1}{8}} = 1.0252
|
||||
\]
|
||||
Donc le taux d'évolution moyen est
|
||||
\[
|
||||
t_m = 1.0252 - 1 = 0.02519999999999989
|
||||
\]
|
||||
\item Coefficient multiplicateur pour revenir en arrière
|
||||
\[
|
||||
CM = (1 + \frac{22}{100})^{-1} = 0.8197
|
||||
\]
|
||||
On en déduit la quantité en 2019
|
||||
\[
|
||||
125 * 0.8197 = 102.46249999999999
|
||||
\]
|
||||
\item L'équation de la droite est
|
||||
\[
|
||||
y = 2.6666666666666665 x -4
|
||||
\]
|
||||
\item Il faut penser à faire la division à par $9$ avant d'utiliser le log car sinon, on ne peut pas utiliser la formule $\log(a^n) = n\times \log(a)$.
|
||||
|
||||
\[x = \frac{\log(4.89)}{\log(0.35)}\]
|
||||
\end{enumerate}
|
||||
\end{solution}
|
||||
|
||||
\begin{exercise}[subtitle={Restaurant}]
|
||||
Un \emph{food truck}, ouvert le midi et le soir, propose deux types de formules :
|
||||
|
||||
\setlength\parindent{10mm}
|
||||
\begin{itemize}
|
||||
\item la formule \emph{Burger} ;
|
||||
\item la formule \emph{Wok}.
|
||||
\end{itemize}
|
||||
\setlength\parindent{0mm}
|
||||
|
||||
\medskip
|
||||
|
||||
Le gérant a remarqué que 14\,\% de ses ventes ont lieu le midi. Le quart des ventes du midi correspondent à la formule \emph{Burger}, alors que 17\,\% des ventes du soir correspondent à la formule \emph{Wok}.
|
||||
|
||||
Le gérant se constitue un fichier en notant, pour chaque vente, la formule choisie et le moment de cette vente (midi ou soir).
|
||||
|
||||
On prélève une fiche de façon équiprobable. On définit les quatre évènements suivants:
|
||||
|
||||
\begin{enumerate}
|
||||
\item $M$ : \og la fiche correspond à une vente du midi\fg{} ;
|
||||
\item $S$ : \og la fiche correspond à une vente du soir\fg {};
|
||||
\item $W$ : \og la fiche correspond à une formule \emph{Wok} \fg{} ;
|
||||
\item $B$ : \og la fiche correspond à une formule \emph{Burger} \fg.
|
||||
\end{enumerate}
|
||||
\setlength\parindent{0mm}
|
||||
|
||||
\medskip
|
||||
|
||||
\begin{enumerate}
|
||||
\item Recopier puis compléter l'arbre pondéré
|
||||
|
||||
\begin{center}
|
||||
\begin{tikzpicture}[sloped]
|
||||
\node {.}
|
||||
child {node {$M$}
|
||||
child {node {$W$}
|
||||
edge from parent
|
||||
node[above] {...}
|
||||
}
|
||||
child {node {$B$}
|
||||
edge from parent
|
||||
node[above] {...}
|
||||
}
|
||||
edge from parent
|
||||
node[above] {...}
|
||||
}
|
||||
child[missing] {}
|
||||
child { node {$S$}
|
||||
child {node {$W$}
|
||||
edge from parent
|
||||
node[above] {...}
|
||||
}
|
||||
child {node {$B$}
|
||||
edge from parent
|
||||
node[above] {...}
|
||||
}
|
||||
edge from parent
|
||||
node[above] {...}
|
||||
} ;
|
||||
\end{tikzpicture}
|
||||
\end{center}
|
||||
|
||||
\item Calculer la probabilité de l'évènement $M \cap W$. Interpréter ce résultat dans le contexte de l'exercice.
|
||||
\item Montrer que la probabilité que la fiche choisie corresponde à une formule \emph{Burger} est égale à $0.7488$.
|
||||
\item On a prélevé une fiche correspondant à la formule \emph{Burger}. Quelle est la probabilité, arrondie au millième, que la vente ait eu lieu le soir?
|
||||
\end{enumerate}
|
||||
\end{exercise}
|
||||
|
||||
\begin{solution}
|
||||
\begin{enumerate}
|
||||
\item
|
||||
\begin{center}
|
||||
\begin{tikzpicture}[sloped]
|
||||
\node {.}
|
||||
child {node {$M$}
|
||||
child {node {$W$}
|
||||
edge from parent
|
||||
node[above] {$0.75$}
|
||||
}
|
||||
child {node {$B$}
|
||||
edge from parent
|
||||
node[above] {$0.25$}
|
||||
}
|
||||
edge from parent
|
||||
node[above] {$0.14$}
|
||||
}
|
||||
child[missing] {}
|
||||
child { node {$S$}
|
||||
child {node {$W$}
|
||||
edge from parent
|
||||
node[above] {$0.17$}
|
||||
}
|
||||
child {node {$B$}
|
||||
edge from parent
|
||||
node[above] {$0.83$}
|
||||
}
|
||||
edge from parent
|
||||
node[above] {$0.86$}
|
||||
} ;
|
||||
\end{tikzpicture}
|
||||
\end{center}
|
||||
\item On calcule la probabilité que la vente soit un wok et ait eu lieu à midi
|
||||
\[ P(M\cap W) = P(M) \times P_M(W) = 0.14 \times 0.75 = 0.105 \]
|
||||
\item Probabilité que la vente soit un burger.
|
||||
\[
|
||||
P(B) = P(M\cap B) + P(S\cap B) = 0.14 \times 0.75 + 0.86 \times 0.17 = 0.7488
|
||||
\]
|
||||
\item On cherche à calculer la quantité $P_B(S)$. Pour cela on utilise la formule de Bayes
|
||||
\[
|
||||
P_B(S) = \frac{P(B\cap S)}{P(B)} = \frac{P_S(B) \times P(S)}{P(B)} = \frac{0.83\times 0.86}{0.7488} = 0.953258547008547 \approx 0.953
|
||||
\]
|
||||
\end{enumerate}
|
||||
\end{solution}
|
||||
|
||||
\begin{exercise}[subtitle={Continent plastique}]
|
||||
\textit{Les quantités évoqués dans cette exercice sont générés au hasard et sont donc complètement farfelus.}
|
||||
\medskip
|
||||
Le \og continent de plastique\fg{} est la plus grande des plaques de déchets plastiques évoluant sur les océans. Elle occupe actuellement dans l'océan Pacifique une surface dont l'aire est évaluée à plus de $1,6$ million de km$^2$, entre Hawaï et la Californie.
|
||||
|
||||
En 2017, des scientifiques ont estimé qu'il y avait $11$ millions de tonnes de déchets plastiques qui était déversé chaque année dans les océans et que cette quantité augmentait de $12\n\%$ par chaque année.
|
||||
|
||||
On modélise l'évolution de la masse de ces déchets plastiques déversée chaque année, si rien n'est fait pour la réduire, par une suite géométrique $\left(u_n\right)$. L'arrondi au centième du terme $u_n$ représente la masse de ces déchets déversée chaque année, exprimée en million de tonnes, pour l'année $(2017 + n)$.
|
||||
|
||||
\medskip
|
||||
|
||||
\begin{enumerate}
|
||||
\item Expliquer pourquoi la suite $u_n$ est géométrique?
|
||||
\item Calculer $u_1$ et $u_2$.
|
||||
\item Exprimer $u_n$ en fonction de $n$.
|
||||
\item Au début de l'année 2017, il y avait $300$ millions de tonnes de déchets plastique. Calculer la quantité totale de déchets plastiques en 2030.
|
||||
\item On souhaite déterminer en quelle année la masse totale de ces déchets plastiques aura pour la première fois augmenté de $50$\,\% par rapport à sa valeur de 2017.
|
||||
\begin{enumerate}
|
||||
\item Recopier et compléter l'algorithme ci-dessous pour que la variable $N$ contienne la réponse au problème posé.
|
||||
|
||||
\begin{center}
|
||||
\begin{tabularx}{0.4\linewidth}{|X|}\hline
|
||||
$N = 2017$\\
|
||||
$U = 11$ \\
|
||||
$S = 300 + U$ \\
|
||||
while $S < 450$: \\
|
||||
\hspace{1cm} $N = \ldots$\\
|
||||
\hspace{1cm} $U = \ldots$\\
|
||||
\hspace{1cm} $S = \ldots$\\
|
||||
\hline
|
||||
\end{tabularx}
|
||||
\end{center}
|
||||
\item Que contiennent les variables $S$, $U$ et $N$ après exécution de cet algorithme ?
|
||||
|
||||
Interpréter les résultats dans le contexte de l'exercice.
|
||||
\end{enumerate}
|
||||
\end{enumerate}
|
||||
\end{exercise}
|
||||
|
||||
\begin{solution}
|
||||
\begin{enumerate}
|
||||
\item Une augmentation de $12\,\%$ revient à multiplier la quantité par $1.12$. La suite est donc bien géométrique. Son premier terme est $u_0 = 11$ et sa raison est $q = 1.12$
|
||||
\item
|
||||
\[
|
||||
u_1 = u_0 * 1.12 = 12.32
|
||||
\]
|
||||
\[
|
||||
u_2 = u_0 * 1.12^2 = 13.7984
|
||||
\]
|
||||
\item
|
||||
\[
|
||||
u_n = u_0 \times q^n = 11 \times 1.12^n
|
||||
\]
|
||||
\item On calcule la quantité totale déversée entre 2017 et 2030.
|
||||
\[
|
||||
\sum_{n = 0}^{13} u_n = u_0 \times \frac{1-q^{13}}{1-q} = 11 \times \frac{1 - 1.12^{13}}{1 - 1.12} = 308.32
|
||||
\]
|
||||
On en déduit la quantité totale de déchets en 2030
|
||||
\[
|
||||
300 + 308.32 = 608.3199999999999
|
||||
\]
|
||||
\item
|
||||
\begin{enumerate}
|
||||
\item ~
|
||||
\begin{center}
|
||||
\begin{tabularx}{0.4\linewidth}{|X|}\hline
|
||||
$N \gets 2017$\\
|
||||
$U \gets 11$ \\
|
||||
$S \gets 300 + U$ \\
|
||||
Tant que $S < 450$ \\
|
||||
\hspace{1cm} $N \gets N + 1$\\
|
||||
\hspace{1cm} $U \gets U * 1.12$\\
|
||||
\hspace{1cm} $S \gets S + u$\\
|
||||
Fin Tant que\\\hline
|
||||
\end{tabularx}
|
||||
\end{center}
|
||||
\item \textit{Pas de correction automatisé}
|
||||
\end{enumerate}
|
||||
\end{enumerate}
|
||||
\end{solution}
|
||||
|
||||
\end{document}
|
||||
|
||||
%%% Local Variables:
|
||||
%%% mode: latex
|
||||
%%% TeX-master: "master"
|
||||
%%% End:
|
262
TST/DS/DS_21_04_07/TST1/corr_14_210407_DS8.tex
Normal file
262
TST/DS/DS_21_04_07/TST1/corr_14_210407_DS8.tex
Normal file
@ -0,0 +1,262 @@
|
||||
\documentclass[a4paper,10pt]{article}
|
||||
\usepackage{myXsim}
|
||||
|
||||
% Title Page
|
||||
\title{DS8 \hfill HIPOLITO DA SILVA Andréa}
|
||||
\tribe{TST}
|
||||
\date{\hfillÀ render pour le Mercredi 7 avril}
|
||||
|
||||
\xsimsetup{
|
||||
solution/print = true
|
||||
}
|
||||
|
||||
\begin{document}
|
||||
\maketitle
|
||||
|
||||
\begin{exercise}[subtitle={Automatismes}]
|
||||
\textit{Toutes les questions de cette exercice sont indépendantes et peuvent être répondus séparément}
|
||||
\begin{enumerate}
|
||||
\item De janvier à septembre, une quantité a augmenté de $23\,\%$. Faire un schéma pour représenter la situation puis calculer le taux d'évolution moyen mensuel.
|
||||
\item Une quantité augmente de $23\,\%$ par ans. En 2020, elle est de 149\euro. Quelle était sa valeur en 2019? Faire un schéma pour représenter la situation.
|
||||
\item Déterminer l'équation de la droite \\
|
||||
\begin{tikzpicture}[xscale=0.8, yscale=0.5]
|
||||
\tkzInit[xmin=-5,xmax=5,xstep=1,
|
||||
ymin=-5,ymax=5,ystep=1]
|
||||
\tkzGrid
|
||||
\tkzAxeXY
|
||||
\tkzFct[domain=-5:5,color=red,very thick]%
|
||||
{0.6666666666666666*\x -1};
|
||||
\end{tikzpicture}
|
||||
\item Résoudre l'équation $7 \times 0.38^x = 21$
|
||||
\end{enumerate}
|
||||
\end{exercise}
|
||||
|
||||
\begin{solution}
|
||||
\begin{enumerate}
|
||||
\item On veut partager cette évolution en 8 évolutions.
|
||||
\[
|
||||
\left(1 + \frac{23}{100}\right)^{\frac{1}{8}} = 1.0262
|
||||
\]
|
||||
Donc le taux d'évolution moyen est
|
||||
\[
|
||||
t_m = 1.0262 - 1 = 0.0262
|
||||
\]
|
||||
\item Coefficient multiplicateur pour revenir en arrière
|
||||
\[
|
||||
CM = (1 + \frac{23}{100})^{-1} = 0.813
|
||||
\]
|
||||
On en déduit la quantité en 2019
|
||||
\[
|
||||
149 * 0.813 = 121.13699999999999
|
||||
\]
|
||||
\item L'équation de la droite est
|
||||
\[
|
||||
y = 0.6666666666666666 x -1
|
||||
\]
|
||||
\item Il faut penser à faire la division à par $7$ avant d'utiliser le log car sinon, on ne peut pas utiliser la formule $\log(a^n) = n\times \log(a)$.
|
||||
|
||||
\[x = \frac{\log(3.0)}{\log(0.38)}\]
|
||||
\end{enumerate}
|
||||
\end{solution}
|
||||
|
||||
\begin{exercise}[subtitle={Restaurant}]
|
||||
Un \emph{food truck}, ouvert le midi et le soir, propose deux types de formules :
|
||||
|
||||
\setlength\parindent{10mm}
|
||||
\begin{itemize}
|
||||
\item la formule \emph{Burger} ;
|
||||
\item la formule \emph{Wok}.
|
||||
\end{itemize}
|
||||
\setlength\parindent{0mm}
|
||||
|
||||
\medskip
|
||||
|
||||
Le gérant a remarqué que 35\,\% de ses ventes ont lieu le midi. Le quart des ventes du midi correspondent à la formule \emph{Burger}, alors que 13\,\% des ventes du soir correspondent à la formule \emph{Wok}.
|
||||
|
||||
Le gérant se constitue un fichier en notant, pour chaque vente, la formule choisie et le moment de cette vente (midi ou soir).
|
||||
|
||||
On prélève une fiche de façon équiprobable. On définit les quatre évènements suivants:
|
||||
|
||||
\begin{enumerate}
|
||||
\item $M$ : \og la fiche correspond à une vente du midi\fg{} ;
|
||||
\item $S$ : \og la fiche correspond à une vente du soir\fg {};
|
||||
\item $W$ : \og la fiche correspond à une formule \emph{Wok} \fg{} ;
|
||||
\item $B$ : \og la fiche correspond à une formule \emph{Burger} \fg.
|
||||
\end{enumerate}
|
||||
\setlength\parindent{0mm}
|
||||
|
||||
\medskip
|
||||
|
||||
\begin{enumerate}
|
||||
\item Recopier puis compléter l'arbre pondéré
|
||||
|
||||
\begin{center}
|
||||
\begin{tikzpicture}[sloped]
|
||||
\node {.}
|
||||
child {node {$M$}
|
||||
child {node {$W$}
|
||||
edge from parent
|
||||
node[above] {...}
|
||||
}
|
||||
child {node {$B$}
|
||||
edge from parent
|
||||
node[above] {...}
|
||||
}
|
||||
edge from parent
|
||||
node[above] {...}
|
||||
}
|
||||
child[missing] {}
|
||||
child { node {$S$}
|
||||
child {node {$W$}
|
||||
edge from parent
|
||||
node[above] {...}
|
||||
}
|
||||
child {node {$B$}
|
||||
edge from parent
|
||||
node[above] {...}
|
||||
}
|
||||
edge from parent
|
||||
node[above] {...}
|
||||
} ;
|
||||
\end{tikzpicture}
|
||||
\end{center}
|
||||
|
||||
\item Calculer la probabilité de l'évènement $M \cap W$. Interpréter ce résultat dans le contexte de l'exercice.
|
||||
\item Montrer que la probabilité que la fiche choisie corresponde à une formule \emph{Burger} est égale à $0.653$.
|
||||
\item On a prélevé une fiche correspondant à la formule \emph{Burger}. Quelle est la probabilité, arrondie au millième, que la vente ait eu lieu le soir?
|
||||
\end{enumerate}
|
||||
\end{exercise}
|
||||
|
||||
\begin{solution}
|
||||
\begin{enumerate}
|
||||
\item
|
||||
\begin{center}
|
||||
\begin{tikzpicture}[sloped]
|
||||
\node {.}
|
||||
child {node {$M$}
|
||||
child {node {$W$}
|
||||
edge from parent
|
||||
node[above] {$0.75$}
|
||||
}
|
||||
child {node {$B$}
|
||||
edge from parent
|
||||
node[above] {$0.25$}
|
||||
}
|
||||
edge from parent
|
||||
node[above] {$0.35$}
|
||||
}
|
||||
child[missing] {}
|
||||
child { node {$S$}
|
||||
child {node {$W$}
|
||||
edge from parent
|
||||
node[above] {$0.13$}
|
||||
}
|
||||
child {node {$B$}
|
||||
edge from parent
|
||||
node[above] {$0.87$}
|
||||
}
|
||||
edge from parent
|
||||
node[above] {$0.65$}
|
||||
} ;
|
||||
\end{tikzpicture}
|
||||
\end{center}
|
||||
\item On calcule la probabilité que la vente soit un wok et ait eu lieu à midi
|
||||
\[ P(M\cap W) = P(M) \times P_M(W) = 0.35 \times 0.75 = 0.2625 \]
|
||||
\item Probabilité que la vente soit un burger.
|
||||
\[
|
||||
P(B) = P(M\cap B) + P(S\cap B) = 0.35 \times 0.75 + 0.65 \times 0.13 = 0.653
|
||||
\]
|
||||
\item On cherche à calculer la quantité $P_B(S)$. Pour cela on utilise la formule de Bayes
|
||||
\[
|
||||
P_B(S) = \frac{P(B\cap S)}{P(B)} = \frac{P_S(B) \times P(S)}{P(B)} = \frac{0.87\times 0.65}{0.653} = 0.8660030627871362 \approx 0.866
|
||||
\]
|
||||
\end{enumerate}
|
||||
\end{solution}
|
||||
|
||||
\begin{exercise}[subtitle={Continent plastique}]
|
||||
\textit{Les quantités évoqués dans cette exercice sont générés au hasard et sont donc complètement farfelus.}
|
||||
\medskip
|
||||
Le \og continent de plastique\fg{} est la plus grande des plaques de déchets plastiques évoluant sur les océans. Elle occupe actuellement dans l'océan Pacifique une surface dont l'aire est évaluée à plus de $1,6$ million de km$^2$, entre Hawaï et la Californie.
|
||||
|
||||
En 2017, des scientifiques ont estimé qu'il y avait $4$ millions de tonnes de déchets plastiques qui était déversé chaque année dans les océans et que cette quantité augmentait de $23\n\%$ par chaque année.
|
||||
|
||||
On modélise l'évolution de la masse de ces déchets plastiques déversée chaque année, si rien n'est fait pour la réduire, par une suite géométrique $\left(u_n\right)$. L'arrondi au centième du terme $u_n$ représente la masse de ces déchets déversée chaque année, exprimée en million de tonnes, pour l'année $(2017 + n)$.
|
||||
|
||||
\medskip
|
||||
|
||||
\begin{enumerate}
|
||||
\item Expliquer pourquoi la suite $u_n$ est géométrique?
|
||||
\item Calculer $u_1$ et $u_2$.
|
||||
\item Exprimer $u_n$ en fonction de $n$.
|
||||
\item Au début de l'année 2017, il y avait $300$ millions de tonnes de déchets plastique. Calculer la quantité totale de déchets plastiques en 2030.
|
||||
\item On souhaite déterminer en quelle année la masse totale de ces déchets plastiques aura pour la première fois augmenté de $50$\,\% par rapport à sa valeur de 2017.
|
||||
\begin{enumerate}
|
||||
\item Recopier et compléter l'algorithme ci-dessous pour que la variable $N$ contienne la réponse au problème posé.
|
||||
|
||||
\begin{center}
|
||||
\begin{tabularx}{0.4\linewidth}{|X|}\hline
|
||||
$N = 2017$\\
|
||||
$U = 4$ \\
|
||||
$S = 300 + U$ \\
|
||||
while $S < 450$: \\
|
||||
\hspace{1cm} $N = \ldots$\\
|
||||
\hspace{1cm} $U = \ldots$\\
|
||||
\hspace{1cm} $S = \ldots$\\
|
||||
\hline
|
||||
\end{tabularx}
|
||||
\end{center}
|
||||
\item Que contiennent les variables $S$, $U$ et $N$ après exécution de cet algorithme ?
|
||||
|
||||
Interpréter les résultats dans le contexte de l'exercice.
|
||||
\end{enumerate}
|
||||
\end{enumerate}
|
||||
\end{exercise}
|
||||
|
||||
\begin{solution}
|
||||
\begin{enumerate}
|
||||
\item Une augmentation de $23\,\%$ revient à multiplier la quantité par $1.23$. La suite est donc bien géométrique. Son premier terme est $u_0 = 4$ et sa raison est $q = 1.23$
|
||||
\item
|
||||
\[
|
||||
u_1 = u_0 * 1.23 = 4.92
|
||||
\]
|
||||
\[
|
||||
u_2 = u_0 * 1.23^2 = 6.0516
|
||||
\]
|
||||
\item
|
||||
\[
|
||||
u_n = u_0 \times q^n = 4 \times 1.23^n
|
||||
\]
|
||||
\item On calcule la quantité totale déversée entre 2017 et 2030.
|
||||
\[
|
||||
\sum_{n = 0}^{13} u_n = u_0 \times \frac{1-q^{13}}{1-q} = 4 \times \frac{1 - 1.23^{13}}{1 - 1.23} = 239.12
|
||||
\]
|
||||
On en déduit la quantité totale de déchets en 2030
|
||||
\[
|
||||
300 + 239.12 = 539.12
|
||||
\]
|
||||
\item
|
||||
\begin{enumerate}
|
||||
\item ~
|
||||
\begin{center}
|
||||
\begin{tabularx}{0.4\linewidth}{|X|}\hline
|
||||
$N \gets 2017$\\
|
||||
$U \gets 4$ \\
|
||||
$S \gets 300 + U$ \\
|
||||
Tant que $S < 450$ \\
|
||||
\hspace{1cm} $N \gets N + 1$\\
|
||||
\hspace{1cm} $U \gets U * 1.23$\\
|
||||
\hspace{1cm} $S \gets S + u$\\
|
||||
Fin Tant que\\\hline
|
||||
\end{tabularx}
|
||||
\end{center}
|
||||
\item \textit{Pas de correction automatisé}
|
||||
\end{enumerate}
|
||||
\end{enumerate}
|
||||
\end{solution}
|
||||
|
||||
\end{document}
|
||||
|
||||
%%% Local Variables:
|
||||
%%% mode: latex
|
||||
%%% TeX-master: "master"
|
||||
%%% End:
|
262
TST/DS/DS_21_04_07/TST1/corr_15_210407_DS8.tex
Normal file
262
TST/DS/DS_21_04_07/TST1/corr_15_210407_DS8.tex
Normal file
@ -0,0 +1,262 @@
|
||||
\documentclass[a4paper,10pt]{article}
|
||||
\usepackage{myXsim}
|
||||
|
||||
% Title Page
|
||||
\title{DS8 \hfill HUMBERT Rayan}
|
||||
\tribe{TST}
|
||||
\date{\hfillÀ render pour le Mercredi 7 avril}
|
||||
|
||||
\xsimsetup{
|
||||
solution/print = true
|
||||
}
|
||||
|
||||
\begin{document}
|
||||
\maketitle
|
||||
|
||||
\begin{exercise}[subtitle={Automatismes}]
|
||||
\textit{Toutes les questions de cette exercice sont indépendantes et peuvent être répondus séparément}
|
||||
\begin{enumerate}
|
||||
\item De janvier à septembre, une quantité a augmenté de $17\,\%$. Faire un schéma pour représenter la situation puis calculer le taux d'évolution moyen mensuel.
|
||||
\item Une quantité augmente de $17\,\%$ par ans. En 2020, elle est de 149\euro. Quelle était sa valeur en 2019? Faire un schéma pour représenter la situation.
|
||||
\item Déterminer l'équation de la droite \\
|
||||
\begin{tikzpicture}[xscale=0.8, yscale=0.5]
|
||||
\tkzInit[xmin=-5,xmax=5,xstep=1,
|
||||
ymin=-5,ymax=5,ystep=1]
|
||||
\tkzGrid
|
||||
\tkzAxeXY
|
||||
\tkzFct[domain=-5:5,color=red,very thick]%
|
||||
{0.5*\x -1};
|
||||
\end{tikzpicture}
|
||||
\item Résoudre l'équation $5 \times 0.02^x = 46$
|
||||
\end{enumerate}
|
||||
\end{exercise}
|
||||
|
||||
\begin{solution}
|
||||
\begin{enumerate}
|
||||
\item On veut partager cette évolution en 8 évolutions.
|
||||
\[
|
||||
\left(1 + \frac{17}{100}\right)^{\frac{1}{8}} = 1.0198
|
||||
\]
|
||||
Donc le taux d'évolution moyen est
|
||||
\[
|
||||
t_m = 1.0198 - 1 = 0.01980000000000004
|
||||
\]
|
||||
\item Coefficient multiplicateur pour revenir en arrière
|
||||
\[
|
||||
CM = (1 + \frac{17}{100})^{-1} = 0.8547
|
||||
\]
|
||||
On en déduit la quantité en 2019
|
||||
\[
|
||||
149 * 0.8547 = 127.3503
|
||||
\]
|
||||
\item L'équation de la droite est
|
||||
\[
|
||||
y = 0.5 x -1
|
||||
\]
|
||||
\item Il faut penser à faire la division à par $5$ avant d'utiliser le log car sinon, on ne peut pas utiliser la formule $\log(a^n) = n\times \log(a)$.
|
||||
|
||||
\[x = \frac{\log(9.2)}{\log(0.02)}\]
|
||||
\end{enumerate}
|
||||
\end{solution}
|
||||
|
||||
\begin{exercise}[subtitle={Restaurant}]
|
||||
Un \emph{food truck}, ouvert le midi et le soir, propose deux types de formules :
|
||||
|
||||
\setlength\parindent{10mm}
|
||||
\begin{itemize}
|
||||
\item la formule \emph{Burger} ;
|
||||
\item la formule \emph{Wok}.
|
||||
\end{itemize}
|
||||
\setlength\parindent{0mm}
|
||||
|
||||
\medskip
|
||||
|
||||
Le gérant a remarqué que 65\,\% de ses ventes ont lieu le midi. Le quart des ventes du midi correspondent à la formule \emph{Burger}, alors que 93\,\% des ventes du soir correspondent à la formule \emph{Wok}.
|
||||
|
||||
Le gérant se constitue un fichier en notant, pour chaque vente, la formule choisie et le moment de cette vente (midi ou soir).
|
||||
|
||||
On prélève une fiche de façon équiprobable. On définit les quatre évènements suivants:
|
||||
|
||||
\begin{enumerate}
|
||||
\item $M$ : \og la fiche correspond à une vente du midi\fg{} ;
|
||||
\item $S$ : \og la fiche correspond à une vente du soir\fg {};
|
||||
\item $W$ : \og la fiche correspond à une formule \emph{Wok} \fg{} ;
|
||||
\item $B$ : \og la fiche correspond à une formule \emph{Burger} \fg.
|
||||
\end{enumerate}
|
||||
\setlength\parindent{0mm}
|
||||
|
||||
\medskip
|
||||
|
||||
\begin{enumerate}
|
||||
\item Recopier puis compléter l'arbre pondéré
|
||||
|
||||
\begin{center}
|
||||
\begin{tikzpicture}[sloped]
|
||||
\node {.}
|
||||
child {node {$M$}
|
||||
child {node {$W$}
|
||||
edge from parent
|
||||
node[above] {...}
|
||||
}
|
||||
child {node {$B$}
|
||||
edge from parent
|
||||
node[above] {...}
|
||||
}
|
||||
edge from parent
|
||||
node[above] {...}
|
||||
}
|
||||
child[missing] {}
|
||||
child { node {$S$}
|
||||
child {node {$W$}
|
||||
edge from parent
|
||||
node[above] {...}
|
||||
}
|
||||
child {node {$B$}
|
||||
edge from parent
|
||||
node[above] {...}
|
||||
}
|
||||
edge from parent
|
||||
node[above] {...}
|
||||
} ;
|
||||
\end{tikzpicture}
|
||||
\end{center}
|
||||
|
||||
\item Calculer la probabilité de l'évènement $M \cap W$. Interpréter ce résultat dans le contexte de l'exercice.
|
||||
\item Montrer que la probabilité que la fiche choisie corresponde à une formule \emph{Burger} est égale à $0.187$.
|
||||
\item On a prélevé une fiche correspondant à la formule \emph{Burger}. Quelle est la probabilité, arrondie au millième, que la vente ait eu lieu le soir?
|
||||
\end{enumerate}
|
||||
\end{exercise}
|
||||
|
||||
\begin{solution}
|
||||
\begin{enumerate}
|
||||
\item
|
||||
\begin{center}
|
||||
\begin{tikzpicture}[sloped]
|
||||
\node {.}
|
||||
child {node {$M$}
|
||||
child {node {$W$}
|
||||
edge from parent
|
||||
node[above] {$0.75$}
|
||||
}
|
||||
child {node {$B$}
|
||||
edge from parent
|
||||
node[above] {$0.25$}
|
||||
}
|
||||
edge from parent
|
||||
node[above] {$0.65$}
|
||||
}
|
||||
child[missing] {}
|
||||
child { node {$S$}
|
||||
child {node {$W$}
|
||||
edge from parent
|
||||
node[above] {$0.93$}
|
||||
}
|
||||
child {node {$B$}
|
||||
edge from parent
|
||||
node[above] {$0.07$}
|
||||
}
|
||||
edge from parent
|
||||
node[above] {$0.35$}
|
||||
} ;
|
||||
\end{tikzpicture}
|
||||
\end{center}
|
||||
\item On calcule la probabilité que la vente soit un wok et ait eu lieu à midi
|
||||
\[ P(M\cap W) = P(M) \times P_M(W) = 0.65 \times 0.75 = 0.4875 \]
|
||||
\item Probabilité que la vente soit un burger.
|
||||
\[
|
||||
P(B) = P(M\cap B) + P(S\cap B) = 0.65 \times 0.75 + 0.35 \times 0.93 = 0.187
|
||||
\]
|
||||
\item On cherche à calculer la quantité $P_B(S)$. Pour cela on utilise la formule de Bayes
|
||||
\[
|
||||
P_B(S) = \frac{P(B\cap S)}{P(B)} = \frac{P_S(B) \times P(S)}{P(B)} = \frac{0.07\times 0.35}{0.187} = 0.13101604278074866 \approx 0.131
|
||||
\]
|
||||
\end{enumerate}
|
||||
\end{solution}
|
||||
|
||||
\begin{exercise}[subtitle={Continent plastique}]
|
||||
\textit{Les quantités évoqués dans cette exercice sont générés au hasard et sont donc complètement farfelus.}
|
||||
\medskip
|
||||
Le \og continent de plastique\fg{} est la plus grande des plaques de déchets plastiques évoluant sur les océans. Elle occupe actuellement dans l'océan Pacifique une surface dont l'aire est évaluée à plus de $1,6$ million de km$^2$, entre Hawaï et la Californie.
|
||||
|
||||
En 2017, des scientifiques ont estimé qu'il y avait $15$ millions de tonnes de déchets plastiques qui était déversé chaque année dans les océans et que cette quantité augmentait de $12\n\%$ par chaque année.
|
||||
|
||||
On modélise l'évolution de la masse de ces déchets plastiques déversée chaque année, si rien n'est fait pour la réduire, par une suite géométrique $\left(u_n\right)$. L'arrondi au centième du terme $u_n$ représente la masse de ces déchets déversée chaque année, exprimée en million de tonnes, pour l'année $(2017 + n)$.
|
||||
|
||||
\medskip
|
||||
|
||||
\begin{enumerate}
|
||||
\item Expliquer pourquoi la suite $u_n$ est géométrique?
|
||||
\item Calculer $u_1$ et $u_2$.
|
||||
\item Exprimer $u_n$ en fonction de $n$.
|
||||
\item Au début de l'année 2017, il y avait $300$ millions de tonnes de déchets plastique. Calculer la quantité totale de déchets plastiques en 2030.
|
||||
\item On souhaite déterminer en quelle année la masse totale de ces déchets plastiques aura pour la première fois augmenté de $50$\,\% par rapport à sa valeur de 2017.
|
||||
\begin{enumerate}
|
||||
\item Recopier et compléter l'algorithme ci-dessous pour que la variable $N$ contienne la réponse au problème posé.
|
||||
|
||||
\begin{center}
|
||||
\begin{tabularx}{0.4\linewidth}{|X|}\hline
|
||||
$N = 2017$\\
|
||||
$U = 15$ \\
|
||||
$S = 300 + U$ \\
|
||||
while $S < 450$: \\
|
||||
\hspace{1cm} $N = \ldots$\\
|
||||
\hspace{1cm} $U = \ldots$\\
|
||||
\hspace{1cm} $S = \ldots$\\
|
||||
\hline
|
||||
\end{tabularx}
|
||||
\end{center}
|
||||
\item Que contiennent les variables $S$, $U$ et $N$ après exécution de cet algorithme ?
|
||||
|
||||
Interpréter les résultats dans le contexte de l'exercice.
|
||||
\end{enumerate}
|
||||
\end{enumerate}
|
||||
\end{exercise}
|
||||
|
||||
\begin{solution}
|
||||
\begin{enumerate}
|
||||
\item Une augmentation de $12\,\%$ revient à multiplier la quantité par $1.12$. La suite est donc bien géométrique. Son premier terme est $u_0 = 15$ et sa raison est $q = 1.12$
|
||||
\item
|
||||
\[
|
||||
u_1 = u_0 * 1.12 = 16.8
|
||||
\]
|
||||
\[
|
||||
u_2 = u_0 * 1.12^2 = 18.816
|
||||
\]
|
||||
\item
|
||||
\[
|
||||
u_n = u_0 \times q^n = 15 \times 1.12^n
|
||||
\]
|
||||
\item On calcule la quantité totale déversée entre 2017 et 2030.
|
||||
\[
|
||||
\sum_{n = 0}^{13} u_n = u_0 \times \frac{1-q^{13}}{1-q} = 15 \times \frac{1 - 1.12^{13}}{1 - 1.12} = 420.44
|
||||
\]
|
||||
On en déduit la quantité totale de déchets en 2030
|
||||
\[
|
||||
300 + 420.44 = 720.44
|
||||
\]
|
||||
\item
|
||||
\begin{enumerate}
|
||||
\item ~
|
||||
\begin{center}
|
||||
\begin{tabularx}{0.4\linewidth}{|X|}\hline
|
||||
$N \gets 2017$\\
|
||||
$U \gets 15$ \\
|
||||
$S \gets 300 + U$ \\
|
||||
Tant que $S < 450$ \\
|
||||
\hspace{1cm} $N \gets N + 1$\\
|
||||
\hspace{1cm} $U \gets U * 1.12$\\
|
||||
\hspace{1cm} $S \gets S + u$\\
|
||||
Fin Tant que\\\hline
|
||||
\end{tabularx}
|
||||
\end{center}
|
||||
\item \textit{Pas de correction automatisé}
|
||||
\end{enumerate}
|
||||
\end{enumerate}
|
||||
\end{solution}
|
||||
|
||||
\end{document}
|
||||
|
||||
%%% Local Variables:
|
||||
%%% mode: latex
|
||||
%%% TeX-master: "master"
|
||||
%%% End:
|
262
TST/DS/DS_21_04_07/TST1/corr_16_210407_DS8.tex
Normal file
262
TST/DS/DS_21_04_07/TST1/corr_16_210407_DS8.tex
Normal file
@ -0,0 +1,262 @@
|
||||
\documentclass[a4paper,10pt]{article}
|
||||
\usepackage{myXsim}
|
||||
|
||||
% Title Page
|
||||
\title{DS8 \hfill MASSON Grace}
|
||||
\tribe{TST}
|
||||
\date{\hfillÀ render pour le Mercredi 7 avril}
|
||||
|
||||
\xsimsetup{
|
||||
solution/print = true
|
||||
}
|
||||
|
||||
\begin{document}
|
||||
\maketitle
|
||||
|
||||
\begin{exercise}[subtitle={Automatismes}]
|
||||
\textit{Toutes les questions de cette exercice sont indépendantes et peuvent être répondus séparément}
|
||||
\begin{enumerate}
|
||||
\item De janvier à septembre, une quantité a augmenté de $24\,\%$. Faire un schéma pour représenter la situation puis calculer le taux d'évolution moyen mensuel.
|
||||
\item Une quantité augmente de $24\,\%$ par ans. En 2020, elle est de 145\euro. Quelle était sa valeur en 2019? Faire un schéma pour représenter la situation.
|
||||
\item Déterminer l'équation de la droite \\
|
||||
\begin{tikzpicture}[xscale=0.8, yscale=0.5]
|
||||
\tkzInit[xmin=-5,xmax=5,xstep=1,
|
||||
ymin=-5,ymax=5,ystep=1]
|
||||
\tkzGrid
|
||||
\tkzAxeXY
|
||||
\tkzFct[domain=-5:5,color=red,very thick]%
|
||||
{3.0*\x -3};
|
||||
\end{tikzpicture}
|
||||
\item Résoudre l'équation $5 \times 0.53^x = 18$
|
||||
\end{enumerate}
|
||||
\end{exercise}
|
||||
|
||||
\begin{solution}
|
||||
\begin{enumerate}
|
||||
\item On veut partager cette évolution en 8 évolutions.
|
||||
\[
|
||||
\left(1 + \frac{24}{100}\right)^{\frac{1}{8}} = 1.0273
|
||||
\]
|
||||
Donc le taux d'évolution moyen est
|
||||
\[
|
||||
t_m = 1.0273 - 1 = 0.027300000000000102
|
||||
\]
|
||||
\item Coefficient multiplicateur pour revenir en arrière
|
||||
\[
|
||||
CM = (1 + \frac{24}{100})^{-1} = 0.8065
|
||||
\]
|
||||
On en déduit la quantité en 2019
|
||||
\[
|
||||
145 * 0.8065 = 116.9425
|
||||
\]
|
||||
\item L'équation de la droite est
|
||||
\[
|
||||
y = 3.0 x -3
|
||||
\]
|
||||
\item Il faut penser à faire la division à par $5$ avant d'utiliser le log car sinon, on ne peut pas utiliser la formule $\log(a^n) = n\times \log(a)$.
|
||||
|
||||
\[x = \frac{\log(3.6)}{\log(0.53)}\]
|
||||
\end{enumerate}
|
||||
\end{solution}
|
||||
|
||||
\begin{exercise}[subtitle={Restaurant}]
|
||||
Un \emph{food truck}, ouvert le midi et le soir, propose deux types de formules :
|
||||
|
||||
\setlength\parindent{10mm}
|
||||
\begin{itemize}
|
||||
\item la formule \emph{Burger} ;
|
||||
\item la formule \emph{Wok}.
|
||||
\end{itemize}
|
||||
\setlength\parindent{0mm}
|
||||
|
||||
\medskip
|
||||
|
||||
Le gérant a remarqué que 73\,\% de ses ventes ont lieu le midi. Le quart des ventes du midi correspondent à la formule \emph{Burger}, alors que 56\,\% des ventes du soir correspondent à la formule \emph{Wok}.
|
||||
|
||||
Le gérant se constitue un fichier en notant, pour chaque vente, la formule choisie et le moment de cette vente (midi ou soir).
|
||||
|
||||
On prélève une fiche de façon équiprobable. On définit les quatre évènements suivants:
|
||||
|
||||
\begin{enumerate}
|
||||
\item $M$ : \og la fiche correspond à une vente du midi\fg{} ;
|
||||
\item $S$ : \og la fiche correspond à une vente du soir\fg {};
|
||||
\item $W$ : \og la fiche correspond à une formule \emph{Wok} \fg{} ;
|
||||
\item $B$ : \og la fiche correspond à une formule \emph{Burger} \fg.
|
||||
\end{enumerate}
|
||||
\setlength\parindent{0mm}
|
||||
|
||||
\medskip
|
||||
|
||||
\begin{enumerate}
|
||||
\item Recopier puis compléter l'arbre pondéré
|
||||
|
||||
\begin{center}
|
||||
\begin{tikzpicture}[sloped]
|
||||
\node {.}
|
||||
child {node {$M$}
|
||||
child {node {$W$}
|
||||
edge from parent
|
||||
node[above] {...}
|
||||
}
|
||||
child {node {$B$}
|
||||
edge from parent
|
||||
node[above] {...}
|
||||
}
|
||||
edge from parent
|
||||
node[above] {...}
|
||||
}
|
||||
child[missing] {}
|
||||
child { node {$S$}
|
||||
child {node {$W$}
|
||||
edge from parent
|
||||
node[above] {...}
|
||||
}
|
||||
child {node {$B$}
|
||||
edge from parent
|
||||
node[above] {...}
|
||||
}
|
||||
edge from parent
|
||||
node[above] {...}
|
||||
} ;
|
||||
\end{tikzpicture}
|
||||
\end{center}
|
||||
|
||||
\item Calculer la probabilité de l'évènement $M \cap W$. Interpréter ce résultat dans le contexte de l'exercice.
|
||||
\item Montrer que la probabilité que la fiche choisie corresponde à une formule \emph{Burger} est égale à $0.2986$.
|
||||
\item On a prélevé une fiche correspondant à la formule \emph{Burger}. Quelle est la probabilité, arrondie au millième, que la vente ait eu lieu le soir?
|
||||
\end{enumerate}
|
||||
\end{exercise}
|
||||
|
||||
\begin{solution}
|
||||
\begin{enumerate}
|
||||
\item
|
||||
\begin{center}
|
||||
\begin{tikzpicture}[sloped]
|
||||
\node {.}
|
||||
child {node {$M$}
|
||||
child {node {$W$}
|
||||
edge from parent
|
||||
node[above] {$0.75$}
|
||||
}
|
||||
child {node {$B$}
|
||||
edge from parent
|
||||
node[above] {$0.25$}
|
||||
}
|
||||
edge from parent
|
||||
node[above] {$0.73$}
|
||||
}
|
||||
child[missing] {}
|
||||
child { node {$S$}
|
||||
child {node {$W$}
|
||||
edge from parent
|
||||
node[above] {$0.57$}
|
||||
}
|
||||
child {node {$B$}
|
||||
edge from parent
|
||||
node[above] {$0.43$}
|
||||
}
|
||||
edge from parent
|
||||
node[above] {$0.27$}
|
||||
} ;
|
||||
\end{tikzpicture}
|
||||
\end{center}
|
||||
\item On calcule la probabilité que la vente soit un wok et ait eu lieu à midi
|
||||
\[ P(M\cap W) = P(M) \times P_M(W) = 0.73 \times 0.75 = 0.5475 \]
|
||||
\item Probabilité que la vente soit un burger.
|
||||
\[
|
||||
P(B) = P(M\cap B) + P(S\cap B) = 0.73 \times 0.75 + 0.27 \times 0.57 = 0.2986
|
||||
\]
|
||||
\item On cherche à calculer la quantité $P_B(S)$. Pour cela on utilise la formule de Bayes
|
||||
\[
|
||||
P_B(S) = \frac{P(B\cap S)}{P(B)} = \frac{P_S(B) \times P(S)}{P(B)} = \frac{0.43\times 0.27}{0.2986} = 0.38881446751507037 \approx 0.389
|
||||
\]
|
||||
\end{enumerate}
|
||||
\end{solution}
|
||||
|
||||
\begin{exercise}[subtitle={Continent plastique}]
|
||||
\textit{Les quantités évoqués dans cette exercice sont générés au hasard et sont donc complètement farfelus.}
|
||||
\medskip
|
||||
Le \og continent de plastique\fg{} est la plus grande des plaques de déchets plastiques évoluant sur les océans. Elle occupe actuellement dans l'océan Pacifique une surface dont l'aire est évaluée à plus de $1,6$ million de km$^2$, entre Hawaï et la Californie.
|
||||
|
||||
En 2017, des scientifiques ont estimé qu'il y avait $4$ millions de tonnes de déchets plastiques qui était déversé chaque année dans les océans et que cette quantité augmentait de $19\n\%$ par chaque année.
|
||||
|
||||
On modélise l'évolution de la masse de ces déchets plastiques déversée chaque année, si rien n'est fait pour la réduire, par une suite géométrique $\left(u_n\right)$. L'arrondi au centième du terme $u_n$ représente la masse de ces déchets déversée chaque année, exprimée en million de tonnes, pour l'année $(2017 + n)$.
|
||||
|
||||
\medskip
|
||||
|
||||
\begin{enumerate}
|
||||
\item Expliquer pourquoi la suite $u_n$ est géométrique?
|
||||
\item Calculer $u_1$ et $u_2$.
|
||||
\item Exprimer $u_n$ en fonction de $n$.
|
||||
\item Au début de l'année 2017, il y avait $300$ millions de tonnes de déchets plastique. Calculer la quantité totale de déchets plastiques en 2030.
|
||||
\item On souhaite déterminer en quelle année la masse totale de ces déchets plastiques aura pour la première fois augmenté de $50$\,\% par rapport à sa valeur de 2017.
|
||||
\begin{enumerate}
|
||||
\item Recopier et compléter l'algorithme ci-dessous pour que la variable $N$ contienne la réponse au problème posé.
|
||||
|
||||
\begin{center}
|
||||
\begin{tabularx}{0.4\linewidth}{|X|}\hline
|
||||
$N = 2017$\\
|
||||
$U = 4$ \\
|
||||
$S = 300 + U$ \\
|
||||
while $S < 450$: \\
|
||||
\hspace{1cm} $N = \ldots$\\
|
||||
\hspace{1cm} $U = \ldots$\\
|
||||
\hspace{1cm} $S = \ldots$\\
|
||||
\hline
|
||||
\end{tabularx}
|
||||
\end{center}
|
||||
\item Que contiennent les variables $S$, $U$ et $N$ après exécution de cet algorithme ?
|
||||
|
||||
Interpréter les résultats dans le contexte de l'exercice.
|
||||
\end{enumerate}
|
||||
\end{enumerate}
|
||||
\end{exercise}
|
||||
|
||||
\begin{solution}
|
||||
\begin{enumerate}
|
||||
\item Une augmentation de $19\,\%$ revient à multiplier la quantité par $1.19$. La suite est donc bien géométrique. Son premier terme est $u_0 = 4$ et sa raison est $q = 1.19$
|
||||
\item
|
||||
\[
|
||||
u_1 = u_0 * 1.19 = 4.76
|
||||
\]
|
||||
\[
|
||||
u_2 = u_0 * 1.19^2 = 5.6644
|
||||
\]
|
||||
\item
|
||||
\[
|
||||
u_n = u_0 \times q^n = 4 \times 1.19^n
|
||||
\]
|
||||
\item On calcule la quantité totale déversée entre 2017 et 2030.
|
||||
\[
|
||||
\sum_{n = 0}^{13} u_n = u_0 \times \frac{1-q^{13}}{1-q} = 4 \times \frac{1 - 1.19^{13}}{1 - 1.19} = 180.98
|
||||
\]
|
||||
On en déduit la quantité totale de déchets en 2030
|
||||
\[
|
||||
300 + 180.98 = 480.98
|
||||
\]
|
||||
\item
|
||||
\begin{enumerate}
|
||||
\item ~
|
||||
\begin{center}
|
||||
\begin{tabularx}{0.4\linewidth}{|X|}\hline
|
||||
$N \gets 2017$\\
|
||||
$U \gets 4$ \\
|
||||
$S \gets 300 + U$ \\
|
||||
Tant que $S < 450$ \\
|
||||
\hspace{1cm} $N \gets N + 1$\\
|
||||
\hspace{1cm} $U \gets U * 1.19$\\
|
||||
\hspace{1cm} $S \gets S + u$\\
|
||||
Fin Tant que\\\hline
|
||||
\end{tabularx}
|
||||
\end{center}
|
||||
\item \textit{Pas de correction automatisé}
|
||||
\end{enumerate}
|
||||
\end{enumerate}
|
||||
\end{solution}
|
||||
|
||||
\end{document}
|
||||
|
||||
%%% Local Variables:
|
||||
%%% mode: latex
|
||||
%%% TeX-master: "master"
|
||||
%%% End:
|
262
TST/DS/DS_21_04_07/TST1/corr_17_210407_DS8.tex
Normal file
262
TST/DS/DS_21_04_07/TST1/corr_17_210407_DS8.tex
Normal file
@ -0,0 +1,262 @@
|
||||
\documentclass[a4paper,10pt]{article}
|
||||
\usepackage{myXsim}
|
||||
|
||||
% Title Page
|
||||
\title{DS8 \hfill MOKHTARI Nissrine}
|
||||
\tribe{TST}
|
||||
\date{\hfillÀ render pour le Mercredi 7 avril}
|
||||
|
||||
\xsimsetup{
|
||||
solution/print = true
|
||||
}
|
||||
|
||||
\begin{document}
|
||||
\maketitle
|
||||
|
||||
\begin{exercise}[subtitle={Automatismes}]
|
||||
\textit{Toutes les questions de cette exercice sont indépendantes et peuvent être répondus séparément}
|
||||
\begin{enumerate}
|
||||
\item De janvier à septembre, une quantité a augmenté de $11\,\%$. Faire un schéma pour représenter la situation puis calculer le taux d'évolution moyen mensuel.
|
||||
\item Une quantité augmente de $11\,\%$ par ans. En 2020, elle est de 138\euro. Quelle était sa valeur en 2019? Faire un schéma pour représenter la situation.
|
||||
\item Déterminer l'équation de la droite \\
|
||||
\begin{tikzpicture}[xscale=0.8, yscale=0.5]
|
||||
\tkzInit[xmin=-5,xmax=5,xstep=1,
|
||||
ymin=-5,ymax=5,ystep=1]
|
||||
\tkzGrid
|
||||
\tkzAxeXY
|
||||
\tkzFct[domain=-5:5,color=red,very thick]%
|
||||
{2.0*\x -3};
|
||||
\end{tikzpicture}
|
||||
\item Résoudre l'équation $8 \times 0.17^x = 35$
|
||||
\end{enumerate}
|
||||
\end{exercise}
|
||||
|
||||
\begin{solution}
|
||||
\begin{enumerate}
|
||||
\item On veut partager cette évolution en 8 évolutions.
|
||||
\[
|
||||
\left(1 + \frac{11}{100}\right)^{\frac{1}{8}} = 1.0131
|
||||
\]
|
||||
Donc le taux d'évolution moyen est
|
||||
\[
|
||||
t_m = 1.0131 - 1 = 0.01309999999999989
|
||||
\]
|
||||
\item Coefficient multiplicateur pour revenir en arrière
|
||||
\[
|
||||
CM = (1 + \frac{11}{100})^{-1} = 0.9009
|
||||
\]
|
||||
On en déduit la quantité en 2019
|
||||
\[
|
||||
138 * 0.9009 = 124.3242
|
||||
\]
|
||||
\item L'équation de la droite est
|
||||
\[
|
||||
y = 2.0 x -3
|
||||
\]
|
||||
\item Il faut penser à faire la division à par $8$ avant d'utiliser le log car sinon, on ne peut pas utiliser la formule $\log(a^n) = n\times \log(a)$.
|
||||
|
||||
\[x = \frac{\log(4.38)}{\log(0.17)}\]
|
||||
\end{enumerate}
|
||||
\end{solution}
|
||||
|
||||
\begin{exercise}[subtitle={Restaurant}]
|
||||
Un \emph{food truck}, ouvert le midi et le soir, propose deux types de formules :
|
||||
|
||||
\setlength\parindent{10mm}
|
||||
\begin{itemize}
|
||||
\item la formule \emph{Burger} ;
|
||||
\item la formule \emph{Wok}.
|
||||
\end{itemize}
|
||||
\setlength\parindent{0mm}
|
||||
|
||||
\medskip
|
||||
|
||||
Le gérant a remarqué que 62\,\% de ses ventes ont lieu le midi. Le quart des ventes du midi correspondent à la formule \emph{Burger}, alors que 28\,\% des ventes du soir correspondent à la formule \emph{Wok}.
|
||||
|
||||
Le gérant se constitue un fichier en notant, pour chaque vente, la formule choisie et le moment de cette vente (midi ou soir).
|
||||
|
||||
On prélève une fiche de façon équiprobable. On définit les quatre évènements suivants:
|
||||
|
||||
\begin{enumerate}
|
||||
\item $M$ : \og la fiche correspond à une vente du midi\fg{} ;
|
||||
\item $S$ : \og la fiche correspond à une vente du soir\fg {};
|
||||
\item $W$ : \og la fiche correspond à une formule \emph{Wok} \fg{} ;
|
||||
\item $B$ : \og la fiche correspond à une formule \emph{Burger} \fg.
|
||||
\end{enumerate}
|
||||
\setlength\parindent{0mm}
|
||||
|
||||
\medskip
|
||||
|
||||
\begin{enumerate}
|
||||
\item Recopier puis compléter l'arbre pondéré
|
||||
|
||||
\begin{center}
|
||||
\begin{tikzpicture}[sloped]
|
||||
\node {.}
|
||||
child {node {$M$}
|
||||
child {node {$W$}
|
||||
edge from parent
|
||||
node[above] {...}
|
||||
}
|
||||
child {node {$B$}
|
||||
edge from parent
|
||||
node[above] {...}
|
||||
}
|
||||
edge from parent
|
||||
node[above] {...}
|
||||
}
|
||||
child[missing] {}
|
||||
child { node {$S$}
|
||||
child {node {$W$}
|
||||
edge from parent
|
||||
node[above] {...}
|
||||
}
|
||||
child {node {$B$}
|
||||
edge from parent
|
||||
node[above] {...}
|
||||
}
|
||||
edge from parent
|
||||
node[above] {...}
|
||||
} ;
|
||||
\end{tikzpicture}
|
||||
\end{center}
|
||||
|
||||
\item Calculer la probabilité de l'évènement $M \cap W$. Interpréter ce résultat dans le contexte de l'exercice.
|
||||
\item Montrer que la probabilité que la fiche choisie corresponde à une formule \emph{Burger} est égale à $0.4248$.
|
||||
\item On a prélevé une fiche correspondant à la formule \emph{Burger}. Quelle est la probabilité, arrondie au millième, que la vente ait eu lieu le soir?
|
||||
\end{enumerate}
|
||||
\end{exercise}
|
||||
|
||||
\begin{solution}
|
||||
\begin{enumerate}
|
||||
\item
|
||||
\begin{center}
|
||||
\begin{tikzpicture}[sloped]
|
||||
\node {.}
|
||||
child {node {$M$}
|
||||
child {node {$W$}
|
||||
edge from parent
|
||||
node[above] {$0.75$}
|
||||
}
|
||||
child {node {$B$}
|
||||
edge from parent
|
||||
node[above] {$0.25$}
|
||||
}
|
||||
edge from parent
|
||||
node[above] {$0.62$}
|
||||
}
|
||||
child[missing] {}
|
||||
child { node {$S$}
|
||||
child {node {$W$}
|
||||
edge from parent
|
||||
node[above] {$0.29$}
|
||||
}
|
||||
child {node {$B$}
|
||||
edge from parent
|
||||
node[above] {$0.71$}
|
||||
}
|
||||
edge from parent
|
||||
node[above] {$0.38$}
|
||||
} ;
|
||||
\end{tikzpicture}
|
||||
\end{center}
|
||||
\item On calcule la probabilité que la vente soit un wok et ait eu lieu à midi
|
||||
\[ P(M\cap W) = P(M) \times P_M(W) = 0.62 \times 0.75 = 0.465 \]
|
||||
\item Probabilité que la vente soit un burger.
|
||||
\[
|
||||
P(B) = P(M\cap B) + P(S\cap B) = 0.62 \times 0.75 + 0.38 \times 0.29 = 0.4248
|
||||
\]
|
||||
\item On cherche à calculer la quantité $P_B(S)$. Pour cela on utilise la formule de Bayes
|
||||
\[
|
||||
P_B(S) = \frac{P(B\cap S)}{P(B)} = \frac{P_S(B) \times P(S)}{P(B)} = \frac{0.71\times 0.38}{0.4248} = 0.6351224105461393 \approx 0.635
|
||||
\]
|
||||
\end{enumerate}
|
||||
\end{solution}
|
||||
|
||||
\begin{exercise}[subtitle={Continent plastique}]
|
||||
\textit{Les quantités évoqués dans cette exercice sont générés au hasard et sont donc complètement farfelus.}
|
||||
\medskip
|
||||
Le \og continent de plastique\fg{} est la plus grande des plaques de déchets plastiques évoluant sur les océans. Elle occupe actuellement dans l'océan Pacifique une surface dont l'aire est évaluée à plus de $1,6$ million de km$^2$, entre Hawaï et la Californie.
|
||||
|
||||
En 2017, des scientifiques ont estimé qu'il y avait $4$ millions de tonnes de déchets plastiques qui était déversé chaque année dans les océans et que cette quantité augmentait de $30\n\%$ par chaque année.
|
||||
|
||||
On modélise l'évolution de la masse de ces déchets plastiques déversée chaque année, si rien n'est fait pour la réduire, par une suite géométrique $\left(u_n\right)$. L'arrondi au centième du terme $u_n$ représente la masse de ces déchets déversée chaque année, exprimée en million de tonnes, pour l'année $(2017 + n)$.
|
||||
|
||||
\medskip
|
||||
|
||||
\begin{enumerate}
|
||||
\item Expliquer pourquoi la suite $u_n$ est géométrique?
|
||||
\item Calculer $u_1$ et $u_2$.
|
||||
\item Exprimer $u_n$ en fonction de $n$.
|
||||
\item Au début de l'année 2017, il y avait $300$ millions de tonnes de déchets plastique. Calculer la quantité totale de déchets plastiques en 2030.
|
||||
\item On souhaite déterminer en quelle année la masse totale de ces déchets plastiques aura pour la première fois augmenté de $50$\,\% par rapport à sa valeur de 2017.
|
||||
\begin{enumerate}
|
||||
\item Recopier et compléter l'algorithme ci-dessous pour que la variable $N$ contienne la réponse au problème posé.
|
||||
|
||||
\begin{center}
|
||||
\begin{tabularx}{0.4\linewidth}{|X|}\hline
|
||||
$N = 2017$\\
|
||||
$U = 4$ \\
|
||||
$S = 300 + U$ \\
|
||||
while $S < 450$: \\
|
||||
\hspace{1cm} $N = \ldots$\\
|
||||
\hspace{1cm} $U = \ldots$\\
|
||||
\hspace{1cm} $S = \ldots$\\
|
||||
\hline
|
||||
\end{tabularx}
|
||||
\end{center}
|
||||
\item Que contiennent les variables $S$, $U$ et $N$ après exécution de cet algorithme ?
|
||||
|
||||
Interpréter les résultats dans le contexte de l'exercice.
|
||||
\end{enumerate}
|
||||
\end{enumerate}
|
||||
\end{exercise}
|
||||
|
||||
\begin{solution}
|
||||
\begin{enumerate}
|
||||
\item Une augmentation de $30\,\%$ revient à multiplier la quantité par $1.3$. La suite est donc bien géométrique. Son premier terme est $u_0 = 4$ et sa raison est $q = 1.3$
|
||||
\item
|
||||
\[
|
||||
u_1 = u_0 * 1.3 = 5.2
|
||||
\]
|
||||
\[
|
||||
u_2 = u_0 * 1.3^2 = 6.76
|
||||
\]
|
||||
\item
|
||||
\[
|
||||
u_n = u_0 \times q^n = 4 \times 1.3^n
|
||||
\]
|
||||
\item On calcule la quantité totale déversée entre 2017 et 2030.
|
||||
\[
|
||||
\sum_{n = 0}^{13} u_n = u_0 \times \frac{1-q^{13}}{1-q} = 4 \times \frac{1 - 1.3^{13}}{1 - 1.3} = 390.5
|
||||
\]
|
||||
On en déduit la quantité totale de déchets en 2030
|
||||
\[
|
||||
300 + 390.5 = 690.5
|
||||
\]
|
||||
\item
|
||||
\begin{enumerate}
|
||||
\item ~
|
||||
\begin{center}
|
||||
\begin{tabularx}{0.4\linewidth}{|X|}\hline
|
||||
$N \gets 2017$\\
|
||||
$U \gets 4$ \\
|
||||
$S \gets 300 + U$ \\
|
||||
Tant que $S < 450$ \\
|
||||
\hspace{1cm} $N \gets N + 1$\\
|
||||
\hspace{1cm} $U \gets U * 1.3$\\
|
||||
\hspace{1cm} $S \gets S + u$\\
|
||||
Fin Tant que\\\hline
|
||||
\end{tabularx}
|
||||
\end{center}
|
||||
\item \textit{Pas de correction automatisé}
|
||||
\end{enumerate}
|
||||
\end{enumerate}
|
||||
\end{solution}
|
||||
|
||||
\end{document}
|
||||
|
||||
%%% Local Variables:
|
||||
%%% mode: latex
|
||||
%%% TeX-master: "master"
|
||||
%%% End:
|
262
TST/DS/DS_21_04_07/TST1/corr_18_210407_DS8.tex
Normal file
262
TST/DS/DS_21_04_07/TST1/corr_18_210407_DS8.tex
Normal file
@ -0,0 +1,262 @@
|
||||
\documentclass[a4paper,10pt]{article}
|
||||
\usepackage{myXsim}
|
||||
|
||||
% Title Page
|
||||
\title{DS8 \hfill MOUFAQ Amine}
|
||||
\tribe{TST}
|
||||
\date{\hfillÀ render pour le Mercredi 7 avril}
|
||||
|
||||
\xsimsetup{
|
||||
solution/print = true
|
||||
}
|
||||
|
||||
\begin{document}
|
||||
\maketitle
|
||||
|
||||
\begin{exercise}[subtitle={Automatismes}]
|
||||
\textit{Toutes les questions de cette exercice sont indépendantes et peuvent être répondus séparément}
|
||||
\begin{enumerate}
|
||||
\item De janvier à septembre, une quantité a augmenté de $16\,\%$. Faire un schéma pour représenter la situation puis calculer le taux d'évolution moyen mensuel.
|
||||
\item Une quantité augmente de $16\,\%$ par ans. En 2020, elle est de 115\euro. Quelle était sa valeur en 2019? Faire un schéma pour représenter la situation.
|
||||
\item Déterminer l'équation de la droite \\
|
||||
\begin{tikzpicture}[xscale=0.8, yscale=0.5]
|
||||
\tkzInit[xmin=-5,xmax=5,xstep=1,
|
||||
ymin=-5,ymax=5,ystep=1]
|
||||
\tkzGrid
|
||||
\tkzAxeXY
|
||||
\tkzFct[domain=-5:5,color=red,very thick]%
|
||||
{0.5*\x -1};
|
||||
\end{tikzpicture}
|
||||
\item Résoudre l'équation $6 \times 0.24^x = 41$
|
||||
\end{enumerate}
|
||||
\end{exercise}
|
||||
|
||||
\begin{solution}
|
||||
\begin{enumerate}
|
||||
\item On veut partager cette évolution en 8 évolutions.
|
||||
\[
|
||||
\left(1 + \frac{16}{100}\right)^{\frac{1}{8}} = 1.0187
|
||||
\]
|
||||
Donc le taux d'évolution moyen est
|
||||
\[
|
||||
t_m = 1.0187 - 1 = 0.01869999999999994
|
||||
\]
|
||||
\item Coefficient multiplicateur pour revenir en arrière
|
||||
\[
|
||||
CM = (1 + \frac{16}{100})^{-1} = 0.8621
|
||||
\]
|
||||
On en déduit la quantité en 2019
|
||||
\[
|
||||
115 * 0.8621 = 99.1415
|
||||
\]
|
||||
\item L'équation de la droite est
|
||||
\[
|
||||
y = 0.5 x -1
|
||||
\]
|
||||
\item Il faut penser à faire la division à par $6$ avant d'utiliser le log car sinon, on ne peut pas utiliser la formule $\log(a^n) = n\times \log(a)$.
|
||||
|
||||
\[x = \frac{\log(6.83)}{\log(0.24)}\]
|
||||
\end{enumerate}
|
||||
\end{solution}
|
||||
|
||||
\begin{exercise}[subtitle={Restaurant}]
|
||||
Un \emph{food truck}, ouvert le midi et le soir, propose deux types de formules :
|
||||
|
||||
\setlength\parindent{10mm}
|
||||
\begin{itemize}
|
||||
\item la formule \emph{Burger} ;
|
||||
\item la formule \emph{Wok}.
|
||||
\end{itemize}
|
||||
\setlength\parindent{0mm}
|
||||
|
||||
\medskip
|
||||
|
||||
Le gérant a remarqué que 78\,\% de ses ventes ont lieu le midi. Le quart des ventes du midi correspondent à la formule \emph{Burger}, alors que 37\,\% des ventes du soir correspondent à la formule \emph{Wok}.
|
||||
|
||||
Le gérant se constitue un fichier en notant, pour chaque vente, la formule choisie et le moment de cette vente (midi ou soir).
|
||||
|
||||
On prélève une fiche de façon équiprobable. On définit les quatre évènements suivants:
|
||||
|
||||
\begin{enumerate}
|
||||
\item $M$ : \og la fiche correspond à une vente du midi\fg{} ;
|
||||
\item $S$ : \og la fiche correspond à une vente du soir\fg {};
|
||||
\item $W$ : \og la fiche correspond à une formule \emph{Wok} \fg{} ;
|
||||
\item $B$ : \og la fiche correspond à une formule \emph{Burger} \fg.
|
||||
\end{enumerate}
|
||||
\setlength\parindent{0mm}
|
||||
|
||||
\medskip
|
||||
|
||||
\begin{enumerate}
|
||||
\item Recopier puis compléter l'arbre pondéré
|
||||
|
||||
\begin{center}
|
||||
\begin{tikzpicture}[sloped]
|
||||
\node {.}
|
||||
child {node {$M$}
|
||||
child {node {$W$}
|
||||
edge from parent
|
||||
node[above] {...}
|
||||
}
|
||||
child {node {$B$}
|
||||
edge from parent
|
||||
node[above] {...}
|
||||
}
|
||||
edge from parent
|
||||
node[above] {...}
|
||||
}
|
||||
child[missing] {}
|
||||
child { node {$S$}
|
||||
child {node {$W$}
|
||||
edge from parent
|
||||
node[above] {...}
|
||||
}
|
||||
child {node {$B$}
|
||||
edge from parent
|
||||
node[above] {...}
|
||||
}
|
||||
edge from parent
|
||||
node[above] {...}
|
||||
} ;
|
||||
\end{tikzpicture}
|
||||
\end{center}
|
||||
|
||||
\item Calculer la probabilité de l'évènement $M \cap W$. Interpréter ce résultat dans le contexte de l'exercice.
|
||||
\item Montrer que la probabilité que la fiche choisie corresponde à une formule \emph{Burger} est égale à $0.3336$.
|
||||
\item On a prélevé une fiche correspondant à la formule \emph{Burger}. Quelle est la probabilité, arrondie au millième, que la vente ait eu lieu le soir?
|
||||
\end{enumerate}
|
||||
\end{exercise}
|
||||
|
||||
\begin{solution}
|
||||
\begin{enumerate}
|
||||
\item
|
||||
\begin{center}
|
||||
\begin{tikzpicture}[sloped]
|
||||
\node {.}
|
||||
child {node {$M$}
|
||||
child {node {$W$}
|
||||
edge from parent
|
||||
node[above] {$0.75$}
|
||||
}
|
||||
child {node {$B$}
|
||||
edge from parent
|
||||
node[above] {$0.25$}
|
||||
}
|
||||
edge from parent
|
||||
node[above] {$0.78$}
|
||||
}
|
||||
child[missing] {}
|
||||
child { node {$S$}
|
||||
child {node {$W$}
|
||||
edge from parent
|
||||
node[above] {$0.37$}
|
||||
}
|
||||
child {node {$B$}
|
||||
edge from parent
|
||||
node[above] {$0.63$}
|
||||
}
|
||||
edge from parent
|
||||
node[above] {$0.22$}
|
||||
} ;
|
||||
\end{tikzpicture}
|
||||
\end{center}
|
||||
\item On calcule la probabilité que la vente soit un wok et ait eu lieu à midi
|
||||
\[ P(M\cap W) = P(M) \times P_M(W) = 0.78 \times 0.75 = 0.585 \]
|
||||
\item Probabilité que la vente soit un burger.
|
||||
\[
|
||||
P(B) = P(M\cap B) + P(S\cap B) = 0.78 \times 0.75 + 0.22 \times 0.37 = 0.3336
|
||||
\]
|
||||
\item On cherche à calculer la quantité $P_B(S)$. Pour cela on utilise la formule de Bayes
|
||||
\[
|
||||
P_B(S) = \frac{P(B\cap S)}{P(B)} = \frac{P_S(B) \times P(S)}{P(B)} = \frac{0.63\times 0.22}{0.3336} = 0.41546762589928055 \approx 0.415
|
||||
\]
|
||||
\end{enumerate}
|
||||
\end{solution}
|
||||
|
||||
\begin{exercise}[subtitle={Continent plastique}]
|
||||
\textit{Les quantités évoqués dans cette exercice sont générés au hasard et sont donc complètement farfelus.}
|
||||
\medskip
|
||||
Le \og continent de plastique\fg{} est la plus grande des plaques de déchets plastiques évoluant sur les océans. Elle occupe actuellement dans l'océan Pacifique une surface dont l'aire est évaluée à plus de $1,6$ million de km$^2$, entre Hawaï et la Californie.
|
||||
|
||||
En 2017, des scientifiques ont estimé qu'il y avait $10$ millions de tonnes de déchets plastiques qui était déversé chaque année dans les océans et que cette quantité augmentait de $24\n\%$ par chaque année.
|
||||
|
||||
On modélise l'évolution de la masse de ces déchets plastiques déversée chaque année, si rien n'est fait pour la réduire, par une suite géométrique $\left(u_n\right)$. L'arrondi au centième du terme $u_n$ représente la masse de ces déchets déversée chaque année, exprimée en million de tonnes, pour l'année $(2017 + n)$.
|
||||
|
||||
\medskip
|
||||
|
||||
\begin{enumerate}
|
||||
\item Expliquer pourquoi la suite $u_n$ est géométrique?
|
||||
\item Calculer $u_1$ et $u_2$.
|
||||
\item Exprimer $u_n$ en fonction de $n$.
|
||||
\item Au début de l'année 2017, il y avait $300$ millions de tonnes de déchets plastique. Calculer la quantité totale de déchets plastiques en 2030.
|
||||
\item On souhaite déterminer en quelle année la masse totale de ces déchets plastiques aura pour la première fois augmenté de $50$\,\% par rapport à sa valeur de 2017.
|
||||
\begin{enumerate}
|
||||
\item Recopier et compléter l'algorithme ci-dessous pour que la variable $N$ contienne la réponse au problème posé.
|
||||
|
||||
\begin{center}
|
||||
\begin{tabularx}{0.4\linewidth}{|X|}\hline
|
||||
$N = 2017$\\
|
||||
$U = 10$ \\
|
||||
$S = 300 + U$ \\
|
||||
while $S < 450$: \\
|
||||
\hspace{1cm} $N = \ldots$\\
|
||||
\hspace{1cm} $U = \ldots$\\
|
||||
\hspace{1cm} $S = \ldots$\\
|
||||
\hline
|
||||
\end{tabularx}
|
||||
\end{center}
|
||||
\item Que contiennent les variables $S$, $U$ et $N$ après exécution de cet algorithme ?
|
||||
|
||||
Interpréter les résultats dans le contexte de l'exercice.
|
||||
\end{enumerate}
|
||||
\end{enumerate}
|
||||
\end{exercise}
|
||||
|
||||
\begin{solution}
|
||||
\begin{enumerate}
|
||||
\item Une augmentation de $24\,\%$ revient à multiplier la quantité par $1.24$. La suite est donc bien géométrique. Son premier terme est $u_0 = 10$ et sa raison est $q = 1.24$
|
||||
\item
|
||||
\[
|
||||
u_1 = u_0 * 1.24 = 12.4
|
||||
\]
|
||||
\[
|
||||
u_2 = u_0 * 1.24^2 = 15.376
|
||||
\]
|
||||
\item
|
||||
\[
|
||||
u_n = u_0 \times q^n = 10 \times 1.24^n
|
||||
\]
|
||||
\item On calcule la quantité totale déversée entre 2017 et 2030.
|
||||
\[
|
||||
\sum_{n = 0}^{13} u_n = u_0 \times \frac{1-q^{13}}{1-q} = 10 \times \frac{1 - 1.24^{13}}{1 - 1.24} = 641.1
|
||||
\]
|
||||
On en déduit la quantité totale de déchets en 2030
|
||||
\[
|
||||
300 + 641.1 = 941.1
|
||||
\]
|
||||
\item
|
||||
\begin{enumerate}
|
||||
\item ~
|
||||
\begin{center}
|
||||
\begin{tabularx}{0.4\linewidth}{|X|}\hline
|
||||
$N \gets 2017$\\
|
||||
$U \gets 10$ \\
|
||||
$S \gets 300 + U$ \\
|
||||
Tant que $S < 450$ \\
|
||||
\hspace{1cm} $N \gets N + 1$\\
|
||||
\hspace{1cm} $U \gets U * 1.24$\\
|
||||
\hspace{1cm} $S \gets S + u$\\
|
||||
Fin Tant que\\\hline
|
||||
\end{tabularx}
|
||||
\end{center}
|
||||
\item \textit{Pas de correction automatisé}
|
||||
\end{enumerate}
|
||||
\end{enumerate}
|
||||
\end{solution}
|
||||
|
||||
\end{document}
|
||||
|
||||
%%% Local Variables:
|
||||
%%% mode: latex
|
||||
%%% TeX-master: "master"
|
||||
%%% End:
|
262
TST/DS/DS_21_04_07/TST1/corr_19_210407_DS8.tex
Normal file
262
TST/DS/DS_21_04_07/TST1/corr_19_210407_DS8.tex
Normal file
@ -0,0 +1,262 @@
|
||||
\documentclass[a4paper,10pt]{article}
|
||||
\usepackage{myXsim}
|
||||
|
||||
% Title Page
|
||||
\title{DS8 \hfill ONAL Yakub}
|
||||
\tribe{TST}
|
||||
\date{\hfillÀ render pour le Mercredi 7 avril}
|
||||
|
||||
\xsimsetup{
|
||||
solution/print = true
|
||||
}
|
||||
|
||||
\begin{document}
|
||||
\maketitle
|
||||
|
||||
\begin{exercise}[subtitle={Automatismes}]
|
||||
\textit{Toutes les questions de cette exercice sont indépendantes et peuvent être répondus séparément}
|
||||
\begin{enumerate}
|
||||
\item De janvier à septembre, une quantité a augmenté de $27\,\%$. Faire un schéma pour représenter la situation puis calculer le taux d'évolution moyen mensuel.
|
||||
\item Une quantité augmente de $27\,\%$ par ans. En 2020, elle est de 132\euro. Quelle était sa valeur en 2019? Faire un schéma pour représenter la situation.
|
||||
\item Déterminer l'équation de la droite \\
|
||||
\begin{tikzpicture}[xscale=0.8, yscale=0.5]
|
||||
\tkzInit[xmin=-5,xmax=5,xstep=1,
|
||||
ymin=-5,ymax=5,ystep=1]
|
||||
\tkzGrid
|
||||
\tkzAxeXY
|
||||
\tkzFct[domain=-5:5,color=red,very thick]%
|
||||
{1.0*\x -1};
|
||||
\end{tikzpicture}
|
||||
\item Résoudre l'équation $10 \times 0.18^x = 7$
|
||||
\end{enumerate}
|
||||
\end{exercise}
|
||||
|
||||
\begin{solution}
|
||||
\begin{enumerate}
|
||||
\item On veut partager cette évolution en 8 évolutions.
|
||||
\[
|
||||
\left(1 + \frac{27}{100}\right)^{\frac{1}{8}} = 1.0303
|
||||
\]
|
||||
Donc le taux d'évolution moyen est
|
||||
\[
|
||||
t_m = 1.0303 - 1 = 0.030299999999999994
|
||||
\]
|
||||
\item Coefficient multiplicateur pour revenir en arrière
|
||||
\[
|
||||
CM = (1 + \frac{27}{100})^{-1} = 0.7874
|
||||
\]
|
||||
On en déduit la quantité en 2019
|
||||
\[
|
||||
132 * 0.7874 = 103.9368
|
||||
\]
|
||||
\item L'équation de la droite est
|
||||
\[
|
||||
y = 1.0 x -1
|
||||
\]
|
||||
\item Il faut penser à faire la division à par $10$ avant d'utiliser le log car sinon, on ne peut pas utiliser la formule $\log(a^n) = n\times \log(a)$.
|
||||
|
||||
\[x = \frac{\log(0.7)}{\log(0.18)}\]
|
||||
\end{enumerate}
|
||||
\end{solution}
|
||||
|
||||
\begin{exercise}[subtitle={Restaurant}]
|
||||
Un \emph{food truck}, ouvert le midi et le soir, propose deux types de formules :
|
||||
|
||||
\setlength\parindent{10mm}
|
||||
\begin{itemize}
|
||||
\item la formule \emph{Burger} ;
|
||||
\item la formule \emph{Wok}.
|
||||
\end{itemize}
|
||||
\setlength\parindent{0mm}
|
||||
|
||||
\medskip
|
||||
|
||||
Le gérant a remarqué que 78\,\% de ses ventes ont lieu le midi. Le quart des ventes du midi correspondent à la formule \emph{Burger}, alors que 28\,\% des ventes du soir correspondent à la formule \emph{Wok}.
|
||||
|
||||
Le gérant se constitue un fichier en notant, pour chaque vente, la formule choisie et le moment de cette vente (midi ou soir).
|
||||
|
||||
On prélève une fiche de façon équiprobable. On définit les quatre évènements suivants:
|
||||
|
||||
\begin{enumerate}
|
||||
\item $M$ : \og la fiche correspond à une vente du midi\fg{} ;
|
||||
\item $S$ : \og la fiche correspond à une vente du soir\fg {};
|
||||
\item $W$ : \og la fiche correspond à une formule \emph{Wok} \fg{} ;
|
||||
\item $B$ : \og la fiche correspond à une formule \emph{Burger} \fg.
|
||||
\end{enumerate}
|
||||
\setlength\parindent{0mm}
|
||||
|
||||
\medskip
|
||||
|
||||
\begin{enumerate}
|
||||
\item Recopier puis compléter l'arbre pondéré
|
||||
|
||||
\begin{center}
|
||||
\begin{tikzpicture}[sloped]
|
||||
\node {.}
|
||||
child {node {$M$}
|
||||
child {node {$W$}
|
||||
edge from parent
|
||||
node[above] {...}
|
||||
}
|
||||
child {node {$B$}
|
||||
edge from parent
|
||||
node[above] {...}
|
||||
}
|
||||
edge from parent
|
||||
node[above] {...}
|
||||
}
|
||||
child[missing] {}
|
||||
child { node {$S$}
|
||||
child {node {$W$}
|
||||
edge from parent
|
||||
node[above] {...}
|
||||
}
|
||||
child {node {$B$}
|
||||
edge from parent
|
||||
node[above] {...}
|
||||
}
|
||||
edge from parent
|
||||
node[above] {...}
|
||||
} ;
|
||||
\end{tikzpicture}
|
||||
\end{center}
|
||||
|
||||
\item Calculer la probabilité de l'évènement $M \cap W$. Interpréter ce résultat dans le contexte de l'exercice.
|
||||
\item Montrer que la probabilité que la fiche choisie corresponde à une formule \emph{Burger} est égale à $0.3534$.
|
||||
\item On a prélevé une fiche correspondant à la formule \emph{Burger}. Quelle est la probabilité, arrondie au millième, que la vente ait eu lieu le soir?
|
||||
\end{enumerate}
|
||||
\end{exercise}
|
||||
|
||||
\begin{solution}
|
||||
\begin{enumerate}
|
||||
\item
|
||||
\begin{center}
|
||||
\begin{tikzpicture}[sloped]
|
||||
\node {.}
|
||||
child {node {$M$}
|
||||
child {node {$W$}
|
||||
edge from parent
|
||||
node[above] {$0.75$}
|
||||
}
|
||||
child {node {$B$}
|
||||
edge from parent
|
||||
node[above] {$0.25$}
|
||||
}
|
||||
edge from parent
|
||||
node[above] {$0.78$}
|
||||
}
|
||||
child[missing] {}
|
||||
child { node {$S$}
|
||||
child {node {$W$}
|
||||
edge from parent
|
||||
node[above] {$0.28$}
|
||||
}
|
||||
child {node {$B$}
|
||||
edge from parent
|
||||
node[above] {$0.72$}
|
||||
}
|
||||
edge from parent
|
||||
node[above] {$0.22$}
|
||||
} ;
|
||||
\end{tikzpicture}
|
||||
\end{center}
|
||||
\item On calcule la probabilité que la vente soit un wok et ait eu lieu à midi
|
||||
\[ P(M\cap W) = P(M) \times P_M(W) = 0.78 \times 0.75 = 0.585 \]
|
||||
\item Probabilité que la vente soit un burger.
|
||||
\[
|
||||
P(B) = P(M\cap B) + P(S\cap B) = 0.78 \times 0.75 + 0.22 \times 0.28 = 0.3534
|
||||
\]
|
||||
\item On cherche à calculer la quantité $P_B(S)$. Pour cela on utilise la formule de Bayes
|
||||
\[
|
||||
P_B(S) = \frac{P(B\cap S)}{P(B)} = \frac{P_S(B) \times P(S)}{P(B)} = \frac{0.72\times 0.22}{0.3534} = 0.4482173174872665 \approx 0.448
|
||||
\]
|
||||
\end{enumerate}
|
||||
\end{solution}
|
||||
|
||||
\begin{exercise}[subtitle={Continent plastique}]
|
||||
\textit{Les quantités évoqués dans cette exercice sont générés au hasard et sont donc complètement farfelus.}
|
||||
\medskip
|
||||
Le \og continent de plastique\fg{} est la plus grande des plaques de déchets plastiques évoluant sur les océans. Elle occupe actuellement dans l'océan Pacifique une surface dont l'aire est évaluée à plus de $1,6$ million de km$^2$, entre Hawaï et la Californie.
|
||||
|
||||
En 2017, des scientifiques ont estimé qu'il y avait $3$ millions de tonnes de déchets plastiques qui était déversé chaque année dans les océans et que cette quantité augmentait de $21\n\%$ par chaque année.
|
||||
|
||||
On modélise l'évolution de la masse de ces déchets plastiques déversée chaque année, si rien n'est fait pour la réduire, par une suite géométrique $\left(u_n\right)$. L'arrondi au centième du terme $u_n$ représente la masse de ces déchets déversée chaque année, exprimée en million de tonnes, pour l'année $(2017 + n)$.
|
||||
|
||||
\medskip
|
||||
|
||||
\begin{enumerate}
|
||||
\item Expliquer pourquoi la suite $u_n$ est géométrique?
|
||||
\item Calculer $u_1$ et $u_2$.
|
||||
\item Exprimer $u_n$ en fonction de $n$.
|
||||
\item Au début de l'année 2017, il y avait $300$ millions de tonnes de déchets plastique. Calculer la quantité totale de déchets plastiques en 2030.
|
||||
\item On souhaite déterminer en quelle année la masse totale de ces déchets plastiques aura pour la première fois augmenté de $50$\,\% par rapport à sa valeur de 2017.
|
||||
\begin{enumerate}
|
||||
\item Recopier et compléter l'algorithme ci-dessous pour que la variable $N$ contienne la réponse au problème posé.
|
||||
|
||||
\begin{center}
|
||||
\begin{tabularx}{0.4\linewidth}{|X|}\hline
|
||||
$N = 2017$\\
|
||||
$U = 3$ \\
|
||||
$S = 300 + U$ \\
|
||||
while $S < 450$: \\
|
||||
\hspace{1cm} $N = \ldots$\\
|
||||
\hspace{1cm} $U = \ldots$\\
|
||||
\hspace{1cm} $S = \ldots$\\
|
||||
\hline
|
||||
\end{tabularx}
|
||||
\end{center}
|
||||
\item Que contiennent les variables $S$, $U$ et $N$ après exécution de cet algorithme ?
|
||||
|
||||
Interpréter les résultats dans le contexte de l'exercice.
|
||||
\end{enumerate}
|
||||
\end{enumerate}
|
||||
\end{exercise}
|
||||
|
||||
\begin{solution}
|
||||
\begin{enumerate}
|
||||
\item Une augmentation de $21\,\%$ revient à multiplier la quantité par $1.21$. La suite est donc bien géométrique. Son premier terme est $u_0 = 3$ et sa raison est $q = 1.21$
|
||||
\item
|
||||
\[
|
||||
u_1 = u_0 * 1.21 = 3.63
|
||||
\]
|
||||
\[
|
||||
u_2 = u_0 * 1.21^2 = 4.3923
|
||||
\]
|
||||
\item
|
||||
\[
|
||||
u_n = u_0 \times q^n = 3 \times 1.21^n
|
||||
\]
|
||||
\item On calcule la quantité totale déversée entre 2017 et 2030.
|
||||
\[
|
||||
\sum_{n = 0}^{13} u_n = u_0 \times \frac{1-q^{13}}{1-q} = 3 \times \frac{1 - 1.21^{13}}{1 - 1.21} = 155.97
|
||||
\]
|
||||
On en déduit la quantité totale de déchets en 2030
|
||||
\[
|
||||
300 + 155.97 = 455.97
|
||||
\]
|
||||
\item
|
||||
\begin{enumerate}
|
||||
\item ~
|
||||
\begin{center}
|
||||
\begin{tabularx}{0.4\linewidth}{|X|}\hline
|
||||
$N \gets 2017$\\
|
||||
$U \gets 3$ \\
|
||||
$S \gets 300 + U$ \\
|
||||
Tant que $S < 450$ \\
|
||||
\hspace{1cm} $N \gets N + 1$\\
|
||||
\hspace{1cm} $U \gets U * 1.21$\\
|
||||
\hspace{1cm} $S \gets S + u$\\
|
||||
Fin Tant que\\\hline
|
||||
\end{tabularx}
|
||||
\end{center}
|
||||
\item \textit{Pas de correction automatisé}
|
||||
\end{enumerate}
|
||||
\end{enumerate}
|
||||
\end{solution}
|
||||
|
||||
\end{document}
|
||||
|
||||
%%% Local Variables:
|
||||
%%% mode: latex
|
||||
%%% TeX-master: "master"
|
||||
%%% End:
|
262
TST/DS/DS_21_04_07/TST1/corr_20_210407_DS8.tex
Normal file
262
TST/DS/DS_21_04_07/TST1/corr_20_210407_DS8.tex
Normal file
@ -0,0 +1,262 @@
|
||||
\documentclass[a4paper,10pt]{article}
|
||||
\usepackage{myXsim}
|
||||
|
||||
% Title Page
|
||||
\title{DS8 \hfill SORIANO Laura}
|
||||
\tribe{TST}
|
||||
\date{\hfillÀ render pour le Mercredi 7 avril}
|
||||
|
||||
\xsimsetup{
|
||||
solution/print = true
|
||||
}
|
||||
|
||||
\begin{document}
|
||||
\maketitle
|
||||
|
||||
\begin{exercise}[subtitle={Automatismes}]
|
||||
\textit{Toutes les questions de cette exercice sont indépendantes et peuvent être répondus séparément}
|
||||
\begin{enumerate}
|
||||
\item De janvier à septembre, une quantité a augmenté de $26\,\%$. Faire un schéma pour représenter la situation puis calculer le taux d'évolution moyen mensuel.
|
||||
\item Une quantité augmente de $26\,\%$ par ans. En 2020, elle est de 122\euro. Quelle était sa valeur en 2019? Faire un schéma pour représenter la situation.
|
||||
\item Déterminer l'équation de la droite \\
|
||||
\begin{tikzpicture}[xscale=0.8, yscale=0.5]
|
||||
\tkzInit[xmin=-5,xmax=5,xstep=1,
|
||||
ymin=-5,ymax=5,ystep=1]
|
||||
\tkzGrid
|
||||
\tkzAxeXY
|
||||
\tkzFct[domain=-5:5,color=red,very thick]%
|
||||
{3.0*\x -3};
|
||||
\end{tikzpicture}
|
||||
\item Résoudre l'équation $2 \times 0.02^x = 10$
|
||||
\end{enumerate}
|
||||
\end{exercise}
|
||||
|
||||
\begin{solution}
|
||||
\begin{enumerate}
|
||||
\item On veut partager cette évolution en 8 évolutions.
|
||||
\[
|
||||
\left(1 + \frac{26}{100}\right)^{\frac{1}{8}} = 1.0293
|
||||
\]
|
||||
Donc le taux d'évolution moyen est
|
||||
\[
|
||||
t_m = 1.0293 - 1 = 0.029300000000000104
|
||||
\]
|
||||
\item Coefficient multiplicateur pour revenir en arrière
|
||||
\[
|
||||
CM = (1 + \frac{26}{100})^{-1} = 0.7937
|
||||
\]
|
||||
On en déduit la quantité en 2019
|
||||
\[
|
||||
122 * 0.7937 = 96.8314
|
||||
\]
|
||||
\item L'équation de la droite est
|
||||
\[
|
||||
y = 3.0 x -3
|
||||
\]
|
||||
\item Il faut penser à faire la division à par $2$ avant d'utiliser le log car sinon, on ne peut pas utiliser la formule $\log(a^n) = n\times \log(a)$.
|
||||
|
||||
\[x = \frac{\log(5.0)}{\log(0.02)}\]
|
||||
\end{enumerate}
|
||||
\end{solution}
|
||||
|
||||
\begin{exercise}[subtitle={Restaurant}]
|
||||
Un \emph{food truck}, ouvert le midi et le soir, propose deux types de formules :
|
||||
|
||||
\setlength\parindent{10mm}
|
||||
\begin{itemize}
|
||||
\item la formule \emph{Burger} ;
|
||||
\item la formule \emph{Wok}.
|
||||
\end{itemize}
|
||||
\setlength\parindent{0mm}
|
||||
|
||||
\medskip
|
||||
|
||||
Le gérant a remarqué que 0\,\% de ses ventes ont lieu le midi. Le quart des ventes du midi correspondent à la formule \emph{Burger}, alors que 21\,\% des ventes du soir correspondent à la formule \emph{Wok}.
|
||||
|
||||
Le gérant se constitue un fichier en notant, pour chaque vente, la formule choisie et le moment de cette vente (midi ou soir).
|
||||
|
||||
On prélève une fiche de façon équiprobable. On définit les quatre évènements suivants:
|
||||
|
||||
\begin{enumerate}
|
||||
\item $M$ : \og la fiche correspond à une vente du midi\fg{} ;
|
||||
\item $S$ : \og la fiche correspond à une vente du soir\fg {};
|
||||
\item $W$ : \og la fiche correspond à une formule \emph{Wok} \fg{} ;
|
||||
\item $B$ : \og la fiche correspond à une formule \emph{Burger} \fg.
|
||||
\end{enumerate}
|
||||
\setlength\parindent{0mm}
|
||||
|
||||
\medskip
|
||||
|
||||
\begin{enumerate}
|
||||
\item Recopier puis compléter l'arbre pondéré
|
||||
|
||||
\begin{center}
|
||||
\begin{tikzpicture}[sloped]
|
||||
\node {.}
|
||||
child {node {$M$}
|
||||
child {node {$W$}
|
||||
edge from parent
|
||||
node[above] {...}
|
||||
}
|
||||
child {node {$B$}
|
||||
edge from parent
|
||||
node[above] {...}
|
||||
}
|
||||
edge from parent
|
||||
node[above] {...}
|
||||
}
|
||||
child[missing] {}
|
||||
child { node {$S$}
|
||||
child {node {$W$}
|
||||
edge from parent
|
||||
node[above] {...}
|
||||
}
|
||||
child {node {$B$}
|
||||
edge from parent
|
||||
node[above] {...}
|
||||
}
|
||||
edge from parent
|
||||
node[above] {...}
|
||||
} ;
|
||||
\end{tikzpicture}
|
||||
\end{center}
|
||||
|
||||
\item Calculer la probabilité de l'évènement $M \cap W$. Interpréter ce résultat dans le contexte de l'exercice.
|
||||
\item Montrer que la probabilité que la fiche choisie corresponde à une formule \emph{Burger} est égale à $0.79$.
|
||||
\item On a prélevé une fiche correspondant à la formule \emph{Burger}. Quelle est la probabilité, arrondie au millième, que la vente ait eu lieu le soir?
|
||||
\end{enumerate}
|
||||
\end{exercise}
|
||||
|
||||
\begin{solution}
|
||||
\begin{enumerate}
|
||||
\item
|
||||
\begin{center}
|
||||
\begin{tikzpicture}[sloped]
|
||||
\node {.}
|
||||
child {node {$M$}
|
||||
child {node {$W$}
|
||||
edge from parent
|
||||
node[above] {$0.75$}
|
||||
}
|
||||
child {node {$B$}
|
||||
edge from parent
|
||||
node[above] {$0.25$}
|
||||
}
|
||||
edge from parent
|
||||
node[above] {$0.0$}
|
||||
}
|
||||
child[missing] {}
|
||||
child { node {$S$}
|
||||
child {node {$W$}
|
||||
edge from parent
|
||||
node[above] {$0.21$}
|
||||
}
|
||||
child {node {$B$}
|
||||
edge from parent
|
||||
node[above] {$0.79$}
|
||||
}
|
||||
edge from parent
|
||||
node[above] {$1.0$}
|
||||
} ;
|
||||
\end{tikzpicture}
|
||||
\end{center}
|
||||
\item On calcule la probabilité que la vente soit un wok et ait eu lieu à midi
|
||||
\[ P(M\cap W) = P(M) \times P_M(W) = 0.0 \times 0.75 = 0.0 \]
|
||||
\item Probabilité que la vente soit un burger.
|
||||
\[
|
||||
P(B) = P(M\cap B) + P(S\cap B) = 0.0 \times 0.75 + 1.0 \times 0.21 = 0.79
|
||||
\]
|
||||
\item On cherche à calculer la quantité $P_B(S)$. Pour cela on utilise la formule de Bayes
|
||||
\[
|
||||
P_B(S) = \frac{P(B\cap S)}{P(B)} = \frac{P_S(B) \times P(S)}{P(B)} = \frac{0.79\times 1.0}{0.79} = 1.0 \approx 1.0
|
||||
\]
|
||||
\end{enumerate}
|
||||
\end{solution}
|
||||
|
||||
\begin{exercise}[subtitle={Continent plastique}]
|
||||
\textit{Les quantités évoqués dans cette exercice sont générés au hasard et sont donc complètement farfelus.}
|
||||
\medskip
|
||||
Le \og continent de plastique\fg{} est la plus grande des plaques de déchets plastiques évoluant sur les océans. Elle occupe actuellement dans l'océan Pacifique une surface dont l'aire est évaluée à plus de $1,6$ million de km$^2$, entre Hawaï et la Californie.
|
||||
|
||||
En 2017, des scientifiques ont estimé qu'il y avait $11$ millions de tonnes de déchets plastiques qui était déversé chaque année dans les océans et que cette quantité augmentait de $16\n\%$ par chaque année.
|
||||
|
||||
On modélise l'évolution de la masse de ces déchets plastiques déversée chaque année, si rien n'est fait pour la réduire, par une suite géométrique $\left(u_n\right)$. L'arrondi au centième du terme $u_n$ représente la masse de ces déchets déversée chaque année, exprimée en million de tonnes, pour l'année $(2017 + n)$.
|
||||
|
||||
\medskip
|
||||
|
||||
\begin{enumerate}
|
||||
\item Expliquer pourquoi la suite $u_n$ est géométrique?
|
||||
\item Calculer $u_1$ et $u_2$.
|
||||
\item Exprimer $u_n$ en fonction de $n$.
|
||||
\item Au début de l'année 2017, il y avait $300$ millions de tonnes de déchets plastique. Calculer la quantité totale de déchets plastiques en 2030.
|
||||
\item On souhaite déterminer en quelle année la masse totale de ces déchets plastiques aura pour la première fois augmenté de $50$\,\% par rapport à sa valeur de 2017.
|
||||
\begin{enumerate}
|
||||
\item Recopier et compléter l'algorithme ci-dessous pour que la variable $N$ contienne la réponse au problème posé.
|
||||
|
||||
\begin{center}
|
||||
\begin{tabularx}{0.4\linewidth}{|X|}\hline
|
||||
$N = 2017$\\
|
||||
$U = 11$ \\
|
||||
$S = 300 + U$ \\
|
||||
while $S < 450$: \\
|
||||
\hspace{1cm} $N = \ldots$\\
|
||||
\hspace{1cm} $U = \ldots$\\
|
||||
\hspace{1cm} $S = \ldots$\\
|
||||
\hline
|
||||
\end{tabularx}
|
||||
\end{center}
|
||||
\item Que contiennent les variables $S$, $U$ et $N$ après exécution de cet algorithme ?
|
||||
|
||||
Interpréter les résultats dans le contexte de l'exercice.
|
||||
\end{enumerate}
|
||||
\end{enumerate}
|
||||
\end{exercise}
|
||||
|
||||
\begin{solution}
|
||||
\begin{enumerate}
|
||||
\item Une augmentation de $16\,\%$ revient à multiplier la quantité par $1.16$. La suite est donc bien géométrique. Son premier terme est $u_0 = 11$ et sa raison est $q = 1.16$
|
||||
\item
|
||||
\[
|
||||
u_1 = u_0 * 1.16 = 12.76
|
||||
\]
|
||||
\[
|
||||
u_2 = u_0 * 1.16^2 = 14.8016
|
||||
\]
|
||||
\item
|
||||
\[
|
||||
u_n = u_0 \times q^n = 11 \times 1.16^n
|
||||
\]
|
||||
\item On calcule la quantité totale déversée entre 2017 et 2030.
|
||||
\[
|
||||
\sum_{n = 0}^{13} u_n = u_0 \times \frac{1-q^{13}}{1-q} = 11 \times \frac{1 - 1.16^{13}}{1 - 1.16} = 404.65
|
||||
\]
|
||||
On en déduit la quantité totale de déchets en 2030
|
||||
\[
|
||||
300 + 404.65 = 704.65
|
||||
\]
|
||||
\item
|
||||
\begin{enumerate}
|
||||
\item ~
|
||||
\begin{center}
|
||||
\begin{tabularx}{0.4\linewidth}{|X|}\hline
|
||||
$N \gets 2017$\\
|
||||
$U \gets 11$ \\
|
||||
$S \gets 300 + U$ \\
|
||||
Tant que $S < 450$ \\
|
||||
\hspace{1cm} $N \gets N + 1$\\
|
||||
\hspace{1cm} $U \gets U * 1.16$\\
|
||||
\hspace{1cm} $S \gets S + u$\\
|
||||
Fin Tant que\\\hline
|
||||
\end{tabularx}
|
||||
\end{center}
|
||||
\item \textit{Pas de correction automatisé}
|
||||
\end{enumerate}
|
||||
\end{enumerate}
|
||||
\end{solution}
|
||||
|
||||
\end{document}
|
||||
|
||||
%%% Local Variables:
|
||||
%%% mode: latex
|
||||
%%% TeX-master: "master"
|
||||
%%% End:
|
262
TST/DS/DS_21_04_07/TST1/corr_21_210407_DS8.tex
Normal file
262
TST/DS/DS_21_04_07/TST1/corr_21_210407_DS8.tex
Normal file
@ -0,0 +1,262 @@
|
||||
\documentclass[a4paper,10pt]{article}
|
||||
\usepackage{myXsim}
|
||||
|
||||
% Title Page
|
||||
\title{DS8 \hfill VECCHIO Léa}
|
||||
\tribe{TST}
|
||||
\date{\hfillÀ render pour le Mercredi 7 avril}
|
||||
|
||||
\xsimsetup{
|
||||
solution/print = true
|
||||
}
|
||||
|
||||
\begin{document}
|
||||
\maketitle
|
||||
|
||||
\begin{exercise}[subtitle={Automatismes}]
|
||||
\textit{Toutes les questions de cette exercice sont indépendantes et peuvent être répondus séparément}
|
||||
\begin{enumerate}
|
||||
\item De janvier à septembre, une quantité a augmenté de $28\,\%$. Faire un schéma pour représenter la situation puis calculer le taux d'évolution moyen mensuel.
|
||||
\item Une quantité augmente de $28\,\%$ par ans. En 2020, elle est de 113\euro. Quelle était sa valeur en 2019? Faire un schéma pour représenter la situation.
|
||||
\item Déterminer l'équation de la droite \\
|
||||
\begin{tikzpicture}[xscale=0.8, yscale=0.5]
|
||||
\tkzInit[xmin=-5,xmax=5,xstep=1,
|
||||
ymin=-5,ymax=5,ystep=1]
|
||||
\tkzGrid
|
||||
\tkzAxeXY
|
||||
\tkzFct[domain=-5:5,color=red,very thick]%
|
||||
{2.0*\x -2};
|
||||
\end{tikzpicture}
|
||||
\item Résoudre l'équation $8 \times 0.84^x = 26$
|
||||
\end{enumerate}
|
||||
\end{exercise}
|
||||
|
||||
\begin{solution}
|
||||
\begin{enumerate}
|
||||
\item On veut partager cette évolution en 8 évolutions.
|
||||
\[
|
||||
\left(1 + \frac{28}{100}\right)^{\frac{1}{8}} = 1.0313
|
||||
\]
|
||||
Donc le taux d'évolution moyen est
|
||||
\[
|
||||
t_m = 1.0313 - 1 = 0.031300000000000106
|
||||
\]
|
||||
\item Coefficient multiplicateur pour revenir en arrière
|
||||
\[
|
||||
CM = (1 + \frac{28}{100})^{-1} = 0.7812
|
||||
\]
|
||||
On en déduit la quantité en 2019
|
||||
\[
|
||||
113 * 0.7812 = 88.2756
|
||||
\]
|
||||
\item L'équation de la droite est
|
||||
\[
|
||||
y = 2.0 x -2
|
||||
\]
|
||||
\item Il faut penser à faire la division à par $8$ avant d'utiliser le log car sinon, on ne peut pas utiliser la formule $\log(a^n) = n\times \log(a)$.
|
||||
|
||||
\[x = \frac{\log(3.25)}{\log(0.84)}\]
|
||||
\end{enumerate}
|
||||
\end{solution}
|
||||
|
||||
\begin{exercise}[subtitle={Restaurant}]
|
||||
Un \emph{food truck}, ouvert le midi et le soir, propose deux types de formules :
|
||||
|
||||
\setlength\parindent{10mm}
|
||||
\begin{itemize}
|
||||
\item la formule \emph{Burger} ;
|
||||
\item la formule \emph{Wok}.
|
||||
\end{itemize}
|
||||
\setlength\parindent{0mm}
|
||||
|
||||
\medskip
|
||||
|
||||
Le gérant a remarqué que 18\,\% de ses ventes ont lieu le midi. Le quart des ventes du midi correspondent à la formule \emph{Burger}, alors que 16\,\% des ventes du soir correspondent à la formule \emph{Wok}.
|
||||
|
||||
Le gérant se constitue un fichier en notant, pour chaque vente, la formule choisie et le moment de cette vente (midi ou soir).
|
||||
|
||||
On prélève une fiche de façon équiprobable. On définit les quatre évènements suivants:
|
||||
|
||||
\begin{enumerate}
|
||||
\item $M$ : \og la fiche correspond à une vente du midi\fg{} ;
|
||||
\item $S$ : \og la fiche correspond à une vente du soir\fg {};
|
||||
\item $W$ : \og la fiche correspond à une formule \emph{Wok} \fg{} ;
|
||||
\item $B$ : \og la fiche correspond à une formule \emph{Burger} \fg.
|
||||
\end{enumerate}
|
||||
\setlength\parindent{0mm}
|
||||
|
||||
\medskip
|
||||
|
||||
\begin{enumerate}
|
||||
\item Recopier puis compléter l'arbre pondéré
|
||||
|
||||
\begin{center}
|
||||
\begin{tikzpicture}[sloped]
|
||||
\node {.}
|
||||
child {node {$M$}
|
||||
child {node {$W$}
|
||||
edge from parent
|
||||
node[above] {...}
|
||||
}
|
||||
child {node {$B$}
|
||||
edge from parent
|
||||
node[above] {...}
|
||||
}
|
||||
edge from parent
|
||||
node[above] {...}
|
||||
}
|
||||
child[missing] {}
|
||||
child { node {$S$}
|
||||
child {node {$W$}
|
||||
edge from parent
|
||||
node[above] {...}
|
||||
}
|
||||
child {node {$B$}
|
||||
edge from parent
|
||||
node[above] {...}
|
||||
}
|
||||
edge from parent
|
||||
node[above] {...}
|
||||
} ;
|
||||
\end{tikzpicture}
|
||||
\end{center}
|
||||
|
||||
\item Calculer la probabilité de l'évènement $M \cap W$. Interpréter ce résultat dans le contexte de l'exercice.
|
||||
\item Montrer que la probabilité que la fiche choisie corresponde à une formule \emph{Burger} est égale à $0.7338$.
|
||||
\item On a prélevé une fiche correspondant à la formule \emph{Burger}. Quelle est la probabilité, arrondie au millième, que la vente ait eu lieu le soir?
|
||||
\end{enumerate}
|
||||
\end{exercise}
|
||||
|
||||
\begin{solution}
|
||||
\begin{enumerate}
|
||||
\item
|
||||
\begin{center}
|
||||
\begin{tikzpicture}[sloped]
|
||||
\node {.}
|
||||
child {node {$M$}
|
||||
child {node {$W$}
|
||||
edge from parent
|
||||
node[above] {$0.75$}
|
||||
}
|
||||
child {node {$B$}
|
||||
edge from parent
|
||||
node[above] {$0.25$}
|
||||
}
|
||||
edge from parent
|
||||
node[above] {$0.18$}
|
||||
}
|
||||
child[missing] {}
|
||||
child { node {$S$}
|
||||
child {node {$W$}
|
||||
edge from parent
|
||||
node[above] {$0.16$}
|
||||
}
|
||||
child {node {$B$}
|
||||
edge from parent
|
||||
node[above] {$0.84$}
|
||||
}
|
||||
edge from parent
|
||||
node[above] {$0.82$}
|
||||
} ;
|
||||
\end{tikzpicture}
|
||||
\end{center}
|
||||
\item On calcule la probabilité que la vente soit un wok et ait eu lieu à midi
|
||||
\[ P(M\cap W) = P(M) \times P_M(W) = 0.18 \times 0.75 = 0.135 \]
|
||||
\item Probabilité que la vente soit un burger.
|
||||
\[
|
||||
P(B) = P(M\cap B) + P(S\cap B) = 0.18 \times 0.75 + 0.82 \times 0.16 = 0.7338
|
||||
\]
|
||||
\item On cherche à calculer la quantité $P_B(S)$. Pour cela on utilise la formule de Bayes
|
||||
\[
|
||||
P_B(S) = \frac{P(B\cap S)}{P(B)} = \frac{P_S(B) \times P(S)}{P(B)} = \frac{0.84\times 0.82}{0.7338} = 0.9386753883892068 \approx 0.939
|
||||
\]
|
||||
\end{enumerate}
|
||||
\end{solution}
|
||||
|
||||
\begin{exercise}[subtitle={Continent plastique}]
|
||||
\textit{Les quantités évoqués dans cette exercice sont générés au hasard et sont donc complètement farfelus.}
|
||||
\medskip
|
||||
Le \og continent de plastique\fg{} est la plus grande des plaques de déchets plastiques évoluant sur les océans. Elle occupe actuellement dans l'océan Pacifique une surface dont l'aire est évaluée à plus de $1,6$ million de km$^2$, entre Hawaï et la Californie.
|
||||
|
||||
En 2017, des scientifiques ont estimé qu'il y avait $2$ millions de tonnes de déchets plastiques qui était déversé chaque année dans les océans et que cette quantité augmentait de $24\n\%$ par chaque année.
|
||||
|
||||
On modélise l'évolution de la masse de ces déchets plastiques déversée chaque année, si rien n'est fait pour la réduire, par une suite géométrique $\left(u_n\right)$. L'arrondi au centième du terme $u_n$ représente la masse de ces déchets déversée chaque année, exprimée en million de tonnes, pour l'année $(2017 + n)$.
|
||||
|
||||
\medskip
|
||||
|
||||
\begin{enumerate}
|
||||
\item Expliquer pourquoi la suite $u_n$ est géométrique?
|
||||
\item Calculer $u_1$ et $u_2$.
|
||||
\item Exprimer $u_n$ en fonction de $n$.
|
||||
\item Au début de l'année 2017, il y avait $300$ millions de tonnes de déchets plastique. Calculer la quantité totale de déchets plastiques en 2030.
|
||||
\item On souhaite déterminer en quelle année la masse totale de ces déchets plastiques aura pour la première fois augmenté de $50$\,\% par rapport à sa valeur de 2017.
|
||||
\begin{enumerate}
|
||||
\item Recopier et compléter l'algorithme ci-dessous pour que la variable $N$ contienne la réponse au problème posé.
|
||||
|
||||
\begin{center}
|
||||
\begin{tabularx}{0.4\linewidth}{|X|}\hline
|
||||
$N = 2017$\\
|
||||
$U = 2$ \\
|
||||
$S = 300 + U$ \\
|
||||
while $S < 450$: \\
|
||||
\hspace{1cm} $N = \ldots$\\
|
||||
\hspace{1cm} $U = \ldots$\\
|
||||
\hspace{1cm} $S = \ldots$\\
|
||||
\hline
|
||||
\end{tabularx}
|
||||
\end{center}
|
||||
\item Que contiennent les variables $S$, $U$ et $N$ après exécution de cet algorithme ?
|
||||
|
||||
Interpréter les résultats dans le contexte de l'exercice.
|
||||
\end{enumerate}
|
||||
\end{enumerate}
|
||||
\end{exercise}
|
||||
|
||||
\begin{solution}
|
||||
\begin{enumerate}
|
||||
\item Une augmentation de $24\,\%$ revient à multiplier la quantité par $1.24$. La suite est donc bien géométrique. Son premier terme est $u_0 = 2$ et sa raison est $q = 1.24$
|
||||
\item
|
||||
\[
|
||||
u_1 = u_0 * 1.24 = 2.48
|
||||
\]
|
||||
\[
|
||||
u_2 = u_0 * 1.24^2 = 3.0752
|
||||
\]
|
||||
\item
|
||||
\[
|
||||
u_n = u_0 \times q^n = 2 \times 1.24^n
|
||||
\]
|
||||
\item On calcule la quantité totale déversée entre 2017 et 2030.
|
||||
\[
|
||||
\sum_{n = 0}^{13} u_n = u_0 \times \frac{1-q^{13}}{1-q} = 2 \times \frac{1 - 1.24^{13}}{1 - 1.24} = 128.22
|
||||
\]
|
||||
On en déduit la quantité totale de déchets en 2030
|
||||
\[
|
||||
300 + 128.22 = 428.22
|
||||
\]
|
||||
\item
|
||||
\begin{enumerate}
|
||||
\item ~
|
||||
\begin{center}
|
||||
\begin{tabularx}{0.4\linewidth}{|X|}\hline
|
||||
$N \gets 2017$\\
|
||||
$U \gets 2$ \\
|
||||
$S \gets 300 + U$ \\
|
||||
Tant que $S < 450$ \\
|
||||
\hspace{1cm} $N \gets N + 1$\\
|
||||
\hspace{1cm} $U \gets U * 1.24$\\
|
||||
\hspace{1cm} $S \gets S + u$\\
|
||||
Fin Tant que\\\hline
|
||||
\end{tabularx}
|
||||
\end{center}
|
||||
\item \textit{Pas de correction automatisé}
|
||||
\end{enumerate}
|
||||
\end{enumerate}
|
||||
\end{solution}
|
||||
|
||||
\end{document}
|
||||
|
||||
%%% Local Variables:
|
||||
%%% mode: latex
|
||||
%%% TeX-master: "master"
|
||||
%%% End:
|
BIN
TST/DS/DS_21_04_07/TST1/corr_all_210407_DS8.pdf
Normal file
BIN
TST/DS/DS_21_04_07/TST1/corr_all_210407_DS8.pdf
Normal file
Binary file not shown.
262
TST/DS/DS_21_04_07/TST3/01_210407_DS8.tex
Normal file
262
TST/DS/DS_21_04_07/TST3/01_210407_DS8.tex
Normal file
@ -0,0 +1,262 @@
|
||||
\documentclass[a4paper,10pt]{article}
|
||||
\usepackage{myXsim}
|
||||
|
||||
% Title Page
|
||||
\title{DS8 \hfill BELARBI Samira}
|
||||
\tribe{TST}
|
||||
\date{\hfillÀ render pour le Mercredi 7 avril}
|
||||
|
||||
\xsimsetup{
|
||||
solution/print = false
|
||||
}
|
||||
|
||||
\begin{document}
|
||||
\maketitle
|
||||
|
||||
\begin{exercise}[subtitle={Automatismes}]
|
||||
\textit{Toutes les questions de cette exercice sont indépendantes et peuvent être répondus séparément}
|
||||
\begin{enumerate}
|
||||
\item De janvier à septembre, une quantité a augmenté de $21\,\%$. Faire un schéma pour représenter la situation puis calculer le taux d'évolution moyen mensuel.
|
||||
\item Une quantité augmente de $21\,\%$ par ans. En 2020, elle est de 143\euro. Quelle était sa valeur en 2019? Faire un schéma pour représenter la situation.
|
||||
\item Déterminer l'équation de la droite \\
|
||||
\begin{tikzpicture}[xscale=0.8, yscale=0.5]
|
||||
\tkzInit[xmin=-5,xmax=5,xstep=1,
|
||||
ymin=-5,ymax=5,ystep=1]
|
||||
\tkzGrid
|
||||
\tkzAxeXY
|
||||
\tkzFct[domain=-5:5,color=red,very thick]%
|
||||
{2.6666666666666665*\x -4};
|
||||
\end{tikzpicture}
|
||||
\item Résoudre l'équation $8 \times 0.76^x = 26$
|
||||
\end{enumerate}
|
||||
\end{exercise}
|
||||
|
||||
\begin{solution}
|
||||
\begin{enumerate}
|
||||
\item On veut partager cette évolution en 8 évolutions.
|
||||
\[
|
||||
\left(1 + \frac{21}{100}\right)^{\frac{1}{8}} = 1.0241
|
||||
\]
|
||||
Donc le taux d'évolution moyen est
|
||||
\[
|
||||
t_m = 1.0241 - 1 = 0.02410000000000001
|
||||
\]
|
||||
\item Coefficient multiplicateur pour revenir en arrière
|
||||
\[
|
||||
CM = (1 + \frac{21}{100})^{-1} = 0.8264
|
||||
\]
|
||||
On en déduit la quantité en 2019
|
||||
\[
|
||||
143 * 0.8264 = 118.1752
|
||||
\]
|
||||
\item L'équation de la droite est
|
||||
\[
|
||||
y = 2.6666666666666665 x -4
|
||||
\]
|
||||
\item Il faut penser à faire la division à par $8$ avant d'utiliser le log car sinon, on ne peut pas utiliser la formule $\log(a^n) = n\times \log(a)$.
|
||||
|
||||
\[x = \frac{\log(3.25)}{\log(0.76)}\]
|
||||
\end{enumerate}
|
||||
\end{solution}
|
||||
|
||||
\begin{exercise}[subtitle={Restaurant}]
|
||||
Un \emph{food truck}, ouvert le midi et le soir, propose deux types de formules :
|
||||
|
||||
\setlength\parindent{10mm}
|
||||
\begin{itemize}
|
||||
\item la formule \emph{Burger} ;
|
||||
\item la formule \emph{Wok}.
|
||||
\end{itemize}
|
||||
\setlength\parindent{0mm}
|
||||
|
||||
\medskip
|
||||
|
||||
Le gérant a remarqué que 21\,\% de ses ventes ont lieu le midi. Le quart des ventes du midi correspondent à la formule \emph{Burger}, alors que 27\,\% des ventes du soir correspondent à la formule \emph{Wok}.
|
||||
|
||||
Le gérant se constitue un fichier en notant, pour chaque vente, la formule choisie et le moment de cette vente (midi ou soir).
|
||||
|
||||
On prélève une fiche de façon équiprobable. On définit les quatre évènements suivants:
|
||||
|
||||
\begin{enumerate}
|
||||
\item $M$ : \og la fiche correspond à une vente du midi\fg{} ;
|
||||
\item $S$ : \og la fiche correspond à une vente du soir\fg {};
|
||||
\item $W$ : \og la fiche correspond à une formule \emph{Wok} \fg{} ;
|
||||
\item $B$ : \og la fiche correspond à une formule \emph{Burger} \fg.
|
||||
\end{enumerate}
|
||||
\setlength\parindent{0mm}
|
||||
|
||||
\medskip
|
||||
|
||||
\begin{enumerate}
|
||||
\item Recopier puis compléter l'arbre pondéré
|
||||
|
||||
\begin{center}
|
||||
\begin{tikzpicture}[sloped]
|
||||
\node {.}
|
||||
child {node {$M$}
|
||||
child {node {$W$}
|
||||
edge from parent
|
||||
node[above] {...}
|
||||
}
|
||||
child {node {$B$}
|
||||
edge from parent
|
||||
node[above] {...}
|
||||
}
|
||||
edge from parent
|
||||
node[above] {...}
|
||||
}
|
||||
child[missing] {}
|
||||
child { node {$S$}
|
||||
child {node {$W$}
|
||||
edge from parent
|
||||
node[above] {...}
|
||||
}
|
||||
child {node {$B$}
|
||||
edge from parent
|
||||
node[above] {...}
|
||||
}
|
||||
edge from parent
|
||||
node[above] {...}
|
||||
} ;
|
||||
\end{tikzpicture}
|
||||
\end{center}
|
||||
|
||||
\item Calculer la probabilité de l'évènement $M \cap W$. Interpréter ce résultat dans le contexte de l'exercice.
|
||||
\item Montrer que la probabilité que la fiche choisie corresponde à une formule \emph{Burger} est égale à $0.6292$.
|
||||
\item On a prélevé une fiche correspondant à la formule \emph{Burger}. Quelle est la probabilité, arrondie au millième, que la vente ait eu lieu le soir?
|
||||
\end{enumerate}
|
||||
\end{exercise}
|
||||
|
||||
\begin{solution}
|
||||
\begin{enumerate}
|
||||
\item
|
||||
\begin{center}
|
||||
\begin{tikzpicture}[sloped]
|
||||
\node {.}
|
||||
child {node {$M$}
|
||||
child {node {$W$}
|
||||
edge from parent
|
||||
node[above] {$0.75$}
|
||||
}
|
||||
child {node {$B$}
|
||||
edge from parent
|
||||
node[above] {$0.25$}
|
||||
}
|
||||
edge from parent
|
||||
node[above] {$0.21$}
|
||||
}
|
||||
child[missing] {}
|
||||
child { node {$S$}
|
||||
child {node {$W$}
|
||||
edge from parent
|
||||
node[above] {$0.27$}
|
||||
}
|
||||
child {node {$B$}
|
||||
edge from parent
|
||||
node[above] {$0.73$}
|
||||
}
|
||||
edge from parent
|
||||
node[above] {$0.79$}
|
||||
} ;
|
||||
\end{tikzpicture}
|
||||
\end{center}
|
||||
\item On calcule la probabilité que la vente soit un wok et ait eu lieu à midi
|
||||
\[ P(M\cap W) = P(M) \times P_M(W) = 0.21 \times 0.75 = 0.1575 \]
|
||||
\item Probabilité que la vente soit un burger.
|
||||
\[
|
||||
P(B) = P(M\cap B) + P(S\cap B) = 0.21 \times 0.75 + 0.79 \times 0.27 = 0.6292
|
||||
\]
|
||||
\item On cherche à calculer la quantité $P_B(S)$. Pour cela on utilise la formule de Bayes
|
||||
\[
|
||||
P_B(S) = \frac{P(B\cap S)}{P(B)} = \frac{P_S(B) \times P(S)}{P(B)} = \frac{0.73\times 0.79}{0.6292} = 0.9165607120152575 \approx 0.917
|
||||
\]
|
||||
\end{enumerate}
|
||||
\end{solution}
|
||||
|
||||
\begin{exercise}[subtitle={Continent plastique}]
|
||||
\textit{Les quantités évoqués dans cette exercice sont générés au hasard et sont donc complètement farfelus.}
|
||||
\medskip
|
||||
Le \og continent de plastique\fg{} est la plus grande des plaques de déchets plastiques évoluant sur les océans. Elle occupe actuellement dans l'océan Pacifique une surface dont l'aire est évaluée à plus de $1,6$ million de km$^2$, entre Hawaï et la Californie.
|
||||
|
||||
En 2017, des scientifiques ont estimé qu'il y avait $20$ millions de tonnes de déchets plastiques qui était déversé chaque année dans les océans et que cette quantité augmentait de $28\n\%$ par chaque année.
|
||||
|
||||
On modélise l'évolution de la masse de ces déchets plastiques déversée chaque année, si rien n'est fait pour la réduire, par une suite géométrique $\left(u_n\right)$. L'arrondi au centième du terme $u_n$ représente la masse de ces déchets déversée chaque année, exprimée en million de tonnes, pour l'année $(2017 + n)$.
|
||||
|
||||
\medskip
|
||||
|
||||
\begin{enumerate}
|
||||
\item Expliquer pourquoi la suite $u_n$ est géométrique?
|
||||
\item Calculer $u_1$ et $u_2$.
|
||||
\item Exprimer $u_n$ en fonction de $n$.
|
||||
\item Au début de l'année 2017, il y avait $300$ millions de tonnes de déchets plastique. Calculer la quantité totale de déchets plastiques en 2030.
|
||||
\item On souhaite déterminer en quelle année la masse totale de ces déchets plastiques aura pour la première fois augmenté de $50$\,\% par rapport à sa valeur de 2017.
|
||||
\begin{enumerate}
|
||||
\item Recopier et compléter l'algorithme ci-dessous pour que la variable $N$ contienne la réponse au problème posé.
|
||||
|
||||
\begin{center}
|
||||
\begin{tabularx}{0.4\linewidth}{|X|}\hline
|
||||
$N = 2017$\\
|
||||
$U = 20$ \\
|
||||
$S = 300 + U$ \\
|
||||
while $S < 450$: \\
|
||||
\hspace{1cm} $N = \ldots$\\
|
||||
\hspace{1cm} $U = \ldots$\\
|
||||
\hspace{1cm} $S = \ldots$\\
|
||||
\hline
|
||||
\end{tabularx}
|
||||
\end{center}
|
||||
\item Que contiennent les variables $S$, $U$ et $N$ après exécution de cet algorithme ?
|
||||
|
||||
Interpréter les résultats dans le contexte de l'exercice.
|
||||
\end{enumerate}
|
||||
\end{enumerate}
|
||||
\end{exercise}
|
||||
|
||||
\begin{solution}
|
||||
\begin{enumerate}
|
||||
\item Une augmentation de $28\,\%$ revient à multiplier la quantité par $1.28$. La suite est donc bien géométrique. Son premier terme est $u_0 = 20$ et sa raison est $q = 1.28$
|
||||
\item
|
||||
\[
|
||||
u_1 = u_0 * 1.28 = 25.6
|
||||
\]
|
||||
\[
|
||||
u_2 = u_0 * 1.28^2 = 32.768
|
||||
\]
|
||||
\item
|
||||
\[
|
||||
u_n = u_0 \times q^n = 20 \times 1.28^n
|
||||
\]
|
||||
\item On calcule la quantité totale déversée entre 2017 et 2030.
|
||||
\[
|
||||
\sum_{n = 0}^{13} u_n = u_0 \times \frac{1-q^{13}}{1-q} = 20 \times \frac{1 - 1.28^{13}}{1 - 1.28} = 1697.06
|
||||
\]
|
||||
On en déduit la quantité totale de déchets en 2030
|
||||
\[
|
||||
300 + 1697.06 = 1997.06
|
||||
\]
|
||||
\item
|
||||
\begin{enumerate}
|
||||
\item ~
|
||||
\begin{center}
|
||||
\begin{tabularx}{0.4\linewidth}{|X|}\hline
|
||||
$N \gets 2017$\\
|
||||
$U \gets 20$ \\
|
||||
$S \gets 300 + U$ \\
|
||||
Tant que $S < 450$ \\
|
||||
\hspace{1cm} $N \gets N + 1$\\
|
||||
\hspace{1cm} $U \gets U * 1.28$\\
|
||||
\hspace{1cm} $S \gets S + u$\\
|
||||
Fin Tant que\\\hline
|
||||
\end{tabularx}
|
||||
\end{center}
|
||||
\item \textit{Pas de correction automatisé}
|
||||
\end{enumerate}
|
||||
\end{enumerate}
|
||||
\end{solution}
|
||||
|
||||
\end{document}
|
||||
|
||||
%%% Local Variables:
|
||||
%%% mode: latex
|
||||
%%% TeX-master: "master"
|
||||
%%% End:
|
262
TST/DS/DS_21_04_07/TST3/02_210407_DS8.tex
Normal file
262
TST/DS/DS_21_04_07/TST3/02_210407_DS8.tex
Normal file
@ -0,0 +1,262 @@
|
||||
\documentclass[a4paper,10pt]{article}
|
||||
\usepackage{myXsim}
|
||||
|
||||
% Title Page
|
||||
\title{DS8 \hfill BERTAN Ufuk}
|
||||
\tribe{TST}
|
||||
\date{\hfillÀ render pour le Mercredi 7 avril}
|
||||
|
||||
\xsimsetup{
|
||||
solution/print = false
|
||||
}
|
||||
|
||||
\begin{document}
|
||||
\maketitle
|
||||
|
||||
\begin{exercise}[subtitle={Automatismes}]
|
||||
\textit{Toutes les questions de cette exercice sont indépendantes et peuvent être répondus séparément}
|
||||
\begin{enumerate}
|
||||
\item De janvier à septembre, une quantité a augmenté de $22\,\%$. Faire un schéma pour représenter la situation puis calculer le taux d'évolution moyen mensuel.
|
||||
\item Une quantité augmente de $22\,\%$ par ans. En 2020, elle est de 134\euro. Quelle était sa valeur en 2019? Faire un schéma pour représenter la situation.
|
||||
\item Déterminer l'équation de la droite \\
|
||||
\begin{tikzpicture}[xscale=0.8, yscale=0.5]
|
||||
\tkzInit[xmin=-5,xmax=5,xstep=1,
|
||||
ymin=-5,ymax=5,ystep=1]
|
||||
\tkzGrid
|
||||
\tkzAxeXY
|
||||
\tkzFct[domain=-5:5,color=red,very thick]%
|
||||
{2.0*\x -2};
|
||||
\end{tikzpicture}
|
||||
\item Résoudre l'équation $5 \times 0.99^x = 21$
|
||||
\end{enumerate}
|
||||
\end{exercise}
|
||||
|
||||
\begin{solution}
|
||||
\begin{enumerate}
|
||||
\item On veut partager cette évolution en 8 évolutions.
|
||||
\[
|
||||
\left(1 + \frac{22}{100}\right)^{\frac{1}{8}} = 1.0252
|
||||
\]
|
||||
Donc le taux d'évolution moyen est
|
||||
\[
|
||||
t_m = 1.0252 - 1 = 0.02519999999999989
|
||||
\]
|
||||
\item Coefficient multiplicateur pour revenir en arrière
|
||||
\[
|
||||
CM = (1 + \frac{22}{100})^{-1} = 0.8197
|
||||
\]
|
||||
On en déduit la quantité en 2019
|
||||
\[
|
||||
134 * 0.8197 = 109.8398
|
||||
\]
|
||||
\item L'équation de la droite est
|
||||
\[
|
||||
y = 2.0 x -2
|
||||
\]
|
||||
\item Il faut penser à faire la division à par $5$ avant d'utiliser le log car sinon, on ne peut pas utiliser la formule $\log(a^n) = n\times \log(a)$.
|
||||
|
||||
\[x = \frac{\log(4.2)}{\log(0.99)}\]
|
||||
\end{enumerate}
|
||||
\end{solution}
|
||||
|
||||
\begin{exercise}[subtitle={Restaurant}]
|
||||
Un \emph{food truck}, ouvert le midi et le soir, propose deux types de formules :
|
||||
|
||||
\setlength\parindent{10mm}
|
||||
\begin{itemize}
|
||||
\item la formule \emph{Burger} ;
|
||||
\item la formule \emph{Wok}.
|
||||
\end{itemize}
|
||||
\setlength\parindent{0mm}
|
||||
|
||||
\medskip
|
||||
|
||||
Le gérant a remarqué que 64\,\% de ses ventes ont lieu le midi. Le quart des ventes du midi correspondent à la formule \emph{Burger}, alors que 100\,\% des ventes du soir correspondent à la formule \emph{Wok}.
|
||||
|
||||
Le gérant se constitue un fichier en notant, pour chaque vente, la formule choisie et le moment de cette vente (midi ou soir).
|
||||
|
||||
On prélève une fiche de façon équiprobable. On définit les quatre évènements suivants:
|
||||
|
||||
\begin{enumerate}
|
||||
\item $M$ : \og la fiche correspond à une vente du midi\fg{} ;
|
||||
\item $S$ : \og la fiche correspond à une vente du soir\fg {};
|
||||
\item $W$ : \og la fiche correspond à une formule \emph{Wok} \fg{} ;
|
||||
\item $B$ : \og la fiche correspond à une formule \emph{Burger} \fg.
|
||||
\end{enumerate}
|
||||
\setlength\parindent{0mm}
|
||||
|
||||
\medskip
|
||||
|
||||
\begin{enumerate}
|
||||
\item Recopier puis compléter l'arbre pondéré
|
||||
|
||||
\begin{center}
|
||||
\begin{tikzpicture}[sloped]
|
||||
\node {.}
|
||||
child {node {$M$}
|
||||
child {node {$W$}
|
||||
edge from parent
|
||||
node[above] {...}
|
||||
}
|
||||
child {node {$B$}
|
||||
edge from parent
|
||||
node[above] {...}
|
||||
}
|
||||
edge from parent
|
||||
node[above] {...}
|
||||
}
|
||||
child[missing] {}
|
||||
child { node {$S$}
|
||||
child {node {$W$}
|
||||
edge from parent
|
||||
node[above] {...}
|
||||
}
|
||||
child {node {$B$}
|
||||
edge from parent
|
||||
node[above] {...}
|
||||
}
|
||||
edge from parent
|
||||
node[above] {...}
|
||||
} ;
|
||||
\end{tikzpicture}
|
||||
\end{center}
|
||||
|
||||
\item Calculer la probabilité de l'évènement $M \cap W$. Interpréter ce résultat dans le contexte de l'exercice.
|
||||
\item Montrer que la probabilité que la fiche choisie corresponde à une formule \emph{Burger} est égale à $0.16$.
|
||||
\item On a prélevé une fiche correspondant à la formule \emph{Burger}. Quelle est la probabilité, arrondie au millième, que la vente ait eu lieu le soir?
|
||||
\end{enumerate}
|
||||
\end{exercise}
|
||||
|
||||
\begin{solution}
|
||||
\begin{enumerate}
|
||||
\item
|
||||
\begin{center}
|
||||
\begin{tikzpicture}[sloped]
|
||||
\node {.}
|
||||
child {node {$M$}
|
||||
child {node {$W$}
|
||||
edge from parent
|
||||
node[above] {$0.75$}
|
||||
}
|
||||
child {node {$B$}
|
||||
edge from parent
|
||||
node[above] {$0.25$}
|
||||
}
|
||||
edge from parent
|
||||
node[above] {$0.64$}
|
||||
}
|
||||
child[missing] {}
|
||||
child { node {$S$}
|
||||
child {node {$W$}
|
||||
edge from parent
|
||||
node[above] {$1.0$}
|
||||
}
|
||||
child {node {$B$}
|
||||
edge from parent
|
||||
node[above] {$0.0$}
|
||||
}
|
||||
edge from parent
|
||||
node[above] {$0.36$}
|
||||
} ;
|
||||
\end{tikzpicture}
|
||||
\end{center}
|
||||
\item On calcule la probabilité que la vente soit un wok et ait eu lieu à midi
|
||||
\[ P(M\cap W) = P(M) \times P_M(W) = 0.64 \times 0.75 = 0.48 \]
|
||||
\item Probabilité que la vente soit un burger.
|
||||
\[
|
||||
P(B) = P(M\cap B) + P(S\cap B) = 0.64 \times 0.75 + 0.36 \times 1.0 = 0.16
|
||||
\]
|
||||
\item On cherche à calculer la quantité $P_B(S)$. Pour cela on utilise la formule de Bayes
|
||||
\[
|
||||
P_B(S) = \frac{P(B\cap S)}{P(B)} = \frac{P_S(B) \times P(S)}{P(B)} = \frac{0.0\times 0.36}{0.16} = 0.0 \approx 0.0
|
||||
\]
|
||||
\end{enumerate}
|
||||
\end{solution}
|
||||
|
||||
\begin{exercise}[subtitle={Continent plastique}]
|
||||
\textit{Les quantités évoqués dans cette exercice sont générés au hasard et sont donc complètement farfelus.}
|
||||
\medskip
|
||||
Le \og continent de plastique\fg{} est la plus grande des plaques de déchets plastiques évoluant sur les océans. Elle occupe actuellement dans l'océan Pacifique une surface dont l'aire est évaluée à plus de $1,6$ million de km$^2$, entre Hawaï et la Californie.
|
||||
|
||||
En 2017, des scientifiques ont estimé qu'il y avait $2$ millions de tonnes de déchets plastiques qui était déversé chaque année dans les océans et que cette quantité augmentait de $27\n\%$ par chaque année.
|
||||
|
||||
On modélise l'évolution de la masse de ces déchets plastiques déversée chaque année, si rien n'est fait pour la réduire, par une suite géométrique $\left(u_n\right)$. L'arrondi au centième du terme $u_n$ représente la masse de ces déchets déversée chaque année, exprimée en million de tonnes, pour l'année $(2017 + n)$.
|
||||
|
||||
\medskip
|
||||
|
||||
\begin{enumerate}
|
||||
\item Expliquer pourquoi la suite $u_n$ est géométrique?
|
||||
\item Calculer $u_1$ et $u_2$.
|
||||
\item Exprimer $u_n$ en fonction de $n$.
|
||||
\item Au début de l'année 2017, il y avait $300$ millions de tonnes de déchets plastique. Calculer la quantité totale de déchets plastiques en 2030.
|
||||
\item On souhaite déterminer en quelle année la masse totale de ces déchets plastiques aura pour la première fois augmenté de $50$\,\% par rapport à sa valeur de 2017.
|
||||
\begin{enumerate}
|
||||
\item Recopier et compléter l'algorithme ci-dessous pour que la variable $N$ contienne la réponse au problème posé.
|
||||
|
||||
\begin{center}
|
||||
\begin{tabularx}{0.4\linewidth}{|X|}\hline
|
||||
$N = 2017$\\
|
||||
$U = 2$ \\
|
||||
$S = 300 + U$ \\
|
||||
while $S < 450$: \\
|
||||
\hspace{1cm} $N = \ldots$\\
|
||||
\hspace{1cm} $U = \ldots$\\
|
||||
\hspace{1cm} $S = \ldots$\\
|
||||
\hline
|
||||
\end{tabularx}
|
||||
\end{center}
|
||||
\item Que contiennent les variables $S$, $U$ et $N$ après exécution de cet algorithme ?
|
||||
|
||||
Interpréter les résultats dans le contexte de l'exercice.
|
||||
\end{enumerate}
|
||||
\end{enumerate}
|
||||
\end{exercise}
|
||||
|
||||
\begin{solution}
|
||||
\begin{enumerate}
|
||||
\item Une augmentation de $27\,\%$ revient à multiplier la quantité par $1.27$. La suite est donc bien géométrique. Son premier terme est $u_0 = 2$ et sa raison est $q = 1.27$
|
||||
\item
|
||||
\[
|
||||
u_1 = u_0 * 1.27 = 2.54
|
||||
\]
|
||||
\[
|
||||
u_2 = u_0 * 1.27^2 = 3.2258
|
||||
\]
|
||||
\item
|
||||
\[
|
||||
u_n = u_0 \times q^n = 2 \times 1.27^n
|
||||
\]
|
||||
\item On calcule la quantité totale déversée entre 2017 et 2030.
|
||||
\[
|
||||
\sum_{n = 0}^{13} u_n = u_0 \times \frac{1-q^{13}}{1-q} = 2 \times \frac{1 - 1.27^{13}}{1 - 1.27} = 158.21
|
||||
\]
|
||||
On en déduit la quantité totale de déchets en 2030
|
||||
\[
|
||||
300 + 158.21 = 458.21000000000004
|
||||
\]
|
||||
\item
|
||||
\begin{enumerate}
|
||||
\item ~
|
||||
\begin{center}
|
||||
\begin{tabularx}{0.4\linewidth}{|X|}\hline
|
||||
$N \gets 2017$\\
|
||||
$U \gets 2$ \\
|
||||
$S \gets 300 + U$ \\
|
||||
Tant que $S < 450$ \\
|
||||
\hspace{1cm} $N \gets N + 1$\\
|
||||
\hspace{1cm} $U \gets U * 1.27$\\
|
||||
\hspace{1cm} $S \gets S + u$\\
|
||||
Fin Tant que\\\hline
|
||||
\end{tabularx}
|
||||
\end{center}
|
||||
\item \textit{Pas de correction automatisé}
|
||||
\end{enumerate}
|
||||
\end{enumerate}
|
||||
\end{solution}
|
||||
|
||||
\end{document}
|
||||
|
||||
%%% Local Variables:
|
||||
%%% mode: latex
|
||||
%%% TeX-master: "master"
|
||||
%%% End:
|
262
TST/DS/DS_21_04_07/TST3/03_210407_DS8.tex
Normal file
262
TST/DS/DS_21_04_07/TST3/03_210407_DS8.tex
Normal file
@ -0,0 +1,262 @@
|
||||
\documentclass[a4paper,10pt]{article}
|
||||
\usepackage{myXsim}
|
||||
|
||||
% Title Page
|
||||
\title{DS8 \hfill BOUALIA Bilel}
|
||||
\tribe{TST}
|
||||
\date{\hfillÀ render pour le Mercredi 7 avril}
|
||||
|
||||
\xsimsetup{
|
||||
solution/print = false
|
||||
}
|
||||
|
||||
\begin{document}
|
||||
\maketitle
|
||||
|
||||
\begin{exercise}[subtitle={Automatismes}]
|
||||
\textit{Toutes les questions de cette exercice sont indépendantes et peuvent être répondus séparément}
|
||||
\begin{enumerate}
|
||||
\item De janvier à septembre, une quantité a augmenté de $17\,\%$. Faire un schéma pour représenter la situation puis calculer le taux d'évolution moyen mensuel.
|
||||
\item Une quantité augmente de $17\,\%$ par ans. En 2020, elle est de 135\euro. Quelle était sa valeur en 2019? Faire un schéma pour représenter la situation.
|
||||
\item Déterminer l'équation de la droite \\
|
||||
\begin{tikzpicture}[xscale=0.8, yscale=0.5]
|
||||
\tkzInit[xmin=-5,xmax=5,xstep=1,
|
||||
ymin=-5,ymax=5,ystep=1]
|
||||
\tkzGrid
|
||||
\tkzAxeXY
|
||||
\tkzFct[domain=-5:5,color=red,very thick]%
|
||||
{2.0*\x -4};
|
||||
\end{tikzpicture}
|
||||
\item Résoudre l'équation $3 \times 0.2^x = 2$
|
||||
\end{enumerate}
|
||||
\end{exercise}
|
||||
|
||||
\begin{solution}
|
||||
\begin{enumerate}
|
||||
\item On veut partager cette évolution en 8 évolutions.
|
||||
\[
|
||||
\left(1 + \frac{17}{100}\right)^{\frac{1}{8}} = 1.0198
|
||||
\]
|
||||
Donc le taux d'évolution moyen est
|
||||
\[
|
||||
t_m = 1.0198 - 1 = 0.01980000000000004
|
||||
\]
|
||||
\item Coefficient multiplicateur pour revenir en arrière
|
||||
\[
|
||||
CM = (1 + \frac{17}{100})^{-1} = 0.8547
|
||||
\]
|
||||
On en déduit la quantité en 2019
|
||||
\[
|
||||
135 * 0.8547 = 115.3845
|
||||
\]
|
||||
\item L'équation de la droite est
|
||||
\[
|
||||
y = 2.0 x -4
|
||||
\]
|
||||
\item Il faut penser à faire la division à par $3$ avant d'utiliser le log car sinon, on ne peut pas utiliser la formule $\log(a^n) = n\times \log(a)$.
|
||||
|
||||
\[x = \frac{\log(0.67)}{\log(0.2)}\]
|
||||
\end{enumerate}
|
||||
\end{solution}
|
||||
|
||||
\begin{exercise}[subtitle={Restaurant}]
|
||||
Un \emph{food truck}, ouvert le midi et le soir, propose deux types de formules :
|
||||
|
||||
\setlength\parindent{10mm}
|
||||
\begin{itemize}
|
||||
\item la formule \emph{Burger} ;
|
||||
\item la formule \emph{Wok}.
|
||||
\end{itemize}
|
||||
\setlength\parindent{0mm}
|
||||
|
||||
\medskip
|
||||
|
||||
Le gérant a remarqué que 89\,\% de ses ventes ont lieu le midi. Le quart des ventes du midi correspondent à la formule \emph{Burger}, alors que 12\,\% des ventes du soir correspondent à la formule \emph{Wok}.
|
||||
|
||||
Le gérant se constitue un fichier en notant, pour chaque vente, la formule choisie et le moment de cette vente (midi ou soir).
|
||||
|
||||
On prélève une fiche de façon équiprobable. On définit les quatre évènements suivants:
|
||||
|
||||
\begin{enumerate}
|
||||
\item $M$ : \og la fiche correspond à une vente du midi\fg{} ;
|
||||
\item $S$ : \og la fiche correspond à une vente du soir\fg {};
|
||||
\item $W$ : \og la fiche correspond à une formule \emph{Wok} \fg{} ;
|
||||
\item $B$ : \og la fiche correspond à une formule \emph{Burger} \fg.
|
||||
\end{enumerate}
|
||||
\setlength\parindent{0mm}
|
||||
|
||||
\medskip
|
||||
|
||||
\begin{enumerate}
|
||||
\item Recopier puis compléter l'arbre pondéré
|
||||
|
||||
\begin{center}
|
||||
\begin{tikzpicture}[sloped]
|
||||
\node {.}
|
||||
child {node {$M$}
|
||||
child {node {$W$}
|
||||
edge from parent
|
||||
node[above] {...}
|
||||
}
|
||||
child {node {$B$}
|
||||
edge from parent
|
||||
node[above] {...}
|
||||
}
|
||||
edge from parent
|
||||
node[above] {...}
|
||||
}
|
||||
child[missing] {}
|
||||
child { node {$S$}
|
||||
child {node {$W$}
|
||||
edge from parent
|
||||
node[above] {...}
|
||||
}
|
||||
child {node {$B$}
|
||||
edge from parent
|
||||
node[above] {...}
|
||||
}
|
||||
edge from parent
|
||||
node[above] {...}
|
||||
} ;
|
||||
\end{tikzpicture}
|
||||
\end{center}
|
||||
|
||||
\item Calculer la probabilité de l'évènement $M \cap W$. Interpréter ce résultat dans le contexte de l'exercice.
|
||||
\item Montrer que la probabilité que la fiche choisie corresponde à une formule \emph{Burger} est égale à $0.3193$.
|
||||
\item On a prélevé une fiche correspondant à la formule \emph{Burger}. Quelle est la probabilité, arrondie au millième, que la vente ait eu lieu le soir?
|
||||
\end{enumerate}
|
||||
\end{exercise}
|
||||
|
||||
\begin{solution}
|
||||
\begin{enumerate}
|
||||
\item
|
||||
\begin{center}
|
||||
\begin{tikzpicture}[sloped]
|
||||
\node {.}
|
||||
child {node {$M$}
|
||||
child {node {$W$}
|
||||
edge from parent
|
||||
node[above] {$0.75$}
|
||||
}
|
||||
child {node {$B$}
|
||||
edge from parent
|
||||
node[above] {$0.25$}
|
||||
}
|
||||
edge from parent
|
||||
node[above] {$0.89$}
|
||||
}
|
||||
child[missing] {}
|
||||
child { node {$S$}
|
||||
child {node {$W$}
|
||||
edge from parent
|
||||
node[above] {$0.12$}
|
||||
}
|
||||
child {node {$B$}
|
||||
edge from parent
|
||||
node[above] {$0.88$}
|
||||
}
|
||||
edge from parent
|
||||
node[above] {$0.11$}
|
||||
} ;
|
||||
\end{tikzpicture}
|
||||
\end{center}
|
||||
\item On calcule la probabilité que la vente soit un wok et ait eu lieu à midi
|
||||
\[ P(M\cap W) = P(M) \times P_M(W) = 0.89 \times 0.75 = 0.6675 \]
|
||||
\item Probabilité que la vente soit un burger.
|
||||
\[
|
||||
P(B) = P(M\cap B) + P(S\cap B) = 0.89 \times 0.75 + 0.11 \times 0.12 = 0.3193
|
||||
\]
|
||||
\item On cherche à calculer la quantité $P_B(S)$. Pour cela on utilise la formule de Bayes
|
||||
\[
|
||||
P_B(S) = \frac{P(B\cap S)}{P(B)} = \frac{P_S(B) \times P(S)}{P(B)} = \frac{0.88\times 0.11}{0.3193} = 0.303163169433135 \approx 0.303
|
||||
\]
|
||||
\end{enumerate}
|
||||
\end{solution}
|
||||
|
||||
\begin{exercise}[subtitle={Continent plastique}]
|
||||
\textit{Les quantités évoqués dans cette exercice sont générés au hasard et sont donc complètement farfelus.}
|
||||
\medskip
|
||||
Le \og continent de plastique\fg{} est la plus grande des plaques de déchets plastiques évoluant sur les océans. Elle occupe actuellement dans l'océan Pacifique une surface dont l'aire est évaluée à plus de $1,6$ million de km$^2$, entre Hawaï et la Californie.
|
||||
|
||||
En 2017, des scientifiques ont estimé qu'il y avait $10$ millions de tonnes de déchets plastiques qui était déversé chaque année dans les océans et que cette quantité augmentait de $27\n\%$ par chaque année.
|
||||
|
||||
On modélise l'évolution de la masse de ces déchets plastiques déversée chaque année, si rien n'est fait pour la réduire, par une suite géométrique $\left(u_n\right)$. L'arrondi au centième du terme $u_n$ représente la masse de ces déchets déversée chaque année, exprimée en million de tonnes, pour l'année $(2017 + n)$.
|
||||
|
||||
\medskip
|
||||
|
||||
\begin{enumerate}
|
||||
\item Expliquer pourquoi la suite $u_n$ est géométrique?
|
||||
\item Calculer $u_1$ et $u_2$.
|
||||
\item Exprimer $u_n$ en fonction de $n$.
|
||||
\item Au début de l'année 2017, il y avait $300$ millions de tonnes de déchets plastique. Calculer la quantité totale de déchets plastiques en 2030.
|
||||
\item On souhaite déterminer en quelle année la masse totale de ces déchets plastiques aura pour la première fois augmenté de $50$\,\% par rapport à sa valeur de 2017.
|
||||
\begin{enumerate}
|
||||
\item Recopier et compléter l'algorithme ci-dessous pour que la variable $N$ contienne la réponse au problème posé.
|
||||
|
||||
\begin{center}
|
||||
\begin{tabularx}{0.4\linewidth}{|X|}\hline
|
||||
$N = 2017$\\
|
||||
$U = 10$ \\
|
||||
$S = 300 + U$ \\
|
||||
while $S < 450$: \\
|
||||
\hspace{1cm} $N = \ldots$\\
|
||||
\hspace{1cm} $U = \ldots$\\
|
||||
\hspace{1cm} $S = \ldots$\\
|
||||
\hline
|
||||
\end{tabularx}
|
||||
\end{center}
|
||||
\item Que contiennent les variables $S$, $U$ et $N$ après exécution de cet algorithme ?
|
||||
|
||||
Interpréter les résultats dans le contexte de l'exercice.
|
||||
\end{enumerate}
|
||||
\end{enumerate}
|
||||
\end{exercise}
|
||||
|
||||
\begin{solution}
|
||||
\begin{enumerate}
|
||||
\item Une augmentation de $27\,\%$ revient à multiplier la quantité par $1.27$. La suite est donc bien géométrique. Son premier terme est $u_0 = 10$ et sa raison est $q = 1.27$
|
||||
\item
|
||||
\[
|
||||
u_1 = u_0 * 1.27 = 12.7
|
||||
\]
|
||||
\[
|
||||
u_2 = u_0 * 1.27^2 = 16.129
|
||||
\]
|
||||
\item
|
||||
\[
|
||||
u_n = u_0 \times q^n = 10 \times 1.27^n
|
||||
\]
|
||||
\item On calcule la quantité totale déversée entre 2017 et 2030.
|
||||
\[
|
||||
\sum_{n = 0}^{13} u_n = u_0 \times \frac{1-q^{13}}{1-q} = 10 \times \frac{1 - 1.27^{13}}{1 - 1.27} = 791.07
|
||||
\]
|
||||
On en déduit la quantité totale de déchets en 2030
|
||||
\[
|
||||
300 + 791.07 = 1091.0700000000002
|
||||
\]
|
||||
\item
|
||||
\begin{enumerate}
|
||||
\item ~
|
||||
\begin{center}
|
||||
\begin{tabularx}{0.4\linewidth}{|X|}\hline
|
||||
$N \gets 2017$\\
|
||||
$U \gets 10$ \\
|
||||
$S \gets 300 + U$ \\
|
||||
Tant que $S < 450$ \\
|
||||
\hspace{1cm} $N \gets N + 1$\\
|
||||
\hspace{1cm} $U \gets U * 1.27$\\
|
||||
\hspace{1cm} $S \gets S + u$\\
|
||||
Fin Tant que\\\hline
|
||||
\end{tabularx}
|
||||
\end{center}
|
||||
\item \textit{Pas de correction automatisé}
|
||||
\end{enumerate}
|
||||
\end{enumerate}
|
||||
\end{solution}
|
||||
|
||||
\end{document}
|
||||
|
||||
%%% Local Variables:
|
||||
%%% mode: latex
|
||||
%%% TeX-master: "master"
|
||||
%%% End:
|
262
TST/DS/DS_21_04_07/TST3/04_210407_DS8.tex
Normal file
262
TST/DS/DS_21_04_07/TST3/04_210407_DS8.tex
Normal file
@ -0,0 +1,262 @@
|
||||
\documentclass[a4paper,10pt]{article}
|
||||
\usepackage{myXsim}
|
||||
|
||||
% Title Page
|
||||
\title{DS8 \hfill BOUCHOUX Kevin}
|
||||
\tribe{TST}
|
||||
\date{\hfillÀ render pour le Mercredi 7 avril}
|
||||
|
||||
\xsimsetup{
|
||||
solution/print = false
|
||||
}
|
||||
|
||||
\begin{document}
|
||||
\maketitle
|
||||
|
||||
\begin{exercise}[subtitle={Automatismes}]
|
||||
\textit{Toutes les questions de cette exercice sont indépendantes et peuvent être répondus séparément}
|
||||
\begin{enumerate}
|
||||
\item De janvier à septembre, une quantité a augmenté de $10\,\%$. Faire un schéma pour représenter la situation puis calculer le taux d'évolution moyen mensuel.
|
||||
\item Une quantité augmente de $10\,\%$ par ans. En 2020, elle est de 118\euro. Quelle était sa valeur en 2019? Faire un schéma pour représenter la situation.
|
||||
\item Déterminer l'équation de la droite \\
|
||||
\begin{tikzpicture}[xscale=0.8, yscale=0.5]
|
||||
\tkzInit[xmin=-5,xmax=5,xstep=1,
|
||||
ymin=-5,ymax=5,ystep=1]
|
||||
\tkzGrid
|
||||
\tkzAxeXY
|
||||
\tkzFct[domain=-5:5,color=red,very thick]%
|
||||
{3.0*\x -3};
|
||||
\end{tikzpicture}
|
||||
\item Résoudre l'équation $2 \times 0.93^x = 44$
|
||||
\end{enumerate}
|
||||
\end{exercise}
|
||||
|
||||
\begin{solution}
|
||||
\begin{enumerate}
|
||||
\item On veut partager cette évolution en 8 évolutions.
|
||||
\[
|
||||
\left(1 + \frac{10}{100}\right)^{\frac{1}{8}} = 1.012
|
||||
\]
|
||||
Donc le taux d'évolution moyen est
|
||||
\[
|
||||
t_m = 1.012 - 1 = 0.01200000000000001
|
||||
\]
|
||||
\item Coefficient multiplicateur pour revenir en arrière
|
||||
\[
|
||||
CM = (1 + \frac{10}{100})^{-1} = 0.9091
|
||||
\]
|
||||
On en déduit la quantité en 2019
|
||||
\[
|
||||
118 * 0.9091 = 107.27380000000001
|
||||
\]
|
||||
\item L'équation de la droite est
|
||||
\[
|
||||
y = 3.0 x -3
|
||||
\]
|
||||
\item Il faut penser à faire la division à par $2$ avant d'utiliser le log car sinon, on ne peut pas utiliser la formule $\log(a^n) = n\times \log(a)$.
|
||||
|
||||
\[x = \frac{\log(22.0)}{\log(0.93)}\]
|
||||
\end{enumerate}
|
||||
\end{solution}
|
||||
|
||||
\begin{exercise}[subtitle={Restaurant}]
|
||||
Un \emph{food truck}, ouvert le midi et le soir, propose deux types de formules :
|
||||
|
||||
\setlength\parindent{10mm}
|
||||
\begin{itemize}
|
||||
\item la formule \emph{Burger} ;
|
||||
\item la formule \emph{Wok}.
|
||||
\end{itemize}
|
||||
\setlength\parindent{0mm}
|
||||
|
||||
\medskip
|
||||
|
||||
Le gérant a remarqué que 83\,\% de ses ventes ont lieu le midi. Le quart des ventes du midi correspondent à la formule \emph{Burger}, alors que 26\,\% des ventes du soir correspondent à la formule \emph{Wok}.
|
||||
|
||||
Le gérant se constitue un fichier en notant, pour chaque vente, la formule choisie et le moment de cette vente (midi ou soir).
|
||||
|
||||
On prélève une fiche de façon équiprobable. On définit les quatre évènements suivants:
|
||||
|
||||
\begin{enumerate}
|
||||
\item $M$ : \og la fiche correspond à une vente du midi\fg{} ;
|
||||
\item $S$ : \og la fiche correspond à une vente du soir\fg {};
|
||||
\item $W$ : \og la fiche correspond à une formule \emph{Wok} \fg{} ;
|
||||
\item $B$ : \og la fiche correspond à une formule \emph{Burger} \fg.
|
||||
\end{enumerate}
|
||||
\setlength\parindent{0mm}
|
||||
|
||||
\medskip
|
||||
|
||||
\begin{enumerate}
|
||||
\item Recopier puis compléter l'arbre pondéré
|
||||
|
||||
\begin{center}
|
||||
\begin{tikzpicture}[sloped]
|
||||
\node {.}
|
||||
child {node {$M$}
|
||||
child {node {$W$}
|
||||
edge from parent
|
||||
node[above] {...}
|
||||
}
|
||||
child {node {$B$}
|
||||
edge from parent
|
||||
node[above] {...}
|
||||
}
|
||||
edge from parent
|
||||
node[above] {...}
|
||||
}
|
||||
child[missing] {}
|
||||
child { node {$S$}
|
||||
child {node {$W$}
|
||||
edge from parent
|
||||
node[above] {...}
|
||||
}
|
||||
child {node {$B$}
|
||||
edge from parent
|
||||
node[above] {...}
|
||||
}
|
||||
edge from parent
|
||||
node[above] {...}
|
||||
} ;
|
||||
\end{tikzpicture}
|
||||
\end{center}
|
||||
|
||||
\item Calculer la probabilité de l'évènement $M \cap W$. Interpréter ce résultat dans le contexte de l'exercice.
|
||||
\item Montrer que la probabilité que la fiche choisie corresponde à une formule \emph{Burger} est égale à $0.3333$.
|
||||
\item On a prélevé une fiche correspondant à la formule \emph{Burger}. Quelle est la probabilité, arrondie au millième, que la vente ait eu lieu le soir?
|
||||
\end{enumerate}
|
||||
\end{exercise}
|
||||
|
||||
\begin{solution}
|
||||
\begin{enumerate}
|
||||
\item
|
||||
\begin{center}
|
||||
\begin{tikzpicture}[sloped]
|
||||
\node {.}
|
||||
child {node {$M$}
|
||||
child {node {$W$}
|
||||
edge from parent
|
||||
node[above] {$0.75$}
|
||||
}
|
||||
child {node {$B$}
|
||||
edge from parent
|
||||
node[above] {$0.25$}
|
||||
}
|
||||
edge from parent
|
||||
node[above] {$0.83$}
|
||||
}
|
||||
child[missing] {}
|
||||
child { node {$S$}
|
||||
child {node {$W$}
|
||||
edge from parent
|
||||
node[above] {$0.26$}
|
||||
}
|
||||
child {node {$B$}
|
||||
edge from parent
|
||||
node[above] {$0.74$}
|
||||
}
|
||||
edge from parent
|
||||
node[above] {$0.17$}
|
||||
} ;
|
||||
\end{tikzpicture}
|
||||
\end{center}
|
||||
\item On calcule la probabilité que la vente soit un wok et ait eu lieu à midi
|
||||
\[ P(M\cap W) = P(M) \times P_M(W) = 0.83 \times 0.75 = 0.6225 \]
|
||||
\item Probabilité que la vente soit un burger.
|
||||
\[
|
||||
P(B) = P(M\cap B) + P(S\cap B) = 0.83 \times 0.75 + 0.17 \times 0.26 = 0.3333
|
||||
\]
|
||||
\item On cherche à calculer la quantité $P_B(S)$. Pour cela on utilise la formule de Bayes
|
||||
\[
|
||||
P_B(S) = \frac{P(B\cap S)}{P(B)} = \frac{P_S(B) \times P(S)}{P(B)} = \frac{0.74\times 0.17}{0.3333} = 0.37743774377437744 \approx 0.377
|
||||
\]
|
||||
\end{enumerate}
|
||||
\end{solution}
|
||||
|
||||
\begin{exercise}[subtitle={Continent plastique}]
|
||||
\textit{Les quantités évoqués dans cette exercice sont générés au hasard et sont donc complètement farfelus.}
|
||||
\medskip
|
||||
Le \og continent de plastique\fg{} est la plus grande des plaques de déchets plastiques évoluant sur les océans. Elle occupe actuellement dans l'océan Pacifique une surface dont l'aire est évaluée à plus de $1,6$ million de km$^2$, entre Hawaï et la Californie.
|
||||
|
||||
En 2017, des scientifiques ont estimé qu'il y avait $18$ millions de tonnes de déchets plastiques qui était déversé chaque année dans les océans et que cette quantité augmentait de $14\n\%$ par chaque année.
|
||||
|
||||
On modélise l'évolution de la masse de ces déchets plastiques déversée chaque année, si rien n'est fait pour la réduire, par une suite géométrique $\left(u_n\right)$. L'arrondi au centième du terme $u_n$ représente la masse de ces déchets déversée chaque année, exprimée en million de tonnes, pour l'année $(2017 + n)$.
|
||||
|
||||
\medskip
|
||||
|
||||
\begin{enumerate}
|
||||
\item Expliquer pourquoi la suite $u_n$ est géométrique?
|
||||
\item Calculer $u_1$ et $u_2$.
|
||||
\item Exprimer $u_n$ en fonction de $n$.
|
||||
\item Au début de l'année 2017, il y avait $300$ millions de tonnes de déchets plastique. Calculer la quantité totale de déchets plastiques en 2030.
|
||||
\item On souhaite déterminer en quelle année la masse totale de ces déchets plastiques aura pour la première fois augmenté de $50$\,\% par rapport à sa valeur de 2017.
|
||||
\begin{enumerate}
|
||||
\item Recopier et compléter l'algorithme ci-dessous pour que la variable $N$ contienne la réponse au problème posé.
|
||||
|
||||
\begin{center}
|
||||
\begin{tabularx}{0.4\linewidth}{|X|}\hline
|
||||
$N = 2017$\\
|
||||
$U = 18$ \\
|
||||
$S = 300 + U$ \\
|
||||
while $S < 450$: \\
|
||||
\hspace{1cm} $N = \ldots$\\
|
||||
\hspace{1cm} $U = \ldots$\\
|
||||
\hspace{1cm} $S = \ldots$\\
|
||||
\hline
|
||||
\end{tabularx}
|
||||
\end{center}
|
||||
\item Que contiennent les variables $S$, $U$ et $N$ après exécution de cet algorithme ?
|
||||
|
||||
Interpréter les résultats dans le contexte de l'exercice.
|
||||
\end{enumerate}
|
||||
\end{enumerate}
|
||||
\end{exercise}
|
||||
|
||||
\begin{solution}
|
||||
\begin{enumerate}
|
||||
\item Une augmentation de $14\,\%$ revient à multiplier la quantité par $1.1400000000000001$. La suite est donc bien géométrique. Son premier terme est $u_0 = 18$ et sa raison est $q = 1.1400000000000001$
|
||||
\item
|
||||
\[
|
||||
u_1 = u_0 * 1.1400000000000001 = 20.520000000000003
|
||||
\]
|
||||
\[
|
||||
u_2 = u_0 * 1.1400000000000001^2 = 23.3928
|
||||
\]
|
||||
\item
|
||||
\[
|
||||
u_n = u_0 \times q^n = 18 \times 1.1400000000000001^n
|
||||
\]
|
||||
\item On calcule la quantité totale déversée entre 2017 et 2030.
|
||||
\[
|
||||
\sum_{n = 0}^{13} u_n = u_0 \times \frac{1-q^{13}}{1-q} = 18 \times \frac{1 - 1.1400000000000001^{13}}{1 - 1.1400000000000001} = 577.6
|
||||
\]
|
||||
On en déduit la quantité totale de déchets en 2030
|
||||
\[
|
||||
300 + 577.6 = 877.6
|
||||
\]
|
||||
\item
|
||||
\begin{enumerate}
|
||||
\item ~
|
||||
\begin{center}
|
||||
\begin{tabularx}{0.4\linewidth}{|X|}\hline
|
||||
$N \gets 2017$\\
|
||||
$U \gets 18$ \\
|
||||
$S \gets 300 + U$ \\
|
||||
Tant que $S < 450$ \\
|
||||
\hspace{1cm} $N \gets N + 1$\\
|
||||
\hspace{1cm} $U \gets U * 1.1400000000000001$\\
|
||||
\hspace{1cm} $S \gets S + u$\\
|
||||
Fin Tant que\\\hline
|
||||
\end{tabularx}
|
||||
\end{center}
|
||||
\item \textit{Pas de correction automatisé}
|
||||
\end{enumerate}
|
||||
\end{enumerate}
|
||||
\end{solution}
|
||||
|
||||
\end{document}
|
||||
|
||||
%%% Local Variables:
|
||||
%%% mode: latex
|
||||
%%% TeX-master: "master"
|
||||
%%% End:
|
262
TST/DS/DS_21_04_07/TST3/05_210407_DS8.tex
Normal file
262
TST/DS/DS_21_04_07/TST3/05_210407_DS8.tex
Normal file
@ -0,0 +1,262 @@
|
||||
\documentclass[a4paper,10pt]{article}
|
||||
\usepackage{myXsim}
|
||||
|
||||
% Title Page
|
||||
\title{DS8 \hfill BUDIN Nathan}
|
||||
\tribe{TST}
|
||||
\date{\hfillÀ render pour le Mercredi 7 avril}
|
||||
|
||||
\xsimsetup{
|
||||
solution/print = false
|
||||
}
|
||||
|
||||
\begin{document}
|
||||
\maketitle
|
||||
|
||||
\begin{exercise}[subtitle={Automatismes}]
|
||||
\textit{Toutes les questions de cette exercice sont indépendantes et peuvent être répondus séparément}
|
||||
\begin{enumerate}
|
||||
\item De janvier à septembre, une quantité a augmenté de $16\,\%$. Faire un schéma pour représenter la situation puis calculer le taux d'évolution moyen mensuel.
|
||||
\item Une quantité augmente de $16\,\%$ par ans. En 2020, elle est de 112\euro. Quelle était sa valeur en 2019? Faire un schéma pour représenter la situation.
|
||||
\item Déterminer l'équation de la droite \\
|
||||
\begin{tikzpicture}[xscale=0.8, yscale=0.5]
|
||||
\tkzInit[xmin=-5,xmax=5,xstep=1,
|
||||
ymin=-5,ymax=5,ystep=1]
|
||||
\tkzGrid
|
||||
\tkzAxeXY
|
||||
\tkzFct[domain=-5:5,color=red,very thick]%
|
||||
{1.0*\x -2};
|
||||
\end{tikzpicture}
|
||||
\item Résoudre l'équation $7 \times 0.22^x = 49$
|
||||
\end{enumerate}
|
||||
\end{exercise}
|
||||
|
||||
\begin{solution}
|
||||
\begin{enumerate}
|
||||
\item On veut partager cette évolution en 8 évolutions.
|
||||
\[
|
||||
\left(1 + \frac{16}{100}\right)^{\frac{1}{8}} = 1.0187
|
||||
\]
|
||||
Donc le taux d'évolution moyen est
|
||||
\[
|
||||
t_m = 1.0187 - 1 = 0.01869999999999994
|
||||
\]
|
||||
\item Coefficient multiplicateur pour revenir en arrière
|
||||
\[
|
||||
CM = (1 + \frac{16}{100})^{-1} = 0.8621
|
||||
\]
|
||||
On en déduit la quantité en 2019
|
||||
\[
|
||||
112 * 0.8621 = 96.5552
|
||||
\]
|
||||
\item L'équation de la droite est
|
||||
\[
|
||||
y = 1.0 x -2
|
||||
\]
|
||||
\item Il faut penser à faire la division à par $7$ avant d'utiliser le log car sinon, on ne peut pas utiliser la formule $\log(a^n) = n\times \log(a)$.
|
||||
|
||||
\[x = \frac{\log(7.0)}{\log(0.22)}\]
|
||||
\end{enumerate}
|
||||
\end{solution}
|
||||
|
||||
\begin{exercise}[subtitle={Restaurant}]
|
||||
Un \emph{food truck}, ouvert le midi et le soir, propose deux types de formules :
|
||||
|
||||
\setlength\parindent{10mm}
|
||||
\begin{itemize}
|
||||
\item la formule \emph{Burger} ;
|
||||
\item la formule \emph{Wok}.
|
||||
\end{itemize}
|
||||
\setlength\parindent{0mm}
|
||||
|
||||
\medskip
|
||||
|
||||
Le gérant a remarqué que 75\,\% de ses ventes ont lieu le midi. Le quart des ventes du midi correspondent à la formule \emph{Burger}, alors que 71\,\% des ventes du soir correspondent à la formule \emph{Wok}.
|
||||
|
||||
Le gérant se constitue un fichier en notant, pour chaque vente, la formule choisie et le moment de cette vente (midi ou soir).
|
||||
|
||||
On prélève une fiche de façon équiprobable. On définit les quatre évènements suivants:
|
||||
|
||||
\begin{enumerate}
|
||||
\item $M$ : \og la fiche correspond à une vente du midi\fg{} ;
|
||||
\item $S$ : \og la fiche correspond à une vente du soir\fg {};
|
||||
\item $W$ : \og la fiche correspond à une formule \emph{Wok} \fg{} ;
|
||||
\item $B$ : \og la fiche correspond à une formule \emph{Burger} \fg.
|
||||
\end{enumerate}
|
||||
\setlength\parindent{0mm}
|
||||
|
||||
\medskip
|
||||
|
||||
\begin{enumerate}
|
||||
\item Recopier puis compléter l'arbre pondéré
|
||||
|
||||
\begin{center}
|
||||
\begin{tikzpicture}[sloped]
|
||||
\node {.}
|
||||
child {node {$M$}
|
||||
child {node {$W$}
|
||||
edge from parent
|
||||
node[above] {...}
|
||||
}
|
||||
child {node {$B$}
|
||||
edge from parent
|
||||
node[above] {...}
|
||||
}
|
||||
edge from parent
|
||||
node[above] {...}
|
||||
}
|
||||
child[missing] {}
|
||||
child { node {$S$}
|
||||
child {node {$W$}
|
||||
edge from parent
|
||||
node[above] {...}
|
||||
}
|
||||
child {node {$B$}
|
||||
edge from parent
|
||||
node[above] {...}
|
||||
}
|
||||
edge from parent
|
||||
node[above] {...}
|
||||
} ;
|
||||
\end{tikzpicture}
|
||||
\end{center}
|
||||
|
||||
\item Calculer la probabilité de l'évènement $M \cap W$. Interpréter ce résultat dans le contexte de l'exercice.
|
||||
\item Montrer que la probabilité que la fiche choisie corresponde à une formule \emph{Burger} est égale à $0.26$.
|
||||
\item On a prélevé une fiche correspondant à la formule \emph{Burger}. Quelle est la probabilité, arrondie au millième, que la vente ait eu lieu le soir?
|
||||
\end{enumerate}
|
||||
\end{exercise}
|
||||
|
||||
\begin{solution}
|
||||
\begin{enumerate}
|
||||
\item
|
||||
\begin{center}
|
||||
\begin{tikzpicture}[sloped]
|
||||
\node {.}
|
||||
child {node {$M$}
|
||||
child {node {$W$}
|
||||
edge from parent
|
||||
node[above] {$0.75$}
|
||||
}
|
||||
child {node {$B$}
|
||||
edge from parent
|
||||
node[above] {$0.25$}
|
||||
}
|
||||
edge from parent
|
||||
node[above] {$0.75$}
|
||||
}
|
||||
child[missing] {}
|
||||
child { node {$S$}
|
||||
child {node {$W$}
|
||||
edge from parent
|
||||
node[above] {$0.71$}
|
||||
}
|
||||
child {node {$B$}
|
||||
edge from parent
|
||||
node[above] {$0.29$}
|
||||
}
|
||||
edge from parent
|
||||
node[above] {$0.25$}
|
||||
} ;
|
||||
\end{tikzpicture}
|
||||
\end{center}
|
||||
\item On calcule la probabilité que la vente soit un wok et ait eu lieu à midi
|
||||
\[ P(M\cap W) = P(M) \times P_M(W) = 0.75 \times 0.75 = 0.5625 \]
|
||||
\item Probabilité que la vente soit un burger.
|
||||
\[
|
||||
P(B) = P(M\cap B) + P(S\cap B) = 0.75 \times 0.75 + 0.25 \times 0.71 = 0.26
|
||||
\]
|
||||
\item On cherche à calculer la quantité $P_B(S)$. Pour cela on utilise la formule de Bayes
|
||||
\[
|
||||
P_B(S) = \frac{P(B\cap S)}{P(B)} = \frac{P_S(B) \times P(S)}{P(B)} = \frac{0.29\times 0.25}{0.26} = 0.2788461538461538 \approx 0.279
|
||||
\]
|
||||
\end{enumerate}
|
||||
\end{solution}
|
||||
|
||||
\begin{exercise}[subtitle={Continent plastique}]
|
||||
\textit{Les quantités évoqués dans cette exercice sont générés au hasard et sont donc complètement farfelus.}
|
||||
\medskip
|
||||
Le \og continent de plastique\fg{} est la plus grande des plaques de déchets plastiques évoluant sur les océans. Elle occupe actuellement dans l'océan Pacifique une surface dont l'aire est évaluée à plus de $1,6$ million de km$^2$, entre Hawaï et la Californie.
|
||||
|
||||
En 2017, des scientifiques ont estimé qu'il y avait $12$ millions de tonnes de déchets plastiques qui était déversé chaque année dans les océans et que cette quantité augmentait de $25\n\%$ par chaque année.
|
||||
|
||||
On modélise l'évolution de la masse de ces déchets plastiques déversée chaque année, si rien n'est fait pour la réduire, par une suite géométrique $\left(u_n\right)$. L'arrondi au centième du terme $u_n$ représente la masse de ces déchets déversée chaque année, exprimée en million de tonnes, pour l'année $(2017 + n)$.
|
||||
|
||||
\medskip
|
||||
|
||||
\begin{enumerate}
|
||||
\item Expliquer pourquoi la suite $u_n$ est géométrique?
|
||||
\item Calculer $u_1$ et $u_2$.
|
||||
\item Exprimer $u_n$ en fonction de $n$.
|
||||
\item Au début de l'année 2017, il y avait $300$ millions de tonnes de déchets plastique. Calculer la quantité totale de déchets plastiques en 2030.
|
||||
\item On souhaite déterminer en quelle année la masse totale de ces déchets plastiques aura pour la première fois augmenté de $50$\,\% par rapport à sa valeur de 2017.
|
||||
\begin{enumerate}
|
||||
\item Recopier et compléter l'algorithme ci-dessous pour que la variable $N$ contienne la réponse au problème posé.
|
||||
|
||||
\begin{center}
|
||||
\begin{tabularx}{0.4\linewidth}{|X|}\hline
|
||||
$N = 2017$\\
|
||||
$U = 12$ \\
|
||||
$S = 300 + U$ \\
|
||||
while $S < 450$: \\
|
||||
\hspace{1cm} $N = \ldots$\\
|
||||
\hspace{1cm} $U = \ldots$\\
|
||||
\hspace{1cm} $S = \ldots$\\
|
||||
\hline
|
||||
\end{tabularx}
|
||||
\end{center}
|
||||
\item Que contiennent les variables $S$, $U$ et $N$ après exécution de cet algorithme ?
|
||||
|
||||
Interpréter les résultats dans le contexte de l'exercice.
|
||||
\end{enumerate}
|
||||
\end{enumerate}
|
||||
\end{exercise}
|
||||
|
||||
\begin{solution}
|
||||
\begin{enumerate}
|
||||
\item Une augmentation de $25\,\%$ revient à multiplier la quantité par $1.25$. La suite est donc bien géométrique. Son premier terme est $u_0 = 12$ et sa raison est $q = 1.25$
|
||||
\item
|
||||
\[
|
||||
u_1 = u_0 * 1.25 = 15.0
|
||||
\]
|
||||
\[
|
||||
u_2 = u_0 * 1.25^2 = 18.75
|
||||
\]
|
||||
\item
|
||||
\[
|
||||
u_n = u_0 \times q^n = 12 \times 1.25^n
|
||||
\]
|
||||
\item On calcule la quantité totale déversée entre 2017 et 2030.
|
||||
\[
|
||||
\sum_{n = 0}^{13} u_n = u_0 \times \frac{1-q^{13}}{1-q} = 12 \times \frac{1 - 1.25^{13}}{1 - 1.25} = 825.11
|
||||
\]
|
||||
On en déduit la quantité totale de déchets en 2030
|
||||
\[
|
||||
300 + 825.11 = 1125.1100000000001
|
||||
\]
|
||||
\item
|
||||
\begin{enumerate}
|
||||
\item ~
|
||||
\begin{center}
|
||||
\begin{tabularx}{0.4\linewidth}{|X|}\hline
|
||||
$N \gets 2017$\\
|
||||
$U \gets 12$ \\
|
||||
$S \gets 300 + U$ \\
|
||||
Tant que $S < 450$ \\
|
||||
\hspace{1cm} $N \gets N + 1$\\
|
||||
\hspace{1cm} $U \gets U * 1.25$\\
|
||||
\hspace{1cm} $S \gets S + u$\\
|
||||
Fin Tant que\\\hline
|
||||
\end{tabularx}
|
||||
\end{center}
|
||||
\item \textit{Pas de correction automatisé}
|
||||
\end{enumerate}
|
||||
\end{enumerate}
|
||||
\end{solution}
|
||||
|
||||
\end{document}
|
||||
|
||||
%%% Local Variables:
|
||||
%%% mode: latex
|
||||
%%% TeX-master: "master"
|
||||
%%% End:
|
262
TST/DS/DS_21_04_07/TST3/06_210407_DS8.tex
Normal file
262
TST/DS/DS_21_04_07/TST3/06_210407_DS8.tex
Normal file
@ -0,0 +1,262 @@
|
||||
\documentclass[a4paper,10pt]{article}
|
||||
\usepackage{myXsim}
|
||||
|
||||
% Title Page
|
||||
\title{DS8 \hfill DARICHE Kaïs}
|
||||
\tribe{TST}
|
||||
\date{\hfillÀ render pour le Mercredi 7 avril}
|
||||
|
||||
\xsimsetup{
|
||||
solution/print = false
|
||||
}
|
||||
|
||||
\begin{document}
|
||||
\maketitle
|
||||
|
||||
\begin{exercise}[subtitle={Automatismes}]
|
||||
\textit{Toutes les questions de cette exercice sont indépendantes et peuvent être répondus séparément}
|
||||
\begin{enumerate}
|
||||
\item De janvier à septembre, une quantité a augmenté de $27\,\%$. Faire un schéma pour représenter la situation puis calculer le taux d'évolution moyen mensuel.
|
||||
\item Une quantité augmente de $27\,\%$ par ans. En 2020, elle est de 143\euro. Quelle était sa valeur en 2019? Faire un schéma pour représenter la situation.
|
||||
\item Déterminer l'équation de la droite \\
|
||||
\begin{tikzpicture}[xscale=0.8, yscale=0.5]
|
||||
\tkzInit[xmin=-5,xmax=5,xstep=1,
|
||||
ymin=-5,ymax=5,ystep=1]
|
||||
\tkzGrid
|
||||
\tkzAxeXY
|
||||
\tkzFct[domain=-5:5,color=red,very thick]%
|
||||
{3.0*\x -3};
|
||||
\end{tikzpicture}
|
||||
\item Résoudre l'équation $7 \times 0.19^x = 21$
|
||||
\end{enumerate}
|
||||
\end{exercise}
|
||||
|
||||
\begin{solution}
|
||||
\begin{enumerate}
|
||||
\item On veut partager cette évolution en 8 évolutions.
|
||||
\[
|
||||
\left(1 + \frac{27}{100}\right)^{\frac{1}{8}} = 1.0303
|
||||
\]
|
||||
Donc le taux d'évolution moyen est
|
||||
\[
|
||||
t_m = 1.0303 - 1 = 0.030299999999999994
|
||||
\]
|
||||
\item Coefficient multiplicateur pour revenir en arrière
|
||||
\[
|
||||
CM = (1 + \frac{27}{100})^{-1} = 0.7874
|
||||
\]
|
||||
On en déduit la quantité en 2019
|
||||
\[
|
||||
143 * 0.7874 = 112.59819999999999
|
||||
\]
|
||||
\item L'équation de la droite est
|
||||
\[
|
||||
y = 3.0 x -3
|
||||
\]
|
||||
\item Il faut penser à faire la division à par $7$ avant d'utiliser le log car sinon, on ne peut pas utiliser la formule $\log(a^n) = n\times \log(a)$.
|
||||
|
||||
\[x = \frac{\log(3.0)}{\log(0.19)}\]
|
||||
\end{enumerate}
|
||||
\end{solution}
|
||||
|
||||
\begin{exercise}[subtitle={Restaurant}]
|
||||
Un \emph{food truck}, ouvert le midi et le soir, propose deux types de formules :
|
||||
|
||||
\setlength\parindent{10mm}
|
||||
\begin{itemize}
|
||||
\item la formule \emph{Burger} ;
|
||||
\item la formule \emph{Wok}.
|
||||
\end{itemize}
|
||||
\setlength\parindent{0mm}
|
||||
|
||||
\medskip
|
||||
|
||||
Le gérant a remarqué que 15\,\% de ses ventes ont lieu le midi. Le quart des ventes du midi correspondent à la formule \emph{Burger}, alors que 13\,\% des ventes du soir correspondent à la formule \emph{Wok}.
|
||||
|
||||
Le gérant se constitue un fichier en notant, pour chaque vente, la formule choisie et le moment de cette vente (midi ou soir).
|
||||
|
||||
On prélève une fiche de façon équiprobable. On définit les quatre évènements suivants:
|
||||
|
||||
\begin{enumerate}
|
||||
\item $M$ : \og la fiche correspond à une vente du midi\fg{} ;
|
||||
\item $S$ : \og la fiche correspond à une vente du soir\fg {};
|
||||
\item $W$ : \og la fiche correspond à une formule \emph{Wok} \fg{} ;
|
||||
\item $B$ : \og la fiche correspond à une formule \emph{Burger} \fg.
|
||||
\end{enumerate}
|
||||
\setlength\parindent{0mm}
|
||||
|
||||
\medskip
|
||||
|
||||
\begin{enumerate}
|
||||
\item Recopier puis compléter l'arbre pondéré
|
||||
|
||||
\begin{center}
|
||||
\begin{tikzpicture}[sloped]
|
||||
\node {.}
|
||||
child {node {$M$}
|
||||
child {node {$W$}
|
||||
edge from parent
|
||||
node[above] {...}
|
||||
}
|
||||
child {node {$B$}
|
||||
edge from parent
|
||||
node[above] {...}
|
||||
}
|
||||
edge from parent
|
||||
node[above] {...}
|
||||
}
|
||||
child[missing] {}
|
||||
child { node {$S$}
|
||||
child {node {$W$}
|
||||
edge from parent
|
||||
node[above] {...}
|
||||
}
|
||||
child {node {$B$}
|
||||
edge from parent
|
||||
node[above] {...}
|
||||
}
|
||||
edge from parent
|
||||
node[above] {...}
|
||||
} ;
|
||||
\end{tikzpicture}
|
||||
\end{center}
|
||||
|
||||
\item Calculer la probabilité de l'évènement $M \cap W$. Interpréter ce résultat dans le contexte de l'exercice.
|
||||
\item Montrer que la probabilité que la fiche choisie corresponde à une formule \emph{Burger} est égale à $0.777$.
|
||||
\item On a prélevé une fiche correspondant à la formule \emph{Burger}. Quelle est la probabilité, arrondie au millième, que la vente ait eu lieu le soir?
|
||||
\end{enumerate}
|
||||
\end{exercise}
|
||||
|
||||
\begin{solution}
|
||||
\begin{enumerate}
|
||||
\item
|
||||
\begin{center}
|
||||
\begin{tikzpicture}[sloped]
|
||||
\node {.}
|
||||
child {node {$M$}
|
||||
child {node {$W$}
|
||||
edge from parent
|
||||
node[above] {$0.75$}
|
||||
}
|
||||
child {node {$B$}
|
||||
edge from parent
|
||||
node[above] {$0.25$}
|
||||
}
|
||||
edge from parent
|
||||
node[above] {$0.15$}
|
||||
}
|
||||
child[missing] {}
|
||||
child { node {$S$}
|
||||
child {node {$W$}
|
||||
edge from parent
|
||||
node[above] {$0.13$}
|
||||
}
|
||||
child {node {$B$}
|
||||
edge from parent
|
||||
node[above] {$0.87$}
|
||||
}
|
||||
edge from parent
|
||||
node[above] {$0.85$}
|
||||
} ;
|
||||
\end{tikzpicture}
|
||||
\end{center}
|
||||
\item On calcule la probabilité que la vente soit un wok et ait eu lieu à midi
|
||||
\[ P(M\cap W) = P(M) \times P_M(W) = 0.15 \times 0.75 = 0.1125 \]
|
||||
\item Probabilité que la vente soit un burger.
|
||||
\[
|
||||
P(B) = P(M\cap B) + P(S\cap B) = 0.15 \times 0.75 + 0.85 \times 0.13 = 0.777
|
||||
\]
|
||||
\item On cherche à calculer la quantité $P_B(S)$. Pour cela on utilise la formule de Bayes
|
||||
\[
|
||||
P_B(S) = \frac{P(B\cap S)}{P(B)} = \frac{P_S(B) \times P(S)}{P(B)} = \frac{0.87\times 0.85}{0.777} = 0.9517374517374516 \approx 0.952
|
||||
\]
|
||||
\end{enumerate}
|
||||
\end{solution}
|
||||
|
||||
\begin{exercise}[subtitle={Continent plastique}]
|
||||
\textit{Les quantités évoqués dans cette exercice sont générés au hasard et sont donc complètement farfelus.}
|
||||
\medskip
|
||||
Le \og continent de plastique\fg{} est la plus grande des plaques de déchets plastiques évoluant sur les océans. Elle occupe actuellement dans l'océan Pacifique une surface dont l'aire est évaluée à plus de $1,6$ million de km$^2$, entre Hawaï et la Californie.
|
||||
|
||||
En 2017, des scientifiques ont estimé qu'il y avait $2$ millions de tonnes de déchets plastiques qui était déversé chaque année dans les océans et que cette quantité augmentait de $30\n\%$ par chaque année.
|
||||
|
||||
On modélise l'évolution de la masse de ces déchets plastiques déversée chaque année, si rien n'est fait pour la réduire, par une suite géométrique $\left(u_n\right)$. L'arrondi au centième du terme $u_n$ représente la masse de ces déchets déversée chaque année, exprimée en million de tonnes, pour l'année $(2017 + n)$.
|
||||
|
||||
\medskip
|
||||
|
||||
\begin{enumerate}
|
||||
\item Expliquer pourquoi la suite $u_n$ est géométrique?
|
||||
\item Calculer $u_1$ et $u_2$.
|
||||
\item Exprimer $u_n$ en fonction de $n$.
|
||||
\item Au début de l'année 2017, il y avait $300$ millions de tonnes de déchets plastique. Calculer la quantité totale de déchets plastiques en 2030.
|
||||
\item On souhaite déterminer en quelle année la masse totale de ces déchets plastiques aura pour la première fois augmenté de $50$\,\% par rapport à sa valeur de 2017.
|
||||
\begin{enumerate}
|
||||
\item Recopier et compléter l'algorithme ci-dessous pour que la variable $N$ contienne la réponse au problème posé.
|
||||
|
||||
\begin{center}
|
||||
\begin{tabularx}{0.4\linewidth}{|X|}\hline
|
||||
$N = 2017$\\
|
||||
$U = 2$ \\
|
||||
$S = 300 + U$ \\
|
||||
while $S < 450$: \\
|
||||
\hspace{1cm} $N = \ldots$\\
|
||||
\hspace{1cm} $U = \ldots$\\
|
||||
\hspace{1cm} $S = \ldots$\\
|
||||
\hline
|
||||
\end{tabularx}
|
||||
\end{center}
|
||||
\item Que contiennent les variables $S$, $U$ et $N$ après exécution de cet algorithme ?
|
||||
|
||||
Interpréter les résultats dans le contexte de l'exercice.
|
||||
\end{enumerate}
|
||||
\end{enumerate}
|
||||
\end{exercise}
|
||||
|
||||
\begin{solution}
|
||||
\begin{enumerate}
|
||||
\item Une augmentation de $30\,\%$ revient à multiplier la quantité par $1.3$. La suite est donc bien géométrique. Son premier terme est $u_0 = 2$ et sa raison est $q = 1.3$
|
||||
\item
|
||||
\[
|
||||
u_1 = u_0 * 1.3 = 2.6
|
||||
\]
|
||||
\[
|
||||
u_2 = u_0 * 1.3^2 = 3.38
|
||||
\]
|
||||
\item
|
||||
\[
|
||||
u_n = u_0 \times q^n = 2 \times 1.3^n
|
||||
\]
|
||||
\item On calcule la quantité totale déversée entre 2017 et 2030.
|
||||
\[
|
||||
\sum_{n = 0}^{13} u_n = u_0 \times \frac{1-q^{13}}{1-q} = 2 \times \frac{1 - 1.3^{13}}{1 - 1.3} = 195.25
|
||||
\]
|
||||
On en déduit la quantité totale de déchets en 2030
|
||||
\[
|
||||
300 + 195.25 = 495.25
|
||||
\]
|
||||
\item
|
||||
\begin{enumerate}
|
||||
\item ~
|
||||
\begin{center}
|
||||
\begin{tabularx}{0.4\linewidth}{|X|}\hline
|
||||
$N \gets 2017$\\
|
||||
$U \gets 2$ \\
|
||||
$S \gets 300 + U$ \\
|
||||
Tant que $S < 450$ \\
|
||||
\hspace{1cm} $N \gets N + 1$\\
|
||||
\hspace{1cm} $U \gets U * 1.3$\\
|
||||
\hspace{1cm} $S \gets S + u$\\
|
||||
Fin Tant que\\\hline
|
||||
\end{tabularx}
|
||||
\end{center}
|
||||
\item \textit{Pas de correction automatisé}
|
||||
\end{enumerate}
|
||||
\end{enumerate}
|
||||
\end{solution}
|
||||
|
||||
\end{document}
|
||||
|
||||
%%% Local Variables:
|
||||
%%% mode: latex
|
||||
%%% TeX-master: "master"
|
||||
%%% End:
|
262
TST/DS/DS_21_04_07/TST3/07_210407_DS8.tex
Normal file
262
TST/DS/DS_21_04_07/TST3/07_210407_DS8.tex
Normal file
@ -0,0 +1,262 @@
|
||||
\documentclass[a4paper,10pt]{article}
|
||||
\usepackage{myXsim}
|
||||
|
||||
% Title Page
|
||||
\title{DS8 \hfill DEBRAS Noémie}
|
||||
\tribe{TST}
|
||||
\date{\hfillÀ render pour le Mercredi 7 avril}
|
||||
|
||||
\xsimsetup{
|
||||
solution/print = false
|
||||
}
|
||||
|
||||
\begin{document}
|
||||
\maketitle
|
||||
|
||||
\begin{exercise}[subtitle={Automatismes}]
|
||||
\textit{Toutes les questions de cette exercice sont indépendantes et peuvent être répondus séparément}
|
||||
\begin{enumerate}
|
||||
\item De janvier à septembre, une quantité a augmenté de $24\,\%$. Faire un schéma pour représenter la situation puis calculer le taux d'évolution moyen mensuel.
|
||||
\item Une quantité augmente de $24\,\%$ par ans. En 2020, elle est de 110\euro. Quelle était sa valeur en 2019? Faire un schéma pour représenter la situation.
|
||||
\item Déterminer l'équation de la droite \\
|
||||
\begin{tikzpicture}[xscale=0.8, yscale=0.5]
|
||||
\tkzInit[xmin=-5,xmax=5,xstep=1,
|
||||
ymin=-5,ymax=5,ystep=1]
|
||||
\tkzGrid
|
||||
\tkzAxeXY
|
||||
\tkzFct[domain=-5:5,color=red,very thick]%
|
||||
{0.6666666666666666*\x -1};
|
||||
\end{tikzpicture}
|
||||
\item Résoudre l'équation $8 \times 0.49^x = 9$
|
||||
\end{enumerate}
|
||||
\end{exercise}
|
||||
|
||||
\begin{solution}
|
||||
\begin{enumerate}
|
||||
\item On veut partager cette évolution en 8 évolutions.
|
||||
\[
|
||||
\left(1 + \frac{24}{100}\right)^{\frac{1}{8}} = 1.0273
|
||||
\]
|
||||
Donc le taux d'évolution moyen est
|
||||
\[
|
||||
t_m = 1.0273 - 1 = 0.027300000000000102
|
||||
\]
|
||||
\item Coefficient multiplicateur pour revenir en arrière
|
||||
\[
|
||||
CM = (1 + \frac{24}{100})^{-1} = 0.8065
|
||||
\]
|
||||
On en déduit la quantité en 2019
|
||||
\[
|
||||
110 * 0.8065 = 88.715
|
||||
\]
|
||||
\item L'équation de la droite est
|
||||
\[
|
||||
y = 0.6666666666666666 x -1
|
||||
\]
|
||||
\item Il faut penser à faire la division à par $8$ avant d'utiliser le log car sinon, on ne peut pas utiliser la formule $\log(a^n) = n\times \log(a)$.
|
||||
|
||||
\[x = \frac{\log(1.12)}{\log(0.49)}\]
|
||||
\end{enumerate}
|
||||
\end{solution}
|
||||
|
||||
\begin{exercise}[subtitle={Restaurant}]
|
||||
Un \emph{food truck}, ouvert le midi et le soir, propose deux types de formules :
|
||||
|
||||
\setlength\parindent{10mm}
|
||||
\begin{itemize}
|
||||
\item la formule \emph{Burger} ;
|
||||
\item la formule \emph{Wok}.
|
||||
\end{itemize}
|
||||
\setlength\parindent{0mm}
|
||||
|
||||
\medskip
|
||||
|
||||
Le gérant a remarqué que 49\,\% de ses ventes ont lieu le midi. Le quart des ventes du midi correspondent à la formule \emph{Burger}, alors que 61\,\% des ventes du soir correspondent à la formule \emph{Wok}.
|
||||
|
||||
Le gérant se constitue un fichier en notant, pour chaque vente, la formule choisie et le moment de cette vente (midi ou soir).
|
||||
|
||||
On prélève une fiche de façon équiprobable. On définit les quatre évènements suivants:
|
||||
|
||||
\begin{enumerate}
|
||||
\item $M$ : \og la fiche correspond à une vente du midi\fg{} ;
|
||||
\item $S$ : \og la fiche correspond à une vente du soir\fg {};
|
||||
\item $W$ : \og la fiche correspond à une formule \emph{Wok} \fg{} ;
|
||||
\item $B$ : \og la fiche correspond à une formule \emph{Burger} \fg.
|
||||
\end{enumerate}
|
||||
\setlength\parindent{0mm}
|
||||
|
||||
\medskip
|
||||
|
||||
\begin{enumerate}
|
||||
\item Recopier puis compléter l'arbre pondéré
|
||||
|
||||
\begin{center}
|
||||
\begin{tikzpicture}[sloped]
|
||||
\node {.}
|
||||
child {node {$M$}
|
||||
child {node {$W$}
|
||||
edge from parent
|
||||
node[above] {...}
|
||||
}
|
||||
child {node {$B$}
|
||||
edge from parent
|
||||
node[above] {...}
|
||||
}
|
||||
edge from parent
|
||||
node[above] {...}
|
||||
}
|
||||
child[missing] {}
|
||||
child { node {$S$}
|
||||
child {node {$W$}
|
||||
edge from parent
|
||||
node[above] {...}
|
||||
}
|
||||
child {node {$B$}
|
||||
edge from parent
|
||||
node[above] {...}
|
||||
}
|
||||
edge from parent
|
||||
node[above] {...}
|
||||
} ;
|
||||
\end{tikzpicture}
|
||||
\end{center}
|
||||
|
||||
\item Calculer la probabilité de l'évènement $M \cap W$. Interpréter ce résultat dans le contexte de l'exercice.
|
||||
\item Montrer que la probabilité que la fiche choisie corresponde à une formule \emph{Burger} est égale à $0.3214$.
|
||||
\item On a prélevé une fiche correspondant à la formule \emph{Burger}. Quelle est la probabilité, arrondie au millième, que la vente ait eu lieu le soir?
|
||||
\end{enumerate}
|
||||
\end{exercise}
|
||||
|
||||
\begin{solution}
|
||||
\begin{enumerate}
|
||||
\item
|
||||
\begin{center}
|
||||
\begin{tikzpicture}[sloped]
|
||||
\node {.}
|
||||
child {node {$M$}
|
||||
child {node {$W$}
|
||||
edge from parent
|
||||
node[above] {$0.75$}
|
||||
}
|
||||
child {node {$B$}
|
||||
edge from parent
|
||||
node[above] {$0.25$}
|
||||
}
|
||||
edge from parent
|
||||
node[above] {$0.49$}
|
||||
}
|
||||
child[missing] {}
|
||||
child { node {$S$}
|
||||
child {node {$W$}
|
||||
edge from parent
|
||||
node[above] {$0.61$}
|
||||
}
|
||||
child {node {$B$}
|
||||
edge from parent
|
||||
node[above] {$0.39$}
|
||||
}
|
||||
edge from parent
|
||||
node[above] {$0.51$}
|
||||
} ;
|
||||
\end{tikzpicture}
|
||||
\end{center}
|
||||
\item On calcule la probabilité que la vente soit un wok et ait eu lieu à midi
|
||||
\[ P(M\cap W) = P(M) \times P_M(W) = 0.49 \times 0.75 = 0.3675 \]
|
||||
\item Probabilité que la vente soit un burger.
|
||||
\[
|
||||
P(B) = P(M\cap B) + P(S\cap B) = 0.49 \times 0.75 + 0.51 \times 0.61 = 0.3214
|
||||
\]
|
||||
\item On cherche à calculer la quantité $P_B(S)$. Pour cela on utilise la formule de Bayes
|
||||
\[
|
||||
P_B(S) = \frac{P(B\cap S)}{P(B)} = \frac{P_S(B) \times P(S)}{P(B)} = \frac{0.39\times 0.51}{0.3214} = 0.618855009334163 \approx 0.619
|
||||
\]
|
||||
\end{enumerate}
|
||||
\end{solution}
|
||||
|
||||
\begin{exercise}[subtitle={Continent plastique}]
|
||||
\textit{Les quantités évoqués dans cette exercice sont générés au hasard et sont donc complètement farfelus.}
|
||||
\medskip
|
||||
Le \og continent de plastique\fg{} est la plus grande des plaques de déchets plastiques évoluant sur les océans. Elle occupe actuellement dans l'océan Pacifique une surface dont l'aire est évaluée à plus de $1,6$ million de km$^2$, entre Hawaï et la Californie.
|
||||
|
||||
En 2017, des scientifiques ont estimé qu'il y avait $15$ millions de tonnes de déchets plastiques qui était déversé chaque année dans les océans et que cette quantité augmentait de $21\n\%$ par chaque année.
|
||||
|
||||
On modélise l'évolution de la masse de ces déchets plastiques déversée chaque année, si rien n'est fait pour la réduire, par une suite géométrique $\left(u_n\right)$. L'arrondi au centième du terme $u_n$ représente la masse de ces déchets déversée chaque année, exprimée en million de tonnes, pour l'année $(2017 + n)$.
|
||||
|
||||
\medskip
|
||||
|
||||
\begin{enumerate}
|
||||
\item Expliquer pourquoi la suite $u_n$ est géométrique?
|
||||
\item Calculer $u_1$ et $u_2$.
|
||||
\item Exprimer $u_n$ en fonction de $n$.
|
||||
\item Au début de l'année 2017, il y avait $300$ millions de tonnes de déchets plastique. Calculer la quantité totale de déchets plastiques en 2030.
|
||||
\item On souhaite déterminer en quelle année la masse totale de ces déchets plastiques aura pour la première fois augmenté de $50$\,\% par rapport à sa valeur de 2017.
|
||||
\begin{enumerate}
|
||||
\item Recopier et compléter l'algorithme ci-dessous pour que la variable $N$ contienne la réponse au problème posé.
|
||||
|
||||
\begin{center}
|
||||
\begin{tabularx}{0.4\linewidth}{|X|}\hline
|
||||
$N = 2017$\\
|
||||
$U = 15$ \\
|
||||
$S = 300 + U$ \\
|
||||
while $S < 450$: \\
|
||||
\hspace{1cm} $N = \ldots$\\
|
||||
\hspace{1cm} $U = \ldots$\\
|
||||
\hspace{1cm} $S = \ldots$\\
|
||||
\hline
|
||||
\end{tabularx}
|
||||
\end{center}
|
||||
\item Que contiennent les variables $S$, $U$ et $N$ après exécution de cet algorithme ?
|
||||
|
||||
Interpréter les résultats dans le contexte de l'exercice.
|
||||
\end{enumerate}
|
||||
\end{enumerate}
|
||||
\end{exercise}
|
||||
|
||||
\begin{solution}
|
||||
\begin{enumerate}
|
||||
\item Une augmentation de $21\,\%$ revient à multiplier la quantité par $1.21$. La suite est donc bien géométrique. Son premier terme est $u_0 = 15$ et sa raison est $q = 1.21$
|
||||
\item
|
||||
\[
|
||||
u_1 = u_0 * 1.21 = 18.15
|
||||
\]
|
||||
\[
|
||||
u_2 = u_0 * 1.21^2 = 21.9615
|
||||
\]
|
||||
\item
|
||||
\[
|
||||
u_n = u_0 \times q^n = 15 \times 1.21^n
|
||||
\]
|
||||
\item On calcule la quantité totale déversée entre 2017 et 2030.
|
||||
\[
|
||||
\sum_{n = 0}^{13} u_n = u_0 \times \frac{1-q^{13}}{1-q} = 15 \times \frac{1 - 1.21^{13}}{1 - 1.21} = 779.87
|
||||
\]
|
||||
On en déduit la quantité totale de déchets en 2030
|
||||
\[
|
||||
300 + 779.87 = 1079.87
|
||||
\]
|
||||
\item
|
||||
\begin{enumerate}
|
||||
\item ~
|
||||
\begin{center}
|
||||
\begin{tabularx}{0.4\linewidth}{|X|}\hline
|
||||
$N \gets 2017$\\
|
||||
$U \gets 15$ \\
|
||||
$S \gets 300 + U$ \\
|
||||
Tant que $S < 450$ \\
|
||||
\hspace{1cm} $N \gets N + 1$\\
|
||||
\hspace{1cm} $U \gets U * 1.21$\\
|
||||
\hspace{1cm} $S \gets S + u$\\
|
||||
Fin Tant que\\\hline
|
||||
\end{tabularx}
|
||||
\end{center}
|
||||
\item \textit{Pas de correction automatisé}
|
||||
\end{enumerate}
|
||||
\end{enumerate}
|
||||
\end{solution}
|
||||
|
||||
\end{document}
|
||||
|
||||
%%% Local Variables:
|
||||
%%% mode: latex
|
||||
%%% TeX-master: "master"
|
||||
%%% End:
|
262
TST/DS/DS_21_04_07/TST3/08_210407_DS8.tex
Normal file
262
TST/DS/DS_21_04_07/TST3/08_210407_DS8.tex
Normal file
@ -0,0 +1,262 @@
|
||||
\documentclass[a4paper,10pt]{article}
|
||||
\usepackage{myXsim}
|
||||
|
||||
% Title Page
|
||||
\title{DS8 \hfill GERMAIN Anaïs}
|
||||
\tribe{TST}
|
||||
\date{\hfillÀ render pour le Mercredi 7 avril}
|
||||
|
||||
\xsimsetup{
|
||||
solution/print = false
|
||||
}
|
||||
|
||||
\begin{document}
|
||||
\maketitle
|
||||
|
||||
\begin{exercise}[subtitle={Automatismes}]
|
||||
\textit{Toutes les questions de cette exercice sont indépendantes et peuvent être répondus séparément}
|
||||
\begin{enumerate}
|
||||
\item De janvier à septembre, une quantité a augmenté de $27\,\%$. Faire un schéma pour représenter la situation puis calculer le taux d'évolution moyen mensuel.
|
||||
\item Une quantité augmente de $27\,\%$ par ans. En 2020, elle est de 132\euro. Quelle était sa valeur en 2019? Faire un schéma pour représenter la situation.
|
||||
\item Déterminer l'équation de la droite \\
|
||||
\begin{tikzpicture}[xscale=0.8, yscale=0.5]
|
||||
\tkzInit[xmin=-5,xmax=5,xstep=1,
|
||||
ymin=-5,ymax=5,ystep=1]
|
||||
\tkzGrid
|
||||
\tkzAxeXY
|
||||
\tkzFct[domain=-5:5,color=red,very thick]%
|
||||
{2.0*\x -4};
|
||||
\end{tikzpicture}
|
||||
\item Résoudre l'équation $7 \times 0.05^x = 18$
|
||||
\end{enumerate}
|
||||
\end{exercise}
|
||||
|
||||
\begin{solution}
|
||||
\begin{enumerate}
|
||||
\item On veut partager cette évolution en 8 évolutions.
|
||||
\[
|
||||
\left(1 + \frac{27}{100}\right)^{\frac{1}{8}} = 1.0303
|
||||
\]
|
||||
Donc le taux d'évolution moyen est
|
||||
\[
|
||||
t_m = 1.0303 - 1 = 0.030299999999999994
|
||||
\]
|
||||
\item Coefficient multiplicateur pour revenir en arrière
|
||||
\[
|
||||
CM = (1 + \frac{27}{100})^{-1} = 0.7874
|
||||
\]
|
||||
On en déduit la quantité en 2019
|
||||
\[
|
||||
132 * 0.7874 = 103.9368
|
||||
\]
|
||||
\item L'équation de la droite est
|
||||
\[
|
||||
y = 2.0 x -4
|
||||
\]
|
||||
\item Il faut penser à faire la division à par $7$ avant d'utiliser le log car sinon, on ne peut pas utiliser la formule $\log(a^n) = n\times \log(a)$.
|
||||
|
||||
\[x = \frac{\log(2.57)}{\log(0.05)}\]
|
||||
\end{enumerate}
|
||||
\end{solution}
|
||||
|
||||
\begin{exercise}[subtitle={Restaurant}]
|
||||
Un \emph{food truck}, ouvert le midi et le soir, propose deux types de formules :
|
||||
|
||||
\setlength\parindent{10mm}
|
||||
\begin{itemize}
|
||||
\item la formule \emph{Burger} ;
|
||||
\item la formule \emph{Wok}.
|
||||
\end{itemize}
|
||||
\setlength\parindent{0mm}
|
||||
|
||||
\medskip
|
||||
|
||||
Le gérant a remarqué que 38\,\% de ses ventes ont lieu le midi. Le quart des ventes du midi correspondent à la formule \emph{Burger}, alors que 63\,\% des ventes du soir correspondent à la formule \emph{Wok}.
|
||||
|
||||
Le gérant se constitue un fichier en notant, pour chaque vente, la formule choisie et le moment de cette vente (midi ou soir).
|
||||
|
||||
On prélève une fiche de façon équiprobable. On définit les quatre évènements suivants:
|
||||
|
||||
\begin{enumerate}
|
||||
\item $M$ : \og la fiche correspond à une vente du midi\fg{} ;
|
||||
\item $S$ : \og la fiche correspond à une vente du soir\fg {};
|
||||
\item $W$ : \og la fiche correspond à une formule \emph{Wok} \fg{} ;
|
||||
\item $B$ : \og la fiche correspond à une formule \emph{Burger} \fg.
|
||||
\end{enumerate}
|
||||
\setlength\parindent{0mm}
|
||||
|
||||
\medskip
|
||||
|
||||
\begin{enumerate}
|
||||
\item Recopier puis compléter l'arbre pondéré
|
||||
|
||||
\begin{center}
|
||||
\begin{tikzpicture}[sloped]
|
||||
\node {.}
|
||||
child {node {$M$}
|
||||
child {node {$W$}
|
||||
edge from parent
|
||||
node[above] {...}
|
||||
}
|
||||
child {node {$B$}
|
||||
edge from parent
|
||||
node[above] {...}
|
||||
}
|
||||
edge from parent
|
||||
node[above] {...}
|
||||
}
|
||||
child[missing] {}
|
||||
child { node {$S$}
|
||||
child {node {$W$}
|
||||
edge from parent
|
||||
node[above] {...}
|
||||
}
|
||||
child {node {$B$}
|
||||
edge from parent
|
||||
node[above] {...}
|
||||
}
|
||||
edge from parent
|
||||
node[above] {...}
|
||||
} ;
|
||||
\end{tikzpicture}
|
||||
\end{center}
|
||||
|
||||
\item Calculer la probabilité de l'évènement $M \cap W$. Interpréter ce résultat dans le contexte de l'exercice.
|
||||
\item Montrer que la probabilité que la fiche choisie corresponde à une formule \emph{Burger} est égale à $0.3244$.
|
||||
\item On a prélevé une fiche correspondant à la formule \emph{Burger}. Quelle est la probabilité, arrondie au millième, que la vente ait eu lieu le soir?
|
||||
\end{enumerate}
|
||||
\end{exercise}
|
||||
|
||||
\begin{solution}
|
||||
\begin{enumerate}
|
||||
\item
|
||||
\begin{center}
|
||||
\begin{tikzpicture}[sloped]
|
||||
\node {.}
|
||||
child {node {$M$}
|
||||
child {node {$W$}
|
||||
edge from parent
|
||||
node[above] {$0.75$}
|
||||
}
|
||||
child {node {$B$}
|
||||
edge from parent
|
||||
node[above] {$0.25$}
|
||||
}
|
||||
edge from parent
|
||||
node[above] {$0.38$}
|
||||
}
|
||||
child[missing] {}
|
||||
child { node {$S$}
|
||||
child {node {$W$}
|
||||
edge from parent
|
||||
node[above] {$0.63$}
|
||||
}
|
||||
child {node {$B$}
|
||||
edge from parent
|
||||
node[above] {$0.37$}
|
||||
}
|
||||
edge from parent
|
||||
node[above] {$0.62$}
|
||||
} ;
|
||||
\end{tikzpicture}
|
||||
\end{center}
|
||||
\item On calcule la probabilité que la vente soit un wok et ait eu lieu à midi
|
||||
\[ P(M\cap W) = P(M) \times P_M(W) = 0.38 \times 0.75 = 0.285 \]
|
||||
\item Probabilité que la vente soit un burger.
|
||||
\[
|
||||
P(B) = P(M\cap B) + P(S\cap B) = 0.38 \times 0.75 + 0.62 \times 0.63 = 0.3244
|
||||
\]
|
||||
\item On cherche à calculer la quantité $P_B(S)$. Pour cela on utilise la formule de Bayes
|
||||
\[
|
||||
P_B(S) = \frac{P(B\cap S)}{P(B)} = \frac{P_S(B) \times P(S)}{P(B)} = \frac{0.37\times 0.62}{0.3244} = 0.7071516646115905 \approx 0.707
|
||||
\]
|
||||
\end{enumerate}
|
||||
\end{solution}
|
||||
|
||||
\begin{exercise}[subtitle={Continent plastique}]
|
||||
\textit{Les quantités évoqués dans cette exercice sont générés au hasard et sont donc complètement farfelus.}
|
||||
\medskip
|
||||
Le \og continent de plastique\fg{} est la plus grande des plaques de déchets plastiques évoluant sur les océans. Elle occupe actuellement dans l'océan Pacifique une surface dont l'aire est évaluée à plus de $1,6$ million de km$^2$, entre Hawaï et la Californie.
|
||||
|
||||
En 2017, des scientifiques ont estimé qu'il y avait $18$ millions de tonnes de déchets plastiques qui était déversé chaque année dans les océans et que cette quantité augmentait de $28\n\%$ par chaque année.
|
||||
|
||||
On modélise l'évolution de la masse de ces déchets plastiques déversée chaque année, si rien n'est fait pour la réduire, par une suite géométrique $\left(u_n\right)$. L'arrondi au centième du terme $u_n$ représente la masse de ces déchets déversée chaque année, exprimée en million de tonnes, pour l'année $(2017 + n)$.
|
||||
|
||||
\medskip
|
||||
|
||||
\begin{enumerate}
|
||||
\item Expliquer pourquoi la suite $u_n$ est géométrique?
|
||||
\item Calculer $u_1$ et $u_2$.
|
||||
\item Exprimer $u_n$ en fonction de $n$.
|
||||
\item Au début de l'année 2017, il y avait $300$ millions de tonnes de déchets plastique. Calculer la quantité totale de déchets plastiques en 2030.
|
||||
\item On souhaite déterminer en quelle année la masse totale de ces déchets plastiques aura pour la première fois augmenté de $50$\,\% par rapport à sa valeur de 2017.
|
||||
\begin{enumerate}
|
||||
\item Recopier et compléter l'algorithme ci-dessous pour que la variable $N$ contienne la réponse au problème posé.
|
||||
|
||||
\begin{center}
|
||||
\begin{tabularx}{0.4\linewidth}{|X|}\hline
|
||||
$N = 2017$\\
|
||||
$U = 18$ \\
|
||||
$S = 300 + U$ \\
|
||||
while $S < 450$: \\
|
||||
\hspace{1cm} $N = \ldots$\\
|
||||
\hspace{1cm} $U = \ldots$\\
|
||||
\hspace{1cm} $S = \ldots$\\
|
||||
\hline
|
||||
\end{tabularx}
|
||||
\end{center}
|
||||
\item Que contiennent les variables $S$, $U$ et $N$ après exécution de cet algorithme ?
|
||||
|
||||
Interpréter les résultats dans le contexte de l'exercice.
|
||||
\end{enumerate}
|
||||
\end{enumerate}
|
||||
\end{exercise}
|
||||
|
||||
\begin{solution}
|
||||
\begin{enumerate}
|
||||
\item Une augmentation de $28\,\%$ revient à multiplier la quantité par $1.28$. La suite est donc bien géométrique. Son premier terme est $u_0 = 18$ et sa raison est $q = 1.28$
|
||||
\item
|
||||
\[
|
||||
u_1 = u_0 * 1.28 = 23.04
|
||||
\]
|
||||
\[
|
||||
u_2 = u_0 * 1.28^2 = 29.4912
|
||||
\]
|
||||
\item
|
||||
\[
|
||||
u_n = u_0 \times q^n = 18 \times 1.28^n
|
||||
\]
|
||||
\item On calcule la quantité totale déversée entre 2017 et 2030.
|
||||
\[
|
||||
\sum_{n = 0}^{13} u_n = u_0 \times \frac{1-q^{13}}{1-q} = 18 \times \frac{1 - 1.28^{13}}{1 - 1.28} = 1527.35
|
||||
\]
|
||||
On en déduit la quantité totale de déchets en 2030
|
||||
\[
|
||||
300 + 1527.35 = 1827.35
|
||||
\]
|
||||
\item
|
||||
\begin{enumerate}
|
||||
\item ~
|
||||
\begin{center}
|
||||
\begin{tabularx}{0.4\linewidth}{|X|}\hline
|
||||
$N \gets 2017$\\
|
||||
$U \gets 18$ \\
|
||||
$S \gets 300 + U$ \\
|
||||
Tant que $S < 450$ \\
|
||||
\hspace{1cm} $N \gets N + 1$\\
|
||||
\hspace{1cm} $U \gets U * 1.28$\\
|
||||
\hspace{1cm} $S \gets S + u$\\
|
||||
Fin Tant que\\\hline
|
||||
\end{tabularx}
|
||||
\end{center}
|
||||
\item \textit{Pas de correction automatisé}
|
||||
\end{enumerate}
|
||||
\end{enumerate}
|
||||
\end{solution}
|
||||
|
||||
\end{document}
|
||||
|
||||
%%% Local Variables:
|
||||
%%% mode: latex
|
||||
%%% TeX-master: "master"
|
||||
%%% End:
|
262
TST/DS/DS_21_04_07/TST3/09_210407_DS8.tex
Normal file
262
TST/DS/DS_21_04_07/TST3/09_210407_DS8.tex
Normal file
@ -0,0 +1,262 @@
|
||||
\documentclass[a4paper,10pt]{article}
|
||||
\usepackage{myXsim}
|
||||
|
||||
% Title Page
|
||||
\title{DS8 \hfill HADJRAS Mohcine}
|
||||
\tribe{TST}
|
||||
\date{\hfillÀ render pour le Mercredi 7 avril}
|
||||
|
||||
\xsimsetup{
|
||||
solution/print = false
|
||||
}
|
||||
|
||||
\begin{document}
|
||||
\maketitle
|
||||
|
||||
\begin{exercise}[subtitle={Automatismes}]
|
||||
\textit{Toutes les questions de cette exercice sont indépendantes et peuvent être répondus séparément}
|
||||
\begin{enumerate}
|
||||
\item De janvier à septembre, une quantité a augmenté de $19\,\%$. Faire un schéma pour représenter la situation puis calculer le taux d'évolution moyen mensuel.
|
||||
\item Une quantité augmente de $19\,\%$ par ans. En 2020, elle est de 113\euro. Quelle était sa valeur en 2019? Faire un schéma pour représenter la situation.
|
||||
\item Déterminer l'équation de la droite \\
|
||||
\begin{tikzpicture}[xscale=0.8, yscale=0.5]
|
||||
\tkzInit[xmin=-5,xmax=5,xstep=1,
|
||||
ymin=-5,ymax=5,ystep=1]
|
||||
\tkzGrid
|
||||
\tkzAxeXY
|
||||
\tkzFct[domain=-5:5,color=red,very thick]%
|
||||
{0.5*\x -1};
|
||||
\end{tikzpicture}
|
||||
\item Résoudre l'équation $4 \times 0.54^x = 6$
|
||||
\end{enumerate}
|
||||
\end{exercise}
|
||||
|
||||
\begin{solution}
|
||||
\begin{enumerate}
|
||||
\item On veut partager cette évolution en 8 évolutions.
|
||||
\[
|
||||
\left(1 + \frac{19}{100}\right)^{\frac{1}{8}} = 1.022
|
||||
\]
|
||||
Donc le taux d'évolution moyen est
|
||||
\[
|
||||
t_m = 1.022 - 1 = 0.02200000000000002
|
||||
\]
|
||||
\item Coefficient multiplicateur pour revenir en arrière
|
||||
\[
|
||||
CM = (1 + \frac{19}{100})^{-1} = 0.8403
|
||||
\]
|
||||
On en déduit la quantité en 2019
|
||||
\[
|
||||
113 * 0.8403 = 94.9539
|
||||
\]
|
||||
\item L'équation de la droite est
|
||||
\[
|
||||
y = 0.5 x -1
|
||||
\]
|
||||
\item Il faut penser à faire la division à par $4$ avant d'utiliser le log car sinon, on ne peut pas utiliser la formule $\log(a^n) = n\times \log(a)$.
|
||||
|
||||
\[x = \frac{\log(1.5)}{\log(0.54)}\]
|
||||
\end{enumerate}
|
||||
\end{solution}
|
||||
|
||||
\begin{exercise}[subtitle={Restaurant}]
|
||||
Un \emph{food truck}, ouvert le midi et le soir, propose deux types de formules :
|
||||
|
||||
\setlength\parindent{10mm}
|
||||
\begin{itemize}
|
||||
\item la formule \emph{Burger} ;
|
||||
\item la formule \emph{Wok}.
|
||||
\end{itemize}
|
||||
\setlength\parindent{0mm}
|
||||
|
||||
\medskip
|
||||
|
||||
Le gérant a remarqué que 68\,\% de ses ventes ont lieu le midi. Le quart des ventes du midi correspondent à la formule \emph{Burger}, alors que 43\,\% des ventes du soir correspondent à la formule \emph{Wok}.
|
||||
|
||||
Le gérant se constitue un fichier en notant, pour chaque vente, la formule choisie et le moment de cette vente (midi ou soir).
|
||||
|
||||
On prélève une fiche de façon équiprobable. On définit les quatre évènements suivants:
|
||||
|
||||
\begin{enumerate}
|
||||
\item $M$ : \og la fiche correspond à une vente du midi\fg{} ;
|
||||
\item $S$ : \og la fiche correspond à une vente du soir\fg {};
|
||||
\item $W$ : \og la fiche correspond à une formule \emph{Wok} \fg{} ;
|
||||
\item $B$ : \og la fiche correspond à une formule \emph{Burger} \fg.
|
||||
\end{enumerate}
|
||||
\setlength\parindent{0mm}
|
||||
|
||||
\medskip
|
||||
|
||||
\begin{enumerate}
|
||||
\item Recopier puis compléter l'arbre pondéré
|
||||
|
||||
\begin{center}
|
||||
\begin{tikzpicture}[sloped]
|
||||
\node {.}
|
||||
child {node {$M$}
|
||||
child {node {$W$}
|
||||
edge from parent
|
||||
node[above] {...}
|
||||
}
|
||||
child {node {$B$}
|
||||
edge from parent
|
||||
node[above] {...}
|
||||
}
|
||||
edge from parent
|
||||
node[above] {...}
|
||||
}
|
||||
child[missing] {}
|
||||
child { node {$S$}
|
||||
child {node {$W$}
|
||||
edge from parent
|
||||
node[above] {...}
|
||||
}
|
||||
child {node {$B$}
|
||||
edge from parent
|
||||
node[above] {...}
|
||||
}
|
||||
edge from parent
|
||||
node[above] {...}
|
||||
} ;
|
||||
\end{tikzpicture}
|
||||
\end{center}
|
||||
|
||||
\item Calculer la probabilité de l'évènement $M \cap W$. Interpréter ce résultat dans le contexte de l'exercice.
|
||||
\item Montrer que la probabilité que la fiche choisie corresponde à une formule \emph{Burger} est égale à $0.3524$.
|
||||
\item On a prélevé une fiche correspondant à la formule \emph{Burger}. Quelle est la probabilité, arrondie au millième, que la vente ait eu lieu le soir?
|
||||
\end{enumerate}
|
||||
\end{exercise}
|
||||
|
||||
\begin{solution}
|
||||
\begin{enumerate}
|
||||
\item
|
||||
\begin{center}
|
||||
\begin{tikzpicture}[sloped]
|
||||
\node {.}
|
||||
child {node {$M$}
|
||||
child {node {$W$}
|
||||
edge from parent
|
||||
node[above] {$0.75$}
|
||||
}
|
||||
child {node {$B$}
|
||||
edge from parent
|
||||
node[above] {$0.25$}
|
||||
}
|
||||
edge from parent
|
||||
node[above] {$0.68$}
|
||||
}
|
||||
child[missing] {}
|
||||
child { node {$S$}
|
||||
child {node {$W$}
|
||||
edge from parent
|
||||
node[above] {$0.43$}
|
||||
}
|
||||
child {node {$B$}
|
||||
edge from parent
|
||||
node[above] {$0.57$}
|
||||
}
|
||||
edge from parent
|
||||
node[above] {$0.32$}
|
||||
} ;
|
||||
\end{tikzpicture}
|
||||
\end{center}
|
||||
\item On calcule la probabilité que la vente soit un wok et ait eu lieu à midi
|
||||
\[ P(M\cap W) = P(M) \times P_M(W) = 0.68 \times 0.75 = 0.51 \]
|
||||
\item Probabilité que la vente soit un burger.
|
||||
\[
|
||||
P(B) = P(M\cap B) + P(S\cap B) = 0.68 \times 0.75 + 0.32 \times 0.43 = 0.3524
|
||||
\]
|
||||
\item On cherche à calculer la quantité $P_B(S)$. Pour cela on utilise la formule de Bayes
|
||||
\[
|
||||
P_B(S) = \frac{P(B\cap S)}{P(B)} = \frac{P_S(B) \times P(S)}{P(B)} = \frac{0.57\times 0.32}{0.3524} = 0.5175936435868331 \approx 0.518
|
||||
\]
|
||||
\end{enumerate}
|
||||
\end{solution}
|
||||
|
||||
\begin{exercise}[subtitle={Continent plastique}]
|
||||
\textit{Les quantités évoqués dans cette exercice sont générés au hasard et sont donc complètement farfelus.}
|
||||
\medskip
|
||||
Le \og continent de plastique\fg{} est la plus grande des plaques de déchets plastiques évoluant sur les océans. Elle occupe actuellement dans l'océan Pacifique une surface dont l'aire est évaluée à plus de $1,6$ million de km$^2$, entre Hawaï et la Californie.
|
||||
|
||||
En 2017, des scientifiques ont estimé qu'il y avait $4$ millions de tonnes de déchets plastiques qui était déversé chaque année dans les océans et que cette quantité augmentait de $10\n\%$ par chaque année.
|
||||
|
||||
On modélise l'évolution de la masse de ces déchets plastiques déversée chaque année, si rien n'est fait pour la réduire, par une suite géométrique $\left(u_n\right)$. L'arrondi au centième du terme $u_n$ représente la masse de ces déchets déversée chaque année, exprimée en million de tonnes, pour l'année $(2017 + n)$.
|
||||
|
||||
\medskip
|
||||
|
||||
\begin{enumerate}
|
||||
\item Expliquer pourquoi la suite $u_n$ est géométrique?
|
||||
\item Calculer $u_1$ et $u_2$.
|
||||
\item Exprimer $u_n$ en fonction de $n$.
|
||||
\item Au début de l'année 2017, il y avait $300$ millions de tonnes de déchets plastique. Calculer la quantité totale de déchets plastiques en 2030.
|
||||
\item On souhaite déterminer en quelle année la masse totale de ces déchets plastiques aura pour la première fois augmenté de $50$\,\% par rapport à sa valeur de 2017.
|
||||
\begin{enumerate}
|
||||
\item Recopier et compléter l'algorithme ci-dessous pour que la variable $N$ contienne la réponse au problème posé.
|
||||
|
||||
\begin{center}
|
||||
\begin{tabularx}{0.4\linewidth}{|X|}\hline
|
||||
$N = 2017$\\
|
||||
$U = 4$ \\
|
||||
$S = 300 + U$ \\
|
||||
while $S < 450$: \\
|
||||
\hspace{1cm} $N = \ldots$\\
|
||||
\hspace{1cm} $U = \ldots$\\
|
||||
\hspace{1cm} $S = \ldots$\\
|
||||
\hline
|
||||
\end{tabularx}
|
||||
\end{center}
|
||||
\item Que contiennent les variables $S$, $U$ et $N$ après exécution de cet algorithme ?
|
||||
|
||||
Interpréter les résultats dans le contexte de l'exercice.
|
||||
\end{enumerate}
|
||||
\end{enumerate}
|
||||
\end{exercise}
|
||||
|
||||
\begin{solution}
|
||||
\begin{enumerate}
|
||||
\item Une augmentation de $10\,\%$ revient à multiplier la quantité par $1.1$. La suite est donc bien géométrique. Son premier terme est $u_0 = 4$ et sa raison est $q = 1.1$
|
||||
\item
|
||||
\[
|
||||
u_1 = u_0 * 1.1 = 4.4
|
||||
\]
|
||||
\[
|
||||
u_2 = u_0 * 1.1^2 = 4.84
|
||||
\]
|
||||
\item
|
||||
\[
|
||||
u_n = u_0 \times q^n = 4 \times 1.1^n
|
||||
\]
|
||||
\item On calcule la quantité totale déversée entre 2017 et 2030.
|
||||
\[
|
||||
\sum_{n = 0}^{13} u_n = u_0 \times \frac{1-q^{13}}{1-q} = 4 \times \frac{1 - 1.1^{13}}{1 - 1.1} = 98.09
|
||||
\]
|
||||
On en déduit la quantité totale de déchets en 2030
|
||||
\[
|
||||
300 + 98.09 = 398.09000000000003
|
||||
\]
|
||||
\item
|
||||
\begin{enumerate}
|
||||
\item ~
|
||||
\begin{center}
|
||||
\begin{tabularx}{0.4\linewidth}{|X|}\hline
|
||||
$N \gets 2017$\\
|
||||
$U \gets 4$ \\
|
||||
$S \gets 300 + U$ \\
|
||||
Tant que $S < 450$ \\
|
||||
\hspace{1cm} $N \gets N + 1$\\
|
||||
\hspace{1cm} $U \gets U * 1.1$\\
|
||||
\hspace{1cm} $S \gets S + u$\\
|
||||
Fin Tant que\\\hline
|
||||
\end{tabularx}
|
||||
\end{center}
|
||||
\item \textit{Pas de correction automatisé}
|
||||
\end{enumerate}
|
||||
\end{enumerate}
|
||||
\end{solution}
|
||||
|
||||
\end{document}
|
||||
|
||||
%%% Local Variables:
|
||||
%%% mode: latex
|
||||
%%% TeX-master: "master"
|
||||
%%% End:
|
262
TST/DS/DS_21_04_07/TST3/10_210407_DS8.tex
Normal file
262
TST/DS/DS_21_04_07/TST3/10_210407_DS8.tex
Normal file
@ -0,0 +1,262 @@
|
||||
\documentclass[a4paper,10pt]{article}
|
||||
\usepackage{myXsim}
|
||||
|
||||
% Title Page
|
||||
\title{DS8 \hfill HENRIST Maxime}
|
||||
\tribe{TST}
|
||||
\date{\hfillÀ render pour le Mercredi 7 avril}
|
||||
|
||||
\xsimsetup{
|
||||
solution/print = false
|
||||
}
|
||||
|
||||
\begin{document}
|
||||
\maketitle
|
||||
|
||||
\begin{exercise}[subtitle={Automatismes}]
|
||||
\textit{Toutes les questions de cette exercice sont indépendantes et peuvent être répondus séparément}
|
||||
\begin{enumerate}
|
||||
\item De janvier à septembre, une quantité a augmenté de $19\,\%$. Faire un schéma pour représenter la situation puis calculer le taux d'évolution moyen mensuel.
|
||||
\item Une quantité augmente de $19\,\%$ par ans. En 2020, elle est de 133\euro. Quelle était sa valeur en 2019? Faire un schéma pour représenter la situation.
|
||||
\item Déterminer l'équation de la droite \\
|
||||
\begin{tikzpicture}[xscale=0.8, yscale=0.5]
|
||||
\tkzInit[xmin=-5,xmax=5,xstep=1,
|
||||
ymin=-5,ymax=5,ystep=1]
|
||||
\tkzGrid
|
||||
\tkzAxeXY
|
||||
\tkzFct[domain=-5:5,color=red,very thick]%
|
||||
{1.0*\x -2};
|
||||
\end{tikzpicture}
|
||||
\item Résoudre l'équation $7 \times 0.82^x = 19$
|
||||
\end{enumerate}
|
||||
\end{exercise}
|
||||
|
||||
\begin{solution}
|
||||
\begin{enumerate}
|
||||
\item On veut partager cette évolution en 8 évolutions.
|
||||
\[
|
||||
\left(1 + \frac{19}{100}\right)^{\frac{1}{8}} = 1.022
|
||||
\]
|
||||
Donc le taux d'évolution moyen est
|
||||
\[
|
||||
t_m = 1.022 - 1 = 0.02200000000000002
|
||||
\]
|
||||
\item Coefficient multiplicateur pour revenir en arrière
|
||||
\[
|
||||
CM = (1 + \frac{19}{100})^{-1} = 0.8403
|
||||
\]
|
||||
On en déduit la quantité en 2019
|
||||
\[
|
||||
133 * 0.8403 = 111.7599
|
||||
\]
|
||||
\item L'équation de la droite est
|
||||
\[
|
||||
y = 1.0 x -2
|
||||
\]
|
||||
\item Il faut penser à faire la division à par $7$ avant d'utiliser le log car sinon, on ne peut pas utiliser la formule $\log(a^n) = n\times \log(a)$.
|
||||
|
||||
\[x = \frac{\log(2.71)}{\log(0.82)}\]
|
||||
\end{enumerate}
|
||||
\end{solution}
|
||||
|
||||
\begin{exercise}[subtitle={Restaurant}]
|
||||
Un \emph{food truck}, ouvert le midi et le soir, propose deux types de formules :
|
||||
|
||||
\setlength\parindent{10mm}
|
||||
\begin{itemize}
|
||||
\item la formule \emph{Burger} ;
|
||||
\item la formule \emph{Wok}.
|
||||
\end{itemize}
|
||||
\setlength\parindent{0mm}
|
||||
|
||||
\medskip
|
||||
|
||||
Le gérant a remarqué que 20\,\% de ses ventes ont lieu le midi. Le quart des ventes du midi correspondent à la formule \emph{Burger}, alors que 56\,\% des ventes du soir correspondent à la formule \emph{Wok}.
|
||||
|
||||
Le gérant se constitue un fichier en notant, pour chaque vente, la formule choisie et le moment de cette vente (midi ou soir).
|
||||
|
||||
On prélève une fiche de façon équiprobable. On définit les quatre évènements suivants:
|
||||
|
||||
\begin{enumerate}
|
||||
\item $M$ : \og la fiche correspond à une vente du midi\fg{} ;
|
||||
\item $S$ : \og la fiche correspond à une vente du soir\fg {};
|
||||
\item $W$ : \og la fiche correspond à une formule \emph{Wok} \fg{} ;
|
||||
\item $B$ : \og la fiche correspond à une formule \emph{Burger} \fg.
|
||||
\end{enumerate}
|
||||
\setlength\parindent{0mm}
|
||||
|
||||
\medskip
|
||||
|
||||
\begin{enumerate}
|
||||
\item Recopier puis compléter l'arbre pondéré
|
||||
|
||||
\begin{center}
|
||||
\begin{tikzpicture}[sloped]
|
||||
\node {.}
|
||||
child {node {$M$}
|
||||
child {node {$W$}
|
||||
edge from parent
|
||||
node[above] {...}
|
||||
}
|
||||
child {node {$B$}
|
||||
edge from parent
|
||||
node[above] {...}
|
||||
}
|
||||
edge from parent
|
||||
node[above] {...}
|
||||
}
|
||||
child[missing] {}
|
||||
child { node {$S$}
|
||||
child {node {$W$}
|
||||
edge from parent
|
||||
node[above] {...}
|
||||
}
|
||||
child {node {$B$}
|
||||
edge from parent
|
||||
node[above] {...}
|
||||
}
|
||||
edge from parent
|
||||
node[above] {...}
|
||||
} ;
|
||||
\end{tikzpicture}
|
||||
\end{center}
|
||||
|
||||
\item Calculer la probabilité de l'évènement $M \cap W$. Interpréter ce résultat dans le contexte de l'exercice.
|
||||
\item Montrer que la probabilité que la fiche choisie corresponde à une formule \emph{Burger} est égale à $0.394$.
|
||||
\item On a prélevé une fiche correspondant à la formule \emph{Burger}. Quelle est la probabilité, arrondie au millième, que la vente ait eu lieu le soir?
|
||||
\end{enumerate}
|
||||
\end{exercise}
|
||||
|
||||
\begin{solution}
|
||||
\begin{enumerate}
|
||||
\item
|
||||
\begin{center}
|
||||
\begin{tikzpicture}[sloped]
|
||||
\node {.}
|
||||
child {node {$M$}
|
||||
child {node {$W$}
|
||||
edge from parent
|
||||
node[above] {$0.75$}
|
||||
}
|
||||
child {node {$B$}
|
||||
edge from parent
|
||||
node[above] {$0.25$}
|
||||
}
|
||||
edge from parent
|
||||
node[above] {$0.2$}
|
||||
}
|
||||
child[missing] {}
|
||||
child { node {$S$}
|
||||
child {node {$W$}
|
||||
edge from parent
|
||||
node[above] {$0.57$}
|
||||
}
|
||||
child {node {$B$}
|
||||
edge from parent
|
||||
node[above] {$0.43$}
|
||||
}
|
||||
edge from parent
|
||||
node[above] {$0.8$}
|
||||
} ;
|
||||
\end{tikzpicture}
|
||||
\end{center}
|
||||
\item On calcule la probabilité que la vente soit un wok et ait eu lieu à midi
|
||||
\[ P(M\cap W) = P(M) \times P_M(W) = 0.2 \times 0.75 = 0.15 \]
|
||||
\item Probabilité que la vente soit un burger.
|
||||
\[
|
||||
P(B) = P(M\cap B) + P(S\cap B) = 0.2 \times 0.75 + 0.8 \times 0.57 = 0.394
|
||||
\]
|
||||
\item On cherche à calculer la quantité $P_B(S)$. Pour cela on utilise la formule de Bayes
|
||||
\[
|
||||
P_B(S) = \frac{P(B\cap S)}{P(B)} = \frac{P_S(B) \times P(S)}{P(B)} = \frac{0.43\times 0.8}{0.394} = 0.8730964467005077 \approx 0.873
|
||||
\]
|
||||
\end{enumerate}
|
||||
\end{solution}
|
||||
|
||||
\begin{exercise}[subtitle={Continent plastique}]
|
||||
\textit{Les quantités évoqués dans cette exercice sont générés au hasard et sont donc complètement farfelus.}
|
||||
\medskip
|
||||
Le \og continent de plastique\fg{} est la plus grande des plaques de déchets plastiques évoluant sur les océans. Elle occupe actuellement dans l'océan Pacifique une surface dont l'aire est évaluée à plus de $1,6$ million de km$^2$, entre Hawaï et la Californie.
|
||||
|
||||
En 2017, des scientifiques ont estimé qu'il y avait $5$ millions de tonnes de déchets plastiques qui était déversé chaque année dans les océans et que cette quantité augmentait de $16\n\%$ par chaque année.
|
||||
|
||||
On modélise l'évolution de la masse de ces déchets plastiques déversée chaque année, si rien n'est fait pour la réduire, par une suite géométrique $\left(u_n\right)$. L'arrondi au centième du terme $u_n$ représente la masse de ces déchets déversée chaque année, exprimée en million de tonnes, pour l'année $(2017 + n)$.
|
||||
|
||||
\medskip
|
||||
|
||||
\begin{enumerate}
|
||||
\item Expliquer pourquoi la suite $u_n$ est géométrique?
|
||||
\item Calculer $u_1$ et $u_2$.
|
||||
\item Exprimer $u_n$ en fonction de $n$.
|
||||
\item Au début de l'année 2017, il y avait $300$ millions de tonnes de déchets plastique. Calculer la quantité totale de déchets plastiques en 2030.
|
||||
\item On souhaite déterminer en quelle année la masse totale de ces déchets plastiques aura pour la première fois augmenté de $50$\,\% par rapport à sa valeur de 2017.
|
||||
\begin{enumerate}
|
||||
\item Recopier et compléter l'algorithme ci-dessous pour que la variable $N$ contienne la réponse au problème posé.
|
||||
|
||||
\begin{center}
|
||||
\begin{tabularx}{0.4\linewidth}{|X|}\hline
|
||||
$N = 2017$\\
|
||||
$U = 5$ \\
|
||||
$S = 300 + U$ \\
|
||||
while $S < 450$: \\
|
||||
\hspace{1cm} $N = \ldots$\\
|
||||
\hspace{1cm} $U = \ldots$\\
|
||||
\hspace{1cm} $S = \ldots$\\
|
||||
\hline
|
||||
\end{tabularx}
|
||||
\end{center}
|
||||
\item Que contiennent les variables $S$, $U$ et $N$ après exécution de cet algorithme ?
|
||||
|
||||
Interpréter les résultats dans le contexte de l'exercice.
|
||||
\end{enumerate}
|
||||
\end{enumerate}
|
||||
\end{exercise}
|
||||
|
||||
\begin{solution}
|
||||
\begin{enumerate}
|
||||
\item Une augmentation de $16\,\%$ revient à multiplier la quantité par $1.16$. La suite est donc bien géométrique. Son premier terme est $u_0 = 5$ et sa raison est $q = 1.16$
|
||||
\item
|
||||
\[
|
||||
u_1 = u_0 * 1.16 = 5.8
|
||||
\]
|
||||
\[
|
||||
u_2 = u_0 * 1.16^2 = 6.728
|
||||
\]
|
||||
\item
|
||||
\[
|
||||
u_n = u_0 \times q^n = 5 \times 1.16^n
|
||||
\]
|
||||
\item On calcule la quantité totale déversée entre 2017 et 2030.
|
||||
\[
|
||||
\sum_{n = 0}^{13} u_n = u_0 \times \frac{1-q^{13}}{1-q} = 5 \times \frac{1 - 1.16^{13}}{1 - 1.16} = 183.93
|
||||
\]
|
||||
On en déduit la quantité totale de déchets en 2030
|
||||
\[
|
||||
300 + 183.93 = 483.93
|
||||
\]
|
||||
\item
|
||||
\begin{enumerate}
|
||||
\item ~
|
||||
\begin{center}
|
||||
\begin{tabularx}{0.4\linewidth}{|X|}\hline
|
||||
$N \gets 2017$\\
|
||||
$U \gets 5$ \\
|
||||
$S \gets 300 + U$ \\
|
||||
Tant que $S < 450$ \\
|
||||
\hspace{1cm} $N \gets N + 1$\\
|
||||
\hspace{1cm} $U \gets U * 1.16$\\
|
||||
\hspace{1cm} $S \gets S + u$\\
|
||||
Fin Tant que\\\hline
|
||||
\end{tabularx}
|
||||
\end{center}
|
||||
\item \textit{Pas de correction automatisé}
|
||||
\end{enumerate}
|
||||
\end{enumerate}
|
||||
\end{solution}
|
||||
|
||||
\end{document}
|
||||
|
||||
%%% Local Variables:
|
||||
%%% mode: latex
|
||||
%%% TeX-master: "master"
|
||||
%%% End:
|
262
TST/DS/DS_21_04_07/TST3/11_210407_DS8.tex
Normal file
262
TST/DS/DS_21_04_07/TST3/11_210407_DS8.tex
Normal file
@ -0,0 +1,262 @@
|
||||
\documentclass[a4paper,10pt]{article}
|
||||
\usepackage{myXsim}
|
||||
|
||||
% Title Page
|
||||
\title{DS8 \hfill INFANTES Antoine}
|
||||
\tribe{TST}
|
||||
\date{\hfillÀ render pour le Mercredi 7 avril}
|
||||
|
||||
\xsimsetup{
|
||||
solution/print = false
|
||||
}
|
||||
|
||||
\begin{document}
|
||||
\maketitle
|
||||
|
||||
\begin{exercise}[subtitle={Automatismes}]
|
||||
\textit{Toutes les questions de cette exercice sont indépendantes et peuvent être répondus séparément}
|
||||
\begin{enumerate}
|
||||
\item De janvier à septembre, une quantité a augmenté de $18\,\%$. Faire un schéma pour représenter la situation puis calculer le taux d'évolution moyen mensuel.
|
||||
\item Une quantité augmente de $18\,\%$ par ans. En 2020, elle est de 130\euro. Quelle était sa valeur en 2019? Faire un schéma pour représenter la situation.
|
||||
\item Déterminer l'équation de la droite \\
|
||||
\begin{tikzpicture}[xscale=0.8, yscale=0.5]
|
||||
\tkzInit[xmin=-5,xmax=5,xstep=1,
|
||||
ymin=-5,ymax=5,ystep=1]
|
||||
\tkzGrid
|
||||
\tkzAxeXY
|
||||
\tkzFct[domain=-5:5,color=red,very thick]%
|
||||
{2.6666666666666665*\x -4};
|
||||
\end{tikzpicture}
|
||||
\item Résoudre l'équation $3 \times 0.31^x = 15$
|
||||
\end{enumerate}
|
||||
\end{exercise}
|
||||
|
||||
\begin{solution}
|
||||
\begin{enumerate}
|
||||
\item On veut partager cette évolution en 8 évolutions.
|
||||
\[
|
||||
\left(1 + \frac{18}{100}\right)^{\frac{1}{8}} = 1.0209
|
||||
\]
|
||||
Donc le taux d'évolution moyen est
|
||||
\[
|
||||
t_m = 1.0209 - 1 = 0.02089999999999992
|
||||
\]
|
||||
\item Coefficient multiplicateur pour revenir en arrière
|
||||
\[
|
||||
CM = (1 + \frac{18}{100})^{-1} = 0.8475
|
||||
\]
|
||||
On en déduit la quantité en 2019
|
||||
\[
|
||||
130 * 0.8475 = 110.175
|
||||
\]
|
||||
\item L'équation de la droite est
|
||||
\[
|
||||
y = 2.6666666666666665 x -4
|
||||
\]
|
||||
\item Il faut penser à faire la division à par $3$ avant d'utiliser le log car sinon, on ne peut pas utiliser la formule $\log(a^n) = n\times \log(a)$.
|
||||
|
||||
\[x = \frac{\log(5.0)}{\log(0.31)}\]
|
||||
\end{enumerate}
|
||||
\end{solution}
|
||||
|
||||
\begin{exercise}[subtitle={Restaurant}]
|
||||
Un \emph{food truck}, ouvert le midi et le soir, propose deux types de formules :
|
||||
|
||||
\setlength\parindent{10mm}
|
||||
\begin{itemize}
|
||||
\item la formule \emph{Burger} ;
|
||||
\item la formule \emph{Wok}.
|
||||
\end{itemize}
|
||||
\setlength\parindent{0mm}
|
||||
|
||||
\medskip
|
||||
|
||||
Le gérant a remarqué que 33\,\% de ses ventes ont lieu le midi. Le quart des ventes du midi correspondent à la formule \emph{Burger}, alors que 13\,\% des ventes du soir correspondent à la formule \emph{Wok}.
|
||||
|
||||
Le gérant se constitue un fichier en notant, pour chaque vente, la formule choisie et le moment de cette vente (midi ou soir).
|
||||
|
||||
On prélève une fiche de façon équiprobable. On définit les quatre évènements suivants:
|
||||
|
||||
\begin{enumerate}
|
||||
\item $M$ : \og la fiche correspond à une vente du midi\fg{} ;
|
||||
\item $S$ : \og la fiche correspond à une vente du soir\fg {};
|
||||
\item $W$ : \og la fiche correspond à une formule \emph{Wok} \fg{} ;
|
||||
\item $B$ : \og la fiche correspond à une formule \emph{Burger} \fg.
|
||||
\end{enumerate}
|
||||
\setlength\parindent{0mm}
|
||||
|
||||
\medskip
|
||||
|
||||
\begin{enumerate}
|
||||
\item Recopier puis compléter l'arbre pondéré
|
||||
|
||||
\begin{center}
|
||||
\begin{tikzpicture}[sloped]
|
||||
\node {.}
|
||||
child {node {$M$}
|
||||
child {node {$W$}
|
||||
edge from parent
|
||||
node[above] {...}
|
||||
}
|
||||
child {node {$B$}
|
||||
edge from parent
|
||||
node[above] {...}
|
||||
}
|
||||
edge from parent
|
||||
node[above] {...}
|
||||
}
|
||||
child[missing] {}
|
||||
child { node {$S$}
|
||||
child {node {$W$}
|
||||
edge from parent
|
||||
node[above] {...}
|
||||
}
|
||||
child {node {$B$}
|
||||
edge from parent
|
||||
node[above] {...}
|
||||
}
|
||||
edge from parent
|
||||
node[above] {...}
|
||||
} ;
|
||||
\end{tikzpicture}
|
||||
\end{center}
|
||||
|
||||
\item Calculer la probabilité de l'évènement $M \cap W$. Interpréter ce résultat dans le contexte de l'exercice.
|
||||
\item Montrer que la probabilité que la fiche choisie corresponde à une formule \emph{Burger} est égale à $0.6654$.
|
||||
\item On a prélevé une fiche correspondant à la formule \emph{Burger}. Quelle est la probabilité, arrondie au millième, que la vente ait eu lieu le soir?
|
||||
\end{enumerate}
|
||||
\end{exercise}
|
||||
|
||||
\begin{solution}
|
||||
\begin{enumerate}
|
||||
\item
|
||||
\begin{center}
|
||||
\begin{tikzpicture}[sloped]
|
||||
\node {.}
|
||||
child {node {$M$}
|
||||
child {node {$W$}
|
||||
edge from parent
|
||||
node[above] {$0.75$}
|
||||
}
|
||||
child {node {$B$}
|
||||
edge from parent
|
||||
node[above] {$0.25$}
|
||||
}
|
||||
edge from parent
|
||||
node[above] {$0.33$}
|
||||
}
|
||||
child[missing] {}
|
||||
child { node {$S$}
|
||||
child {node {$W$}
|
||||
edge from parent
|
||||
node[above] {$0.13$}
|
||||
}
|
||||
child {node {$B$}
|
||||
edge from parent
|
||||
node[above] {$0.87$}
|
||||
}
|
||||
edge from parent
|
||||
node[above] {$0.67$}
|
||||
} ;
|
||||
\end{tikzpicture}
|
||||
\end{center}
|
||||
\item On calcule la probabilité que la vente soit un wok et ait eu lieu à midi
|
||||
\[ P(M\cap W) = P(M) \times P_M(W) = 0.33 \times 0.75 = 0.2475 \]
|
||||
\item Probabilité que la vente soit un burger.
|
||||
\[
|
||||
P(B) = P(M\cap B) + P(S\cap B) = 0.33 \times 0.75 + 0.67 \times 0.13 = 0.6654
|
||||
\]
|
||||
\item On cherche à calculer la quantité $P_B(S)$. Pour cela on utilise la formule de Bayes
|
||||
\[
|
||||
P_B(S) = \frac{P(B\cap S)}{P(B)} = \frac{P_S(B) \times P(S)}{P(B)} = \frac{0.87\times 0.67}{0.6654} = 0.8760144274120831 \approx 0.876
|
||||
\]
|
||||
\end{enumerate}
|
||||
\end{solution}
|
||||
|
||||
\begin{exercise}[subtitle={Continent plastique}]
|
||||
\textit{Les quantités évoqués dans cette exercice sont générés au hasard et sont donc complètement farfelus.}
|
||||
\medskip
|
||||
Le \og continent de plastique\fg{} est la plus grande des plaques de déchets plastiques évoluant sur les océans. Elle occupe actuellement dans l'océan Pacifique une surface dont l'aire est évaluée à plus de $1,6$ million de km$^2$, entre Hawaï et la Californie.
|
||||
|
||||
En 2017, des scientifiques ont estimé qu'il y avait $12$ millions de tonnes de déchets plastiques qui était déversé chaque année dans les océans et que cette quantité augmentait de $26\n\%$ par chaque année.
|
||||
|
||||
On modélise l'évolution de la masse de ces déchets plastiques déversée chaque année, si rien n'est fait pour la réduire, par une suite géométrique $\left(u_n\right)$. L'arrondi au centième du terme $u_n$ représente la masse de ces déchets déversée chaque année, exprimée en million de tonnes, pour l'année $(2017 + n)$.
|
||||
|
||||
\medskip
|
||||
|
||||
\begin{enumerate}
|
||||
\item Expliquer pourquoi la suite $u_n$ est géométrique?
|
||||
\item Calculer $u_1$ et $u_2$.
|
||||
\item Exprimer $u_n$ en fonction de $n$.
|
||||
\item Au début de l'année 2017, il y avait $300$ millions de tonnes de déchets plastique. Calculer la quantité totale de déchets plastiques en 2030.
|
||||
\item On souhaite déterminer en quelle année la masse totale de ces déchets plastiques aura pour la première fois augmenté de $50$\,\% par rapport à sa valeur de 2017.
|
||||
\begin{enumerate}
|
||||
\item Recopier et compléter l'algorithme ci-dessous pour que la variable $N$ contienne la réponse au problème posé.
|
||||
|
||||
\begin{center}
|
||||
\begin{tabularx}{0.4\linewidth}{|X|}\hline
|
||||
$N = 2017$\\
|
||||
$U = 12$ \\
|
||||
$S = 300 + U$ \\
|
||||
while $S < 450$: \\
|
||||
\hspace{1cm} $N = \ldots$\\
|
||||
\hspace{1cm} $U = \ldots$\\
|
||||
\hspace{1cm} $S = \ldots$\\
|
||||
\hline
|
||||
\end{tabularx}
|
||||
\end{center}
|
||||
\item Que contiennent les variables $S$, $U$ et $N$ après exécution de cet algorithme ?
|
||||
|
||||
Interpréter les résultats dans le contexte de l'exercice.
|
||||
\end{enumerate}
|
||||
\end{enumerate}
|
||||
\end{exercise}
|
||||
|
||||
\begin{solution}
|
||||
\begin{enumerate}
|
||||
\item Une augmentation de $26\,\%$ revient à multiplier la quantité par $1.26$. La suite est donc bien géométrique. Son premier terme est $u_0 = 12$ et sa raison est $q = 1.26$
|
||||
\item
|
||||
\[
|
||||
u_1 = u_0 * 1.26 = 15.120000000000001
|
||||
\]
|
||||
\[
|
||||
u_2 = u_0 * 1.26^2 = 19.0512
|
||||
\]
|
||||
\item
|
||||
\[
|
||||
u_n = u_0 \times q^n = 12 \times 1.26^n
|
||||
\]
|
||||
\item On calcule la quantité totale déversée entre 2017 et 2030.
|
||||
\[
|
||||
\sum_{n = 0}^{13} u_n = u_0 \times \frac{1-q^{13}}{1-q} = 12 \times \frac{1 - 1.26^{13}}{1 - 1.26} = 885.01
|
||||
\]
|
||||
On en déduit la quantité totale de déchets en 2030
|
||||
\[
|
||||
300 + 885.01 = 1185.01
|
||||
\]
|
||||
\item
|
||||
\begin{enumerate}
|
||||
\item ~
|
||||
\begin{center}
|
||||
\begin{tabularx}{0.4\linewidth}{|X|}\hline
|
||||
$N \gets 2017$\\
|
||||
$U \gets 12$ \\
|
||||
$S \gets 300 + U$ \\
|
||||
Tant que $S < 450$ \\
|
||||
\hspace{1cm} $N \gets N + 1$\\
|
||||
\hspace{1cm} $U \gets U * 1.26$\\
|
||||
\hspace{1cm} $S \gets S + u$\\
|
||||
Fin Tant que\\\hline
|
||||
\end{tabularx}
|
||||
\end{center}
|
||||
\item \textit{Pas de correction automatisé}
|
||||
\end{enumerate}
|
||||
\end{enumerate}
|
||||
\end{solution}
|
||||
|
||||
\end{document}
|
||||
|
||||
%%% Local Variables:
|
||||
%%% mode: latex
|
||||
%%% TeX-master: "master"
|
||||
%%% End:
|
262
TST/DS/DS_21_04_07/TST3/12_210407_DS8.tex
Normal file
262
TST/DS/DS_21_04_07/TST3/12_210407_DS8.tex
Normal file
@ -0,0 +1,262 @@
|
||||
\documentclass[a4paper,10pt]{article}
|
||||
\usepackage{myXsim}
|
||||
|
||||
% Title Page
|
||||
\title{DS8 \hfill MAGRO Robin}
|
||||
\tribe{TST}
|
||||
\date{\hfillÀ render pour le Mercredi 7 avril}
|
||||
|
||||
\xsimsetup{
|
||||
solution/print = false
|
||||
}
|
||||
|
||||
\begin{document}
|
||||
\maketitle
|
||||
|
||||
\begin{exercise}[subtitle={Automatismes}]
|
||||
\textit{Toutes les questions de cette exercice sont indépendantes et peuvent être répondus séparément}
|
||||
\begin{enumerate}
|
||||
\item De janvier à septembre, une quantité a augmenté de $11\,\%$. Faire un schéma pour représenter la situation puis calculer le taux d'évolution moyen mensuel.
|
||||
\item Une quantité augmente de $11\,\%$ par ans. En 2020, elle est de 143\euro. Quelle était sa valeur en 2019? Faire un schéma pour représenter la situation.
|
||||
\item Déterminer l'équation de la droite \\
|
||||
\begin{tikzpicture}[xscale=0.8, yscale=0.5]
|
||||
\tkzInit[xmin=-5,xmax=5,xstep=1,
|
||||
ymin=-5,ymax=5,ystep=1]
|
||||
\tkzGrid
|
||||
\tkzAxeXY
|
||||
\tkzFct[domain=-5:5,color=red,very thick]%
|
||||
{3.0*\x -3};
|
||||
\end{tikzpicture}
|
||||
\item Résoudre l'équation $5 \times 0.12^x = 40$
|
||||
\end{enumerate}
|
||||
\end{exercise}
|
||||
|
||||
\begin{solution}
|
||||
\begin{enumerate}
|
||||
\item On veut partager cette évolution en 8 évolutions.
|
||||
\[
|
||||
\left(1 + \frac{11}{100}\right)^{\frac{1}{8}} = 1.0131
|
||||
\]
|
||||
Donc le taux d'évolution moyen est
|
||||
\[
|
||||
t_m = 1.0131 - 1 = 0.01309999999999989
|
||||
\]
|
||||
\item Coefficient multiplicateur pour revenir en arrière
|
||||
\[
|
||||
CM = (1 + \frac{11}{100})^{-1} = 0.9009
|
||||
\]
|
||||
On en déduit la quantité en 2019
|
||||
\[
|
||||
143 * 0.9009 = 128.8287
|
||||
\]
|
||||
\item L'équation de la droite est
|
||||
\[
|
||||
y = 3.0 x -3
|
||||
\]
|
||||
\item Il faut penser à faire la division à par $5$ avant d'utiliser le log car sinon, on ne peut pas utiliser la formule $\log(a^n) = n\times \log(a)$.
|
||||
|
||||
\[x = \frac{\log(8.0)}{\log(0.12)}\]
|
||||
\end{enumerate}
|
||||
\end{solution}
|
||||
|
||||
\begin{exercise}[subtitle={Restaurant}]
|
||||
Un \emph{food truck}, ouvert le midi et le soir, propose deux types de formules :
|
||||
|
||||
\setlength\parindent{10mm}
|
||||
\begin{itemize}
|
||||
\item la formule \emph{Burger} ;
|
||||
\item la formule \emph{Wok}.
|
||||
\end{itemize}
|
||||
\setlength\parindent{0mm}
|
||||
|
||||
\medskip
|
||||
|
||||
Le gérant a remarqué que 37\,\% de ses ventes ont lieu le midi. Le quart des ventes du midi correspondent à la formule \emph{Burger}, alors que 32\,\% des ventes du soir correspondent à la formule \emph{Wok}.
|
||||
|
||||
Le gérant se constitue un fichier en notant, pour chaque vente, la formule choisie et le moment de cette vente (midi ou soir).
|
||||
|
||||
On prélève une fiche de façon équiprobable. On définit les quatre évènements suivants:
|
||||
|
||||
\begin{enumerate}
|
||||
\item $M$ : \og la fiche correspond à une vente du midi\fg{} ;
|
||||
\item $S$ : \og la fiche correspond à une vente du soir\fg {};
|
||||
\item $W$ : \og la fiche correspond à une formule \emph{Wok} \fg{} ;
|
||||
\item $B$ : \og la fiche correspond à une formule \emph{Burger} \fg.
|
||||
\end{enumerate}
|
||||
\setlength\parindent{0mm}
|
||||
|
||||
\medskip
|
||||
|
||||
\begin{enumerate}
|
||||
\item Recopier puis compléter l'arbre pondéré
|
||||
|
||||
\begin{center}
|
||||
\begin{tikzpicture}[sloped]
|
||||
\node {.}
|
||||
child {node {$M$}
|
||||
child {node {$W$}
|
||||
edge from parent
|
||||
node[above] {...}
|
||||
}
|
||||
child {node {$B$}
|
||||
edge from parent
|
||||
node[above] {...}
|
||||
}
|
||||
edge from parent
|
||||
node[above] {...}
|
||||
}
|
||||
child[missing] {}
|
||||
child { node {$S$}
|
||||
child {node {$W$}
|
||||
edge from parent
|
||||
node[above] {...}
|
||||
}
|
||||
child {node {$B$}
|
||||
edge from parent
|
||||
node[above] {...}
|
||||
}
|
||||
edge from parent
|
||||
node[above] {...}
|
||||
} ;
|
||||
\end{tikzpicture}
|
||||
\end{center}
|
||||
|
||||
\item Calculer la probabilité de l'évènement $M \cap W$. Interpréter ce résultat dans le contexte de l'exercice.
|
||||
\item Montrer que la probabilité que la fiche choisie corresponde à une formule \emph{Burger} est égale à $0.5209$.
|
||||
\item On a prélevé une fiche correspondant à la formule \emph{Burger}. Quelle est la probabilité, arrondie au millième, que la vente ait eu lieu le soir?
|
||||
\end{enumerate}
|
||||
\end{exercise}
|
||||
|
||||
\begin{solution}
|
||||
\begin{enumerate}
|
||||
\item
|
||||
\begin{center}
|
||||
\begin{tikzpicture}[sloped]
|
||||
\node {.}
|
||||
child {node {$M$}
|
||||
child {node {$W$}
|
||||
edge from parent
|
||||
node[above] {$0.75$}
|
||||
}
|
||||
child {node {$B$}
|
||||
edge from parent
|
||||
node[above] {$0.25$}
|
||||
}
|
||||
edge from parent
|
||||
node[above] {$0.37$}
|
||||
}
|
||||
child[missing] {}
|
||||
child { node {$S$}
|
||||
child {node {$W$}
|
||||
edge from parent
|
||||
node[above] {$0.32$}
|
||||
}
|
||||
child {node {$B$}
|
||||
edge from parent
|
||||
node[above] {$0.68$}
|
||||
}
|
||||
edge from parent
|
||||
node[above] {$0.63$}
|
||||
} ;
|
||||
\end{tikzpicture}
|
||||
\end{center}
|
||||
\item On calcule la probabilité que la vente soit un wok et ait eu lieu à midi
|
||||
\[ P(M\cap W) = P(M) \times P_M(W) = 0.37 \times 0.75 = 0.2775 \]
|
||||
\item Probabilité que la vente soit un burger.
|
||||
\[
|
||||
P(B) = P(M\cap B) + P(S\cap B) = 0.37 \times 0.75 + 0.63 \times 0.32 = 0.5209
|
||||
\]
|
||||
\item On cherche à calculer la quantité $P_B(S)$. Pour cela on utilise la formule de Bayes
|
||||
\[
|
||||
P_B(S) = \frac{P(B\cap S)}{P(B)} = \frac{P_S(B) \times P(S)}{P(B)} = \frac{0.68\times 0.63}{0.5209} = 0.8224227298905741 \approx 0.822
|
||||
\]
|
||||
\end{enumerate}
|
||||
\end{solution}
|
||||
|
||||
\begin{exercise}[subtitle={Continent plastique}]
|
||||
\textit{Les quantités évoqués dans cette exercice sont générés au hasard et sont donc complètement farfelus.}
|
||||
\medskip
|
||||
Le \og continent de plastique\fg{} est la plus grande des plaques de déchets plastiques évoluant sur les océans. Elle occupe actuellement dans l'océan Pacifique une surface dont l'aire est évaluée à plus de $1,6$ million de km$^2$, entre Hawaï et la Californie.
|
||||
|
||||
En 2017, des scientifiques ont estimé qu'il y avait $16$ millions de tonnes de déchets plastiques qui était déversé chaque année dans les océans et que cette quantité augmentait de $17\n\%$ par chaque année.
|
||||
|
||||
On modélise l'évolution de la masse de ces déchets plastiques déversée chaque année, si rien n'est fait pour la réduire, par une suite géométrique $\left(u_n\right)$. L'arrondi au centième du terme $u_n$ représente la masse de ces déchets déversée chaque année, exprimée en million de tonnes, pour l'année $(2017 + n)$.
|
||||
|
||||
\medskip
|
||||
|
||||
\begin{enumerate}
|
||||
\item Expliquer pourquoi la suite $u_n$ est géométrique?
|
||||
\item Calculer $u_1$ et $u_2$.
|
||||
\item Exprimer $u_n$ en fonction de $n$.
|
||||
\item Au début de l'année 2017, il y avait $300$ millions de tonnes de déchets plastique. Calculer la quantité totale de déchets plastiques en 2030.
|
||||
\item On souhaite déterminer en quelle année la masse totale de ces déchets plastiques aura pour la première fois augmenté de $50$\,\% par rapport à sa valeur de 2017.
|
||||
\begin{enumerate}
|
||||
\item Recopier et compléter l'algorithme ci-dessous pour que la variable $N$ contienne la réponse au problème posé.
|
||||
|
||||
\begin{center}
|
||||
\begin{tabularx}{0.4\linewidth}{|X|}\hline
|
||||
$N = 2017$\\
|
||||
$U = 16$ \\
|
||||
$S = 300 + U$ \\
|
||||
while $S < 450$: \\
|
||||
\hspace{1cm} $N = \ldots$\\
|
||||
\hspace{1cm} $U = \ldots$\\
|
||||
\hspace{1cm} $S = \ldots$\\
|
||||
\hline
|
||||
\end{tabularx}
|
||||
\end{center}
|
||||
\item Que contiennent les variables $S$, $U$ et $N$ après exécution de cet algorithme ?
|
||||
|
||||
Interpréter les résultats dans le contexte de l'exercice.
|
||||
\end{enumerate}
|
||||
\end{enumerate}
|
||||
\end{exercise}
|
||||
|
||||
\begin{solution}
|
||||
\begin{enumerate}
|
||||
\item Une augmentation de $17\,\%$ revient à multiplier la quantité par $1.17$. La suite est donc bien géométrique. Son premier terme est $u_0 = 16$ et sa raison est $q = 1.17$
|
||||
\item
|
||||
\[
|
||||
u_1 = u_0 * 1.17 = 18.72
|
||||
\]
|
||||
\[
|
||||
u_2 = u_0 * 1.17^2 = 21.9024
|
||||
\]
|
||||
\item
|
||||
\[
|
||||
u_n = u_0 \times q^n = 16 \times 1.17^n
|
||||
\]
|
||||
\item On calcule la quantité totale déversée entre 2017 et 2030.
|
||||
\[
|
||||
\sum_{n = 0}^{13} u_n = u_0 \times \frac{1-q^{13}}{1-q} = 16 \times \frac{1 - 1.17^{13}}{1 - 1.17} = 630.46
|
||||
\]
|
||||
On en déduit la quantité totale de déchets en 2030
|
||||
\[
|
||||
300 + 630.46 = 930.46
|
||||
\]
|
||||
\item
|
||||
\begin{enumerate}
|
||||
\item ~
|
||||
\begin{center}
|
||||
\begin{tabularx}{0.4\linewidth}{|X|}\hline
|
||||
$N \gets 2017$\\
|
||||
$U \gets 16$ \\
|
||||
$S \gets 300 + U$ \\
|
||||
Tant que $S < 450$ \\
|
||||
\hspace{1cm} $N \gets N + 1$\\
|
||||
\hspace{1cm} $U \gets U * 1.17$\\
|
||||
\hspace{1cm} $S \gets S + u$\\
|
||||
Fin Tant que\\\hline
|
||||
\end{tabularx}
|
||||
\end{center}
|
||||
\item \textit{Pas de correction automatisé}
|
||||
\end{enumerate}
|
||||
\end{enumerate}
|
||||
\end{solution}
|
||||
|
||||
\end{document}
|
||||
|
||||
%%% Local Variables:
|
||||
%%% mode: latex
|
||||
%%% TeX-master: "master"
|
||||
%%% End:
|
262
TST/DS/DS_21_04_07/TST3/13_210407_DS8.tex
Normal file
262
TST/DS/DS_21_04_07/TST3/13_210407_DS8.tex
Normal file
@ -0,0 +1,262 @@
|
||||
\documentclass[a4paper,10pt]{article}
|
||||
\usepackage{myXsim}
|
||||
|
||||
% Title Page
|
||||
\title{DS8 \hfill MORFIN Chloé}
|
||||
\tribe{TST}
|
||||
\date{\hfillÀ render pour le Mercredi 7 avril}
|
||||
|
||||
\xsimsetup{
|
||||
solution/print = false
|
||||
}
|
||||
|
||||
\begin{document}
|
||||
\maketitle
|
||||
|
||||
\begin{exercise}[subtitle={Automatismes}]
|
||||
\textit{Toutes les questions de cette exercice sont indépendantes et peuvent être répondus séparément}
|
||||
\begin{enumerate}
|
||||
\item De janvier à septembre, une quantité a augmenté de $18\,\%$. Faire un schéma pour représenter la situation puis calculer le taux d'évolution moyen mensuel.
|
||||
\item Une quantité augmente de $18\,\%$ par ans. En 2020, elle est de 119\euro. Quelle était sa valeur en 2019? Faire un schéma pour représenter la situation.
|
||||
\item Déterminer l'équation de la droite \\
|
||||
\begin{tikzpicture}[xscale=0.8, yscale=0.5]
|
||||
\tkzInit[xmin=-5,xmax=5,xstep=1,
|
||||
ymin=-5,ymax=5,ystep=1]
|
||||
\tkzGrid
|
||||
\tkzAxeXY
|
||||
\tkzFct[domain=-5:5,color=red,very thick]%
|
||||
{4.0*\x -4};
|
||||
\end{tikzpicture}
|
||||
\item Résoudre l'équation $3 \times 1.0^x = 32$
|
||||
\end{enumerate}
|
||||
\end{exercise}
|
||||
|
||||
\begin{solution}
|
||||
\begin{enumerate}
|
||||
\item On veut partager cette évolution en 8 évolutions.
|
||||
\[
|
||||
\left(1 + \frac{18}{100}\right)^{\frac{1}{8}} = 1.0209
|
||||
\]
|
||||
Donc le taux d'évolution moyen est
|
||||
\[
|
||||
t_m = 1.0209 - 1 = 0.02089999999999992
|
||||
\]
|
||||
\item Coefficient multiplicateur pour revenir en arrière
|
||||
\[
|
||||
CM = (1 + \frac{18}{100})^{-1} = 0.8475
|
||||
\]
|
||||
On en déduit la quantité en 2019
|
||||
\[
|
||||
119 * 0.8475 = 100.8525
|
||||
\]
|
||||
\item L'équation de la droite est
|
||||
\[
|
||||
y = 4.0 x -4
|
||||
\]
|
||||
\item Il faut penser à faire la division à par $3$ avant d'utiliser le log car sinon, on ne peut pas utiliser la formule $\log(a^n) = n\times \log(a)$.
|
||||
|
||||
\[x = \frac{\log(10.67)}{\log(1.0)}\]
|
||||
\end{enumerate}
|
||||
\end{solution}
|
||||
|
||||
\begin{exercise}[subtitle={Restaurant}]
|
||||
Un \emph{food truck}, ouvert le midi et le soir, propose deux types de formules :
|
||||
|
||||
\setlength\parindent{10mm}
|
||||
\begin{itemize}
|
||||
\item la formule \emph{Burger} ;
|
||||
\item la formule \emph{Wok}.
|
||||
\end{itemize}
|
||||
\setlength\parindent{0mm}
|
||||
|
||||
\medskip
|
||||
|
||||
Le gérant a remarqué que 24\,\% de ses ventes ont lieu le midi. Le quart des ventes du midi correspondent à la formule \emph{Burger}, alors que 57\,\% des ventes du soir correspondent à la formule \emph{Wok}.
|
||||
|
||||
Le gérant se constitue un fichier en notant, pour chaque vente, la formule choisie et le moment de cette vente (midi ou soir).
|
||||
|
||||
On prélève une fiche de façon équiprobable. On définit les quatre évènements suivants:
|
||||
|
||||
\begin{enumerate}
|
||||
\item $M$ : \og la fiche correspond à une vente du midi\fg{} ;
|
||||
\item $S$ : \og la fiche correspond à une vente du soir\fg {};
|
||||
\item $W$ : \og la fiche correspond à une formule \emph{Wok} \fg{} ;
|
||||
\item $B$ : \og la fiche correspond à une formule \emph{Burger} \fg.
|
||||
\end{enumerate}
|
||||
\setlength\parindent{0mm}
|
||||
|
||||
\medskip
|
||||
|
||||
\begin{enumerate}
|
||||
\item Recopier puis compléter l'arbre pondéré
|
||||
|
||||
\begin{center}
|
||||
\begin{tikzpicture}[sloped]
|
||||
\node {.}
|
||||
child {node {$M$}
|
||||
child {node {$W$}
|
||||
edge from parent
|
||||
node[above] {...}
|
||||
}
|
||||
child {node {$B$}
|
||||
edge from parent
|
||||
node[above] {...}
|
||||
}
|
||||
edge from parent
|
||||
node[above] {...}
|
||||
}
|
||||
child[missing] {}
|
||||
child { node {$S$}
|
||||
child {node {$W$}
|
||||
edge from parent
|
||||
node[above] {...}
|
||||
}
|
||||
child {node {$B$}
|
||||
edge from parent
|
||||
node[above] {...}
|
||||
}
|
||||
edge from parent
|
||||
node[above] {...}
|
||||
} ;
|
||||
\end{tikzpicture}
|
||||
\end{center}
|
||||
|
||||
\item Calculer la probabilité de l'évènement $M \cap W$. Interpréter ce résultat dans le contexte de l'exercice.
|
||||
\item Montrer que la probabilité que la fiche choisie corresponde à une formule \emph{Burger} est égale à $0.3792$.
|
||||
\item On a prélevé une fiche correspondant à la formule \emph{Burger}. Quelle est la probabilité, arrondie au millième, que la vente ait eu lieu le soir?
|
||||
\end{enumerate}
|
||||
\end{exercise}
|
||||
|
||||
\begin{solution}
|
||||
\begin{enumerate}
|
||||
\item
|
||||
\begin{center}
|
||||
\begin{tikzpicture}[sloped]
|
||||
\node {.}
|
||||
child {node {$M$}
|
||||
child {node {$W$}
|
||||
edge from parent
|
||||
node[above] {$0.75$}
|
||||
}
|
||||
child {node {$B$}
|
||||
edge from parent
|
||||
node[above] {$0.25$}
|
||||
}
|
||||
edge from parent
|
||||
node[above] {$0.24$}
|
||||
}
|
||||
child[missing] {}
|
||||
child { node {$S$}
|
||||
child {node {$W$}
|
||||
edge from parent
|
||||
node[above] {$0.58$}
|
||||
}
|
||||
child {node {$B$}
|
||||
edge from parent
|
||||
node[above] {$0.42$}
|
||||
}
|
||||
edge from parent
|
||||
node[above] {$0.76$}
|
||||
} ;
|
||||
\end{tikzpicture}
|
||||
\end{center}
|
||||
\item On calcule la probabilité que la vente soit un wok et ait eu lieu à midi
|
||||
\[ P(M\cap W) = P(M) \times P_M(W) = 0.24 \times 0.75 = 0.18 \]
|
||||
\item Probabilité que la vente soit un burger.
|
||||
\[
|
||||
P(B) = P(M\cap B) + P(S\cap B) = 0.24 \times 0.75 + 0.76 \times 0.58 = 0.3792
|
||||
\]
|
||||
\item On cherche à calculer la quantité $P_B(S)$. Pour cela on utilise la formule de Bayes
|
||||
\[
|
||||
P_B(S) = \frac{P(B\cap S)}{P(B)} = \frac{P_S(B) \times P(S)}{P(B)} = \frac{0.42\times 0.76}{0.3792} = 0.8417721518987342 \approx 0.842
|
||||
\]
|
||||
\end{enumerate}
|
||||
\end{solution}
|
||||
|
||||
\begin{exercise}[subtitle={Continent plastique}]
|
||||
\textit{Les quantités évoqués dans cette exercice sont générés au hasard et sont donc complètement farfelus.}
|
||||
\medskip
|
||||
Le \og continent de plastique\fg{} est la plus grande des plaques de déchets plastiques évoluant sur les océans. Elle occupe actuellement dans l'océan Pacifique une surface dont l'aire est évaluée à plus de $1,6$ million de km$^2$, entre Hawaï et la Californie.
|
||||
|
||||
En 2017, des scientifiques ont estimé qu'il y avait $17$ millions de tonnes de déchets plastiques qui était déversé chaque année dans les océans et que cette quantité augmentait de $16\n\%$ par chaque année.
|
||||
|
||||
On modélise l'évolution de la masse de ces déchets plastiques déversée chaque année, si rien n'est fait pour la réduire, par une suite géométrique $\left(u_n\right)$. L'arrondi au centième du terme $u_n$ représente la masse de ces déchets déversée chaque année, exprimée en million de tonnes, pour l'année $(2017 + n)$.
|
||||
|
||||
\medskip
|
||||
|
||||
\begin{enumerate}
|
||||
\item Expliquer pourquoi la suite $u_n$ est géométrique?
|
||||
\item Calculer $u_1$ et $u_2$.
|
||||
\item Exprimer $u_n$ en fonction de $n$.
|
||||
\item Au début de l'année 2017, il y avait $300$ millions de tonnes de déchets plastique. Calculer la quantité totale de déchets plastiques en 2030.
|
||||
\item On souhaite déterminer en quelle année la masse totale de ces déchets plastiques aura pour la première fois augmenté de $50$\,\% par rapport à sa valeur de 2017.
|
||||
\begin{enumerate}
|
||||
\item Recopier et compléter l'algorithme ci-dessous pour que la variable $N$ contienne la réponse au problème posé.
|
||||
|
||||
\begin{center}
|
||||
\begin{tabularx}{0.4\linewidth}{|X|}\hline
|
||||
$N = 2017$\\
|
||||
$U = 17$ \\
|
||||
$S = 300 + U$ \\
|
||||
while $S < 450$: \\
|
||||
\hspace{1cm} $N = \ldots$\\
|
||||
\hspace{1cm} $U = \ldots$\\
|
||||
\hspace{1cm} $S = \ldots$\\
|
||||
\hline
|
||||
\end{tabularx}
|
||||
\end{center}
|
||||
\item Que contiennent les variables $S$, $U$ et $N$ après exécution de cet algorithme ?
|
||||
|
||||
Interpréter les résultats dans le contexte de l'exercice.
|
||||
\end{enumerate}
|
||||
\end{enumerate}
|
||||
\end{exercise}
|
||||
|
||||
\begin{solution}
|
||||
\begin{enumerate}
|
||||
\item Une augmentation de $16\,\%$ revient à multiplier la quantité par $1.16$. La suite est donc bien géométrique. Son premier terme est $u_0 = 17$ et sa raison est $q = 1.16$
|
||||
\item
|
||||
\[
|
||||
u_1 = u_0 * 1.16 = 19.72
|
||||
\]
|
||||
\[
|
||||
u_2 = u_0 * 1.16^2 = 22.8752
|
||||
\]
|
||||
\item
|
||||
\[
|
||||
u_n = u_0 \times q^n = 17 \times 1.16^n
|
||||
\]
|
||||
\item On calcule la quantité totale déversée entre 2017 et 2030.
|
||||
\[
|
||||
\sum_{n = 0}^{13} u_n = u_0 \times \frac{1-q^{13}}{1-q} = 17 \times \frac{1 - 1.16^{13}}{1 - 1.16} = 625.37
|
||||
\]
|
||||
On en déduit la quantité totale de déchets en 2030
|
||||
\[
|
||||
300 + 625.37 = 925.37
|
||||
\]
|
||||
\item
|
||||
\begin{enumerate}
|
||||
\item ~
|
||||
\begin{center}
|
||||
\begin{tabularx}{0.4\linewidth}{|X|}\hline
|
||||
$N \gets 2017$\\
|
||||
$U \gets 17$ \\
|
||||
$S \gets 300 + U$ \\
|
||||
Tant que $S < 450$ \\
|
||||
\hspace{1cm} $N \gets N + 1$\\
|
||||
\hspace{1cm} $U \gets U * 1.16$\\
|
||||
\hspace{1cm} $S \gets S + u$\\
|
||||
Fin Tant que\\\hline
|
||||
\end{tabularx}
|
||||
\end{center}
|
||||
\item \textit{Pas de correction automatisé}
|
||||
\end{enumerate}
|
||||
\end{enumerate}
|
||||
\end{solution}
|
||||
|
||||
\end{document}
|
||||
|
||||
%%% Local Variables:
|
||||
%%% mode: latex
|
||||
%%% TeX-master: "master"
|
||||
%%% End:
|
262
TST/DS/DS_21_04_07/TST3/14_210407_DS8.tex
Normal file
262
TST/DS/DS_21_04_07/TST3/14_210407_DS8.tex
Normal file
@ -0,0 +1,262 @@
|
||||
\documentclass[a4paper,10pt]{article}
|
||||
\usepackage{myXsim}
|
||||
|
||||
% Title Page
|
||||
\title{DS8 \hfill PERES RAMALHO Emeric}
|
||||
\tribe{TST}
|
||||
\date{\hfillÀ render pour le Mercredi 7 avril}
|
||||
|
||||
\xsimsetup{
|
||||
solution/print = false
|
||||
}
|
||||
|
||||
\begin{document}
|
||||
\maketitle
|
||||
|
||||
\begin{exercise}[subtitle={Automatismes}]
|
||||
\textit{Toutes les questions de cette exercice sont indépendantes et peuvent être répondus séparément}
|
||||
\begin{enumerate}
|
||||
\item De janvier à septembre, une quantité a augmenté de $23\,\%$. Faire un schéma pour représenter la situation puis calculer le taux d'évolution moyen mensuel.
|
||||
\item Une quantité augmente de $23\,\%$ par ans. En 2020, elle est de 118\euro. Quelle était sa valeur en 2019? Faire un schéma pour représenter la situation.
|
||||
\item Déterminer l'équation de la droite \\
|
||||
\begin{tikzpicture}[xscale=0.8, yscale=0.5]
|
||||
\tkzInit[xmin=-5,xmax=5,xstep=1,
|
||||
ymin=-5,ymax=5,ystep=1]
|
||||
\tkzGrid
|
||||
\tkzAxeXY
|
||||
\tkzFct[domain=-5:5,color=red,very thick]%
|
||||
{1.3333333333333333*\x -2};
|
||||
\end{tikzpicture}
|
||||
\item Résoudre l'équation $5 \times 0.46^x = 2$
|
||||
\end{enumerate}
|
||||
\end{exercise}
|
||||
|
||||
\begin{solution}
|
||||
\begin{enumerate}
|
||||
\item On veut partager cette évolution en 8 évolutions.
|
||||
\[
|
||||
\left(1 + \frac{23}{100}\right)^{\frac{1}{8}} = 1.0262
|
||||
\]
|
||||
Donc le taux d'évolution moyen est
|
||||
\[
|
||||
t_m = 1.0262 - 1 = 0.0262
|
||||
\]
|
||||
\item Coefficient multiplicateur pour revenir en arrière
|
||||
\[
|
||||
CM = (1 + \frac{23}{100})^{-1} = 0.813
|
||||
\]
|
||||
On en déduit la quantité en 2019
|
||||
\[
|
||||
118 * 0.813 = 95.934
|
||||
\]
|
||||
\item L'équation de la droite est
|
||||
\[
|
||||
y = 1.3333333333333333 x -2
|
||||
\]
|
||||
\item Il faut penser à faire la division à par $5$ avant d'utiliser le log car sinon, on ne peut pas utiliser la formule $\log(a^n) = n\times \log(a)$.
|
||||
|
||||
\[x = \frac{\log(0.4)}{\log(0.46)}\]
|
||||
\end{enumerate}
|
||||
\end{solution}
|
||||
|
||||
\begin{exercise}[subtitle={Restaurant}]
|
||||
Un \emph{food truck}, ouvert le midi et le soir, propose deux types de formules :
|
||||
|
||||
\setlength\parindent{10mm}
|
||||
\begin{itemize}
|
||||
\item la formule \emph{Burger} ;
|
||||
\item la formule \emph{Wok}.
|
||||
\end{itemize}
|
||||
\setlength\parindent{0mm}
|
||||
|
||||
\medskip
|
||||
|
||||
Le gérant a remarqué que 11\,\% de ses ventes ont lieu le midi. Le quart des ventes du midi correspondent à la formule \emph{Burger}, alors que 18\,\% des ventes du soir correspondent à la formule \emph{Wok}.
|
||||
|
||||
Le gérant se constitue un fichier en notant, pour chaque vente, la formule choisie et le moment de cette vente (midi ou soir).
|
||||
|
||||
On prélève une fiche de façon équiprobable. On définit les quatre évènements suivants:
|
||||
|
||||
\begin{enumerate}
|
||||
\item $M$ : \og la fiche correspond à une vente du midi\fg{} ;
|
||||
\item $S$ : \og la fiche correspond à une vente du soir\fg {};
|
||||
\item $W$ : \og la fiche correspond à une formule \emph{Wok} \fg{} ;
|
||||
\item $B$ : \og la fiche correspond à une formule \emph{Burger} \fg.
|
||||
\end{enumerate}
|
||||
\setlength\parindent{0mm}
|
||||
|
||||
\medskip
|
||||
|
||||
\begin{enumerate}
|
||||
\item Recopier puis compléter l'arbre pondéré
|
||||
|
||||
\begin{center}
|
||||
\begin{tikzpicture}[sloped]
|
||||
\node {.}
|
||||
child {node {$M$}
|
||||
child {node {$W$}
|
||||
edge from parent
|
||||
node[above] {...}
|
||||
}
|
||||
child {node {$B$}
|
||||
edge from parent
|
||||
node[above] {...}
|
||||
}
|
||||
edge from parent
|
||||
node[above] {...}
|
||||
}
|
||||
child[missing] {}
|
||||
child { node {$S$}
|
||||
child {node {$W$}
|
||||
edge from parent
|
||||
node[above] {...}
|
||||
}
|
||||
child {node {$B$}
|
||||
edge from parent
|
||||
node[above] {...}
|
||||
}
|
||||
edge from parent
|
||||
node[above] {...}
|
||||
} ;
|
||||
\end{tikzpicture}
|
||||
\end{center}
|
||||
|
||||
\item Calculer la probabilité de l'évènement $M \cap W$. Interpréter ce résultat dans le contexte de l'exercice.
|
||||
\item Montrer que la probabilité que la fiche choisie corresponde à une formule \emph{Burger} est égale à $0.7573$.
|
||||
\item On a prélevé une fiche correspondant à la formule \emph{Burger}. Quelle est la probabilité, arrondie au millième, que la vente ait eu lieu le soir?
|
||||
\end{enumerate}
|
||||
\end{exercise}
|
||||
|
||||
\begin{solution}
|
||||
\begin{enumerate}
|
||||
\item
|
||||
\begin{center}
|
||||
\begin{tikzpicture}[sloped]
|
||||
\node {.}
|
||||
child {node {$M$}
|
||||
child {node {$W$}
|
||||
edge from parent
|
||||
node[above] {$0.75$}
|
||||
}
|
||||
child {node {$B$}
|
||||
edge from parent
|
||||
node[above] {$0.25$}
|
||||
}
|
||||
edge from parent
|
||||
node[above] {$0.11$}
|
||||
}
|
||||
child[missing] {}
|
||||
child { node {$S$}
|
||||
child {node {$W$}
|
||||
edge from parent
|
||||
node[above] {$0.18$}
|
||||
}
|
||||
child {node {$B$}
|
||||
edge from parent
|
||||
node[above] {$0.82$}
|
||||
}
|
||||
edge from parent
|
||||
node[above] {$0.89$}
|
||||
} ;
|
||||
\end{tikzpicture}
|
||||
\end{center}
|
||||
\item On calcule la probabilité que la vente soit un wok et ait eu lieu à midi
|
||||
\[ P(M\cap W) = P(M) \times P_M(W) = 0.11 \times 0.75 = 0.0825 \]
|
||||
\item Probabilité que la vente soit un burger.
|
||||
\[
|
||||
P(B) = P(M\cap B) + P(S\cap B) = 0.11 \times 0.75 + 0.89 \times 0.18 = 0.7573
|
||||
\]
|
||||
\item On cherche à calculer la quantité $P_B(S)$. Pour cela on utilise la formule de Bayes
|
||||
\[
|
||||
P_B(S) = \frac{P(B\cap S)}{P(B)} = \frac{P_S(B) \times P(S)}{P(B)} = \frac{0.82\times 0.89}{0.7573} = 0.9636867819886439 \approx 0.964
|
||||
\]
|
||||
\end{enumerate}
|
||||
\end{solution}
|
||||
|
||||
\begin{exercise}[subtitle={Continent plastique}]
|
||||
\textit{Les quantités évoqués dans cette exercice sont générés au hasard et sont donc complètement farfelus.}
|
||||
\medskip
|
||||
Le \og continent de plastique\fg{} est la plus grande des plaques de déchets plastiques évoluant sur les océans. Elle occupe actuellement dans l'océan Pacifique une surface dont l'aire est évaluée à plus de $1,6$ million de km$^2$, entre Hawaï et la Californie.
|
||||
|
||||
En 2017, des scientifiques ont estimé qu'il y avait $19$ millions de tonnes de déchets plastiques qui était déversé chaque année dans les océans et que cette quantité augmentait de $22\n\%$ par chaque année.
|
||||
|
||||
On modélise l'évolution de la masse de ces déchets plastiques déversée chaque année, si rien n'est fait pour la réduire, par une suite géométrique $\left(u_n\right)$. L'arrondi au centième du terme $u_n$ représente la masse de ces déchets déversée chaque année, exprimée en million de tonnes, pour l'année $(2017 + n)$.
|
||||
|
||||
\medskip
|
||||
|
||||
\begin{enumerate}
|
||||
\item Expliquer pourquoi la suite $u_n$ est géométrique?
|
||||
\item Calculer $u_1$ et $u_2$.
|
||||
\item Exprimer $u_n$ en fonction de $n$.
|
||||
\item Au début de l'année 2017, il y avait $300$ millions de tonnes de déchets plastique. Calculer la quantité totale de déchets plastiques en 2030.
|
||||
\item On souhaite déterminer en quelle année la masse totale de ces déchets plastiques aura pour la première fois augmenté de $50$\,\% par rapport à sa valeur de 2017.
|
||||
\begin{enumerate}
|
||||
\item Recopier et compléter l'algorithme ci-dessous pour que la variable $N$ contienne la réponse au problème posé.
|
||||
|
||||
\begin{center}
|
||||
\begin{tabularx}{0.4\linewidth}{|X|}\hline
|
||||
$N = 2017$\\
|
||||
$U = 19$ \\
|
||||
$S = 300 + U$ \\
|
||||
while $S < 450$: \\
|
||||
\hspace{1cm} $N = \ldots$\\
|
||||
\hspace{1cm} $U = \ldots$\\
|
||||
\hspace{1cm} $S = \ldots$\\
|
||||
\hline
|
||||
\end{tabularx}
|
||||
\end{center}
|
||||
\item Que contiennent les variables $S$, $U$ et $N$ après exécution de cet algorithme ?
|
||||
|
||||
Interpréter les résultats dans le contexte de l'exercice.
|
||||
\end{enumerate}
|
||||
\end{enumerate}
|
||||
\end{exercise}
|
||||
|
||||
\begin{solution}
|
||||
\begin{enumerate}
|
||||
\item Une augmentation de $22\,\%$ revient à multiplier la quantité par $1.22$. La suite est donc bien géométrique. Son premier terme est $u_0 = 19$ et sa raison est $q = 1.22$
|
||||
\item
|
||||
\[
|
||||
u_1 = u_0 * 1.22 = 23.18
|
||||
\]
|
||||
\[
|
||||
u_2 = u_0 * 1.22^2 = 28.2796
|
||||
\]
|
||||
\item
|
||||
\[
|
||||
u_n = u_0 \times q^n = 19 \times 1.22^n
|
||||
\]
|
||||
\item On calcule la quantité totale déversée entre 2017 et 2030.
|
||||
\[
|
||||
\sum_{n = 0}^{13} u_n = u_0 \times \frac{1-q^{13}}{1-q} = 19 \times \frac{1 - 1.22^{13}}{1 - 1.22} = 1059.17
|
||||
\]
|
||||
On en déduit la quantité totale de déchets en 2030
|
||||
\[
|
||||
300 + 1059.17 = 1359.17
|
||||
\]
|
||||
\item
|
||||
\begin{enumerate}
|
||||
\item ~
|
||||
\begin{center}
|
||||
\begin{tabularx}{0.4\linewidth}{|X|}\hline
|
||||
$N \gets 2017$\\
|
||||
$U \gets 19$ \\
|
||||
$S \gets 300 + U$ \\
|
||||
Tant que $S < 450$ \\
|
||||
\hspace{1cm} $N \gets N + 1$\\
|
||||
\hspace{1cm} $U \gets U * 1.22$\\
|
||||
\hspace{1cm} $S \gets S + u$\\
|
||||
Fin Tant que\\\hline
|
||||
\end{tabularx}
|
||||
\end{center}
|
||||
\item \textit{Pas de correction automatisé}
|
||||
\end{enumerate}
|
||||
\end{enumerate}
|
||||
\end{solution}
|
||||
|
||||
\end{document}
|
||||
|
||||
%%% Local Variables:
|
||||
%%% mode: latex
|
||||
%%% TeX-master: "master"
|
||||
%%% End:
|
262
TST/DS/DS_21_04_07/TST3/15_210407_DS8.tex
Normal file
262
TST/DS/DS_21_04_07/TST3/15_210407_DS8.tex
Normal file
@ -0,0 +1,262 @@
|
||||
\documentclass[a4paper,10pt]{article}
|
||||
\usepackage{myXsim}
|
||||
|
||||
% Title Page
|
||||
\title{DS8 \hfill RADOUAA Saleh}
|
||||
\tribe{TST}
|
||||
\date{\hfillÀ render pour le Mercredi 7 avril}
|
||||
|
||||
\xsimsetup{
|
||||
solution/print = false
|
||||
}
|
||||
|
||||
\begin{document}
|
||||
\maketitle
|
||||
|
||||
\begin{exercise}[subtitle={Automatismes}]
|
||||
\textit{Toutes les questions de cette exercice sont indépendantes et peuvent être répondus séparément}
|
||||
\begin{enumerate}
|
||||
\item De janvier à septembre, une quantité a augmenté de $27\,\%$. Faire un schéma pour représenter la situation puis calculer le taux d'évolution moyen mensuel.
|
||||
\item Une quantité augmente de $27\,\%$ par ans. En 2020, elle est de 141\euro. Quelle était sa valeur en 2019? Faire un schéma pour représenter la situation.
|
||||
\item Déterminer l'équation de la droite \\
|
||||
\begin{tikzpicture}[xscale=0.8, yscale=0.5]
|
||||
\tkzInit[xmin=-5,xmax=5,xstep=1,
|
||||
ymin=-5,ymax=5,ystep=1]
|
||||
\tkzGrid
|
||||
\tkzAxeXY
|
||||
\tkzFct[domain=-5:5,color=red,very thick]%
|
||||
{2.0*\x -4};
|
||||
\end{tikzpicture}
|
||||
\item Résoudre l'équation $2 \times 0.88^x = 8$
|
||||
\end{enumerate}
|
||||
\end{exercise}
|
||||
|
||||
\begin{solution}
|
||||
\begin{enumerate}
|
||||
\item On veut partager cette évolution en 8 évolutions.
|
||||
\[
|
||||
\left(1 + \frac{27}{100}\right)^{\frac{1}{8}} = 1.0303
|
||||
\]
|
||||
Donc le taux d'évolution moyen est
|
||||
\[
|
||||
t_m = 1.0303 - 1 = 0.030299999999999994
|
||||
\]
|
||||
\item Coefficient multiplicateur pour revenir en arrière
|
||||
\[
|
||||
CM = (1 + \frac{27}{100})^{-1} = 0.7874
|
||||
\]
|
||||
On en déduit la quantité en 2019
|
||||
\[
|
||||
141 * 0.7874 = 111.0234
|
||||
\]
|
||||
\item L'équation de la droite est
|
||||
\[
|
||||
y = 2.0 x -4
|
||||
\]
|
||||
\item Il faut penser à faire la division à par $2$ avant d'utiliser le log car sinon, on ne peut pas utiliser la formule $\log(a^n) = n\times \log(a)$.
|
||||
|
||||
\[x = \frac{\log(4.0)}{\log(0.88)}\]
|
||||
\end{enumerate}
|
||||
\end{solution}
|
||||
|
||||
\begin{exercise}[subtitle={Restaurant}]
|
||||
Un \emph{food truck}, ouvert le midi et le soir, propose deux types de formules :
|
||||
|
||||
\setlength\parindent{10mm}
|
||||
\begin{itemize}
|
||||
\item la formule \emph{Burger} ;
|
||||
\item la formule \emph{Wok}.
|
||||
\end{itemize}
|
||||
\setlength\parindent{0mm}
|
||||
|
||||
\medskip
|
||||
|
||||
Le gérant a remarqué que 1\,\% de ses ventes ont lieu le midi. Le quart des ventes du midi correspondent à la formule \emph{Burger}, alors que 99\,\% des ventes du soir correspondent à la formule \emph{Wok}.
|
||||
|
||||
Le gérant se constitue un fichier en notant, pour chaque vente, la formule choisie et le moment de cette vente (midi ou soir).
|
||||
|
||||
On prélève une fiche de façon équiprobable. On définit les quatre évènements suivants:
|
||||
|
||||
\begin{enumerate}
|
||||
\item $M$ : \og la fiche correspond à une vente du midi\fg{} ;
|
||||
\item $S$ : \og la fiche correspond à une vente du soir\fg {};
|
||||
\item $W$ : \og la fiche correspond à une formule \emph{Wok} \fg{} ;
|
||||
\item $B$ : \og la fiche correspond à une formule \emph{Burger} \fg.
|
||||
\end{enumerate}
|
||||
\setlength\parindent{0mm}
|
||||
|
||||
\medskip
|
||||
|
||||
\begin{enumerate}
|
||||
\item Recopier puis compléter l'arbre pondéré
|
||||
|
||||
\begin{center}
|
||||
\begin{tikzpicture}[sloped]
|
||||
\node {.}
|
||||
child {node {$M$}
|
||||
child {node {$W$}
|
||||
edge from parent
|
||||
node[above] {...}
|
||||
}
|
||||
child {node {$B$}
|
||||
edge from parent
|
||||
node[above] {...}
|
||||
}
|
||||
edge from parent
|
||||
node[above] {...}
|
||||
}
|
||||
child[missing] {}
|
||||
child { node {$S$}
|
||||
child {node {$W$}
|
||||
edge from parent
|
||||
node[above] {...}
|
||||
}
|
||||
child {node {$B$}
|
||||
edge from parent
|
||||
node[above] {...}
|
||||
}
|
||||
edge from parent
|
||||
node[above] {...}
|
||||
} ;
|
||||
\end{tikzpicture}
|
||||
\end{center}
|
||||
|
||||
\item Calculer la probabilité de l'évènement $M \cap W$. Interpréter ce résultat dans le contexte de l'exercice.
|
||||
\item Montrer que la probabilité que la fiche choisie corresponde à une formule \emph{Burger} est égale à $0.0124$.
|
||||
\item On a prélevé une fiche correspondant à la formule \emph{Burger}. Quelle est la probabilité, arrondie au millième, que la vente ait eu lieu le soir?
|
||||
\end{enumerate}
|
||||
\end{exercise}
|
||||
|
||||
\begin{solution}
|
||||
\begin{enumerate}
|
||||
\item
|
||||
\begin{center}
|
||||
\begin{tikzpicture}[sloped]
|
||||
\node {.}
|
||||
child {node {$M$}
|
||||
child {node {$W$}
|
||||
edge from parent
|
||||
node[above] {$0.75$}
|
||||
}
|
||||
child {node {$B$}
|
||||
edge from parent
|
||||
node[above] {$0.25$}
|
||||
}
|
||||
edge from parent
|
||||
node[above] {$0.01$}
|
||||
}
|
||||
child[missing] {}
|
||||
child { node {$S$}
|
||||
child {node {$W$}
|
||||
edge from parent
|
||||
node[above] {$0.99$}
|
||||
}
|
||||
child {node {$B$}
|
||||
edge from parent
|
||||
node[above] {$0.01$}
|
||||
}
|
||||
edge from parent
|
||||
node[above] {$0.99$}
|
||||
} ;
|
||||
\end{tikzpicture}
|
||||
\end{center}
|
||||
\item On calcule la probabilité que la vente soit un wok et ait eu lieu à midi
|
||||
\[ P(M\cap W) = P(M) \times P_M(W) = 0.01 \times 0.75 = 0.0075 \]
|
||||
\item Probabilité que la vente soit un burger.
|
||||
\[
|
||||
P(B) = P(M\cap B) + P(S\cap B) = 0.01 \times 0.75 + 0.99 \times 0.99 = 0.0124
|
||||
\]
|
||||
\item On cherche à calculer la quantité $P_B(S)$. Pour cela on utilise la formule de Bayes
|
||||
\[
|
||||
P_B(S) = \frac{P(B\cap S)}{P(B)} = \frac{P_S(B) \times P(S)}{P(B)} = \frac{0.01\times 0.99}{0.0124} = 0.7983870967741936 \approx 0.798
|
||||
\]
|
||||
\end{enumerate}
|
||||
\end{solution}
|
||||
|
||||
\begin{exercise}[subtitle={Continent plastique}]
|
||||
\textit{Les quantités évoqués dans cette exercice sont générés au hasard et sont donc complètement farfelus.}
|
||||
\medskip
|
||||
Le \og continent de plastique\fg{} est la plus grande des plaques de déchets plastiques évoluant sur les océans. Elle occupe actuellement dans l'océan Pacifique une surface dont l'aire est évaluée à plus de $1,6$ million de km$^2$, entre Hawaï et la Californie.
|
||||
|
||||
En 2017, des scientifiques ont estimé qu'il y avait $5$ millions de tonnes de déchets plastiques qui était déversé chaque année dans les océans et que cette quantité augmentait de $17\n\%$ par chaque année.
|
||||
|
||||
On modélise l'évolution de la masse de ces déchets plastiques déversée chaque année, si rien n'est fait pour la réduire, par une suite géométrique $\left(u_n\right)$. L'arrondi au centième du terme $u_n$ représente la masse de ces déchets déversée chaque année, exprimée en million de tonnes, pour l'année $(2017 + n)$.
|
||||
|
||||
\medskip
|
||||
|
||||
\begin{enumerate}
|
||||
\item Expliquer pourquoi la suite $u_n$ est géométrique?
|
||||
\item Calculer $u_1$ et $u_2$.
|
||||
\item Exprimer $u_n$ en fonction de $n$.
|
||||
\item Au début de l'année 2017, il y avait $300$ millions de tonnes de déchets plastique. Calculer la quantité totale de déchets plastiques en 2030.
|
||||
\item On souhaite déterminer en quelle année la masse totale de ces déchets plastiques aura pour la première fois augmenté de $50$\,\% par rapport à sa valeur de 2017.
|
||||
\begin{enumerate}
|
||||
\item Recopier et compléter l'algorithme ci-dessous pour que la variable $N$ contienne la réponse au problème posé.
|
||||
|
||||
\begin{center}
|
||||
\begin{tabularx}{0.4\linewidth}{|X|}\hline
|
||||
$N = 2017$\\
|
||||
$U = 5$ \\
|
||||
$S = 300 + U$ \\
|
||||
while $S < 450$: \\
|
||||
\hspace{1cm} $N = \ldots$\\
|
||||
\hspace{1cm} $U = \ldots$\\
|
||||
\hspace{1cm} $S = \ldots$\\
|
||||
\hline
|
||||
\end{tabularx}
|
||||
\end{center}
|
||||
\item Que contiennent les variables $S$, $U$ et $N$ après exécution de cet algorithme ?
|
||||
|
||||
Interpréter les résultats dans le contexte de l'exercice.
|
||||
\end{enumerate}
|
||||
\end{enumerate}
|
||||
\end{exercise}
|
||||
|
||||
\begin{solution}
|
||||
\begin{enumerate}
|
||||
\item Une augmentation de $17\,\%$ revient à multiplier la quantité par $1.17$. La suite est donc bien géométrique. Son premier terme est $u_0 = 5$ et sa raison est $q = 1.17$
|
||||
\item
|
||||
\[
|
||||
u_1 = u_0 * 1.17 = 5.85
|
||||
\]
|
||||
\[
|
||||
u_2 = u_0 * 1.17^2 = 6.8445
|
||||
\]
|
||||
\item
|
||||
\[
|
||||
u_n = u_0 \times q^n = 5 \times 1.17^n
|
||||
\]
|
||||
\item On calcule la quantité totale déversée entre 2017 et 2030.
|
||||
\[
|
||||
\sum_{n = 0}^{13} u_n = u_0 \times \frac{1-q^{13}}{1-q} = 5 \times \frac{1 - 1.17^{13}}{1 - 1.17} = 197.02
|
||||
\]
|
||||
On en déduit la quantité totale de déchets en 2030
|
||||
\[
|
||||
300 + 197.02 = 497.02
|
||||
\]
|
||||
\item
|
||||
\begin{enumerate}
|
||||
\item ~
|
||||
\begin{center}
|
||||
\begin{tabularx}{0.4\linewidth}{|X|}\hline
|
||||
$N \gets 2017$\\
|
||||
$U \gets 5$ \\
|
||||
$S \gets 300 + U$ \\
|
||||
Tant que $S < 450$ \\
|
||||
\hspace{1cm} $N \gets N + 1$\\
|
||||
\hspace{1cm} $U \gets U * 1.17$\\
|
||||
\hspace{1cm} $S \gets S + u$\\
|
||||
Fin Tant que\\\hline
|
||||
\end{tabularx}
|
||||
\end{center}
|
||||
\item \textit{Pas de correction automatisé}
|
||||
\end{enumerate}
|
||||
\end{enumerate}
|
||||
\end{solution}
|
||||
|
||||
\end{document}
|
||||
|
||||
%%% Local Variables:
|
||||
%%% mode: latex
|
||||
%%% TeX-master: "master"
|
||||
%%% End:
|
262
TST/DS/DS_21_04_07/TST3/16_210407_DS8.tex
Normal file
262
TST/DS/DS_21_04_07/TST3/16_210407_DS8.tex
Normal file
@ -0,0 +1,262 @@
|
||||
\documentclass[a4paper,10pt]{article}
|
||||
\usepackage{myXsim}
|
||||
|
||||
% Title Page
|
||||
\title{DS8 \hfill TAY Ummuhan}
|
||||
\tribe{TST}
|
||||
\date{\hfillÀ render pour le Mercredi 7 avril}
|
||||
|
||||
\xsimsetup{
|
||||
solution/print = false
|
||||
}
|
||||
|
||||
\begin{document}
|
||||
\maketitle
|
||||
|
||||
\begin{exercise}[subtitle={Automatismes}]
|
||||
\textit{Toutes les questions de cette exercice sont indépendantes et peuvent être répondus séparément}
|
||||
\begin{enumerate}
|
||||
\item De janvier à septembre, une quantité a augmenté de $14\,\%$. Faire un schéma pour représenter la situation puis calculer le taux d'évolution moyen mensuel.
|
||||
\item Une quantité augmente de $14\,\%$ par ans. En 2020, elle est de 116\euro. Quelle était sa valeur en 2019? Faire un schéma pour représenter la situation.
|
||||
\item Déterminer l'équation de la droite \\
|
||||
\begin{tikzpicture}[xscale=0.8, yscale=0.5]
|
||||
\tkzInit[xmin=-5,xmax=5,xstep=1,
|
||||
ymin=-5,ymax=5,ystep=1]
|
||||
\tkzGrid
|
||||
\tkzAxeXY
|
||||
\tkzFct[domain=-5:5,color=red,very thick]%
|
||||
{1.3333333333333333*\x -2};
|
||||
\end{tikzpicture}
|
||||
\item Résoudre l'équation $6 \times 0.14^x = 14$
|
||||
\end{enumerate}
|
||||
\end{exercise}
|
||||
|
||||
\begin{solution}
|
||||
\begin{enumerate}
|
||||
\item On veut partager cette évolution en 8 évolutions.
|
||||
\[
|
||||
\left(1 + \frac{14}{100}\right)^{\frac{1}{8}} = 1.0165
|
||||
\]
|
||||
Donc le taux d'évolution moyen est
|
||||
\[
|
||||
t_m = 1.0165 - 1 = 0.01649999999999996
|
||||
\]
|
||||
\item Coefficient multiplicateur pour revenir en arrière
|
||||
\[
|
||||
CM = (1 + \frac{14}{100})^{-1} = 0.8772
|
||||
\]
|
||||
On en déduit la quantité en 2019
|
||||
\[
|
||||
116 * 0.8772 = 101.7552
|
||||
\]
|
||||
\item L'équation de la droite est
|
||||
\[
|
||||
y = 1.3333333333333333 x -2
|
||||
\]
|
||||
\item Il faut penser à faire la division à par $6$ avant d'utiliser le log car sinon, on ne peut pas utiliser la formule $\log(a^n) = n\times \log(a)$.
|
||||
|
||||
\[x = \frac{\log(2.33)}{\log(0.14)}\]
|
||||
\end{enumerate}
|
||||
\end{solution}
|
||||
|
||||
\begin{exercise}[subtitle={Restaurant}]
|
||||
Un \emph{food truck}, ouvert le midi et le soir, propose deux types de formules :
|
||||
|
||||
\setlength\parindent{10mm}
|
||||
\begin{itemize}
|
||||
\item la formule \emph{Burger} ;
|
||||
\item la formule \emph{Wok}.
|
||||
\end{itemize}
|
||||
\setlength\parindent{0mm}
|
||||
|
||||
\medskip
|
||||
|
||||
Le gérant a remarqué que 28\,\% de ses ventes ont lieu le midi. Le quart des ventes du midi correspondent à la formule \emph{Burger}, alors que 80\,\% des ventes du soir correspondent à la formule \emph{Wok}.
|
||||
|
||||
Le gérant se constitue un fichier en notant, pour chaque vente, la formule choisie et le moment de cette vente (midi ou soir).
|
||||
|
||||
On prélève une fiche de façon équiprobable. On définit les quatre évènements suivants:
|
||||
|
||||
\begin{enumerate}
|
||||
\item $M$ : \og la fiche correspond à une vente du midi\fg{} ;
|
||||
\item $S$ : \og la fiche correspond à une vente du soir\fg {};
|
||||
\item $W$ : \og la fiche correspond à une formule \emph{Wok} \fg{} ;
|
||||
\item $B$ : \og la fiche correspond à une formule \emph{Burger} \fg.
|
||||
\end{enumerate}
|
||||
\setlength\parindent{0mm}
|
||||
|
||||
\medskip
|
||||
|
||||
\begin{enumerate}
|
||||
\item Recopier puis compléter l'arbre pondéré
|
||||
|
||||
\begin{center}
|
||||
\begin{tikzpicture}[sloped]
|
||||
\node {.}
|
||||
child {node {$M$}
|
||||
child {node {$W$}
|
||||
edge from parent
|
||||
node[above] {...}
|
||||
}
|
||||
child {node {$B$}
|
||||
edge from parent
|
||||
node[above] {...}
|
||||
}
|
||||
edge from parent
|
||||
node[above] {...}
|
||||
}
|
||||
child[missing] {}
|
||||
child { node {$S$}
|
||||
child {node {$W$}
|
||||
edge from parent
|
||||
node[above] {...}
|
||||
}
|
||||
child {node {$B$}
|
||||
edge from parent
|
||||
node[above] {...}
|
||||
}
|
||||
edge from parent
|
||||
node[above] {...}
|
||||
} ;
|
||||
\end{tikzpicture}
|
||||
\end{center}
|
||||
|
||||
\item Calculer la probabilité de l'évènement $M \cap W$. Interpréter ce résultat dans le contexte de l'exercice.
|
||||
\item Montrer que la probabilité que la fiche choisie corresponde à une formule \emph{Burger} est égale à $0.214$.
|
||||
\item On a prélevé une fiche correspondant à la formule \emph{Burger}. Quelle est la probabilité, arrondie au millième, que la vente ait eu lieu le soir?
|
||||
\end{enumerate}
|
||||
\end{exercise}
|
||||
|
||||
\begin{solution}
|
||||
\begin{enumerate}
|
||||
\item
|
||||
\begin{center}
|
||||
\begin{tikzpicture}[sloped]
|
||||
\node {.}
|
||||
child {node {$M$}
|
||||
child {node {$W$}
|
||||
edge from parent
|
||||
node[above] {$0.75$}
|
||||
}
|
||||
child {node {$B$}
|
||||
edge from parent
|
||||
node[above] {$0.25$}
|
||||
}
|
||||
edge from parent
|
||||
node[above] {$0.28$}
|
||||
}
|
||||
child[missing] {}
|
||||
child { node {$S$}
|
||||
child {node {$W$}
|
||||
edge from parent
|
||||
node[above] {$0.8$}
|
||||
}
|
||||
child {node {$B$}
|
||||
edge from parent
|
||||
node[above] {$0.2$}
|
||||
}
|
||||
edge from parent
|
||||
node[above] {$0.72$}
|
||||
} ;
|
||||
\end{tikzpicture}
|
||||
\end{center}
|
||||
\item On calcule la probabilité que la vente soit un wok et ait eu lieu à midi
|
||||
\[ P(M\cap W) = P(M) \times P_M(W) = 0.28 \times 0.75 = 0.21 \]
|
||||
\item Probabilité que la vente soit un burger.
|
||||
\[
|
||||
P(B) = P(M\cap B) + P(S\cap B) = 0.28 \times 0.75 + 0.72 \times 0.8 = 0.214
|
||||
\]
|
||||
\item On cherche à calculer la quantité $P_B(S)$. Pour cela on utilise la formule de Bayes
|
||||
\[
|
||||
P_B(S) = \frac{P(B\cap S)}{P(B)} = \frac{P_S(B) \times P(S)}{P(B)} = \frac{0.2\times 0.72}{0.214} = 0.6728971962616822 \approx 0.673
|
||||
\]
|
||||
\end{enumerate}
|
||||
\end{solution}
|
||||
|
||||
\begin{exercise}[subtitle={Continent plastique}]
|
||||
\textit{Les quantités évoqués dans cette exercice sont générés au hasard et sont donc complètement farfelus.}
|
||||
\medskip
|
||||
Le \og continent de plastique\fg{} est la plus grande des plaques de déchets plastiques évoluant sur les océans. Elle occupe actuellement dans l'océan Pacifique une surface dont l'aire est évaluée à plus de $1,6$ million de km$^2$, entre Hawaï et la Californie.
|
||||
|
||||
En 2017, des scientifiques ont estimé qu'il y avait $7$ millions de tonnes de déchets plastiques qui était déversé chaque année dans les océans et que cette quantité augmentait de $26\n\%$ par chaque année.
|
||||
|
||||
On modélise l'évolution de la masse de ces déchets plastiques déversée chaque année, si rien n'est fait pour la réduire, par une suite géométrique $\left(u_n\right)$. L'arrondi au centième du terme $u_n$ représente la masse de ces déchets déversée chaque année, exprimée en million de tonnes, pour l'année $(2017 + n)$.
|
||||
|
||||
\medskip
|
||||
|
||||
\begin{enumerate}
|
||||
\item Expliquer pourquoi la suite $u_n$ est géométrique?
|
||||
\item Calculer $u_1$ et $u_2$.
|
||||
\item Exprimer $u_n$ en fonction de $n$.
|
||||
\item Au début de l'année 2017, il y avait $300$ millions de tonnes de déchets plastique. Calculer la quantité totale de déchets plastiques en 2030.
|
||||
\item On souhaite déterminer en quelle année la masse totale de ces déchets plastiques aura pour la première fois augmenté de $50$\,\% par rapport à sa valeur de 2017.
|
||||
\begin{enumerate}
|
||||
\item Recopier et compléter l'algorithme ci-dessous pour que la variable $N$ contienne la réponse au problème posé.
|
||||
|
||||
\begin{center}
|
||||
\begin{tabularx}{0.4\linewidth}{|X|}\hline
|
||||
$N = 2017$\\
|
||||
$U = 7$ \\
|
||||
$S = 300 + U$ \\
|
||||
while $S < 450$: \\
|
||||
\hspace{1cm} $N = \ldots$\\
|
||||
\hspace{1cm} $U = \ldots$\\
|
||||
\hspace{1cm} $S = \ldots$\\
|
||||
\hline
|
||||
\end{tabularx}
|
||||
\end{center}
|
||||
\item Que contiennent les variables $S$, $U$ et $N$ après exécution de cet algorithme ?
|
||||
|
||||
Interpréter les résultats dans le contexte de l'exercice.
|
||||
\end{enumerate}
|
||||
\end{enumerate}
|
||||
\end{exercise}
|
||||
|
||||
\begin{solution}
|
||||
\begin{enumerate}
|
||||
\item Une augmentation de $26\,\%$ revient à multiplier la quantité par $1.26$. La suite est donc bien géométrique. Son premier terme est $u_0 = 7$ et sa raison est $q = 1.26$
|
||||
\item
|
||||
\[
|
||||
u_1 = u_0 * 1.26 = 8.82
|
||||
\]
|
||||
\[
|
||||
u_2 = u_0 * 1.26^2 = 11.1132
|
||||
\]
|
||||
\item
|
||||
\[
|
||||
u_n = u_0 \times q^n = 7 \times 1.26^n
|
||||
\]
|
||||
\item On calcule la quantité totale déversée entre 2017 et 2030.
|
||||
\[
|
||||
\sum_{n = 0}^{13} u_n = u_0 \times \frac{1-q^{13}}{1-q} = 7 \times \frac{1 - 1.26^{13}}{1 - 1.26} = 516.25
|
||||
\]
|
||||
On en déduit la quantité totale de déchets en 2030
|
||||
\[
|
||||
300 + 516.25 = 816.25
|
||||
\]
|
||||
\item
|
||||
\begin{enumerate}
|
||||
\item ~
|
||||
\begin{center}
|
||||
\begin{tabularx}{0.4\linewidth}{|X|}\hline
|
||||
$N \gets 2017$\\
|
||||
$U \gets 7$ \\
|
||||
$S \gets 300 + U$ \\
|
||||
Tant que $S < 450$ \\
|
||||
\hspace{1cm} $N \gets N + 1$\\
|
||||
\hspace{1cm} $U \gets U * 1.26$\\
|
||||
\hspace{1cm} $S \gets S + u$\\
|
||||
Fin Tant que\\\hline
|
||||
\end{tabularx}
|
||||
\end{center}
|
||||
\item \textit{Pas de correction automatisé}
|
||||
\end{enumerate}
|
||||
\end{enumerate}
|
||||
\end{solution}
|
||||
|
||||
\end{document}
|
||||
|
||||
%%% Local Variables:
|
||||
%%% mode: latex
|
||||
%%% TeX-master: "master"
|
||||
%%% End:
|
262
TST/DS/DS_21_04_07/TST3/17_210407_DS8.tex
Normal file
262
TST/DS/DS_21_04_07/TST3/17_210407_DS8.tex
Normal file
@ -0,0 +1,262 @@
|
||||
\documentclass[a4paper,10pt]{article}
|
||||
\usepackage{myXsim}
|
||||
|
||||
% Title Page
|
||||
\title{DS8 \hfill VIALON-DUPERRON Victorien}
|
||||
\tribe{TST}
|
||||
\date{\hfillÀ render pour le Mercredi 7 avril}
|
||||
|
||||
\xsimsetup{
|
||||
solution/print = false
|
||||
}
|
||||
|
||||
\begin{document}
|
||||
\maketitle
|
||||
|
||||
\begin{exercise}[subtitle={Automatismes}]
|
||||
\textit{Toutes les questions de cette exercice sont indépendantes et peuvent être répondus séparément}
|
||||
\begin{enumerate}
|
||||
\item De janvier à septembre, une quantité a augmenté de $20\,\%$. Faire un schéma pour représenter la situation puis calculer le taux d'évolution moyen mensuel.
|
||||
\item Une quantité augmente de $20\,\%$ par ans. En 2020, elle est de 137\euro. Quelle était sa valeur en 2019? Faire un schéma pour représenter la situation.
|
||||
\item Déterminer l'équation de la droite \\
|
||||
\begin{tikzpicture}[xscale=0.8, yscale=0.5]
|
||||
\tkzInit[xmin=-5,xmax=5,xstep=1,
|
||||
ymin=-5,ymax=5,ystep=1]
|
||||
\tkzGrid
|
||||
\tkzAxeXY
|
||||
\tkzFct[domain=-5:5,color=red,very thick]%
|
||||
{2.6666666666666665*\x -4};
|
||||
\end{tikzpicture}
|
||||
\item Résoudre l'équation $10 \times 0.26^x = 42$
|
||||
\end{enumerate}
|
||||
\end{exercise}
|
||||
|
||||
\begin{solution}
|
||||
\begin{enumerate}
|
||||
\item On veut partager cette évolution en 8 évolutions.
|
||||
\[
|
||||
\left(1 + \frac{20}{100}\right)^{\frac{1}{8}} = 1.0231
|
||||
\]
|
||||
Donc le taux d'évolution moyen est
|
||||
\[
|
||||
t_m = 1.0231 - 1 = 0.0230999999999999
|
||||
\]
|
||||
\item Coefficient multiplicateur pour revenir en arrière
|
||||
\[
|
||||
CM = (1 + \frac{20}{100})^{-1} = 0.8333
|
||||
\]
|
||||
On en déduit la quantité en 2019
|
||||
\[
|
||||
137 * 0.8333 = 114.16210000000001
|
||||
\]
|
||||
\item L'équation de la droite est
|
||||
\[
|
||||
y = 2.6666666666666665 x -4
|
||||
\]
|
||||
\item Il faut penser à faire la division à par $10$ avant d'utiliser le log car sinon, on ne peut pas utiliser la formule $\log(a^n) = n\times \log(a)$.
|
||||
|
||||
\[x = \frac{\log(4.2)}{\log(0.26)}\]
|
||||
\end{enumerate}
|
||||
\end{solution}
|
||||
|
||||
\begin{exercise}[subtitle={Restaurant}]
|
||||
Un \emph{food truck}, ouvert le midi et le soir, propose deux types de formules :
|
||||
|
||||
\setlength\parindent{10mm}
|
||||
\begin{itemize}
|
||||
\item la formule \emph{Burger} ;
|
||||
\item la formule \emph{Wok}.
|
||||
\end{itemize}
|
||||
\setlength\parindent{0mm}
|
||||
|
||||
\medskip
|
||||
|
||||
Le gérant a remarqué que 87\,\% de ses ventes ont lieu le midi. Le quart des ventes du midi correspondent à la formule \emph{Burger}, alors que 47\,\% des ventes du soir correspondent à la formule \emph{Wok}.
|
||||
|
||||
Le gérant se constitue un fichier en notant, pour chaque vente, la formule choisie et le moment de cette vente (midi ou soir).
|
||||
|
||||
On prélève une fiche de façon équiprobable. On définit les quatre évènements suivants:
|
||||
|
||||
\begin{enumerate}
|
||||
\item $M$ : \og la fiche correspond à une vente du midi\fg{} ;
|
||||
\item $S$ : \og la fiche correspond à une vente du soir\fg {};
|
||||
\item $W$ : \og la fiche correspond à une formule \emph{Wok} \fg{} ;
|
||||
\item $B$ : \og la fiche correspond à une formule \emph{Burger} \fg.
|
||||
\end{enumerate}
|
||||
\setlength\parindent{0mm}
|
||||
|
||||
\medskip
|
||||
|
||||
\begin{enumerate}
|
||||
\item Recopier puis compléter l'arbre pondéré
|
||||
|
||||
\begin{center}
|
||||
\begin{tikzpicture}[sloped]
|
||||
\node {.}
|
||||
child {node {$M$}
|
||||
child {node {$W$}
|
||||
edge from parent
|
||||
node[above] {...}
|
||||
}
|
||||
child {node {$B$}
|
||||
edge from parent
|
||||
node[above] {...}
|
||||
}
|
||||
edge from parent
|
||||
node[above] {...}
|
||||
}
|
||||
child[missing] {}
|
||||
child { node {$S$}
|
||||
child {node {$W$}
|
||||
edge from parent
|
||||
node[above] {...}
|
||||
}
|
||||
child {node {$B$}
|
||||
edge from parent
|
||||
node[above] {...}
|
||||
}
|
||||
edge from parent
|
||||
node[above] {...}
|
||||
} ;
|
||||
\end{tikzpicture}
|
||||
\end{center}
|
||||
|
||||
\item Calculer la probabilité de l'évènement $M \cap W$. Interpréter ce résultat dans le contexte de l'exercice.
|
||||
\item Montrer que la probabilité que la fiche choisie corresponde à une formule \emph{Burger} est égale à $0.2864$.
|
||||
\item On a prélevé une fiche correspondant à la formule \emph{Burger}. Quelle est la probabilité, arrondie au millième, que la vente ait eu lieu le soir?
|
||||
\end{enumerate}
|
||||
\end{exercise}
|
||||
|
||||
\begin{solution}
|
||||
\begin{enumerate}
|
||||
\item
|
||||
\begin{center}
|
||||
\begin{tikzpicture}[sloped]
|
||||
\node {.}
|
||||
child {node {$M$}
|
||||
child {node {$W$}
|
||||
edge from parent
|
||||
node[above] {$0.75$}
|
||||
}
|
||||
child {node {$B$}
|
||||
edge from parent
|
||||
node[above] {$0.25$}
|
||||
}
|
||||
edge from parent
|
||||
node[above] {$0.87$}
|
||||
}
|
||||
child[missing] {}
|
||||
child { node {$S$}
|
||||
child {node {$W$}
|
||||
edge from parent
|
||||
node[above] {$0.47$}
|
||||
}
|
||||
child {node {$B$}
|
||||
edge from parent
|
||||
node[above] {$0.53$}
|
||||
}
|
||||
edge from parent
|
||||
node[above] {$0.13$}
|
||||
} ;
|
||||
\end{tikzpicture}
|
||||
\end{center}
|
||||
\item On calcule la probabilité que la vente soit un wok et ait eu lieu à midi
|
||||
\[ P(M\cap W) = P(M) \times P_M(W) = 0.87 \times 0.75 = 0.6525 \]
|
||||
\item Probabilité que la vente soit un burger.
|
||||
\[
|
||||
P(B) = P(M\cap B) + P(S\cap B) = 0.87 \times 0.75 + 0.13 \times 0.47 = 0.2864
|
||||
\]
|
||||
\item On cherche à calculer la quantité $P_B(S)$. Pour cela on utilise la formule de Bayes
|
||||
\[
|
||||
P_B(S) = \frac{P(B\cap S)}{P(B)} = \frac{P_S(B) \times P(S)}{P(B)} = \frac{0.53\times 0.13}{0.2864} = 0.24057262569832405 \approx 0.241
|
||||
\]
|
||||
\end{enumerate}
|
||||
\end{solution}
|
||||
|
||||
\begin{exercise}[subtitle={Continent plastique}]
|
||||
\textit{Les quantités évoqués dans cette exercice sont générés au hasard et sont donc complètement farfelus.}
|
||||
\medskip
|
||||
Le \og continent de plastique\fg{} est la plus grande des plaques de déchets plastiques évoluant sur les océans. Elle occupe actuellement dans l'océan Pacifique une surface dont l'aire est évaluée à plus de $1,6$ million de km$^2$, entre Hawaï et la Californie.
|
||||
|
||||
En 2017, des scientifiques ont estimé qu'il y avait $14$ millions de tonnes de déchets plastiques qui était déversé chaque année dans les océans et que cette quantité augmentait de $17\n\%$ par chaque année.
|
||||
|
||||
On modélise l'évolution de la masse de ces déchets plastiques déversée chaque année, si rien n'est fait pour la réduire, par une suite géométrique $\left(u_n\right)$. L'arrondi au centième du terme $u_n$ représente la masse de ces déchets déversée chaque année, exprimée en million de tonnes, pour l'année $(2017 + n)$.
|
||||
|
||||
\medskip
|
||||
|
||||
\begin{enumerate}
|
||||
\item Expliquer pourquoi la suite $u_n$ est géométrique?
|
||||
\item Calculer $u_1$ et $u_2$.
|
||||
\item Exprimer $u_n$ en fonction de $n$.
|
||||
\item Au début de l'année 2017, il y avait $300$ millions de tonnes de déchets plastique. Calculer la quantité totale de déchets plastiques en 2030.
|
||||
\item On souhaite déterminer en quelle année la masse totale de ces déchets plastiques aura pour la première fois augmenté de $50$\,\% par rapport à sa valeur de 2017.
|
||||
\begin{enumerate}
|
||||
\item Recopier et compléter l'algorithme ci-dessous pour que la variable $N$ contienne la réponse au problème posé.
|
||||
|
||||
\begin{center}
|
||||
\begin{tabularx}{0.4\linewidth}{|X|}\hline
|
||||
$N = 2017$\\
|
||||
$U = 14$ \\
|
||||
$S = 300 + U$ \\
|
||||
while $S < 450$: \\
|
||||
\hspace{1cm} $N = \ldots$\\
|
||||
\hspace{1cm} $U = \ldots$\\
|
||||
\hspace{1cm} $S = \ldots$\\
|
||||
\hline
|
||||
\end{tabularx}
|
||||
\end{center}
|
||||
\item Que contiennent les variables $S$, $U$ et $N$ après exécution de cet algorithme ?
|
||||
|
||||
Interpréter les résultats dans le contexte de l'exercice.
|
||||
\end{enumerate}
|
||||
\end{enumerate}
|
||||
\end{exercise}
|
||||
|
||||
\begin{solution}
|
||||
\begin{enumerate}
|
||||
\item Une augmentation de $17\,\%$ revient à multiplier la quantité par $1.17$. La suite est donc bien géométrique. Son premier terme est $u_0 = 14$ et sa raison est $q = 1.17$
|
||||
\item
|
||||
\[
|
||||
u_1 = u_0 * 1.17 = 16.38
|
||||
\]
|
||||
\[
|
||||
u_2 = u_0 * 1.17^2 = 19.1646
|
||||
\]
|
||||
\item
|
||||
\[
|
||||
u_n = u_0 \times q^n = 14 \times 1.17^n
|
||||
\]
|
||||
\item On calcule la quantité totale déversée entre 2017 et 2030.
|
||||
\[
|
||||
\sum_{n = 0}^{13} u_n = u_0 \times \frac{1-q^{13}}{1-q} = 14 \times \frac{1 - 1.17^{13}}{1 - 1.17} = 551.66
|
||||
\]
|
||||
On en déduit la quantité totale de déchets en 2030
|
||||
\[
|
||||
300 + 551.66 = 851.66
|
||||
\]
|
||||
\item
|
||||
\begin{enumerate}
|
||||
\item ~
|
||||
\begin{center}
|
||||
\begin{tabularx}{0.4\linewidth}{|X|}\hline
|
||||
$N \gets 2017$\\
|
||||
$U \gets 14$ \\
|
||||
$S \gets 300 + U$ \\
|
||||
Tant que $S < 450$ \\
|
||||
\hspace{1cm} $N \gets N + 1$\\
|
||||
\hspace{1cm} $U \gets U * 1.17$\\
|
||||
\hspace{1cm} $S \gets S + u$\\
|
||||
Fin Tant que\\\hline
|
||||
\end{tabularx}
|
||||
\end{center}
|
||||
\item \textit{Pas de correction automatisé}
|
||||
\end{enumerate}
|
||||
\end{enumerate}
|
||||
\end{solution}
|
||||
|
||||
\end{document}
|
||||
|
||||
%%% Local Variables:
|
||||
%%% mode: latex
|
||||
%%% TeX-master: "master"
|
||||
%%% End:
|
262
TST/DS/DS_21_04_07/TST3/18_210407_DS8.tex
Normal file
262
TST/DS/DS_21_04_07/TST3/18_210407_DS8.tex
Normal file
@ -0,0 +1,262 @@
|
||||
\documentclass[a4paper,10pt]{article}
|
||||
\usepackage{myXsim}
|
||||
|
||||
% Title Page
|
||||
\title{DS8 \hfill ZENAGUI Yanis}
|
||||
\tribe{TST}
|
||||
\date{\hfillÀ render pour le Mercredi 7 avril}
|
||||
|
||||
\xsimsetup{
|
||||
solution/print = false
|
||||
}
|
||||
|
||||
\begin{document}
|
||||
\maketitle
|
||||
|
||||
\begin{exercise}[subtitle={Automatismes}]
|
||||
\textit{Toutes les questions de cette exercice sont indépendantes et peuvent être répondus séparément}
|
||||
\begin{enumerate}
|
||||
\item De janvier à septembre, une quantité a augmenté de $29\,\%$. Faire un schéma pour représenter la situation puis calculer le taux d'évolution moyen mensuel.
|
||||
\item Une quantité augmente de $29\,\%$ par ans. En 2020, elle est de 142\euro. Quelle était sa valeur en 2019? Faire un schéma pour représenter la situation.
|
||||
\item Déterminer l'équation de la droite \\
|
||||
\begin{tikzpicture}[xscale=0.8, yscale=0.5]
|
||||
\tkzInit[xmin=-5,xmax=5,xstep=1,
|
||||
ymin=-5,ymax=5,ystep=1]
|
||||
\tkzGrid
|
||||
\tkzAxeXY
|
||||
\tkzFct[domain=-5:5,color=red,very thick]%
|
||||
{2.0*\x -4};
|
||||
\end{tikzpicture}
|
||||
\item Résoudre l'équation $6 \times 0.41^x = 34$
|
||||
\end{enumerate}
|
||||
\end{exercise}
|
||||
|
||||
\begin{solution}
|
||||
\begin{enumerate}
|
||||
\item On veut partager cette évolution en 8 évolutions.
|
||||
\[
|
||||
\left(1 + \frac{29}{100}\right)^{\frac{1}{8}} = 1.0323
|
||||
\]
|
||||
Donc le taux d'évolution moyen est
|
||||
\[
|
||||
t_m = 1.0323 - 1 = 0.032299999999999995
|
||||
\]
|
||||
\item Coefficient multiplicateur pour revenir en arrière
|
||||
\[
|
||||
CM = (1 + \frac{29}{100})^{-1} = 0.7752
|
||||
\]
|
||||
On en déduit la quantité en 2019
|
||||
\[
|
||||
142 * 0.7752 = 110.0784
|
||||
\]
|
||||
\item L'équation de la droite est
|
||||
\[
|
||||
y = 2.0 x -4
|
||||
\]
|
||||
\item Il faut penser à faire la division à par $6$ avant d'utiliser le log car sinon, on ne peut pas utiliser la formule $\log(a^n) = n\times \log(a)$.
|
||||
|
||||
\[x = \frac{\log(5.67)}{\log(0.41)}\]
|
||||
\end{enumerate}
|
||||
\end{solution}
|
||||
|
||||
\begin{exercise}[subtitle={Restaurant}]
|
||||
Un \emph{food truck}, ouvert le midi et le soir, propose deux types de formules :
|
||||
|
||||
\setlength\parindent{10mm}
|
||||
\begin{itemize}
|
||||
\item la formule \emph{Burger} ;
|
||||
\item la formule \emph{Wok}.
|
||||
\end{itemize}
|
||||
\setlength\parindent{0mm}
|
||||
|
||||
\medskip
|
||||
|
||||
Le gérant a remarqué que 30\,\% de ses ventes ont lieu le midi. Le quart des ventes du midi correspondent à la formule \emph{Burger}, alors que 33\,\% des ventes du soir correspondent à la formule \emph{Wok}.
|
||||
|
||||
Le gérant se constitue un fichier en notant, pour chaque vente, la formule choisie et le moment de cette vente (midi ou soir).
|
||||
|
||||
On prélève une fiche de façon équiprobable. On définit les quatre évènements suivants:
|
||||
|
||||
\begin{enumerate}
|
||||
\item $M$ : \og la fiche correspond à une vente du midi\fg{} ;
|
||||
\item $S$ : \og la fiche correspond à une vente du soir\fg {};
|
||||
\item $W$ : \og la fiche correspond à une formule \emph{Wok} \fg{} ;
|
||||
\item $B$ : \og la fiche correspond à une formule \emph{Burger} \fg.
|
||||
\end{enumerate}
|
||||
\setlength\parindent{0mm}
|
||||
|
||||
\medskip
|
||||
|
||||
\begin{enumerate}
|
||||
\item Recopier puis compléter l'arbre pondéré
|
||||
|
||||
\begin{center}
|
||||
\begin{tikzpicture}[sloped]
|
||||
\node {.}
|
||||
child {node {$M$}
|
||||
child {node {$W$}
|
||||
edge from parent
|
||||
node[above] {...}
|
||||
}
|
||||
child {node {$B$}
|
||||
edge from parent
|
||||
node[above] {...}
|
||||
}
|
||||
edge from parent
|
||||
node[above] {...}
|
||||
}
|
||||
child[missing] {}
|
||||
child { node {$S$}
|
||||
child {node {$W$}
|
||||
edge from parent
|
||||
node[above] {...}
|
||||
}
|
||||
child {node {$B$}
|
||||
edge from parent
|
||||
node[above] {...}
|
||||
}
|
||||
edge from parent
|
||||
node[above] {...}
|
||||
} ;
|
||||
\end{tikzpicture}
|
||||
\end{center}
|
||||
|
||||
\item Calculer la probabilité de l'évènement $M \cap W$. Interpréter ce résultat dans le contexte de l'exercice.
|
||||
\item Montrer que la probabilité que la fiche choisie corresponde à une formule \emph{Burger} est égale à $0.544$.
|
||||
\item On a prélevé une fiche correspondant à la formule \emph{Burger}. Quelle est la probabilité, arrondie au millième, que la vente ait eu lieu le soir?
|
||||
\end{enumerate}
|
||||
\end{exercise}
|
||||
|
||||
\begin{solution}
|
||||
\begin{enumerate}
|
||||
\item
|
||||
\begin{center}
|
||||
\begin{tikzpicture}[sloped]
|
||||
\node {.}
|
||||
child {node {$M$}
|
||||
child {node {$W$}
|
||||
edge from parent
|
||||
node[above] {$0.75$}
|
||||
}
|
||||
child {node {$B$}
|
||||
edge from parent
|
||||
node[above] {$0.25$}
|
||||
}
|
||||
edge from parent
|
||||
node[above] {$0.3$}
|
||||
}
|
||||
child[missing] {}
|
||||
child { node {$S$}
|
||||
child {node {$W$}
|
||||
edge from parent
|
||||
node[above] {$0.33$}
|
||||
}
|
||||
child {node {$B$}
|
||||
edge from parent
|
||||
node[above] {$0.67$}
|
||||
}
|
||||
edge from parent
|
||||
node[above] {$0.7$}
|
||||
} ;
|
||||
\end{tikzpicture}
|
||||
\end{center}
|
||||
\item On calcule la probabilité que la vente soit un wok et ait eu lieu à midi
|
||||
\[ P(M\cap W) = P(M) \times P_M(W) = 0.3 \times 0.75 = 0.225 \]
|
||||
\item Probabilité que la vente soit un burger.
|
||||
\[
|
||||
P(B) = P(M\cap B) + P(S\cap B) = 0.3 \times 0.75 + 0.7 \times 0.33 = 0.544
|
||||
\]
|
||||
\item On cherche à calculer la quantité $P_B(S)$. Pour cela on utilise la formule de Bayes
|
||||
\[
|
||||
P_B(S) = \frac{P(B\cap S)}{P(B)} = \frac{P_S(B) \times P(S)}{P(B)} = \frac{0.67\times 0.7}{0.544} = 0.8621323529411764 \approx 0.862
|
||||
\]
|
||||
\end{enumerate}
|
||||
\end{solution}
|
||||
|
||||
\begin{exercise}[subtitle={Continent plastique}]
|
||||
\textit{Les quantités évoqués dans cette exercice sont générés au hasard et sont donc complètement farfelus.}
|
||||
\medskip
|
||||
Le \og continent de plastique\fg{} est la plus grande des plaques de déchets plastiques évoluant sur les océans. Elle occupe actuellement dans l'océan Pacifique une surface dont l'aire est évaluée à plus de $1,6$ million de km$^2$, entre Hawaï et la Californie.
|
||||
|
||||
En 2017, des scientifiques ont estimé qu'il y avait $4$ millions de tonnes de déchets plastiques qui était déversé chaque année dans les océans et que cette quantité augmentait de $18\n\%$ par chaque année.
|
||||
|
||||
On modélise l'évolution de la masse de ces déchets plastiques déversée chaque année, si rien n'est fait pour la réduire, par une suite géométrique $\left(u_n\right)$. L'arrondi au centième du terme $u_n$ représente la masse de ces déchets déversée chaque année, exprimée en million de tonnes, pour l'année $(2017 + n)$.
|
||||
|
||||
\medskip
|
||||
|
||||
\begin{enumerate}
|
||||
\item Expliquer pourquoi la suite $u_n$ est géométrique?
|
||||
\item Calculer $u_1$ et $u_2$.
|
||||
\item Exprimer $u_n$ en fonction de $n$.
|
||||
\item Au début de l'année 2017, il y avait $300$ millions de tonnes de déchets plastique. Calculer la quantité totale de déchets plastiques en 2030.
|
||||
\item On souhaite déterminer en quelle année la masse totale de ces déchets plastiques aura pour la première fois augmenté de $50$\,\% par rapport à sa valeur de 2017.
|
||||
\begin{enumerate}
|
||||
\item Recopier et compléter l'algorithme ci-dessous pour que la variable $N$ contienne la réponse au problème posé.
|
||||
|
||||
\begin{center}
|
||||
\begin{tabularx}{0.4\linewidth}{|X|}\hline
|
||||
$N = 2017$\\
|
||||
$U = 4$ \\
|
||||
$S = 300 + U$ \\
|
||||
while $S < 450$: \\
|
||||
\hspace{1cm} $N = \ldots$\\
|
||||
\hspace{1cm} $U = \ldots$\\
|
||||
\hspace{1cm} $S = \ldots$\\
|
||||
\hline
|
||||
\end{tabularx}
|
||||
\end{center}
|
||||
\item Que contiennent les variables $S$, $U$ et $N$ après exécution de cet algorithme ?
|
||||
|
||||
Interpréter les résultats dans le contexte de l'exercice.
|
||||
\end{enumerate}
|
||||
\end{enumerate}
|
||||
\end{exercise}
|
||||
|
||||
\begin{solution}
|
||||
\begin{enumerate}
|
||||
\item Une augmentation de $18\,\%$ revient à multiplier la quantité par $1.18$. La suite est donc bien géométrique. Son premier terme est $u_0 = 4$ et sa raison est $q = 1.18$
|
||||
\item
|
||||
\[
|
||||
u_1 = u_0 * 1.18 = 4.72
|
||||
\]
|
||||
\[
|
||||
u_2 = u_0 * 1.18^2 = 5.5696
|
||||
\]
|
||||
\item
|
||||
\[
|
||||
u_n = u_0 \times q^n = 4 \times 1.18^n
|
||||
\]
|
||||
\item On calcule la quantité totale déversée entre 2017 et 2030.
|
||||
\[
|
||||
\sum_{n = 0}^{13} u_n = u_0 \times \frac{1-q^{13}}{1-q} = 4 \times \frac{1 - 1.18^{13}}{1 - 1.18} = 168.87
|
||||
\]
|
||||
On en déduit la quantité totale de déchets en 2030
|
||||
\[
|
||||
300 + 168.87 = 468.87
|
||||
\]
|
||||
\item
|
||||
\begin{enumerate}
|
||||
\item ~
|
||||
\begin{center}
|
||||
\begin{tabularx}{0.4\linewidth}{|X|}\hline
|
||||
$N \gets 2017$\\
|
||||
$U \gets 4$ \\
|
||||
$S \gets 300 + U$ \\
|
||||
Tant que $S < 450$ \\
|
||||
\hspace{1cm} $N \gets N + 1$\\
|
||||
\hspace{1cm} $U \gets U * 1.18$\\
|
||||
\hspace{1cm} $S \gets S + u$\\
|
||||
Fin Tant que\\\hline
|
||||
\end{tabularx}
|
||||
\end{center}
|
||||
\item \textit{Pas de correction automatisé}
|
||||
\end{enumerate}
|
||||
\end{enumerate}
|
||||
\end{solution}
|
||||
|
||||
\end{document}
|
||||
|
||||
%%% Local Variables:
|
||||
%%% mode: latex
|
||||
%%% TeX-master: "master"
|
||||
%%% End:
|
262
TST/DS/DS_21_04_07/TST3/19_210407_DS8.tex
Normal file
262
TST/DS/DS_21_04_07/TST3/19_210407_DS8.tex
Normal file
@ -0,0 +1,262 @@
|
||||
\documentclass[a4paper,10pt]{article}
|
||||
\usepackage{myXsim}
|
||||
|
||||
% Title Page
|
||||
\title{DS8 \hfill VIALON-DUPERRON Victorien}
|
||||
\tribe{TST}
|
||||
\date{\hfillÀ render pour le Mercredi 7 avril}
|
||||
|
||||
\xsimsetup{
|
||||
solution/print = false
|
||||
}
|
||||
|
||||
\begin{document}
|
||||
\maketitle
|
||||
|
||||
\begin{exercise}[subtitle={Automatismes}]
|
||||
\textit{Toutes les questions de cette exercice sont indépendantes et peuvent être répondus séparément}
|
||||
\begin{enumerate}
|
||||
\item De janvier à septembre, une quantité a augmenté de $10\,\%$. Faire un schéma pour représenter la situation puis calculer le taux d'évolution moyen mensuel.
|
||||
\item Une quantité augmente de $10\,\%$ par ans. En 2020, elle est de 120\euro. Quelle était sa valeur en 2019? Faire un schéma pour représenter la situation.
|
||||
\item Déterminer l'équation de la droite \\
|
||||
\begin{tikzpicture}[xscale=0.8, yscale=0.5]
|
||||
\tkzInit[xmin=-5,xmax=5,xstep=1,
|
||||
ymin=-5,ymax=5,ystep=1]
|
||||
\tkzGrid
|
||||
\tkzAxeXY
|
||||
\tkzFct[domain=-5:5,color=red,very thick]%
|
||||
{2.0*\x -3};
|
||||
\end{tikzpicture}
|
||||
\item Résoudre l'équation $6 \times 0.69^x = 39$
|
||||
\end{enumerate}
|
||||
\end{exercise}
|
||||
|
||||
\begin{solution}
|
||||
\begin{enumerate}
|
||||
\item On veut partager cette évolution en 8 évolutions.
|
||||
\[
|
||||
\left(1 + \frac{10}{100}\right)^{\frac{1}{8}} = 1.012
|
||||
\]
|
||||
Donc le taux d'évolution moyen est
|
||||
\[
|
||||
t_m = 1.012 - 1 = 0.01200000000000001
|
||||
\]
|
||||
\item Coefficient multiplicateur pour revenir en arrière
|
||||
\[
|
||||
CM = (1 + \frac{10}{100})^{-1} = 0.9091
|
||||
\]
|
||||
On en déduit la quantité en 2019
|
||||
\[
|
||||
120 * 0.9091 = 109.092
|
||||
\]
|
||||
\item L'équation de la droite est
|
||||
\[
|
||||
y = 2.0 x -3
|
||||
\]
|
||||
\item Il faut penser à faire la division à par $6$ avant d'utiliser le log car sinon, on ne peut pas utiliser la formule $\log(a^n) = n\times \log(a)$.
|
||||
|
||||
\[x = \frac{\log(6.5)}{\log(0.69)}\]
|
||||
\end{enumerate}
|
||||
\end{solution}
|
||||
|
||||
\begin{exercise}[subtitle={Restaurant}]
|
||||
Un \emph{food truck}, ouvert le midi et le soir, propose deux types de formules :
|
||||
|
||||
\setlength\parindent{10mm}
|
||||
\begin{itemize}
|
||||
\item la formule \emph{Burger} ;
|
||||
\item la formule \emph{Wok}.
|
||||
\end{itemize}
|
||||
\setlength\parindent{0mm}
|
||||
|
||||
\medskip
|
||||
|
||||
Le gérant a remarqué que 43\,\% de ses ventes ont lieu le midi. Le quart des ventes du midi correspondent à la formule \emph{Burger}, alors que 68\,\% des ventes du soir correspondent à la formule \emph{Wok}.
|
||||
|
||||
Le gérant se constitue un fichier en notant, pour chaque vente, la formule choisie et le moment de cette vente (midi ou soir).
|
||||
|
||||
On prélève une fiche de façon équiprobable. On définit les quatre évènements suivants:
|
||||
|
||||
\begin{enumerate}
|
||||
\item $M$ : \og la fiche correspond à une vente du midi\fg{} ;
|
||||
\item $S$ : \og la fiche correspond à une vente du soir\fg {};
|
||||
\item $W$ : \og la fiche correspond à une formule \emph{Wok} \fg{} ;
|
||||
\item $B$ : \og la fiche correspond à une formule \emph{Burger} \fg.
|
||||
\end{enumerate}
|
||||
\setlength\parindent{0mm}
|
||||
|
||||
\medskip
|
||||
|
||||
\begin{enumerate}
|
||||
\item Recopier puis compléter l'arbre pondéré
|
||||
|
||||
\begin{center}
|
||||
\begin{tikzpicture}[sloped]
|
||||
\node {.}
|
||||
child {node {$M$}
|
||||
child {node {$W$}
|
||||
edge from parent
|
||||
node[above] {...}
|
||||
}
|
||||
child {node {$B$}
|
||||
edge from parent
|
||||
node[above] {...}
|
||||
}
|
||||
edge from parent
|
||||
node[above] {...}
|
||||
}
|
||||
child[missing] {}
|
||||
child { node {$S$}
|
||||
child {node {$W$}
|
||||
edge from parent
|
||||
node[above] {...}
|
||||
}
|
||||
child {node {$B$}
|
||||
edge from parent
|
||||
node[above] {...}
|
||||
}
|
||||
edge from parent
|
||||
node[above] {...}
|
||||
} ;
|
||||
\end{tikzpicture}
|
||||
\end{center}
|
||||
|
||||
\item Calculer la probabilité de l'évènement $M \cap W$. Interpréter ce résultat dans le contexte de l'exercice.
|
||||
\item Montrer que la probabilité que la fiche choisie corresponde à une formule \emph{Burger} est égale à $0.2899$.
|
||||
\item On a prélevé une fiche correspondant à la formule \emph{Burger}. Quelle est la probabilité, arrondie au millième, que la vente ait eu lieu le soir?
|
||||
\end{enumerate}
|
||||
\end{exercise}
|
||||
|
||||
\begin{solution}
|
||||
\begin{enumerate}
|
||||
\item
|
||||
\begin{center}
|
||||
\begin{tikzpicture}[sloped]
|
||||
\node {.}
|
||||
child {node {$M$}
|
||||
child {node {$W$}
|
||||
edge from parent
|
||||
node[above] {$0.75$}
|
||||
}
|
||||
child {node {$B$}
|
||||
edge from parent
|
||||
node[above] {$0.25$}
|
||||
}
|
||||
edge from parent
|
||||
node[above] {$0.43$}
|
||||
}
|
||||
child[missing] {}
|
||||
child { node {$S$}
|
||||
child {node {$W$}
|
||||
edge from parent
|
||||
node[above] {$0.68$}
|
||||
}
|
||||
child {node {$B$}
|
||||
edge from parent
|
||||
node[above] {$0.32$}
|
||||
}
|
||||
edge from parent
|
||||
node[above] {$0.57$}
|
||||
} ;
|
||||
\end{tikzpicture}
|
||||
\end{center}
|
||||
\item On calcule la probabilité que la vente soit un wok et ait eu lieu à midi
|
||||
\[ P(M\cap W) = P(M) \times P_M(W) = 0.43 \times 0.75 = 0.3225 \]
|
||||
\item Probabilité que la vente soit un burger.
|
||||
\[
|
||||
P(B) = P(M\cap B) + P(S\cap B) = 0.43 \times 0.75 + 0.57 \times 0.68 = 0.2899
|
||||
\]
|
||||
\item On cherche à calculer la quantité $P_B(S)$. Pour cela on utilise la formule de Bayes
|
||||
\[
|
||||
P_B(S) = \frac{P(B\cap S)}{P(B)} = \frac{P_S(B) \times P(S)}{P(B)} = \frac{0.32\times 0.57}{0.2899} = 0.6291824767161089 \approx 0.629
|
||||
\]
|
||||
\end{enumerate}
|
||||
\end{solution}
|
||||
|
||||
\begin{exercise}[subtitle={Continent plastique}]
|
||||
\textit{Les quantités évoqués dans cette exercice sont générés au hasard et sont donc complètement farfelus.}
|
||||
\medskip
|
||||
Le \og continent de plastique\fg{} est la plus grande des plaques de déchets plastiques évoluant sur les océans. Elle occupe actuellement dans l'océan Pacifique une surface dont l'aire est évaluée à plus de $1,6$ million de km$^2$, entre Hawaï et la Californie.
|
||||
|
||||
En 2017, des scientifiques ont estimé qu'il y avait $7$ millions de tonnes de déchets plastiques qui était déversé chaque année dans les océans et que cette quantité augmentait de $11\n\%$ par chaque année.
|
||||
|
||||
On modélise l'évolution de la masse de ces déchets plastiques déversée chaque année, si rien n'est fait pour la réduire, par une suite géométrique $\left(u_n\right)$. L'arrondi au centième du terme $u_n$ représente la masse de ces déchets déversée chaque année, exprimée en million de tonnes, pour l'année $(2017 + n)$.
|
||||
|
||||
\medskip
|
||||
|
||||
\begin{enumerate}
|
||||
\item Expliquer pourquoi la suite $u_n$ est géométrique?
|
||||
\item Calculer $u_1$ et $u_2$.
|
||||
\item Exprimer $u_n$ en fonction de $n$.
|
||||
\item Au début de l'année 2017, il y avait $300$ millions de tonnes de déchets plastique. Calculer la quantité totale de déchets plastiques en 2030.
|
||||
\item On souhaite déterminer en quelle année la masse totale de ces déchets plastiques aura pour la première fois augmenté de $50$\,\% par rapport à sa valeur de 2017.
|
||||
\begin{enumerate}
|
||||
\item Recopier et compléter l'algorithme ci-dessous pour que la variable $N$ contienne la réponse au problème posé.
|
||||
|
||||
\begin{center}
|
||||
\begin{tabularx}{0.4\linewidth}{|X|}\hline
|
||||
$N = 2017$\\
|
||||
$U = 7$ \\
|
||||
$S = 300 + U$ \\
|
||||
while $S < 450$: \\
|
||||
\hspace{1cm} $N = \ldots$\\
|
||||
\hspace{1cm} $U = \ldots$\\
|
||||
\hspace{1cm} $S = \ldots$\\
|
||||
\hline
|
||||
\end{tabularx}
|
||||
\end{center}
|
||||
\item Que contiennent les variables $S$, $U$ et $N$ après exécution de cet algorithme ?
|
||||
|
||||
Interpréter les résultats dans le contexte de l'exercice.
|
||||
\end{enumerate}
|
||||
\end{enumerate}
|
||||
\end{exercise}
|
||||
|
||||
\begin{solution}
|
||||
\begin{enumerate}
|
||||
\item Une augmentation de $11\,\%$ revient à multiplier la quantité par $1.11$. La suite est donc bien géométrique. Son premier terme est $u_0 = 7$ et sa raison est $q = 1.11$
|
||||
\item
|
||||
\[
|
||||
u_1 = u_0 * 1.11 = 7.7700000000000005
|
||||
\]
|
||||
\[
|
||||
u_2 = u_0 * 1.11^2 = 8.6247
|
||||
\]
|
||||
\item
|
||||
\[
|
||||
u_n = u_0 \times q^n = 7 \times 1.11^n
|
||||
\]
|
||||
\item On calcule la quantité totale déversée entre 2017 et 2030.
|
||||
\[
|
||||
\sum_{n = 0}^{13} u_n = u_0 \times \frac{1-q^{13}}{1-q} = 7 \times \frac{1 - 1.11^{13}}{1 - 1.11} = 183.48
|
||||
\]
|
||||
On en déduit la quantité totale de déchets en 2030
|
||||
\[
|
||||
300 + 183.48 = 483.48
|
||||
\]
|
||||
\item
|
||||
\begin{enumerate}
|
||||
\item ~
|
||||
\begin{center}
|
||||
\begin{tabularx}{0.4\linewidth}{|X|}\hline
|
||||
$N \gets 2017$\\
|
||||
$U \gets 7$ \\
|
||||
$S \gets 300 + U$ \\
|
||||
Tant que $S < 450$ \\
|
||||
\hspace{1cm} $N \gets N + 1$\\
|
||||
\hspace{1cm} $U \gets U * 1.11$\\
|
||||
\hspace{1cm} $S \gets S + u$\\
|
||||
Fin Tant que\\\hline
|
||||
\end{tabularx}
|
||||
\end{center}
|
||||
\item \textit{Pas de correction automatisé}
|
||||
\end{enumerate}
|
||||
\end{enumerate}
|
||||
\end{solution}
|
||||
|
||||
\end{document}
|
||||
|
||||
%%% Local Variables:
|
||||
%%% mode: latex
|
||||
%%% TeX-master: "master"
|
||||
%%% End:
|
262
TST/DS/DS_21_04_07/TST3/20_210407_DS8.tex
Normal file
262
TST/DS/DS_21_04_07/TST3/20_210407_DS8.tex
Normal file
@ -0,0 +1,262 @@
|
||||
\documentclass[a4paper,10pt]{article}
|
||||
\usepackage{myXsim}
|
||||
|
||||
% Title Page
|
||||
\title{DS8 \hfill ZENAGUI Yanis}
|
||||
\tribe{TST}
|
||||
\date{\hfillÀ render pour le Mercredi 7 avril}
|
||||
|
||||
\xsimsetup{
|
||||
solution/print = false
|
||||
}
|
||||
|
||||
\begin{document}
|
||||
\maketitle
|
||||
|
||||
\begin{exercise}[subtitle={Automatismes}]
|
||||
\textit{Toutes les questions de cette exercice sont indépendantes et peuvent être répondus séparément}
|
||||
\begin{enumerate}
|
||||
\item De janvier à septembre, une quantité a augmenté de $23\,\%$. Faire un schéma pour représenter la situation puis calculer le taux d'évolution moyen mensuel.
|
||||
\item Une quantité augmente de $23\,\%$ par ans. En 2020, elle est de 135\euro. Quelle était sa valeur en 2019? Faire un schéma pour représenter la situation.
|
||||
\item Déterminer l'équation de la droite \\
|
||||
\begin{tikzpicture}[xscale=0.8, yscale=0.5]
|
||||
\tkzInit[xmin=-5,xmax=5,xstep=1,
|
||||
ymin=-5,ymax=5,ystep=1]
|
||||
\tkzGrid
|
||||
\tkzAxeXY
|
||||
\tkzFct[domain=-5:5,color=red,very thick]%
|
||||
{0.5*\x -1};
|
||||
\end{tikzpicture}
|
||||
\item Résoudre l'équation $3 \times 0.55^x = 39$
|
||||
\end{enumerate}
|
||||
\end{exercise}
|
||||
|
||||
\begin{solution}
|
||||
\begin{enumerate}
|
||||
\item On veut partager cette évolution en 8 évolutions.
|
||||
\[
|
||||
\left(1 + \frac{23}{100}\right)^{\frac{1}{8}} = 1.0262
|
||||
\]
|
||||
Donc le taux d'évolution moyen est
|
||||
\[
|
||||
t_m = 1.0262 - 1 = 0.0262
|
||||
\]
|
||||
\item Coefficient multiplicateur pour revenir en arrière
|
||||
\[
|
||||
CM = (1 + \frac{23}{100})^{-1} = 0.813
|
||||
\]
|
||||
On en déduit la quantité en 2019
|
||||
\[
|
||||
135 * 0.813 = 109.755
|
||||
\]
|
||||
\item L'équation de la droite est
|
||||
\[
|
||||
y = 0.5 x -1
|
||||
\]
|
||||
\item Il faut penser à faire la division à par $3$ avant d'utiliser le log car sinon, on ne peut pas utiliser la formule $\log(a^n) = n\times \log(a)$.
|
||||
|
||||
\[x = \frac{\log(13.0)}{\log(0.55)}\]
|
||||
\end{enumerate}
|
||||
\end{solution}
|
||||
|
||||
\begin{exercise}[subtitle={Restaurant}]
|
||||
Un \emph{food truck}, ouvert le midi et le soir, propose deux types de formules :
|
||||
|
||||
\setlength\parindent{10mm}
|
||||
\begin{itemize}
|
||||
\item la formule \emph{Burger} ;
|
||||
\item la formule \emph{Wok}.
|
||||
\end{itemize}
|
||||
\setlength\parindent{0mm}
|
||||
|
||||
\medskip
|
||||
|
||||
Le gérant a remarqué que 9\,\% de ses ventes ont lieu le midi. Le quart des ventes du midi correspondent à la formule \emph{Burger}, alors que 11\,\% des ventes du soir correspondent à la formule \emph{Wok}.
|
||||
|
||||
Le gérant se constitue un fichier en notant, pour chaque vente, la formule choisie et le moment de cette vente (midi ou soir).
|
||||
|
||||
On prélève une fiche de façon équiprobable. On définit les quatre évènements suivants:
|
||||
|
||||
\begin{enumerate}
|
||||
\item $M$ : \og la fiche correspond à une vente du midi\fg{} ;
|
||||
\item $S$ : \og la fiche correspond à une vente du soir\fg {};
|
||||
\item $W$ : \og la fiche correspond à une formule \emph{Wok} \fg{} ;
|
||||
\item $B$ : \og la fiche correspond à une formule \emph{Burger} \fg.
|
||||
\end{enumerate}
|
||||
\setlength\parindent{0mm}
|
||||
|
||||
\medskip
|
||||
|
||||
\begin{enumerate}
|
||||
\item Recopier puis compléter l'arbre pondéré
|
||||
|
||||
\begin{center}
|
||||
\begin{tikzpicture}[sloped]
|
||||
\node {.}
|
||||
child {node {$M$}
|
||||
child {node {$W$}
|
||||
edge from parent
|
||||
node[above] {...}
|
||||
}
|
||||
child {node {$B$}
|
||||
edge from parent
|
||||
node[above] {...}
|
||||
}
|
||||
edge from parent
|
||||
node[above] {...}
|
||||
}
|
||||
child[missing] {}
|
||||
child { node {$S$}
|
||||
child {node {$W$}
|
||||
edge from parent
|
||||
node[above] {...}
|
||||
}
|
||||
child {node {$B$}
|
||||
edge from parent
|
||||
node[above] {...}
|
||||
}
|
||||
edge from parent
|
||||
node[above] {...}
|
||||
} ;
|
||||
\end{tikzpicture}
|
||||
\end{center}
|
||||
|
||||
\item Calculer la probabilité de l'évènement $M \cap W$. Interpréter ce résultat dans le contexte de l'exercice.
|
||||
\item Montrer que la probabilité que la fiche choisie corresponde à une formule \emph{Burger} est égale à $0.8324$.
|
||||
\item On a prélevé une fiche correspondant à la formule \emph{Burger}. Quelle est la probabilité, arrondie au millième, que la vente ait eu lieu le soir?
|
||||
\end{enumerate}
|
||||
\end{exercise}
|
||||
|
||||
\begin{solution}
|
||||
\begin{enumerate}
|
||||
\item
|
||||
\begin{center}
|
||||
\begin{tikzpicture}[sloped]
|
||||
\node {.}
|
||||
child {node {$M$}
|
||||
child {node {$W$}
|
||||
edge from parent
|
||||
node[above] {$0.75$}
|
||||
}
|
||||
child {node {$B$}
|
||||
edge from parent
|
||||
node[above] {$0.25$}
|
||||
}
|
||||
edge from parent
|
||||
node[above] {$0.09$}
|
||||
}
|
||||
child[missing] {}
|
||||
child { node {$S$}
|
||||
child {node {$W$}
|
||||
edge from parent
|
||||
node[above] {$0.11$}
|
||||
}
|
||||
child {node {$B$}
|
||||
edge from parent
|
||||
node[above] {$0.89$}
|
||||
}
|
||||
edge from parent
|
||||
node[above] {$0.91$}
|
||||
} ;
|
||||
\end{tikzpicture}
|
||||
\end{center}
|
||||
\item On calcule la probabilité que la vente soit un wok et ait eu lieu à midi
|
||||
\[ P(M\cap W) = P(M) \times P_M(W) = 0.09 \times 0.75 = 0.0675 \]
|
||||
\item Probabilité que la vente soit un burger.
|
||||
\[
|
||||
P(B) = P(M\cap B) + P(S\cap B) = 0.09 \times 0.75 + 0.91 \times 0.11 = 0.8324
|
||||
\]
|
||||
\item On cherche à calculer la quantité $P_B(S)$. Pour cela on utilise la formule de Bayes
|
||||
\[
|
||||
P_B(S) = \frac{P(B\cap S)}{P(B)} = \frac{P_S(B) \times P(S)}{P(B)} = \frac{0.89\times 0.91}{0.8324} = 0.9729697260932244 \approx 0.973
|
||||
\]
|
||||
\end{enumerate}
|
||||
\end{solution}
|
||||
|
||||
\begin{exercise}[subtitle={Continent plastique}]
|
||||
\textit{Les quantités évoqués dans cette exercice sont générés au hasard et sont donc complètement farfelus.}
|
||||
\medskip
|
||||
Le \og continent de plastique\fg{} est la plus grande des plaques de déchets plastiques évoluant sur les océans. Elle occupe actuellement dans l'océan Pacifique une surface dont l'aire est évaluée à plus de $1,6$ million de km$^2$, entre Hawaï et la Californie.
|
||||
|
||||
En 2017, des scientifiques ont estimé qu'il y avait $4$ millions de tonnes de déchets plastiques qui était déversé chaque année dans les océans et que cette quantité augmentait de $19\n\%$ par chaque année.
|
||||
|
||||
On modélise l'évolution de la masse de ces déchets plastiques déversée chaque année, si rien n'est fait pour la réduire, par une suite géométrique $\left(u_n\right)$. L'arrondi au centième du terme $u_n$ représente la masse de ces déchets déversée chaque année, exprimée en million de tonnes, pour l'année $(2017 + n)$.
|
||||
|
||||
\medskip
|
||||
|
||||
\begin{enumerate}
|
||||
\item Expliquer pourquoi la suite $u_n$ est géométrique?
|
||||
\item Calculer $u_1$ et $u_2$.
|
||||
\item Exprimer $u_n$ en fonction de $n$.
|
||||
\item Au début de l'année 2017, il y avait $300$ millions de tonnes de déchets plastique. Calculer la quantité totale de déchets plastiques en 2030.
|
||||
\item On souhaite déterminer en quelle année la masse totale de ces déchets plastiques aura pour la première fois augmenté de $50$\,\% par rapport à sa valeur de 2017.
|
||||
\begin{enumerate}
|
||||
\item Recopier et compléter l'algorithme ci-dessous pour que la variable $N$ contienne la réponse au problème posé.
|
||||
|
||||
\begin{center}
|
||||
\begin{tabularx}{0.4\linewidth}{|X|}\hline
|
||||
$N = 2017$\\
|
||||
$U = 4$ \\
|
||||
$S = 300 + U$ \\
|
||||
while $S < 450$: \\
|
||||
\hspace{1cm} $N = \ldots$\\
|
||||
\hspace{1cm} $U = \ldots$\\
|
||||
\hspace{1cm} $S = \ldots$\\
|
||||
\hline
|
||||
\end{tabularx}
|
||||
\end{center}
|
||||
\item Que contiennent les variables $S$, $U$ et $N$ après exécution de cet algorithme ?
|
||||
|
||||
Interpréter les résultats dans le contexte de l'exercice.
|
||||
\end{enumerate}
|
||||
\end{enumerate}
|
||||
\end{exercise}
|
||||
|
||||
\begin{solution}
|
||||
\begin{enumerate}
|
||||
\item Une augmentation de $19\,\%$ revient à multiplier la quantité par $1.19$. La suite est donc bien géométrique. Son premier terme est $u_0 = 4$ et sa raison est $q = 1.19$
|
||||
\item
|
||||
\[
|
||||
u_1 = u_0 * 1.19 = 4.76
|
||||
\]
|
||||
\[
|
||||
u_2 = u_0 * 1.19^2 = 5.6644
|
||||
\]
|
||||
\item
|
||||
\[
|
||||
u_n = u_0 \times q^n = 4 \times 1.19^n
|
||||
\]
|
||||
\item On calcule la quantité totale déversée entre 2017 et 2030.
|
||||
\[
|
||||
\sum_{n = 0}^{13} u_n = u_0 \times \frac{1-q^{13}}{1-q} = 4 \times \frac{1 - 1.19^{13}}{1 - 1.19} = 180.98
|
||||
\]
|
||||
On en déduit la quantité totale de déchets en 2030
|
||||
\[
|
||||
300 + 180.98 = 480.98
|
||||
\]
|
||||
\item
|
||||
\begin{enumerate}
|
||||
\item ~
|
||||
\begin{center}
|
||||
\begin{tabularx}{0.4\linewidth}{|X|}\hline
|
||||
$N \gets 2017$\\
|
||||
$U \gets 4$ \\
|
||||
$S \gets 300 + U$ \\
|
||||
Tant que $S < 450$ \\
|
||||
\hspace{1cm} $N \gets N + 1$\\
|
||||
\hspace{1cm} $U \gets U * 1.19$\\
|
||||
\hspace{1cm} $S \gets S + u$\\
|
||||
Fin Tant que\\\hline
|
||||
\end{tabularx}
|
||||
\end{center}
|
||||
\item \textit{Pas de correction automatisé}
|
||||
\end{enumerate}
|
||||
\end{enumerate}
|
||||
\end{solution}
|
||||
|
||||
\end{document}
|
||||
|
||||
%%% Local Variables:
|
||||
%%% mode: latex
|
||||
%%% TeX-master: "master"
|
||||
%%% End:
|
BIN
TST/DS/DS_21_04_07/TST3/all_210407_DS8.pdf
Normal file
BIN
TST/DS/DS_21_04_07/TST3/all_210407_DS8.pdf
Normal file
Binary file not shown.
262
TST/DS/DS_21_04_07/TST3/corr_01_210407_DS8.tex
Normal file
262
TST/DS/DS_21_04_07/TST3/corr_01_210407_DS8.tex
Normal file
@ -0,0 +1,262 @@
|
||||
\documentclass[a4paper,10pt]{article}
|
||||
\usepackage{myXsim}
|
||||
|
||||
% Title Page
|
||||
\title{DS8 \hfill BELARBI Samira}
|
||||
\tribe{TST}
|
||||
\date{\hfillÀ render pour le Mercredi 7 avril}
|
||||
|
||||
\xsimsetup{
|
||||
solution/print = true
|
||||
}
|
||||
|
||||
\begin{document}
|
||||
\maketitle
|
||||
|
||||
\begin{exercise}[subtitle={Automatismes}]
|
||||
\textit{Toutes les questions de cette exercice sont indépendantes et peuvent être répondus séparément}
|
||||
\begin{enumerate}
|
||||
\item De janvier à septembre, une quantité a augmenté de $21\,\%$. Faire un schéma pour représenter la situation puis calculer le taux d'évolution moyen mensuel.
|
||||
\item Une quantité augmente de $21\,\%$ par ans. En 2020, elle est de 143\euro. Quelle était sa valeur en 2019? Faire un schéma pour représenter la situation.
|
||||
\item Déterminer l'équation de la droite \\
|
||||
\begin{tikzpicture}[xscale=0.8, yscale=0.5]
|
||||
\tkzInit[xmin=-5,xmax=5,xstep=1,
|
||||
ymin=-5,ymax=5,ystep=1]
|
||||
\tkzGrid
|
||||
\tkzAxeXY
|
||||
\tkzFct[domain=-5:5,color=red,very thick]%
|
||||
{2.6666666666666665*\x -4};
|
||||
\end{tikzpicture}
|
||||
\item Résoudre l'équation $8 \times 0.76^x = 26$
|
||||
\end{enumerate}
|
||||
\end{exercise}
|
||||
|
||||
\begin{solution}
|
||||
\begin{enumerate}
|
||||
\item On veut partager cette évolution en 8 évolutions.
|
||||
\[
|
||||
\left(1 + \frac{21}{100}\right)^{\frac{1}{8}} = 1.0241
|
||||
\]
|
||||
Donc le taux d'évolution moyen est
|
||||
\[
|
||||
t_m = 1.0241 - 1 = 0.02410000000000001
|
||||
\]
|
||||
\item Coefficient multiplicateur pour revenir en arrière
|
||||
\[
|
||||
CM = (1 + \frac{21}{100})^{-1} = 0.8264
|
||||
\]
|
||||
On en déduit la quantité en 2019
|
||||
\[
|
||||
143 * 0.8264 = 118.1752
|
||||
\]
|
||||
\item L'équation de la droite est
|
||||
\[
|
||||
y = 2.6666666666666665 x -4
|
||||
\]
|
||||
\item Il faut penser à faire la division à par $8$ avant d'utiliser le log car sinon, on ne peut pas utiliser la formule $\log(a^n) = n\times \log(a)$.
|
||||
|
||||
\[x = \frac{\log(3.25)}{\log(0.76)}\]
|
||||
\end{enumerate}
|
||||
\end{solution}
|
||||
|
||||
\begin{exercise}[subtitle={Restaurant}]
|
||||
Un \emph{food truck}, ouvert le midi et le soir, propose deux types de formules :
|
||||
|
||||
\setlength\parindent{10mm}
|
||||
\begin{itemize}
|
||||
\item la formule \emph{Burger} ;
|
||||
\item la formule \emph{Wok}.
|
||||
\end{itemize}
|
||||
\setlength\parindent{0mm}
|
||||
|
||||
\medskip
|
||||
|
||||
Le gérant a remarqué que 21\,\% de ses ventes ont lieu le midi. Le quart des ventes du midi correspondent à la formule \emph{Burger}, alors que 27\,\% des ventes du soir correspondent à la formule \emph{Wok}.
|
||||
|
||||
Le gérant se constitue un fichier en notant, pour chaque vente, la formule choisie et le moment de cette vente (midi ou soir).
|
||||
|
||||
On prélève une fiche de façon équiprobable. On définit les quatre évènements suivants:
|
||||
|
||||
\begin{enumerate}
|
||||
\item $M$ : \og la fiche correspond à une vente du midi\fg{} ;
|
||||
\item $S$ : \og la fiche correspond à une vente du soir\fg {};
|
||||
\item $W$ : \og la fiche correspond à une formule \emph{Wok} \fg{} ;
|
||||
\item $B$ : \og la fiche correspond à une formule \emph{Burger} \fg.
|
||||
\end{enumerate}
|
||||
\setlength\parindent{0mm}
|
||||
|
||||
\medskip
|
||||
|
||||
\begin{enumerate}
|
||||
\item Recopier puis compléter l'arbre pondéré
|
||||
|
||||
\begin{center}
|
||||
\begin{tikzpicture}[sloped]
|
||||
\node {.}
|
||||
child {node {$M$}
|
||||
child {node {$W$}
|
||||
edge from parent
|
||||
node[above] {...}
|
||||
}
|
||||
child {node {$B$}
|
||||
edge from parent
|
||||
node[above] {...}
|
||||
}
|
||||
edge from parent
|
||||
node[above] {...}
|
||||
}
|
||||
child[missing] {}
|
||||
child { node {$S$}
|
||||
child {node {$W$}
|
||||
edge from parent
|
||||
node[above] {...}
|
||||
}
|
||||
child {node {$B$}
|
||||
edge from parent
|
||||
node[above] {...}
|
||||
}
|
||||
edge from parent
|
||||
node[above] {...}
|
||||
} ;
|
||||
\end{tikzpicture}
|
||||
\end{center}
|
||||
|
||||
\item Calculer la probabilité de l'évènement $M \cap W$. Interpréter ce résultat dans le contexte de l'exercice.
|
||||
\item Montrer que la probabilité que la fiche choisie corresponde à une formule \emph{Burger} est égale à $0.6292$.
|
||||
\item On a prélevé une fiche correspondant à la formule \emph{Burger}. Quelle est la probabilité, arrondie au millième, que la vente ait eu lieu le soir?
|
||||
\end{enumerate}
|
||||
\end{exercise}
|
||||
|
||||
\begin{solution}
|
||||
\begin{enumerate}
|
||||
\item
|
||||
\begin{center}
|
||||
\begin{tikzpicture}[sloped]
|
||||
\node {.}
|
||||
child {node {$M$}
|
||||
child {node {$W$}
|
||||
edge from parent
|
||||
node[above] {$0.75$}
|
||||
}
|
||||
child {node {$B$}
|
||||
edge from parent
|
||||
node[above] {$0.25$}
|
||||
}
|
||||
edge from parent
|
||||
node[above] {$0.21$}
|
||||
}
|
||||
child[missing] {}
|
||||
child { node {$S$}
|
||||
child {node {$W$}
|
||||
edge from parent
|
||||
node[above] {$0.27$}
|
||||
}
|
||||
child {node {$B$}
|
||||
edge from parent
|
||||
node[above] {$0.73$}
|
||||
}
|
||||
edge from parent
|
||||
node[above] {$0.79$}
|
||||
} ;
|
||||
\end{tikzpicture}
|
||||
\end{center}
|
||||
\item On calcule la probabilité que la vente soit un wok et ait eu lieu à midi
|
||||
\[ P(M\cap W) = P(M) \times P_M(W) = 0.21 \times 0.75 = 0.1575 \]
|
||||
\item Probabilité que la vente soit un burger.
|
||||
\[
|
||||
P(B) = P(M\cap B) + P(S\cap B) = 0.21 \times 0.75 + 0.79 \times 0.27 = 0.6292
|
||||
\]
|
||||
\item On cherche à calculer la quantité $P_B(S)$. Pour cela on utilise la formule de Bayes
|
||||
\[
|
||||
P_B(S) = \frac{P(B\cap S)}{P(B)} = \frac{P_S(B) \times P(S)}{P(B)} = \frac{0.73\times 0.79}{0.6292} = 0.9165607120152575 \approx 0.917
|
||||
\]
|
||||
\end{enumerate}
|
||||
\end{solution}
|
||||
|
||||
\begin{exercise}[subtitle={Continent plastique}]
|
||||
\textit{Les quantités évoqués dans cette exercice sont générés au hasard et sont donc complètement farfelus.}
|
||||
\medskip
|
||||
Le \og continent de plastique\fg{} est la plus grande des plaques de déchets plastiques évoluant sur les océans. Elle occupe actuellement dans l'océan Pacifique une surface dont l'aire est évaluée à plus de $1,6$ million de km$^2$, entre Hawaï et la Californie.
|
||||
|
||||
En 2017, des scientifiques ont estimé qu'il y avait $20$ millions de tonnes de déchets plastiques qui était déversé chaque année dans les océans et que cette quantité augmentait de $28\n\%$ par chaque année.
|
||||
|
||||
On modélise l'évolution de la masse de ces déchets plastiques déversée chaque année, si rien n'est fait pour la réduire, par une suite géométrique $\left(u_n\right)$. L'arrondi au centième du terme $u_n$ représente la masse de ces déchets déversée chaque année, exprimée en million de tonnes, pour l'année $(2017 + n)$.
|
||||
|
||||
\medskip
|
||||
|
||||
\begin{enumerate}
|
||||
\item Expliquer pourquoi la suite $u_n$ est géométrique?
|
||||
\item Calculer $u_1$ et $u_2$.
|
||||
\item Exprimer $u_n$ en fonction de $n$.
|
||||
\item Au début de l'année 2017, il y avait $300$ millions de tonnes de déchets plastique. Calculer la quantité totale de déchets plastiques en 2030.
|
||||
\item On souhaite déterminer en quelle année la masse totale de ces déchets plastiques aura pour la première fois augmenté de $50$\,\% par rapport à sa valeur de 2017.
|
||||
\begin{enumerate}
|
||||
\item Recopier et compléter l'algorithme ci-dessous pour que la variable $N$ contienne la réponse au problème posé.
|
||||
|
||||
\begin{center}
|
||||
\begin{tabularx}{0.4\linewidth}{|X|}\hline
|
||||
$N = 2017$\\
|
||||
$U = 20$ \\
|
||||
$S = 300 + U$ \\
|
||||
while $S < 450$: \\
|
||||
\hspace{1cm} $N = \ldots$\\
|
||||
\hspace{1cm} $U = \ldots$\\
|
||||
\hspace{1cm} $S = \ldots$\\
|
||||
\hline
|
||||
\end{tabularx}
|
||||
\end{center}
|
||||
\item Que contiennent les variables $S$, $U$ et $N$ après exécution de cet algorithme ?
|
||||
|
||||
Interpréter les résultats dans le contexte de l'exercice.
|
||||
\end{enumerate}
|
||||
\end{enumerate}
|
||||
\end{exercise}
|
||||
|
||||
\begin{solution}
|
||||
\begin{enumerate}
|
||||
\item Une augmentation de $28\,\%$ revient à multiplier la quantité par $1.28$. La suite est donc bien géométrique. Son premier terme est $u_0 = 20$ et sa raison est $q = 1.28$
|
||||
\item
|
||||
\[
|
||||
u_1 = u_0 * 1.28 = 25.6
|
||||
\]
|
||||
\[
|
||||
u_2 = u_0 * 1.28^2 = 32.768
|
||||
\]
|
||||
\item
|
||||
\[
|
||||
u_n = u_0 \times q^n = 20 \times 1.28^n
|
||||
\]
|
||||
\item On calcule la quantité totale déversée entre 2017 et 2030.
|
||||
\[
|
||||
\sum_{n = 0}^{13} u_n = u_0 \times \frac{1-q^{13}}{1-q} = 20 \times \frac{1 - 1.28^{13}}{1 - 1.28} = 1697.06
|
||||
\]
|
||||
On en déduit la quantité totale de déchets en 2030
|
||||
\[
|
||||
300 + 1697.06 = 1997.06
|
||||
\]
|
||||
\item
|
||||
\begin{enumerate}
|
||||
\item ~
|
||||
\begin{center}
|
||||
\begin{tabularx}{0.4\linewidth}{|X|}\hline
|
||||
$N \gets 2017$\\
|
||||
$U \gets 20$ \\
|
||||
$S \gets 300 + U$ \\
|
||||
Tant que $S < 450$ \\
|
||||
\hspace{1cm} $N \gets N + 1$\\
|
||||
\hspace{1cm} $U \gets U * 1.28$\\
|
||||
\hspace{1cm} $S \gets S + u$\\
|
||||
Fin Tant que\\\hline
|
||||
\end{tabularx}
|
||||
\end{center}
|
||||
\item \textit{Pas de correction automatisé}
|
||||
\end{enumerate}
|
||||
\end{enumerate}
|
||||
\end{solution}
|
||||
|
||||
\end{document}
|
||||
|
||||
%%% Local Variables:
|
||||
%%% mode: latex
|
||||
%%% TeX-master: "master"
|
||||
%%% End:
|
262
TST/DS/DS_21_04_07/TST3/corr_02_210407_DS8.tex
Normal file
262
TST/DS/DS_21_04_07/TST3/corr_02_210407_DS8.tex
Normal file
@ -0,0 +1,262 @@
|
||||
\documentclass[a4paper,10pt]{article}
|
||||
\usepackage{myXsim}
|
||||
|
||||
% Title Page
|
||||
\title{DS8 \hfill BERTAN Ufuk}
|
||||
\tribe{TST}
|
||||
\date{\hfillÀ render pour le Mercredi 7 avril}
|
||||
|
||||
\xsimsetup{
|
||||
solution/print = true
|
||||
}
|
||||
|
||||
\begin{document}
|
||||
\maketitle
|
||||
|
||||
\begin{exercise}[subtitle={Automatismes}]
|
||||
\textit{Toutes les questions de cette exercice sont indépendantes et peuvent être répondus séparément}
|
||||
\begin{enumerate}
|
||||
\item De janvier à septembre, une quantité a augmenté de $22\,\%$. Faire un schéma pour représenter la situation puis calculer le taux d'évolution moyen mensuel.
|
||||
\item Une quantité augmente de $22\,\%$ par ans. En 2020, elle est de 134\euro. Quelle était sa valeur en 2019? Faire un schéma pour représenter la situation.
|
||||
\item Déterminer l'équation de la droite \\
|
||||
\begin{tikzpicture}[xscale=0.8, yscale=0.5]
|
||||
\tkzInit[xmin=-5,xmax=5,xstep=1,
|
||||
ymin=-5,ymax=5,ystep=1]
|
||||
\tkzGrid
|
||||
\tkzAxeXY
|
||||
\tkzFct[domain=-5:5,color=red,very thick]%
|
||||
{2.0*\x -2};
|
||||
\end{tikzpicture}
|
||||
\item Résoudre l'équation $5 \times 0.99^x = 21$
|
||||
\end{enumerate}
|
||||
\end{exercise}
|
||||
|
||||
\begin{solution}
|
||||
\begin{enumerate}
|
||||
\item On veut partager cette évolution en 8 évolutions.
|
||||
\[
|
||||
\left(1 + \frac{22}{100}\right)^{\frac{1}{8}} = 1.0252
|
||||
\]
|
||||
Donc le taux d'évolution moyen est
|
||||
\[
|
||||
t_m = 1.0252 - 1 = 0.02519999999999989
|
||||
\]
|
||||
\item Coefficient multiplicateur pour revenir en arrière
|
||||
\[
|
||||
CM = (1 + \frac{22}{100})^{-1} = 0.8197
|
||||
\]
|
||||
On en déduit la quantité en 2019
|
||||
\[
|
||||
134 * 0.8197 = 109.8398
|
||||
\]
|
||||
\item L'équation de la droite est
|
||||
\[
|
||||
y = 2.0 x -2
|
||||
\]
|
||||
\item Il faut penser à faire la division à par $5$ avant d'utiliser le log car sinon, on ne peut pas utiliser la formule $\log(a^n) = n\times \log(a)$.
|
||||
|
||||
\[x = \frac{\log(4.2)}{\log(0.99)}\]
|
||||
\end{enumerate}
|
||||
\end{solution}
|
||||
|
||||
\begin{exercise}[subtitle={Restaurant}]
|
||||
Un \emph{food truck}, ouvert le midi et le soir, propose deux types de formules :
|
||||
|
||||
\setlength\parindent{10mm}
|
||||
\begin{itemize}
|
||||
\item la formule \emph{Burger} ;
|
||||
\item la formule \emph{Wok}.
|
||||
\end{itemize}
|
||||
\setlength\parindent{0mm}
|
||||
|
||||
\medskip
|
||||
|
||||
Le gérant a remarqué que 64\,\% de ses ventes ont lieu le midi. Le quart des ventes du midi correspondent à la formule \emph{Burger}, alors que 100\,\% des ventes du soir correspondent à la formule \emph{Wok}.
|
||||
|
||||
Le gérant se constitue un fichier en notant, pour chaque vente, la formule choisie et le moment de cette vente (midi ou soir).
|
||||
|
||||
On prélève une fiche de façon équiprobable. On définit les quatre évènements suivants:
|
||||
|
||||
\begin{enumerate}
|
||||
\item $M$ : \og la fiche correspond à une vente du midi\fg{} ;
|
||||
\item $S$ : \og la fiche correspond à une vente du soir\fg {};
|
||||
\item $W$ : \og la fiche correspond à une formule \emph{Wok} \fg{} ;
|
||||
\item $B$ : \og la fiche correspond à une formule \emph{Burger} \fg.
|
||||
\end{enumerate}
|
||||
\setlength\parindent{0mm}
|
||||
|
||||
\medskip
|
||||
|
||||
\begin{enumerate}
|
||||
\item Recopier puis compléter l'arbre pondéré
|
||||
|
||||
\begin{center}
|
||||
\begin{tikzpicture}[sloped]
|
||||
\node {.}
|
||||
child {node {$M$}
|
||||
child {node {$W$}
|
||||
edge from parent
|
||||
node[above] {...}
|
||||
}
|
||||
child {node {$B$}
|
||||
edge from parent
|
||||
node[above] {...}
|
||||
}
|
||||
edge from parent
|
||||
node[above] {...}
|
||||
}
|
||||
child[missing] {}
|
||||
child { node {$S$}
|
||||
child {node {$W$}
|
||||
edge from parent
|
||||
node[above] {...}
|
||||
}
|
||||
child {node {$B$}
|
||||
edge from parent
|
||||
node[above] {...}
|
||||
}
|
||||
edge from parent
|
||||
node[above] {...}
|
||||
} ;
|
||||
\end{tikzpicture}
|
||||
\end{center}
|
||||
|
||||
\item Calculer la probabilité de l'évènement $M \cap W$. Interpréter ce résultat dans le contexte de l'exercice.
|
||||
\item Montrer que la probabilité que la fiche choisie corresponde à une formule \emph{Burger} est égale à $0.16$.
|
||||
\item On a prélevé une fiche correspondant à la formule \emph{Burger}. Quelle est la probabilité, arrondie au millième, que la vente ait eu lieu le soir?
|
||||
\end{enumerate}
|
||||
\end{exercise}
|
||||
|
||||
\begin{solution}
|
||||
\begin{enumerate}
|
||||
\item
|
||||
\begin{center}
|
||||
\begin{tikzpicture}[sloped]
|
||||
\node {.}
|
||||
child {node {$M$}
|
||||
child {node {$W$}
|
||||
edge from parent
|
||||
node[above] {$0.75$}
|
||||
}
|
||||
child {node {$B$}
|
||||
edge from parent
|
||||
node[above] {$0.25$}
|
||||
}
|
||||
edge from parent
|
||||
node[above] {$0.64$}
|
||||
}
|
||||
child[missing] {}
|
||||
child { node {$S$}
|
||||
child {node {$W$}
|
||||
edge from parent
|
||||
node[above] {$1.0$}
|
||||
}
|
||||
child {node {$B$}
|
||||
edge from parent
|
||||
node[above] {$0.0$}
|
||||
}
|
||||
edge from parent
|
||||
node[above] {$0.36$}
|
||||
} ;
|
||||
\end{tikzpicture}
|
||||
\end{center}
|
||||
\item On calcule la probabilité que la vente soit un wok et ait eu lieu à midi
|
||||
\[ P(M\cap W) = P(M) \times P_M(W) = 0.64 \times 0.75 = 0.48 \]
|
||||
\item Probabilité que la vente soit un burger.
|
||||
\[
|
||||
P(B) = P(M\cap B) + P(S\cap B) = 0.64 \times 0.75 + 0.36 \times 1.0 = 0.16
|
||||
\]
|
||||
\item On cherche à calculer la quantité $P_B(S)$. Pour cela on utilise la formule de Bayes
|
||||
\[
|
||||
P_B(S) = \frac{P(B\cap S)}{P(B)} = \frac{P_S(B) \times P(S)}{P(B)} = \frac{0.0\times 0.36}{0.16} = 0.0 \approx 0.0
|
||||
\]
|
||||
\end{enumerate}
|
||||
\end{solution}
|
||||
|
||||
\begin{exercise}[subtitle={Continent plastique}]
|
||||
\textit{Les quantités évoqués dans cette exercice sont générés au hasard et sont donc complètement farfelus.}
|
||||
\medskip
|
||||
Le \og continent de plastique\fg{} est la plus grande des plaques de déchets plastiques évoluant sur les océans. Elle occupe actuellement dans l'océan Pacifique une surface dont l'aire est évaluée à plus de $1,6$ million de km$^2$, entre Hawaï et la Californie.
|
||||
|
||||
En 2017, des scientifiques ont estimé qu'il y avait $2$ millions de tonnes de déchets plastiques qui était déversé chaque année dans les océans et que cette quantité augmentait de $27\n\%$ par chaque année.
|
||||
|
||||
On modélise l'évolution de la masse de ces déchets plastiques déversée chaque année, si rien n'est fait pour la réduire, par une suite géométrique $\left(u_n\right)$. L'arrondi au centième du terme $u_n$ représente la masse de ces déchets déversée chaque année, exprimée en million de tonnes, pour l'année $(2017 + n)$.
|
||||
|
||||
\medskip
|
||||
|
||||
\begin{enumerate}
|
||||
\item Expliquer pourquoi la suite $u_n$ est géométrique?
|
||||
\item Calculer $u_1$ et $u_2$.
|
||||
\item Exprimer $u_n$ en fonction de $n$.
|
||||
\item Au début de l'année 2017, il y avait $300$ millions de tonnes de déchets plastique. Calculer la quantité totale de déchets plastiques en 2030.
|
||||
\item On souhaite déterminer en quelle année la masse totale de ces déchets plastiques aura pour la première fois augmenté de $50$\,\% par rapport à sa valeur de 2017.
|
||||
\begin{enumerate}
|
||||
\item Recopier et compléter l'algorithme ci-dessous pour que la variable $N$ contienne la réponse au problème posé.
|
||||
|
||||
\begin{center}
|
||||
\begin{tabularx}{0.4\linewidth}{|X|}\hline
|
||||
$N = 2017$\\
|
||||
$U = 2$ \\
|
||||
$S = 300 + U$ \\
|
||||
while $S < 450$: \\
|
||||
\hspace{1cm} $N = \ldots$\\
|
||||
\hspace{1cm} $U = \ldots$\\
|
||||
\hspace{1cm} $S = \ldots$\\
|
||||
\hline
|
||||
\end{tabularx}
|
||||
\end{center}
|
||||
\item Que contiennent les variables $S$, $U$ et $N$ après exécution de cet algorithme ?
|
||||
|
||||
Interpréter les résultats dans le contexte de l'exercice.
|
||||
\end{enumerate}
|
||||
\end{enumerate}
|
||||
\end{exercise}
|
||||
|
||||
\begin{solution}
|
||||
\begin{enumerate}
|
||||
\item Une augmentation de $27\,\%$ revient à multiplier la quantité par $1.27$. La suite est donc bien géométrique. Son premier terme est $u_0 = 2$ et sa raison est $q = 1.27$
|
||||
\item
|
||||
\[
|
||||
u_1 = u_0 * 1.27 = 2.54
|
||||
\]
|
||||
\[
|
||||
u_2 = u_0 * 1.27^2 = 3.2258
|
||||
\]
|
||||
\item
|
||||
\[
|
||||
u_n = u_0 \times q^n = 2 \times 1.27^n
|
||||
\]
|
||||
\item On calcule la quantité totale déversée entre 2017 et 2030.
|
||||
\[
|
||||
\sum_{n = 0}^{13} u_n = u_0 \times \frac{1-q^{13}}{1-q} = 2 \times \frac{1 - 1.27^{13}}{1 - 1.27} = 158.21
|
||||
\]
|
||||
On en déduit la quantité totale de déchets en 2030
|
||||
\[
|
||||
300 + 158.21 = 458.21000000000004
|
||||
\]
|
||||
\item
|
||||
\begin{enumerate}
|
||||
\item ~
|
||||
\begin{center}
|
||||
\begin{tabularx}{0.4\linewidth}{|X|}\hline
|
||||
$N \gets 2017$\\
|
||||
$U \gets 2$ \\
|
||||
$S \gets 300 + U$ \\
|
||||
Tant que $S < 450$ \\
|
||||
\hspace{1cm} $N \gets N + 1$\\
|
||||
\hspace{1cm} $U \gets U * 1.27$\\
|
||||
\hspace{1cm} $S \gets S + u$\\
|
||||
Fin Tant que\\\hline
|
||||
\end{tabularx}
|
||||
\end{center}
|
||||
\item \textit{Pas de correction automatisé}
|
||||
\end{enumerate}
|
||||
\end{enumerate}
|
||||
\end{solution}
|
||||
|
||||
\end{document}
|
||||
|
||||
%%% Local Variables:
|
||||
%%% mode: latex
|
||||
%%% TeX-master: "master"
|
||||
%%% End:
|
262
TST/DS/DS_21_04_07/TST3/corr_03_210407_DS8.tex
Normal file
262
TST/DS/DS_21_04_07/TST3/corr_03_210407_DS8.tex
Normal file
@ -0,0 +1,262 @@
|
||||
\documentclass[a4paper,10pt]{article}
|
||||
\usepackage{myXsim}
|
||||
|
||||
% Title Page
|
||||
\title{DS8 \hfill BOUALIA Bilel}
|
||||
\tribe{TST}
|
||||
\date{\hfillÀ render pour le Mercredi 7 avril}
|
||||
|
||||
\xsimsetup{
|
||||
solution/print = true
|
||||
}
|
||||
|
||||
\begin{document}
|
||||
\maketitle
|
||||
|
||||
\begin{exercise}[subtitle={Automatismes}]
|
||||
\textit{Toutes les questions de cette exercice sont indépendantes et peuvent être répondus séparément}
|
||||
\begin{enumerate}
|
||||
\item De janvier à septembre, une quantité a augmenté de $17\,\%$. Faire un schéma pour représenter la situation puis calculer le taux d'évolution moyen mensuel.
|
||||
\item Une quantité augmente de $17\,\%$ par ans. En 2020, elle est de 135\euro. Quelle était sa valeur en 2019? Faire un schéma pour représenter la situation.
|
||||
\item Déterminer l'équation de la droite \\
|
||||
\begin{tikzpicture}[xscale=0.8, yscale=0.5]
|
||||
\tkzInit[xmin=-5,xmax=5,xstep=1,
|
||||
ymin=-5,ymax=5,ystep=1]
|
||||
\tkzGrid
|
||||
\tkzAxeXY
|
||||
\tkzFct[domain=-5:5,color=red,very thick]%
|
||||
{2.0*\x -4};
|
||||
\end{tikzpicture}
|
||||
\item Résoudre l'équation $3 \times 0.2^x = 2$
|
||||
\end{enumerate}
|
||||
\end{exercise}
|
||||
|
||||
\begin{solution}
|
||||
\begin{enumerate}
|
||||
\item On veut partager cette évolution en 8 évolutions.
|
||||
\[
|
||||
\left(1 + \frac{17}{100}\right)^{\frac{1}{8}} = 1.0198
|
||||
\]
|
||||
Donc le taux d'évolution moyen est
|
||||
\[
|
||||
t_m = 1.0198 - 1 = 0.01980000000000004
|
||||
\]
|
||||
\item Coefficient multiplicateur pour revenir en arrière
|
||||
\[
|
||||
CM = (1 + \frac{17}{100})^{-1} = 0.8547
|
||||
\]
|
||||
On en déduit la quantité en 2019
|
||||
\[
|
||||
135 * 0.8547 = 115.3845
|
||||
\]
|
||||
\item L'équation de la droite est
|
||||
\[
|
||||
y = 2.0 x -4
|
||||
\]
|
||||
\item Il faut penser à faire la division à par $3$ avant d'utiliser le log car sinon, on ne peut pas utiliser la formule $\log(a^n) = n\times \log(a)$.
|
||||
|
||||
\[x = \frac{\log(0.67)}{\log(0.2)}\]
|
||||
\end{enumerate}
|
||||
\end{solution}
|
||||
|
||||
\begin{exercise}[subtitle={Restaurant}]
|
||||
Un \emph{food truck}, ouvert le midi et le soir, propose deux types de formules :
|
||||
|
||||
\setlength\parindent{10mm}
|
||||
\begin{itemize}
|
||||
\item la formule \emph{Burger} ;
|
||||
\item la formule \emph{Wok}.
|
||||
\end{itemize}
|
||||
\setlength\parindent{0mm}
|
||||
|
||||
\medskip
|
||||
|
||||
Le gérant a remarqué que 89\,\% de ses ventes ont lieu le midi. Le quart des ventes du midi correspondent à la formule \emph{Burger}, alors que 12\,\% des ventes du soir correspondent à la formule \emph{Wok}.
|
||||
|
||||
Le gérant se constitue un fichier en notant, pour chaque vente, la formule choisie et le moment de cette vente (midi ou soir).
|
||||
|
||||
On prélève une fiche de façon équiprobable. On définit les quatre évènements suivants:
|
||||
|
||||
\begin{enumerate}
|
||||
\item $M$ : \og la fiche correspond à une vente du midi\fg{} ;
|
||||
\item $S$ : \og la fiche correspond à une vente du soir\fg {};
|
||||
\item $W$ : \og la fiche correspond à une formule \emph{Wok} \fg{} ;
|
||||
\item $B$ : \og la fiche correspond à une formule \emph{Burger} \fg.
|
||||
\end{enumerate}
|
||||
\setlength\parindent{0mm}
|
||||
|
||||
\medskip
|
||||
|
||||
\begin{enumerate}
|
||||
\item Recopier puis compléter l'arbre pondéré
|
||||
|
||||
\begin{center}
|
||||
\begin{tikzpicture}[sloped]
|
||||
\node {.}
|
||||
child {node {$M$}
|
||||
child {node {$W$}
|
||||
edge from parent
|
||||
node[above] {...}
|
||||
}
|
||||
child {node {$B$}
|
||||
edge from parent
|
||||
node[above] {...}
|
||||
}
|
||||
edge from parent
|
||||
node[above] {...}
|
||||
}
|
||||
child[missing] {}
|
||||
child { node {$S$}
|
||||
child {node {$W$}
|
||||
edge from parent
|
||||
node[above] {...}
|
||||
}
|
||||
child {node {$B$}
|
||||
edge from parent
|
||||
node[above] {...}
|
||||
}
|
||||
edge from parent
|
||||
node[above] {...}
|
||||
} ;
|
||||
\end{tikzpicture}
|
||||
\end{center}
|
||||
|
||||
\item Calculer la probabilité de l'évènement $M \cap W$. Interpréter ce résultat dans le contexte de l'exercice.
|
||||
\item Montrer que la probabilité que la fiche choisie corresponde à une formule \emph{Burger} est égale à $0.3193$.
|
||||
\item On a prélevé une fiche correspondant à la formule \emph{Burger}. Quelle est la probabilité, arrondie au millième, que la vente ait eu lieu le soir?
|
||||
\end{enumerate}
|
||||
\end{exercise}
|
||||
|
||||
\begin{solution}
|
||||
\begin{enumerate}
|
||||
\item
|
||||
\begin{center}
|
||||
\begin{tikzpicture}[sloped]
|
||||
\node {.}
|
||||
child {node {$M$}
|
||||
child {node {$W$}
|
||||
edge from parent
|
||||
node[above] {$0.75$}
|
||||
}
|
||||
child {node {$B$}
|
||||
edge from parent
|
||||
node[above] {$0.25$}
|
||||
}
|
||||
edge from parent
|
||||
node[above] {$0.89$}
|
||||
}
|
||||
child[missing] {}
|
||||
child { node {$S$}
|
||||
child {node {$W$}
|
||||
edge from parent
|
||||
node[above] {$0.12$}
|
||||
}
|
||||
child {node {$B$}
|
||||
edge from parent
|
||||
node[above] {$0.88$}
|
||||
}
|
||||
edge from parent
|
||||
node[above] {$0.11$}
|
||||
} ;
|
||||
\end{tikzpicture}
|
||||
\end{center}
|
||||
\item On calcule la probabilité que la vente soit un wok et ait eu lieu à midi
|
||||
\[ P(M\cap W) = P(M) \times P_M(W) = 0.89 \times 0.75 = 0.6675 \]
|
||||
\item Probabilité que la vente soit un burger.
|
||||
\[
|
||||
P(B) = P(M\cap B) + P(S\cap B) = 0.89 \times 0.75 + 0.11 \times 0.12 = 0.3193
|
||||
\]
|
||||
\item On cherche à calculer la quantité $P_B(S)$. Pour cela on utilise la formule de Bayes
|
||||
\[
|
||||
P_B(S) = \frac{P(B\cap S)}{P(B)} = \frac{P_S(B) \times P(S)}{P(B)} = \frac{0.88\times 0.11}{0.3193} = 0.303163169433135 \approx 0.303
|
||||
\]
|
||||
\end{enumerate}
|
||||
\end{solution}
|
||||
|
||||
\begin{exercise}[subtitle={Continent plastique}]
|
||||
\textit{Les quantités évoqués dans cette exercice sont générés au hasard et sont donc complètement farfelus.}
|
||||
\medskip
|
||||
Le \og continent de plastique\fg{} est la plus grande des plaques de déchets plastiques évoluant sur les océans. Elle occupe actuellement dans l'océan Pacifique une surface dont l'aire est évaluée à plus de $1,6$ million de km$^2$, entre Hawaï et la Californie.
|
||||
|
||||
En 2017, des scientifiques ont estimé qu'il y avait $10$ millions de tonnes de déchets plastiques qui était déversé chaque année dans les océans et que cette quantité augmentait de $27\n\%$ par chaque année.
|
||||
|
||||
On modélise l'évolution de la masse de ces déchets plastiques déversée chaque année, si rien n'est fait pour la réduire, par une suite géométrique $\left(u_n\right)$. L'arrondi au centième du terme $u_n$ représente la masse de ces déchets déversée chaque année, exprimée en million de tonnes, pour l'année $(2017 + n)$.
|
||||
|
||||
\medskip
|
||||
|
||||
\begin{enumerate}
|
||||
\item Expliquer pourquoi la suite $u_n$ est géométrique?
|
||||
\item Calculer $u_1$ et $u_2$.
|
||||
\item Exprimer $u_n$ en fonction de $n$.
|
||||
\item Au début de l'année 2017, il y avait $300$ millions de tonnes de déchets plastique. Calculer la quantité totale de déchets plastiques en 2030.
|
||||
\item On souhaite déterminer en quelle année la masse totale de ces déchets plastiques aura pour la première fois augmenté de $50$\,\% par rapport à sa valeur de 2017.
|
||||
\begin{enumerate}
|
||||
\item Recopier et compléter l'algorithme ci-dessous pour que la variable $N$ contienne la réponse au problème posé.
|
||||
|
||||
\begin{center}
|
||||
\begin{tabularx}{0.4\linewidth}{|X|}\hline
|
||||
$N = 2017$\\
|
||||
$U = 10$ \\
|
||||
$S = 300 + U$ \\
|
||||
while $S < 450$: \\
|
||||
\hspace{1cm} $N = \ldots$\\
|
||||
\hspace{1cm} $U = \ldots$\\
|
||||
\hspace{1cm} $S = \ldots$\\
|
||||
\hline
|
||||
\end{tabularx}
|
||||
\end{center}
|
||||
\item Que contiennent les variables $S$, $U$ et $N$ après exécution de cet algorithme ?
|
||||
|
||||
Interpréter les résultats dans le contexte de l'exercice.
|
||||
\end{enumerate}
|
||||
\end{enumerate}
|
||||
\end{exercise}
|
||||
|
||||
\begin{solution}
|
||||
\begin{enumerate}
|
||||
\item Une augmentation de $27\,\%$ revient à multiplier la quantité par $1.27$. La suite est donc bien géométrique. Son premier terme est $u_0 = 10$ et sa raison est $q = 1.27$
|
||||
\item
|
||||
\[
|
||||
u_1 = u_0 * 1.27 = 12.7
|
||||
\]
|
||||
\[
|
||||
u_2 = u_0 * 1.27^2 = 16.129
|
||||
\]
|
||||
\item
|
||||
\[
|
||||
u_n = u_0 \times q^n = 10 \times 1.27^n
|
||||
\]
|
||||
\item On calcule la quantité totale déversée entre 2017 et 2030.
|
||||
\[
|
||||
\sum_{n = 0}^{13} u_n = u_0 \times \frac{1-q^{13}}{1-q} = 10 \times \frac{1 - 1.27^{13}}{1 - 1.27} = 791.07
|
||||
\]
|
||||
On en déduit la quantité totale de déchets en 2030
|
||||
\[
|
||||
300 + 791.07 = 1091.0700000000002
|
||||
\]
|
||||
\item
|
||||
\begin{enumerate}
|
||||
\item ~
|
||||
\begin{center}
|
||||
\begin{tabularx}{0.4\linewidth}{|X|}\hline
|
||||
$N \gets 2017$\\
|
||||
$U \gets 10$ \\
|
||||
$S \gets 300 + U$ \\
|
||||
Tant que $S < 450$ \\
|
||||
\hspace{1cm} $N \gets N + 1$\\
|
||||
\hspace{1cm} $U \gets U * 1.27$\\
|
||||
\hspace{1cm} $S \gets S + u$\\
|
||||
Fin Tant que\\\hline
|
||||
\end{tabularx}
|
||||
\end{center}
|
||||
\item \textit{Pas de correction automatisé}
|
||||
\end{enumerate}
|
||||
\end{enumerate}
|
||||
\end{solution}
|
||||
|
||||
\end{document}
|
||||
|
||||
%%% Local Variables:
|
||||
%%% mode: latex
|
||||
%%% TeX-master: "master"
|
||||
%%% End:
|
262
TST/DS/DS_21_04_07/TST3/corr_04_210407_DS8.tex
Normal file
262
TST/DS/DS_21_04_07/TST3/corr_04_210407_DS8.tex
Normal file
@ -0,0 +1,262 @@
|
||||
\documentclass[a4paper,10pt]{article}
|
||||
\usepackage{myXsim}
|
||||
|
||||
% Title Page
|
||||
\title{DS8 \hfill BOUCHOUX Kevin}
|
||||
\tribe{TST}
|
||||
\date{\hfillÀ render pour le Mercredi 7 avril}
|
||||
|
||||
\xsimsetup{
|
||||
solution/print = true
|
||||
}
|
||||
|
||||
\begin{document}
|
||||
\maketitle
|
||||
|
||||
\begin{exercise}[subtitle={Automatismes}]
|
||||
\textit{Toutes les questions de cette exercice sont indépendantes et peuvent être répondus séparément}
|
||||
\begin{enumerate}
|
||||
\item De janvier à septembre, une quantité a augmenté de $10\,\%$. Faire un schéma pour représenter la situation puis calculer le taux d'évolution moyen mensuel.
|
||||
\item Une quantité augmente de $10\,\%$ par ans. En 2020, elle est de 118\euro. Quelle était sa valeur en 2019? Faire un schéma pour représenter la situation.
|
||||
\item Déterminer l'équation de la droite \\
|
||||
\begin{tikzpicture}[xscale=0.8, yscale=0.5]
|
||||
\tkzInit[xmin=-5,xmax=5,xstep=1,
|
||||
ymin=-5,ymax=5,ystep=1]
|
||||
\tkzGrid
|
||||
\tkzAxeXY
|
||||
\tkzFct[domain=-5:5,color=red,very thick]%
|
||||
{3.0*\x -3};
|
||||
\end{tikzpicture}
|
||||
\item Résoudre l'équation $2 \times 0.93^x = 44$
|
||||
\end{enumerate}
|
||||
\end{exercise}
|
||||
|
||||
\begin{solution}
|
||||
\begin{enumerate}
|
||||
\item On veut partager cette évolution en 8 évolutions.
|
||||
\[
|
||||
\left(1 + \frac{10}{100}\right)^{\frac{1}{8}} = 1.012
|
||||
\]
|
||||
Donc le taux d'évolution moyen est
|
||||
\[
|
||||
t_m = 1.012 - 1 = 0.01200000000000001
|
||||
\]
|
||||
\item Coefficient multiplicateur pour revenir en arrière
|
||||
\[
|
||||
CM = (1 + \frac{10}{100})^{-1} = 0.9091
|
||||
\]
|
||||
On en déduit la quantité en 2019
|
||||
\[
|
||||
118 * 0.9091 = 107.27380000000001
|
||||
\]
|
||||
\item L'équation de la droite est
|
||||
\[
|
||||
y = 3.0 x -3
|
||||
\]
|
||||
\item Il faut penser à faire la division à par $2$ avant d'utiliser le log car sinon, on ne peut pas utiliser la formule $\log(a^n) = n\times \log(a)$.
|
||||
|
||||
\[x = \frac{\log(22.0)}{\log(0.93)}\]
|
||||
\end{enumerate}
|
||||
\end{solution}
|
||||
|
||||
\begin{exercise}[subtitle={Restaurant}]
|
||||
Un \emph{food truck}, ouvert le midi et le soir, propose deux types de formules :
|
||||
|
||||
\setlength\parindent{10mm}
|
||||
\begin{itemize}
|
||||
\item la formule \emph{Burger} ;
|
||||
\item la formule \emph{Wok}.
|
||||
\end{itemize}
|
||||
\setlength\parindent{0mm}
|
||||
|
||||
\medskip
|
||||
|
||||
Le gérant a remarqué que 83\,\% de ses ventes ont lieu le midi. Le quart des ventes du midi correspondent à la formule \emph{Burger}, alors que 26\,\% des ventes du soir correspondent à la formule \emph{Wok}.
|
||||
|
||||
Le gérant se constitue un fichier en notant, pour chaque vente, la formule choisie et le moment de cette vente (midi ou soir).
|
||||
|
||||
On prélève une fiche de façon équiprobable. On définit les quatre évènements suivants:
|
||||
|
||||
\begin{enumerate}
|
||||
\item $M$ : \og la fiche correspond à une vente du midi\fg{} ;
|
||||
\item $S$ : \og la fiche correspond à une vente du soir\fg {};
|
||||
\item $W$ : \og la fiche correspond à une formule \emph{Wok} \fg{} ;
|
||||
\item $B$ : \og la fiche correspond à une formule \emph{Burger} \fg.
|
||||
\end{enumerate}
|
||||
\setlength\parindent{0mm}
|
||||
|
||||
\medskip
|
||||
|
||||
\begin{enumerate}
|
||||
\item Recopier puis compléter l'arbre pondéré
|
||||
|
||||
\begin{center}
|
||||
\begin{tikzpicture}[sloped]
|
||||
\node {.}
|
||||
child {node {$M$}
|
||||
child {node {$W$}
|
||||
edge from parent
|
||||
node[above] {...}
|
||||
}
|
||||
child {node {$B$}
|
||||
edge from parent
|
||||
node[above] {...}
|
||||
}
|
||||
edge from parent
|
||||
node[above] {...}
|
||||
}
|
||||
child[missing] {}
|
||||
child { node {$S$}
|
||||
child {node {$W$}
|
||||
edge from parent
|
||||
node[above] {...}
|
||||
}
|
||||
child {node {$B$}
|
||||
edge from parent
|
||||
node[above] {...}
|
||||
}
|
||||
edge from parent
|
||||
node[above] {...}
|
||||
} ;
|
||||
\end{tikzpicture}
|
||||
\end{center}
|
||||
|
||||
\item Calculer la probabilité de l'évènement $M \cap W$. Interpréter ce résultat dans le contexte de l'exercice.
|
||||
\item Montrer que la probabilité que la fiche choisie corresponde à une formule \emph{Burger} est égale à $0.3333$.
|
||||
\item On a prélevé une fiche correspondant à la formule \emph{Burger}. Quelle est la probabilité, arrondie au millième, que la vente ait eu lieu le soir?
|
||||
\end{enumerate}
|
||||
\end{exercise}
|
||||
|
||||
\begin{solution}
|
||||
\begin{enumerate}
|
||||
\item
|
||||
\begin{center}
|
||||
\begin{tikzpicture}[sloped]
|
||||
\node {.}
|
||||
child {node {$M$}
|
||||
child {node {$W$}
|
||||
edge from parent
|
||||
node[above] {$0.75$}
|
||||
}
|
||||
child {node {$B$}
|
||||
edge from parent
|
||||
node[above] {$0.25$}
|
||||
}
|
||||
edge from parent
|
||||
node[above] {$0.83$}
|
||||
}
|
||||
child[missing] {}
|
||||
child { node {$S$}
|
||||
child {node {$W$}
|
||||
edge from parent
|
||||
node[above] {$0.26$}
|
||||
}
|
||||
child {node {$B$}
|
||||
edge from parent
|
||||
node[above] {$0.74$}
|
||||
}
|
||||
edge from parent
|
||||
node[above] {$0.17$}
|
||||
} ;
|
||||
\end{tikzpicture}
|
||||
\end{center}
|
||||
\item On calcule la probabilité que la vente soit un wok et ait eu lieu à midi
|
||||
\[ P(M\cap W) = P(M) \times P_M(W) = 0.83 \times 0.75 = 0.6225 \]
|
||||
\item Probabilité que la vente soit un burger.
|
||||
\[
|
||||
P(B) = P(M\cap B) + P(S\cap B) = 0.83 \times 0.75 + 0.17 \times 0.26 = 0.3333
|
||||
\]
|
||||
\item On cherche à calculer la quantité $P_B(S)$. Pour cela on utilise la formule de Bayes
|
||||
\[
|
||||
P_B(S) = \frac{P(B\cap S)}{P(B)} = \frac{P_S(B) \times P(S)}{P(B)} = \frac{0.74\times 0.17}{0.3333} = 0.37743774377437744 \approx 0.377
|
||||
\]
|
||||
\end{enumerate}
|
||||
\end{solution}
|
||||
|
||||
\begin{exercise}[subtitle={Continent plastique}]
|
||||
\textit{Les quantités évoqués dans cette exercice sont générés au hasard et sont donc complètement farfelus.}
|
||||
\medskip
|
||||
Le \og continent de plastique\fg{} est la plus grande des plaques de déchets plastiques évoluant sur les océans. Elle occupe actuellement dans l'océan Pacifique une surface dont l'aire est évaluée à plus de $1,6$ million de km$^2$, entre Hawaï et la Californie.
|
||||
|
||||
En 2017, des scientifiques ont estimé qu'il y avait $18$ millions de tonnes de déchets plastiques qui était déversé chaque année dans les océans et que cette quantité augmentait de $14\n\%$ par chaque année.
|
||||
|
||||
On modélise l'évolution de la masse de ces déchets plastiques déversée chaque année, si rien n'est fait pour la réduire, par une suite géométrique $\left(u_n\right)$. L'arrondi au centième du terme $u_n$ représente la masse de ces déchets déversée chaque année, exprimée en million de tonnes, pour l'année $(2017 + n)$.
|
||||
|
||||
\medskip
|
||||
|
||||
\begin{enumerate}
|
||||
\item Expliquer pourquoi la suite $u_n$ est géométrique?
|
||||
\item Calculer $u_1$ et $u_2$.
|
||||
\item Exprimer $u_n$ en fonction de $n$.
|
||||
\item Au début de l'année 2017, il y avait $300$ millions de tonnes de déchets plastique. Calculer la quantité totale de déchets plastiques en 2030.
|
||||
\item On souhaite déterminer en quelle année la masse totale de ces déchets plastiques aura pour la première fois augmenté de $50$\,\% par rapport à sa valeur de 2017.
|
||||
\begin{enumerate}
|
||||
\item Recopier et compléter l'algorithme ci-dessous pour que la variable $N$ contienne la réponse au problème posé.
|
||||
|
||||
\begin{center}
|
||||
\begin{tabularx}{0.4\linewidth}{|X|}\hline
|
||||
$N = 2017$\\
|
||||
$U = 18$ \\
|
||||
$S = 300 + U$ \\
|
||||
while $S < 450$: \\
|
||||
\hspace{1cm} $N = \ldots$\\
|
||||
\hspace{1cm} $U = \ldots$\\
|
||||
\hspace{1cm} $S = \ldots$\\
|
||||
\hline
|
||||
\end{tabularx}
|
||||
\end{center}
|
||||
\item Que contiennent les variables $S$, $U$ et $N$ après exécution de cet algorithme ?
|
||||
|
||||
Interpréter les résultats dans le contexte de l'exercice.
|
||||
\end{enumerate}
|
||||
\end{enumerate}
|
||||
\end{exercise}
|
||||
|
||||
\begin{solution}
|
||||
\begin{enumerate}
|
||||
\item Une augmentation de $14\,\%$ revient à multiplier la quantité par $1.1400000000000001$. La suite est donc bien géométrique. Son premier terme est $u_0 = 18$ et sa raison est $q = 1.1400000000000001$
|
||||
\item
|
||||
\[
|
||||
u_1 = u_0 * 1.1400000000000001 = 20.520000000000003
|
||||
\]
|
||||
\[
|
||||
u_2 = u_0 * 1.1400000000000001^2 = 23.3928
|
||||
\]
|
||||
\item
|
||||
\[
|
||||
u_n = u_0 \times q^n = 18 \times 1.1400000000000001^n
|
||||
\]
|
||||
\item On calcule la quantité totale déversée entre 2017 et 2030.
|
||||
\[
|
||||
\sum_{n = 0}^{13} u_n = u_0 \times \frac{1-q^{13}}{1-q} = 18 \times \frac{1 - 1.1400000000000001^{13}}{1 - 1.1400000000000001} = 577.6
|
||||
\]
|
||||
On en déduit la quantité totale de déchets en 2030
|
||||
\[
|
||||
300 + 577.6 = 877.6
|
||||
\]
|
||||
\item
|
||||
\begin{enumerate}
|
||||
\item ~
|
||||
\begin{center}
|
||||
\begin{tabularx}{0.4\linewidth}{|X|}\hline
|
||||
$N \gets 2017$\\
|
||||
$U \gets 18$ \\
|
||||
$S \gets 300 + U$ \\
|
||||
Tant que $S < 450$ \\
|
||||
\hspace{1cm} $N \gets N + 1$\\
|
||||
\hspace{1cm} $U \gets U * 1.1400000000000001$\\
|
||||
\hspace{1cm} $S \gets S + u$\\
|
||||
Fin Tant que\\\hline
|
||||
\end{tabularx}
|
||||
\end{center}
|
||||
\item \textit{Pas de correction automatisé}
|
||||
\end{enumerate}
|
||||
\end{enumerate}
|
||||
\end{solution}
|
||||
|
||||
\end{document}
|
||||
|
||||
%%% Local Variables:
|
||||
%%% mode: latex
|
||||
%%% TeX-master: "master"
|
||||
%%% End:
|
262
TST/DS/DS_21_04_07/TST3/corr_05_210407_DS8.tex
Normal file
262
TST/DS/DS_21_04_07/TST3/corr_05_210407_DS8.tex
Normal file
@ -0,0 +1,262 @@
|
||||
\documentclass[a4paper,10pt]{article}
|
||||
\usepackage{myXsim}
|
||||
|
||||
% Title Page
|
||||
\title{DS8 \hfill BUDIN Nathan}
|
||||
\tribe{TST}
|
||||
\date{\hfillÀ render pour le Mercredi 7 avril}
|
||||
|
||||
\xsimsetup{
|
||||
solution/print = true
|
||||
}
|
||||
|
||||
\begin{document}
|
||||
\maketitle
|
||||
|
||||
\begin{exercise}[subtitle={Automatismes}]
|
||||
\textit{Toutes les questions de cette exercice sont indépendantes et peuvent être répondus séparément}
|
||||
\begin{enumerate}
|
||||
\item De janvier à septembre, une quantité a augmenté de $16\,\%$. Faire un schéma pour représenter la situation puis calculer le taux d'évolution moyen mensuel.
|
||||
\item Une quantité augmente de $16\,\%$ par ans. En 2020, elle est de 112\euro. Quelle était sa valeur en 2019? Faire un schéma pour représenter la situation.
|
||||
\item Déterminer l'équation de la droite \\
|
||||
\begin{tikzpicture}[xscale=0.8, yscale=0.5]
|
||||
\tkzInit[xmin=-5,xmax=5,xstep=1,
|
||||
ymin=-5,ymax=5,ystep=1]
|
||||
\tkzGrid
|
||||
\tkzAxeXY
|
||||
\tkzFct[domain=-5:5,color=red,very thick]%
|
||||
{1.0*\x -2};
|
||||
\end{tikzpicture}
|
||||
\item Résoudre l'équation $7 \times 0.22^x = 49$
|
||||
\end{enumerate}
|
||||
\end{exercise}
|
||||
|
||||
\begin{solution}
|
||||
\begin{enumerate}
|
||||
\item On veut partager cette évolution en 8 évolutions.
|
||||
\[
|
||||
\left(1 + \frac{16}{100}\right)^{\frac{1}{8}} = 1.0187
|
||||
\]
|
||||
Donc le taux d'évolution moyen est
|
||||
\[
|
||||
t_m = 1.0187 - 1 = 0.01869999999999994
|
||||
\]
|
||||
\item Coefficient multiplicateur pour revenir en arrière
|
||||
\[
|
||||
CM = (1 + \frac{16}{100})^{-1} = 0.8621
|
||||
\]
|
||||
On en déduit la quantité en 2019
|
||||
\[
|
||||
112 * 0.8621 = 96.5552
|
||||
\]
|
||||
\item L'équation de la droite est
|
||||
\[
|
||||
y = 1.0 x -2
|
||||
\]
|
||||
\item Il faut penser à faire la division à par $7$ avant d'utiliser le log car sinon, on ne peut pas utiliser la formule $\log(a^n) = n\times \log(a)$.
|
||||
|
||||
\[x = \frac{\log(7.0)}{\log(0.22)}\]
|
||||
\end{enumerate}
|
||||
\end{solution}
|
||||
|
||||
\begin{exercise}[subtitle={Restaurant}]
|
||||
Un \emph{food truck}, ouvert le midi et le soir, propose deux types de formules :
|
||||
|
||||
\setlength\parindent{10mm}
|
||||
\begin{itemize}
|
||||
\item la formule \emph{Burger} ;
|
||||
\item la formule \emph{Wok}.
|
||||
\end{itemize}
|
||||
\setlength\parindent{0mm}
|
||||
|
||||
\medskip
|
||||
|
||||
Le gérant a remarqué que 75\,\% de ses ventes ont lieu le midi. Le quart des ventes du midi correspondent à la formule \emph{Burger}, alors que 71\,\% des ventes du soir correspondent à la formule \emph{Wok}.
|
||||
|
||||
Le gérant se constitue un fichier en notant, pour chaque vente, la formule choisie et le moment de cette vente (midi ou soir).
|
||||
|
||||
On prélève une fiche de façon équiprobable. On définit les quatre évènements suivants:
|
||||
|
||||
\begin{enumerate}
|
||||
\item $M$ : \og la fiche correspond à une vente du midi\fg{} ;
|
||||
\item $S$ : \og la fiche correspond à une vente du soir\fg {};
|
||||
\item $W$ : \og la fiche correspond à une formule \emph{Wok} \fg{} ;
|
||||
\item $B$ : \og la fiche correspond à une formule \emph{Burger} \fg.
|
||||
\end{enumerate}
|
||||
\setlength\parindent{0mm}
|
||||
|
||||
\medskip
|
||||
|
||||
\begin{enumerate}
|
||||
\item Recopier puis compléter l'arbre pondéré
|
||||
|
||||
\begin{center}
|
||||
\begin{tikzpicture}[sloped]
|
||||
\node {.}
|
||||
child {node {$M$}
|
||||
child {node {$W$}
|
||||
edge from parent
|
||||
node[above] {...}
|
||||
}
|
||||
child {node {$B$}
|
||||
edge from parent
|
||||
node[above] {...}
|
||||
}
|
||||
edge from parent
|
||||
node[above] {...}
|
||||
}
|
||||
child[missing] {}
|
||||
child { node {$S$}
|
||||
child {node {$W$}
|
||||
edge from parent
|
||||
node[above] {...}
|
||||
}
|
||||
child {node {$B$}
|
||||
edge from parent
|
||||
node[above] {...}
|
||||
}
|
||||
edge from parent
|
||||
node[above] {...}
|
||||
} ;
|
||||
\end{tikzpicture}
|
||||
\end{center}
|
||||
|
||||
\item Calculer la probabilité de l'évènement $M \cap W$. Interpréter ce résultat dans le contexte de l'exercice.
|
||||
\item Montrer que la probabilité que la fiche choisie corresponde à une formule \emph{Burger} est égale à $0.26$.
|
||||
\item On a prélevé une fiche correspondant à la formule \emph{Burger}. Quelle est la probabilité, arrondie au millième, que la vente ait eu lieu le soir?
|
||||
\end{enumerate}
|
||||
\end{exercise}
|
||||
|
||||
\begin{solution}
|
||||
\begin{enumerate}
|
||||
\item
|
||||
\begin{center}
|
||||
\begin{tikzpicture}[sloped]
|
||||
\node {.}
|
||||
child {node {$M$}
|
||||
child {node {$W$}
|
||||
edge from parent
|
||||
node[above] {$0.75$}
|
||||
}
|
||||
child {node {$B$}
|
||||
edge from parent
|
||||
node[above] {$0.25$}
|
||||
}
|
||||
edge from parent
|
||||
node[above] {$0.75$}
|
||||
}
|
||||
child[missing] {}
|
||||
child { node {$S$}
|
||||
child {node {$W$}
|
||||
edge from parent
|
||||
node[above] {$0.71$}
|
||||
}
|
||||
child {node {$B$}
|
||||
edge from parent
|
||||
node[above] {$0.29$}
|
||||
}
|
||||
edge from parent
|
||||
node[above] {$0.25$}
|
||||
} ;
|
||||
\end{tikzpicture}
|
||||
\end{center}
|
||||
\item On calcule la probabilité que la vente soit un wok et ait eu lieu à midi
|
||||
\[ P(M\cap W) = P(M) \times P_M(W) = 0.75 \times 0.75 = 0.5625 \]
|
||||
\item Probabilité que la vente soit un burger.
|
||||
\[
|
||||
P(B) = P(M\cap B) + P(S\cap B) = 0.75 \times 0.75 + 0.25 \times 0.71 = 0.26
|
||||
\]
|
||||
\item On cherche à calculer la quantité $P_B(S)$. Pour cela on utilise la formule de Bayes
|
||||
\[
|
||||
P_B(S) = \frac{P(B\cap S)}{P(B)} = \frac{P_S(B) \times P(S)}{P(B)} = \frac{0.29\times 0.25}{0.26} = 0.2788461538461538 \approx 0.279
|
||||
\]
|
||||
\end{enumerate}
|
||||
\end{solution}
|
||||
|
||||
\begin{exercise}[subtitle={Continent plastique}]
|
||||
\textit{Les quantités évoqués dans cette exercice sont générés au hasard et sont donc complètement farfelus.}
|
||||
\medskip
|
||||
Le \og continent de plastique\fg{} est la plus grande des plaques de déchets plastiques évoluant sur les océans. Elle occupe actuellement dans l'océan Pacifique une surface dont l'aire est évaluée à plus de $1,6$ million de km$^2$, entre Hawaï et la Californie.
|
||||
|
||||
En 2017, des scientifiques ont estimé qu'il y avait $12$ millions de tonnes de déchets plastiques qui était déversé chaque année dans les océans et que cette quantité augmentait de $25\n\%$ par chaque année.
|
||||
|
||||
On modélise l'évolution de la masse de ces déchets plastiques déversée chaque année, si rien n'est fait pour la réduire, par une suite géométrique $\left(u_n\right)$. L'arrondi au centième du terme $u_n$ représente la masse de ces déchets déversée chaque année, exprimée en million de tonnes, pour l'année $(2017 + n)$.
|
||||
|
||||
\medskip
|
||||
|
||||
\begin{enumerate}
|
||||
\item Expliquer pourquoi la suite $u_n$ est géométrique?
|
||||
\item Calculer $u_1$ et $u_2$.
|
||||
\item Exprimer $u_n$ en fonction de $n$.
|
||||
\item Au début de l'année 2017, il y avait $300$ millions de tonnes de déchets plastique. Calculer la quantité totale de déchets plastiques en 2030.
|
||||
\item On souhaite déterminer en quelle année la masse totale de ces déchets plastiques aura pour la première fois augmenté de $50$\,\% par rapport à sa valeur de 2017.
|
||||
\begin{enumerate}
|
||||
\item Recopier et compléter l'algorithme ci-dessous pour que la variable $N$ contienne la réponse au problème posé.
|
||||
|
||||
\begin{center}
|
||||
\begin{tabularx}{0.4\linewidth}{|X|}\hline
|
||||
$N = 2017$\\
|
||||
$U = 12$ \\
|
||||
$S = 300 + U$ \\
|
||||
while $S < 450$: \\
|
||||
\hspace{1cm} $N = \ldots$\\
|
||||
\hspace{1cm} $U = \ldots$\\
|
||||
\hspace{1cm} $S = \ldots$\\
|
||||
\hline
|
||||
\end{tabularx}
|
||||
\end{center}
|
||||
\item Que contiennent les variables $S$, $U$ et $N$ après exécution de cet algorithme ?
|
||||
|
||||
Interpréter les résultats dans le contexte de l'exercice.
|
||||
\end{enumerate}
|
||||
\end{enumerate}
|
||||
\end{exercise}
|
||||
|
||||
\begin{solution}
|
||||
\begin{enumerate}
|
||||
\item Une augmentation de $25\,\%$ revient à multiplier la quantité par $1.25$. La suite est donc bien géométrique. Son premier terme est $u_0 = 12$ et sa raison est $q = 1.25$
|
||||
\item
|
||||
\[
|
||||
u_1 = u_0 * 1.25 = 15.0
|
||||
\]
|
||||
\[
|
||||
u_2 = u_0 * 1.25^2 = 18.75
|
||||
\]
|
||||
\item
|
||||
\[
|
||||
u_n = u_0 \times q^n = 12 \times 1.25^n
|
||||
\]
|
||||
\item On calcule la quantité totale déversée entre 2017 et 2030.
|
||||
\[
|
||||
\sum_{n = 0}^{13} u_n = u_0 \times \frac{1-q^{13}}{1-q} = 12 \times \frac{1 - 1.25^{13}}{1 - 1.25} = 825.11
|
||||
\]
|
||||
On en déduit la quantité totale de déchets en 2030
|
||||
\[
|
||||
300 + 825.11 = 1125.1100000000001
|
||||
\]
|
||||
\item
|
||||
\begin{enumerate}
|
||||
\item ~
|
||||
\begin{center}
|
||||
\begin{tabularx}{0.4\linewidth}{|X|}\hline
|
||||
$N \gets 2017$\\
|
||||
$U \gets 12$ \\
|
||||
$S \gets 300 + U$ \\
|
||||
Tant que $S < 450$ \\
|
||||
\hspace{1cm} $N \gets N + 1$\\
|
||||
\hspace{1cm} $U \gets U * 1.25$\\
|
||||
\hspace{1cm} $S \gets S + u$\\
|
||||
Fin Tant que\\\hline
|
||||
\end{tabularx}
|
||||
\end{center}
|
||||
\item \textit{Pas de correction automatisé}
|
||||
\end{enumerate}
|
||||
\end{enumerate}
|
||||
\end{solution}
|
||||
|
||||
\end{document}
|
||||
|
||||
%%% Local Variables:
|
||||
%%% mode: latex
|
||||
%%% TeX-master: "master"
|
||||
%%% End:
|
262
TST/DS/DS_21_04_07/TST3/corr_06_210407_DS8.tex
Normal file
262
TST/DS/DS_21_04_07/TST3/corr_06_210407_DS8.tex
Normal file
@ -0,0 +1,262 @@
|
||||
\documentclass[a4paper,10pt]{article}
|
||||
\usepackage{myXsim}
|
||||
|
||||
% Title Page
|
||||
\title{DS8 \hfill DARICHE Kaïs}
|
||||
\tribe{TST}
|
||||
\date{\hfillÀ render pour le Mercredi 7 avril}
|
||||
|
||||
\xsimsetup{
|
||||
solution/print = true
|
||||
}
|
||||
|
||||
\begin{document}
|
||||
\maketitle
|
||||
|
||||
\begin{exercise}[subtitle={Automatismes}]
|
||||
\textit{Toutes les questions de cette exercice sont indépendantes et peuvent être répondus séparément}
|
||||
\begin{enumerate}
|
||||
\item De janvier à septembre, une quantité a augmenté de $27\,\%$. Faire un schéma pour représenter la situation puis calculer le taux d'évolution moyen mensuel.
|
||||
\item Une quantité augmente de $27\,\%$ par ans. En 2020, elle est de 143\euro. Quelle était sa valeur en 2019? Faire un schéma pour représenter la situation.
|
||||
\item Déterminer l'équation de la droite \\
|
||||
\begin{tikzpicture}[xscale=0.8, yscale=0.5]
|
||||
\tkzInit[xmin=-5,xmax=5,xstep=1,
|
||||
ymin=-5,ymax=5,ystep=1]
|
||||
\tkzGrid
|
||||
\tkzAxeXY
|
||||
\tkzFct[domain=-5:5,color=red,very thick]%
|
||||
{3.0*\x -3};
|
||||
\end{tikzpicture}
|
||||
\item Résoudre l'équation $7 \times 0.19^x = 21$
|
||||
\end{enumerate}
|
||||
\end{exercise}
|
||||
|
||||
\begin{solution}
|
||||
\begin{enumerate}
|
||||
\item On veut partager cette évolution en 8 évolutions.
|
||||
\[
|
||||
\left(1 + \frac{27}{100}\right)^{\frac{1}{8}} = 1.0303
|
||||
\]
|
||||
Donc le taux d'évolution moyen est
|
||||
\[
|
||||
t_m = 1.0303 - 1 = 0.030299999999999994
|
||||
\]
|
||||
\item Coefficient multiplicateur pour revenir en arrière
|
||||
\[
|
||||
CM = (1 + \frac{27}{100})^{-1} = 0.7874
|
||||
\]
|
||||
On en déduit la quantité en 2019
|
||||
\[
|
||||
143 * 0.7874 = 112.59819999999999
|
||||
\]
|
||||
\item L'équation de la droite est
|
||||
\[
|
||||
y = 3.0 x -3
|
||||
\]
|
||||
\item Il faut penser à faire la division à par $7$ avant d'utiliser le log car sinon, on ne peut pas utiliser la formule $\log(a^n) = n\times \log(a)$.
|
||||
|
||||
\[x = \frac{\log(3.0)}{\log(0.19)}\]
|
||||
\end{enumerate}
|
||||
\end{solution}
|
||||
|
||||
\begin{exercise}[subtitle={Restaurant}]
|
||||
Un \emph{food truck}, ouvert le midi et le soir, propose deux types de formules :
|
||||
|
||||
\setlength\parindent{10mm}
|
||||
\begin{itemize}
|
||||
\item la formule \emph{Burger} ;
|
||||
\item la formule \emph{Wok}.
|
||||
\end{itemize}
|
||||
\setlength\parindent{0mm}
|
||||
|
||||
\medskip
|
||||
|
||||
Le gérant a remarqué que 15\,\% de ses ventes ont lieu le midi. Le quart des ventes du midi correspondent à la formule \emph{Burger}, alors que 13\,\% des ventes du soir correspondent à la formule \emph{Wok}.
|
||||
|
||||
Le gérant se constitue un fichier en notant, pour chaque vente, la formule choisie et le moment de cette vente (midi ou soir).
|
||||
|
||||
On prélève une fiche de façon équiprobable. On définit les quatre évènements suivants:
|
||||
|
||||
\begin{enumerate}
|
||||
\item $M$ : \og la fiche correspond à une vente du midi\fg{} ;
|
||||
\item $S$ : \og la fiche correspond à une vente du soir\fg {};
|
||||
\item $W$ : \og la fiche correspond à une formule \emph{Wok} \fg{} ;
|
||||
\item $B$ : \og la fiche correspond à une formule \emph{Burger} \fg.
|
||||
\end{enumerate}
|
||||
\setlength\parindent{0mm}
|
||||
|
||||
\medskip
|
||||
|
||||
\begin{enumerate}
|
||||
\item Recopier puis compléter l'arbre pondéré
|
||||
|
||||
\begin{center}
|
||||
\begin{tikzpicture}[sloped]
|
||||
\node {.}
|
||||
child {node {$M$}
|
||||
child {node {$W$}
|
||||
edge from parent
|
||||
node[above] {...}
|
||||
}
|
||||
child {node {$B$}
|
||||
edge from parent
|
||||
node[above] {...}
|
||||
}
|
||||
edge from parent
|
||||
node[above] {...}
|
||||
}
|
||||
child[missing] {}
|
||||
child { node {$S$}
|
||||
child {node {$W$}
|
||||
edge from parent
|
||||
node[above] {...}
|
||||
}
|
||||
child {node {$B$}
|
||||
edge from parent
|
||||
node[above] {...}
|
||||
}
|
||||
edge from parent
|
||||
node[above] {...}
|
||||
} ;
|
||||
\end{tikzpicture}
|
||||
\end{center}
|
||||
|
||||
\item Calculer la probabilité de l'évènement $M \cap W$. Interpréter ce résultat dans le contexte de l'exercice.
|
||||
\item Montrer que la probabilité que la fiche choisie corresponde à une formule \emph{Burger} est égale à $0.777$.
|
||||
\item On a prélevé une fiche correspondant à la formule \emph{Burger}. Quelle est la probabilité, arrondie au millième, que la vente ait eu lieu le soir?
|
||||
\end{enumerate}
|
||||
\end{exercise}
|
||||
|
||||
\begin{solution}
|
||||
\begin{enumerate}
|
||||
\item
|
||||
\begin{center}
|
||||
\begin{tikzpicture}[sloped]
|
||||
\node {.}
|
||||
child {node {$M$}
|
||||
child {node {$W$}
|
||||
edge from parent
|
||||
node[above] {$0.75$}
|
||||
}
|
||||
child {node {$B$}
|
||||
edge from parent
|
||||
node[above] {$0.25$}
|
||||
}
|
||||
edge from parent
|
||||
node[above] {$0.15$}
|
||||
}
|
||||
child[missing] {}
|
||||
child { node {$S$}
|
||||
child {node {$W$}
|
||||
edge from parent
|
||||
node[above] {$0.13$}
|
||||
}
|
||||
child {node {$B$}
|
||||
edge from parent
|
||||
node[above] {$0.87$}
|
||||
}
|
||||
edge from parent
|
||||
node[above] {$0.85$}
|
||||
} ;
|
||||
\end{tikzpicture}
|
||||
\end{center}
|
||||
\item On calcule la probabilité que la vente soit un wok et ait eu lieu à midi
|
||||
\[ P(M\cap W) = P(M) \times P_M(W) = 0.15 \times 0.75 = 0.1125 \]
|
||||
\item Probabilité que la vente soit un burger.
|
||||
\[
|
||||
P(B) = P(M\cap B) + P(S\cap B) = 0.15 \times 0.75 + 0.85 \times 0.13 = 0.777
|
||||
\]
|
||||
\item On cherche à calculer la quantité $P_B(S)$. Pour cela on utilise la formule de Bayes
|
||||
\[
|
||||
P_B(S) = \frac{P(B\cap S)}{P(B)} = \frac{P_S(B) \times P(S)}{P(B)} = \frac{0.87\times 0.85}{0.777} = 0.9517374517374516 \approx 0.952
|
||||
\]
|
||||
\end{enumerate}
|
||||
\end{solution}
|
||||
|
||||
\begin{exercise}[subtitle={Continent plastique}]
|
||||
\textit{Les quantités évoqués dans cette exercice sont générés au hasard et sont donc complètement farfelus.}
|
||||
\medskip
|
||||
Le \og continent de plastique\fg{} est la plus grande des plaques de déchets plastiques évoluant sur les océans. Elle occupe actuellement dans l'océan Pacifique une surface dont l'aire est évaluée à plus de $1,6$ million de km$^2$, entre Hawaï et la Californie.
|
||||
|
||||
En 2017, des scientifiques ont estimé qu'il y avait $2$ millions de tonnes de déchets plastiques qui était déversé chaque année dans les océans et que cette quantité augmentait de $30\n\%$ par chaque année.
|
||||
|
||||
On modélise l'évolution de la masse de ces déchets plastiques déversée chaque année, si rien n'est fait pour la réduire, par une suite géométrique $\left(u_n\right)$. L'arrondi au centième du terme $u_n$ représente la masse de ces déchets déversée chaque année, exprimée en million de tonnes, pour l'année $(2017 + n)$.
|
||||
|
||||
\medskip
|
||||
|
||||
\begin{enumerate}
|
||||
\item Expliquer pourquoi la suite $u_n$ est géométrique?
|
||||
\item Calculer $u_1$ et $u_2$.
|
||||
\item Exprimer $u_n$ en fonction de $n$.
|
||||
\item Au début de l'année 2017, il y avait $300$ millions de tonnes de déchets plastique. Calculer la quantité totale de déchets plastiques en 2030.
|
||||
\item On souhaite déterminer en quelle année la masse totale de ces déchets plastiques aura pour la première fois augmenté de $50$\,\% par rapport à sa valeur de 2017.
|
||||
\begin{enumerate}
|
||||
\item Recopier et compléter l'algorithme ci-dessous pour que la variable $N$ contienne la réponse au problème posé.
|
||||
|
||||
\begin{center}
|
||||
\begin{tabularx}{0.4\linewidth}{|X|}\hline
|
||||
$N = 2017$\\
|
||||
$U = 2$ \\
|
||||
$S = 300 + U$ \\
|
||||
while $S < 450$: \\
|
||||
\hspace{1cm} $N = \ldots$\\
|
||||
\hspace{1cm} $U = \ldots$\\
|
||||
\hspace{1cm} $S = \ldots$\\
|
||||
\hline
|
||||
\end{tabularx}
|
||||
\end{center}
|
||||
\item Que contiennent les variables $S$, $U$ et $N$ après exécution de cet algorithme ?
|
||||
|
||||
Interpréter les résultats dans le contexte de l'exercice.
|
||||
\end{enumerate}
|
||||
\end{enumerate}
|
||||
\end{exercise}
|
||||
|
||||
\begin{solution}
|
||||
\begin{enumerate}
|
||||
\item Une augmentation de $30\,\%$ revient à multiplier la quantité par $1.3$. La suite est donc bien géométrique. Son premier terme est $u_0 = 2$ et sa raison est $q = 1.3$
|
||||
\item
|
||||
\[
|
||||
u_1 = u_0 * 1.3 = 2.6
|
||||
\]
|
||||
\[
|
||||
u_2 = u_0 * 1.3^2 = 3.38
|
||||
\]
|
||||
\item
|
||||
\[
|
||||
u_n = u_0 \times q^n = 2 \times 1.3^n
|
||||
\]
|
||||
\item On calcule la quantité totale déversée entre 2017 et 2030.
|
||||
\[
|
||||
\sum_{n = 0}^{13} u_n = u_0 \times \frac{1-q^{13}}{1-q} = 2 \times \frac{1 - 1.3^{13}}{1 - 1.3} = 195.25
|
||||
\]
|
||||
On en déduit la quantité totale de déchets en 2030
|
||||
\[
|
||||
300 + 195.25 = 495.25
|
||||
\]
|
||||
\item
|
||||
\begin{enumerate}
|
||||
\item ~
|
||||
\begin{center}
|
||||
\begin{tabularx}{0.4\linewidth}{|X|}\hline
|
||||
$N \gets 2017$\\
|
||||
$U \gets 2$ \\
|
||||
$S \gets 300 + U$ \\
|
||||
Tant que $S < 450$ \\
|
||||
\hspace{1cm} $N \gets N + 1$\\
|
||||
\hspace{1cm} $U \gets U * 1.3$\\
|
||||
\hspace{1cm} $S \gets S + u$\\
|
||||
Fin Tant que\\\hline
|
||||
\end{tabularx}
|
||||
\end{center}
|
||||
\item \textit{Pas de correction automatisé}
|
||||
\end{enumerate}
|
||||
\end{enumerate}
|
||||
\end{solution}
|
||||
|
||||
\end{document}
|
||||
|
||||
%%% Local Variables:
|
||||
%%% mode: latex
|
||||
%%% TeX-master: "master"
|
||||
%%% End:
|
262
TST/DS/DS_21_04_07/TST3/corr_07_210407_DS8.tex
Normal file
262
TST/DS/DS_21_04_07/TST3/corr_07_210407_DS8.tex
Normal file
@ -0,0 +1,262 @@
|
||||
\documentclass[a4paper,10pt]{article}
|
||||
\usepackage{myXsim}
|
||||
|
||||
% Title Page
|
||||
\title{DS8 \hfill DEBRAS Noémie}
|
||||
\tribe{TST}
|
||||
\date{\hfillÀ render pour le Mercredi 7 avril}
|
||||
|
||||
\xsimsetup{
|
||||
solution/print = true
|
||||
}
|
||||
|
||||
\begin{document}
|
||||
\maketitle
|
||||
|
||||
\begin{exercise}[subtitle={Automatismes}]
|
||||
\textit{Toutes les questions de cette exercice sont indépendantes et peuvent être répondus séparément}
|
||||
\begin{enumerate}
|
||||
\item De janvier à septembre, une quantité a augmenté de $24\,\%$. Faire un schéma pour représenter la situation puis calculer le taux d'évolution moyen mensuel.
|
||||
\item Une quantité augmente de $24\,\%$ par ans. En 2020, elle est de 110\euro. Quelle était sa valeur en 2019? Faire un schéma pour représenter la situation.
|
||||
\item Déterminer l'équation de la droite \\
|
||||
\begin{tikzpicture}[xscale=0.8, yscale=0.5]
|
||||
\tkzInit[xmin=-5,xmax=5,xstep=1,
|
||||
ymin=-5,ymax=5,ystep=1]
|
||||
\tkzGrid
|
||||
\tkzAxeXY
|
||||
\tkzFct[domain=-5:5,color=red,very thick]%
|
||||
{0.6666666666666666*\x -1};
|
||||
\end{tikzpicture}
|
||||
\item Résoudre l'équation $8 \times 0.49^x = 9$
|
||||
\end{enumerate}
|
||||
\end{exercise}
|
||||
|
||||
\begin{solution}
|
||||
\begin{enumerate}
|
||||
\item On veut partager cette évolution en 8 évolutions.
|
||||
\[
|
||||
\left(1 + \frac{24}{100}\right)^{\frac{1}{8}} = 1.0273
|
||||
\]
|
||||
Donc le taux d'évolution moyen est
|
||||
\[
|
||||
t_m = 1.0273 - 1 = 0.027300000000000102
|
||||
\]
|
||||
\item Coefficient multiplicateur pour revenir en arrière
|
||||
\[
|
||||
CM = (1 + \frac{24}{100})^{-1} = 0.8065
|
||||
\]
|
||||
On en déduit la quantité en 2019
|
||||
\[
|
||||
110 * 0.8065 = 88.715
|
||||
\]
|
||||
\item L'équation de la droite est
|
||||
\[
|
||||
y = 0.6666666666666666 x -1
|
||||
\]
|
||||
\item Il faut penser à faire la division à par $8$ avant d'utiliser le log car sinon, on ne peut pas utiliser la formule $\log(a^n) = n\times \log(a)$.
|
||||
|
||||
\[x = \frac{\log(1.12)}{\log(0.49)}\]
|
||||
\end{enumerate}
|
||||
\end{solution}
|
||||
|
||||
\begin{exercise}[subtitle={Restaurant}]
|
||||
Un \emph{food truck}, ouvert le midi et le soir, propose deux types de formules :
|
||||
|
||||
\setlength\parindent{10mm}
|
||||
\begin{itemize}
|
||||
\item la formule \emph{Burger} ;
|
||||
\item la formule \emph{Wok}.
|
||||
\end{itemize}
|
||||
\setlength\parindent{0mm}
|
||||
|
||||
\medskip
|
||||
|
||||
Le gérant a remarqué que 49\,\% de ses ventes ont lieu le midi. Le quart des ventes du midi correspondent à la formule \emph{Burger}, alors que 61\,\% des ventes du soir correspondent à la formule \emph{Wok}.
|
||||
|
||||
Le gérant se constitue un fichier en notant, pour chaque vente, la formule choisie et le moment de cette vente (midi ou soir).
|
||||
|
||||
On prélève une fiche de façon équiprobable. On définit les quatre évènements suivants:
|
||||
|
||||
\begin{enumerate}
|
||||
\item $M$ : \og la fiche correspond à une vente du midi\fg{} ;
|
||||
\item $S$ : \og la fiche correspond à une vente du soir\fg {};
|
||||
\item $W$ : \og la fiche correspond à une formule \emph{Wok} \fg{} ;
|
||||
\item $B$ : \og la fiche correspond à une formule \emph{Burger} \fg.
|
||||
\end{enumerate}
|
||||
\setlength\parindent{0mm}
|
||||
|
||||
\medskip
|
||||
|
||||
\begin{enumerate}
|
||||
\item Recopier puis compléter l'arbre pondéré
|
||||
|
||||
\begin{center}
|
||||
\begin{tikzpicture}[sloped]
|
||||
\node {.}
|
||||
child {node {$M$}
|
||||
child {node {$W$}
|
||||
edge from parent
|
||||
node[above] {...}
|
||||
}
|
||||
child {node {$B$}
|
||||
edge from parent
|
||||
node[above] {...}
|
||||
}
|
||||
edge from parent
|
||||
node[above] {...}
|
||||
}
|
||||
child[missing] {}
|
||||
child { node {$S$}
|
||||
child {node {$W$}
|
||||
edge from parent
|
||||
node[above] {...}
|
||||
}
|
||||
child {node {$B$}
|
||||
edge from parent
|
||||
node[above] {...}
|
||||
}
|
||||
edge from parent
|
||||
node[above] {...}
|
||||
} ;
|
||||
\end{tikzpicture}
|
||||
\end{center}
|
||||
|
||||
\item Calculer la probabilité de l'évènement $M \cap W$. Interpréter ce résultat dans le contexte de l'exercice.
|
||||
\item Montrer que la probabilité que la fiche choisie corresponde à une formule \emph{Burger} est égale à $0.3214$.
|
||||
\item On a prélevé une fiche correspondant à la formule \emph{Burger}. Quelle est la probabilité, arrondie au millième, que la vente ait eu lieu le soir?
|
||||
\end{enumerate}
|
||||
\end{exercise}
|
||||
|
||||
\begin{solution}
|
||||
\begin{enumerate}
|
||||
\item
|
||||
\begin{center}
|
||||
\begin{tikzpicture}[sloped]
|
||||
\node {.}
|
||||
child {node {$M$}
|
||||
child {node {$W$}
|
||||
edge from parent
|
||||
node[above] {$0.75$}
|
||||
}
|
||||
child {node {$B$}
|
||||
edge from parent
|
||||
node[above] {$0.25$}
|
||||
}
|
||||
edge from parent
|
||||
node[above] {$0.49$}
|
||||
}
|
||||
child[missing] {}
|
||||
child { node {$S$}
|
||||
child {node {$W$}
|
||||
edge from parent
|
||||
node[above] {$0.61$}
|
||||
}
|
||||
child {node {$B$}
|
||||
edge from parent
|
||||
node[above] {$0.39$}
|
||||
}
|
||||
edge from parent
|
||||
node[above] {$0.51$}
|
||||
} ;
|
||||
\end{tikzpicture}
|
||||
\end{center}
|
||||
\item On calcule la probabilité que la vente soit un wok et ait eu lieu à midi
|
||||
\[ P(M\cap W) = P(M) \times P_M(W) = 0.49 \times 0.75 = 0.3675 \]
|
||||
\item Probabilité que la vente soit un burger.
|
||||
\[
|
||||
P(B) = P(M\cap B) + P(S\cap B) = 0.49 \times 0.75 + 0.51 \times 0.61 = 0.3214
|
||||
\]
|
||||
\item On cherche à calculer la quantité $P_B(S)$. Pour cela on utilise la formule de Bayes
|
||||
\[
|
||||
P_B(S) = \frac{P(B\cap S)}{P(B)} = \frac{P_S(B) \times P(S)}{P(B)} = \frac{0.39\times 0.51}{0.3214} = 0.618855009334163 \approx 0.619
|
||||
\]
|
||||
\end{enumerate}
|
||||
\end{solution}
|
||||
|
||||
\begin{exercise}[subtitle={Continent plastique}]
|
||||
\textit{Les quantités évoqués dans cette exercice sont générés au hasard et sont donc complètement farfelus.}
|
||||
\medskip
|
||||
Le \og continent de plastique\fg{} est la plus grande des plaques de déchets plastiques évoluant sur les océans. Elle occupe actuellement dans l'océan Pacifique une surface dont l'aire est évaluée à plus de $1,6$ million de km$^2$, entre Hawaï et la Californie.
|
||||
|
||||
En 2017, des scientifiques ont estimé qu'il y avait $15$ millions de tonnes de déchets plastiques qui était déversé chaque année dans les océans et que cette quantité augmentait de $21\n\%$ par chaque année.
|
||||
|
||||
On modélise l'évolution de la masse de ces déchets plastiques déversée chaque année, si rien n'est fait pour la réduire, par une suite géométrique $\left(u_n\right)$. L'arrondi au centième du terme $u_n$ représente la masse de ces déchets déversée chaque année, exprimée en million de tonnes, pour l'année $(2017 + n)$.
|
||||
|
||||
\medskip
|
||||
|
||||
\begin{enumerate}
|
||||
\item Expliquer pourquoi la suite $u_n$ est géométrique?
|
||||
\item Calculer $u_1$ et $u_2$.
|
||||
\item Exprimer $u_n$ en fonction de $n$.
|
||||
\item Au début de l'année 2017, il y avait $300$ millions de tonnes de déchets plastique. Calculer la quantité totale de déchets plastiques en 2030.
|
||||
\item On souhaite déterminer en quelle année la masse totale de ces déchets plastiques aura pour la première fois augmenté de $50$\,\% par rapport à sa valeur de 2017.
|
||||
\begin{enumerate}
|
||||
\item Recopier et compléter l'algorithme ci-dessous pour que la variable $N$ contienne la réponse au problème posé.
|
||||
|
||||
\begin{center}
|
||||
\begin{tabularx}{0.4\linewidth}{|X|}\hline
|
||||
$N = 2017$\\
|
||||
$U = 15$ \\
|
||||
$S = 300 + U$ \\
|
||||
while $S < 450$: \\
|
||||
\hspace{1cm} $N = \ldots$\\
|
||||
\hspace{1cm} $U = \ldots$\\
|
||||
\hspace{1cm} $S = \ldots$\\
|
||||
\hline
|
||||
\end{tabularx}
|
||||
\end{center}
|
||||
\item Que contiennent les variables $S$, $U$ et $N$ après exécution de cet algorithme ?
|
||||
|
||||
Interpréter les résultats dans le contexte de l'exercice.
|
||||
\end{enumerate}
|
||||
\end{enumerate}
|
||||
\end{exercise}
|
||||
|
||||
\begin{solution}
|
||||
\begin{enumerate}
|
||||
\item Une augmentation de $21\,\%$ revient à multiplier la quantité par $1.21$. La suite est donc bien géométrique. Son premier terme est $u_0 = 15$ et sa raison est $q = 1.21$
|
||||
\item
|
||||
\[
|
||||
u_1 = u_0 * 1.21 = 18.15
|
||||
\]
|
||||
\[
|
||||
u_2 = u_0 * 1.21^2 = 21.9615
|
||||
\]
|
||||
\item
|
||||
\[
|
||||
u_n = u_0 \times q^n = 15 \times 1.21^n
|
||||
\]
|
||||
\item On calcule la quantité totale déversée entre 2017 et 2030.
|
||||
\[
|
||||
\sum_{n = 0}^{13} u_n = u_0 \times \frac{1-q^{13}}{1-q} = 15 \times \frac{1 - 1.21^{13}}{1 - 1.21} = 779.87
|
||||
\]
|
||||
On en déduit la quantité totale de déchets en 2030
|
||||
\[
|
||||
300 + 779.87 = 1079.87
|
||||
\]
|
||||
\item
|
||||
\begin{enumerate}
|
||||
\item ~
|
||||
\begin{center}
|
||||
\begin{tabularx}{0.4\linewidth}{|X|}\hline
|
||||
$N \gets 2017$\\
|
||||
$U \gets 15$ \\
|
||||
$S \gets 300 + U$ \\
|
||||
Tant que $S < 450$ \\
|
||||
\hspace{1cm} $N \gets N + 1$\\
|
||||
\hspace{1cm} $U \gets U * 1.21$\\
|
||||
\hspace{1cm} $S \gets S + u$\\
|
||||
Fin Tant que\\\hline
|
||||
\end{tabularx}
|
||||
\end{center}
|
||||
\item \textit{Pas de correction automatisé}
|
||||
\end{enumerate}
|
||||
\end{enumerate}
|
||||
\end{solution}
|
||||
|
||||
\end{document}
|
||||
|
||||
%%% Local Variables:
|
||||
%%% mode: latex
|
||||
%%% TeX-master: "master"
|
||||
%%% End:
|
262
TST/DS/DS_21_04_07/TST3/corr_08_210407_DS8.tex
Normal file
262
TST/DS/DS_21_04_07/TST3/corr_08_210407_DS8.tex
Normal file
@ -0,0 +1,262 @@
|
||||
\documentclass[a4paper,10pt]{article}
|
||||
\usepackage{myXsim}
|
||||
|
||||
% Title Page
|
||||
\title{DS8 \hfill GERMAIN Anaïs}
|
||||
\tribe{TST}
|
||||
\date{\hfillÀ render pour le Mercredi 7 avril}
|
||||
|
||||
\xsimsetup{
|
||||
solution/print = true
|
||||
}
|
||||
|
||||
\begin{document}
|
||||
\maketitle
|
||||
|
||||
\begin{exercise}[subtitle={Automatismes}]
|
||||
\textit{Toutes les questions de cette exercice sont indépendantes et peuvent être répondus séparément}
|
||||
\begin{enumerate}
|
||||
\item De janvier à septembre, une quantité a augmenté de $27\,\%$. Faire un schéma pour représenter la situation puis calculer le taux d'évolution moyen mensuel.
|
||||
\item Une quantité augmente de $27\,\%$ par ans. En 2020, elle est de 132\euro. Quelle était sa valeur en 2019? Faire un schéma pour représenter la situation.
|
||||
\item Déterminer l'équation de la droite \\
|
||||
\begin{tikzpicture}[xscale=0.8, yscale=0.5]
|
||||
\tkzInit[xmin=-5,xmax=5,xstep=1,
|
||||
ymin=-5,ymax=5,ystep=1]
|
||||
\tkzGrid
|
||||
\tkzAxeXY
|
||||
\tkzFct[domain=-5:5,color=red,very thick]%
|
||||
{2.0*\x -4};
|
||||
\end{tikzpicture}
|
||||
\item Résoudre l'équation $7 \times 0.05^x = 18$
|
||||
\end{enumerate}
|
||||
\end{exercise}
|
||||
|
||||
\begin{solution}
|
||||
\begin{enumerate}
|
||||
\item On veut partager cette évolution en 8 évolutions.
|
||||
\[
|
||||
\left(1 + \frac{27}{100}\right)^{\frac{1}{8}} = 1.0303
|
||||
\]
|
||||
Donc le taux d'évolution moyen est
|
||||
\[
|
||||
t_m = 1.0303 - 1 = 0.030299999999999994
|
||||
\]
|
||||
\item Coefficient multiplicateur pour revenir en arrière
|
||||
\[
|
||||
CM = (1 + \frac{27}{100})^{-1} = 0.7874
|
||||
\]
|
||||
On en déduit la quantité en 2019
|
||||
\[
|
||||
132 * 0.7874 = 103.9368
|
||||
\]
|
||||
\item L'équation de la droite est
|
||||
\[
|
||||
y = 2.0 x -4
|
||||
\]
|
||||
\item Il faut penser à faire la division à par $7$ avant d'utiliser le log car sinon, on ne peut pas utiliser la formule $\log(a^n) = n\times \log(a)$.
|
||||
|
||||
\[x = \frac{\log(2.57)}{\log(0.05)}\]
|
||||
\end{enumerate}
|
||||
\end{solution}
|
||||
|
||||
\begin{exercise}[subtitle={Restaurant}]
|
||||
Un \emph{food truck}, ouvert le midi et le soir, propose deux types de formules :
|
||||
|
||||
\setlength\parindent{10mm}
|
||||
\begin{itemize}
|
||||
\item la formule \emph{Burger} ;
|
||||
\item la formule \emph{Wok}.
|
||||
\end{itemize}
|
||||
\setlength\parindent{0mm}
|
||||
|
||||
\medskip
|
||||
|
||||
Le gérant a remarqué que 38\,\% de ses ventes ont lieu le midi. Le quart des ventes du midi correspondent à la formule \emph{Burger}, alors que 63\,\% des ventes du soir correspondent à la formule \emph{Wok}.
|
||||
|
||||
Le gérant se constitue un fichier en notant, pour chaque vente, la formule choisie et le moment de cette vente (midi ou soir).
|
||||
|
||||
On prélève une fiche de façon équiprobable. On définit les quatre évènements suivants:
|
||||
|
||||
\begin{enumerate}
|
||||
\item $M$ : \og la fiche correspond à une vente du midi\fg{} ;
|
||||
\item $S$ : \og la fiche correspond à une vente du soir\fg {};
|
||||
\item $W$ : \og la fiche correspond à une formule \emph{Wok} \fg{} ;
|
||||
\item $B$ : \og la fiche correspond à une formule \emph{Burger} \fg.
|
||||
\end{enumerate}
|
||||
\setlength\parindent{0mm}
|
||||
|
||||
\medskip
|
||||
|
||||
\begin{enumerate}
|
||||
\item Recopier puis compléter l'arbre pondéré
|
||||
|
||||
\begin{center}
|
||||
\begin{tikzpicture}[sloped]
|
||||
\node {.}
|
||||
child {node {$M$}
|
||||
child {node {$W$}
|
||||
edge from parent
|
||||
node[above] {...}
|
||||
}
|
||||
child {node {$B$}
|
||||
edge from parent
|
||||
node[above] {...}
|
||||
}
|
||||
edge from parent
|
||||
node[above] {...}
|
||||
}
|
||||
child[missing] {}
|
||||
child { node {$S$}
|
||||
child {node {$W$}
|
||||
edge from parent
|
||||
node[above] {...}
|
||||
}
|
||||
child {node {$B$}
|
||||
edge from parent
|
||||
node[above] {...}
|
||||
}
|
||||
edge from parent
|
||||
node[above] {...}
|
||||
} ;
|
||||
\end{tikzpicture}
|
||||
\end{center}
|
||||
|
||||
\item Calculer la probabilité de l'évènement $M \cap W$. Interpréter ce résultat dans le contexte de l'exercice.
|
||||
\item Montrer que la probabilité que la fiche choisie corresponde à une formule \emph{Burger} est égale à $0.3244$.
|
||||
\item On a prélevé une fiche correspondant à la formule \emph{Burger}. Quelle est la probabilité, arrondie au millième, que la vente ait eu lieu le soir?
|
||||
\end{enumerate}
|
||||
\end{exercise}
|
||||
|
||||
\begin{solution}
|
||||
\begin{enumerate}
|
||||
\item
|
||||
\begin{center}
|
||||
\begin{tikzpicture}[sloped]
|
||||
\node {.}
|
||||
child {node {$M$}
|
||||
child {node {$W$}
|
||||
edge from parent
|
||||
node[above] {$0.75$}
|
||||
}
|
||||
child {node {$B$}
|
||||
edge from parent
|
||||
node[above] {$0.25$}
|
||||
}
|
||||
edge from parent
|
||||
node[above] {$0.38$}
|
||||
}
|
||||
child[missing] {}
|
||||
child { node {$S$}
|
||||
child {node {$W$}
|
||||
edge from parent
|
||||
node[above] {$0.63$}
|
||||
}
|
||||
child {node {$B$}
|
||||
edge from parent
|
||||
node[above] {$0.37$}
|
||||
}
|
||||
edge from parent
|
||||
node[above] {$0.62$}
|
||||
} ;
|
||||
\end{tikzpicture}
|
||||
\end{center}
|
||||
\item On calcule la probabilité que la vente soit un wok et ait eu lieu à midi
|
||||
\[ P(M\cap W) = P(M) \times P_M(W) = 0.38 \times 0.75 = 0.285 \]
|
||||
\item Probabilité que la vente soit un burger.
|
||||
\[
|
||||
P(B) = P(M\cap B) + P(S\cap B) = 0.38 \times 0.75 + 0.62 \times 0.63 = 0.3244
|
||||
\]
|
||||
\item On cherche à calculer la quantité $P_B(S)$. Pour cela on utilise la formule de Bayes
|
||||
\[
|
||||
P_B(S) = \frac{P(B\cap S)}{P(B)} = \frac{P_S(B) \times P(S)}{P(B)} = \frac{0.37\times 0.62}{0.3244} = 0.7071516646115905 \approx 0.707
|
||||
\]
|
||||
\end{enumerate}
|
||||
\end{solution}
|
||||
|
||||
\begin{exercise}[subtitle={Continent plastique}]
|
||||
\textit{Les quantités évoqués dans cette exercice sont générés au hasard et sont donc complètement farfelus.}
|
||||
\medskip
|
||||
Le \og continent de plastique\fg{} est la plus grande des plaques de déchets plastiques évoluant sur les océans. Elle occupe actuellement dans l'océan Pacifique une surface dont l'aire est évaluée à plus de $1,6$ million de km$^2$, entre Hawaï et la Californie.
|
||||
|
||||
En 2017, des scientifiques ont estimé qu'il y avait $18$ millions de tonnes de déchets plastiques qui était déversé chaque année dans les océans et que cette quantité augmentait de $28\n\%$ par chaque année.
|
||||
|
||||
On modélise l'évolution de la masse de ces déchets plastiques déversée chaque année, si rien n'est fait pour la réduire, par une suite géométrique $\left(u_n\right)$. L'arrondi au centième du terme $u_n$ représente la masse de ces déchets déversée chaque année, exprimée en million de tonnes, pour l'année $(2017 + n)$.
|
||||
|
||||
\medskip
|
||||
|
||||
\begin{enumerate}
|
||||
\item Expliquer pourquoi la suite $u_n$ est géométrique?
|
||||
\item Calculer $u_1$ et $u_2$.
|
||||
\item Exprimer $u_n$ en fonction de $n$.
|
||||
\item Au début de l'année 2017, il y avait $300$ millions de tonnes de déchets plastique. Calculer la quantité totale de déchets plastiques en 2030.
|
||||
\item On souhaite déterminer en quelle année la masse totale de ces déchets plastiques aura pour la première fois augmenté de $50$\,\% par rapport à sa valeur de 2017.
|
||||
\begin{enumerate}
|
||||
\item Recopier et compléter l'algorithme ci-dessous pour que la variable $N$ contienne la réponse au problème posé.
|
||||
|
||||
\begin{center}
|
||||
\begin{tabularx}{0.4\linewidth}{|X|}\hline
|
||||
$N = 2017$\\
|
||||
$U = 18$ \\
|
||||
$S = 300 + U$ \\
|
||||
while $S < 450$: \\
|
||||
\hspace{1cm} $N = \ldots$\\
|
||||
\hspace{1cm} $U = \ldots$\\
|
||||
\hspace{1cm} $S = \ldots$\\
|
||||
\hline
|
||||
\end{tabularx}
|
||||
\end{center}
|
||||
\item Que contiennent les variables $S$, $U$ et $N$ après exécution de cet algorithme ?
|
||||
|
||||
Interpréter les résultats dans le contexte de l'exercice.
|
||||
\end{enumerate}
|
||||
\end{enumerate}
|
||||
\end{exercise}
|
||||
|
||||
\begin{solution}
|
||||
\begin{enumerate}
|
||||
\item Une augmentation de $28\,\%$ revient à multiplier la quantité par $1.28$. La suite est donc bien géométrique. Son premier terme est $u_0 = 18$ et sa raison est $q = 1.28$
|
||||
\item
|
||||
\[
|
||||
u_1 = u_0 * 1.28 = 23.04
|
||||
\]
|
||||
\[
|
||||
u_2 = u_0 * 1.28^2 = 29.4912
|
||||
\]
|
||||
\item
|
||||
\[
|
||||
u_n = u_0 \times q^n = 18 \times 1.28^n
|
||||
\]
|
||||
\item On calcule la quantité totale déversée entre 2017 et 2030.
|
||||
\[
|
||||
\sum_{n = 0}^{13} u_n = u_0 \times \frac{1-q^{13}}{1-q} = 18 \times \frac{1 - 1.28^{13}}{1 - 1.28} = 1527.35
|
||||
\]
|
||||
On en déduit la quantité totale de déchets en 2030
|
||||
\[
|
||||
300 + 1527.35 = 1827.35
|
||||
\]
|
||||
\item
|
||||
\begin{enumerate}
|
||||
\item ~
|
||||
\begin{center}
|
||||
\begin{tabularx}{0.4\linewidth}{|X|}\hline
|
||||
$N \gets 2017$\\
|
||||
$U \gets 18$ \\
|
||||
$S \gets 300 + U$ \\
|
||||
Tant que $S < 450$ \\
|
||||
\hspace{1cm} $N \gets N + 1$\\
|
||||
\hspace{1cm} $U \gets U * 1.28$\\
|
||||
\hspace{1cm} $S \gets S + u$\\
|
||||
Fin Tant que\\\hline
|
||||
\end{tabularx}
|
||||
\end{center}
|
||||
\item \textit{Pas de correction automatisé}
|
||||
\end{enumerate}
|
||||
\end{enumerate}
|
||||
\end{solution}
|
||||
|
||||
\end{document}
|
||||
|
||||
%%% Local Variables:
|
||||
%%% mode: latex
|
||||
%%% TeX-master: "master"
|
||||
%%% End:
|
262
TST/DS/DS_21_04_07/TST3/corr_09_210407_DS8.tex
Normal file
262
TST/DS/DS_21_04_07/TST3/corr_09_210407_DS8.tex
Normal file
@ -0,0 +1,262 @@
|
||||
\documentclass[a4paper,10pt]{article}
|
||||
\usepackage{myXsim}
|
||||
|
||||
% Title Page
|
||||
\title{DS8 \hfill HADJRAS Mohcine}
|
||||
\tribe{TST}
|
||||
\date{\hfillÀ render pour le Mercredi 7 avril}
|
||||
|
||||
\xsimsetup{
|
||||
solution/print = true
|
||||
}
|
||||
|
||||
\begin{document}
|
||||
\maketitle
|
||||
|
||||
\begin{exercise}[subtitle={Automatismes}]
|
||||
\textit{Toutes les questions de cette exercice sont indépendantes et peuvent être répondus séparément}
|
||||
\begin{enumerate}
|
||||
\item De janvier à septembre, une quantité a augmenté de $19\,\%$. Faire un schéma pour représenter la situation puis calculer le taux d'évolution moyen mensuel.
|
||||
\item Une quantité augmente de $19\,\%$ par ans. En 2020, elle est de 113\euro. Quelle était sa valeur en 2019? Faire un schéma pour représenter la situation.
|
||||
\item Déterminer l'équation de la droite \\
|
||||
\begin{tikzpicture}[xscale=0.8, yscale=0.5]
|
||||
\tkzInit[xmin=-5,xmax=5,xstep=1,
|
||||
ymin=-5,ymax=5,ystep=1]
|
||||
\tkzGrid
|
||||
\tkzAxeXY
|
||||
\tkzFct[domain=-5:5,color=red,very thick]%
|
||||
{0.5*\x -1};
|
||||
\end{tikzpicture}
|
||||
\item Résoudre l'équation $4 \times 0.54^x = 6$
|
||||
\end{enumerate}
|
||||
\end{exercise}
|
||||
|
||||
\begin{solution}
|
||||
\begin{enumerate}
|
||||
\item On veut partager cette évolution en 8 évolutions.
|
||||
\[
|
||||
\left(1 + \frac{19}{100}\right)^{\frac{1}{8}} = 1.022
|
||||
\]
|
||||
Donc le taux d'évolution moyen est
|
||||
\[
|
||||
t_m = 1.022 - 1 = 0.02200000000000002
|
||||
\]
|
||||
\item Coefficient multiplicateur pour revenir en arrière
|
||||
\[
|
||||
CM = (1 + \frac{19}{100})^{-1} = 0.8403
|
||||
\]
|
||||
On en déduit la quantité en 2019
|
||||
\[
|
||||
113 * 0.8403 = 94.9539
|
||||
\]
|
||||
\item L'équation de la droite est
|
||||
\[
|
||||
y = 0.5 x -1
|
||||
\]
|
||||
\item Il faut penser à faire la division à par $4$ avant d'utiliser le log car sinon, on ne peut pas utiliser la formule $\log(a^n) = n\times \log(a)$.
|
||||
|
||||
\[x = \frac{\log(1.5)}{\log(0.54)}\]
|
||||
\end{enumerate}
|
||||
\end{solution}
|
||||
|
||||
\begin{exercise}[subtitle={Restaurant}]
|
||||
Un \emph{food truck}, ouvert le midi et le soir, propose deux types de formules :
|
||||
|
||||
\setlength\parindent{10mm}
|
||||
\begin{itemize}
|
||||
\item la formule \emph{Burger} ;
|
||||
\item la formule \emph{Wok}.
|
||||
\end{itemize}
|
||||
\setlength\parindent{0mm}
|
||||
|
||||
\medskip
|
||||
|
||||
Le gérant a remarqué que 68\,\% de ses ventes ont lieu le midi. Le quart des ventes du midi correspondent à la formule \emph{Burger}, alors que 43\,\% des ventes du soir correspondent à la formule \emph{Wok}.
|
||||
|
||||
Le gérant se constitue un fichier en notant, pour chaque vente, la formule choisie et le moment de cette vente (midi ou soir).
|
||||
|
||||
On prélève une fiche de façon équiprobable. On définit les quatre évènements suivants:
|
||||
|
||||
\begin{enumerate}
|
||||
\item $M$ : \og la fiche correspond à une vente du midi\fg{} ;
|
||||
\item $S$ : \og la fiche correspond à une vente du soir\fg {};
|
||||
\item $W$ : \og la fiche correspond à une formule \emph{Wok} \fg{} ;
|
||||
\item $B$ : \og la fiche correspond à une formule \emph{Burger} \fg.
|
||||
\end{enumerate}
|
||||
\setlength\parindent{0mm}
|
||||
|
||||
\medskip
|
||||
|
||||
\begin{enumerate}
|
||||
\item Recopier puis compléter l'arbre pondéré
|
||||
|
||||
\begin{center}
|
||||
\begin{tikzpicture}[sloped]
|
||||
\node {.}
|
||||
child {node {$M$}
|
||||
child {node {$W$}
|
||||
edge from parent
|
||||
node[above] {...}
|
||||
}
|
||||
child {node {$B$}
|
||||
edge from parent
|
||||
node[above] {...}
|
||||
}
|
||||
edge from parent
|
||||
node[above] {...}
|
||||
}
|
||||
child[missing] {}
|
||||
child { node {$S$}
|
||||
child {node {$W$}
|
||||
edge from parent
|
||||
node[above] {...}
|
||||
}
|
||||
child {node {$B$}
|
||||
edge from parent
|
||||
node[above] {...}
|
||||
}
|
||||
edge from parent
|
||||
node[above] {...}
|
||||
} ;
|
||||
\end{tikzpicture}
|
||||
\end{center}
|
||||
|
||||
\item Calculer la probabilité de l'évènement $M \cap W$. Interpréter ce résultat dans le contexte de l'exercice.
|
||||
\item Montrer que la probabilité que la fiche choisie corresponde à une formule \emph{Burger} est égale à $0.3524$.
|
||||
\item On a prélevé une fiche correspondant à la formule \emph{Burger}. Quelle est la probabilité, arrondie au millième, que la vente ait eu lieu le soir?
|
||||
\end{enumerate}
|
||||
\end{exercise}
|
||||
|
||||
\begin{solution}
|
||||
\begin{enumerate}
|
||||
\item
|
||||
\begin{center}
|
||||
\begin{tikzpicture}[sloped]
|
||||
\node {.}
|
||||
child {node {$M$}
|
||||
child {node {$W$}
|
||||
edge from parent
|
||||
node[above] {$0.75$}
|
||||
}
|
||||
child {node {$B$}
|
||||
edge from parent
|
||||
node[above] {$0.25$}
|
||||
}
|
||||
edge from parent
|
||||
node[above] {$0.68$}
|
||||
}
|
||||
child[missing] {}
|
||||
child { node {$S$}
|
||||
child {node {$W$}
|
||||
edge from parent
|
||||
node[above] {$0.43$}
|
||||
}
|
||||
child {node {$B$}
|
||||
edge from parent
|
||||
node[above] {$0.57$}
|
||||
}
|
||||
edge from parent
|
||||
node[above] {$0.32$}
|
||||
} ;
|
||||
\end{tikzpicture}
|
||||
\end{center}
|
||||
\item On calcule la probabilité que la vente soit un wok et ait eu lieu à midi
|
||||
\[ P(M\cap W) = P(M) \times P_M(W) = 0.68 \times 0.75 = 0.51 \]
|
||||
\item Probabilité que la vente soit un burger.
|
||||
\[
|
||||
P(B) = P(M\cap B) + P(S\cap B) = 0.68 \times 0.75 + 0.32 \times 0.43 = 0.3524
|
||||
\]
|
||||
\item On cherche à calculer la quantité $P_B(S)$. Pour cela on utilise la formule de Bayes
|
||||
\[
|
||||
P_B(S) = \frac{P(B\cap S)}{P(B)} = \frac{P_S(B) \times P(S)}{P(B)} = \frac{0.57\times 0.32}{0.3524} = 0.5175936435868331 \approx 0.518
|
||||
\]
|
||||
\end{enumerate}
|
||||
\end{solution}
|
||||
|
||||
\begin{exercise}[subtitle={Continent plastique}]
|
||||
\textit{Les quantités évoqués dans cette exercice sont générés au hasard et sont donc complètement farfelus.}
|
||||
\medskip
|
||||
Le \og continent de plastique\fg{} est la plus grande des plaques de déchets plastiques évoluant sur les océans. Elle occupe actuellement dans l'océan Pacifique une surface dont l'aire est évaluée à plus de $1,6$ million de km$^2$, entre Hawaï et la Californie.
|
||||
|
||||
En 2017, des scientifiques ont estimé qu'il y avait $4$ millions de tonnes de déchets plastiques qui était déversé chaque année dans les océans et que cette quantité augmentait de $10\n\%$ par chaque année.
|
||||
|
||||
On modélise l'évolution de la masse de ces déchets plastiques déversée chaque année, si rien n'est fait pour la réduire, par une suite géométrique $\left(u_n\right)$. L'arrondi au centième du terme $u_n$ représente la masse de ces déchets déversée chaque année, exprimée en million de tonnes, pour l'année $(2017 + n)$.
|
||||
|
||||
\medskip
|
||||
|
||||
\begin{enumerate}
|
||||
\item Expliquer pourquoi la suite $u_n$ est géométrique?
|
||||
\item Calculer $u_1$ et $u_2$.
|
||||
\item Exprimer $u_n$ en fonction de $n$.
|
||||
\item Au début de l'année 2017, il y avait $300$ millions de tonnes de déchets plastique. Calculer la quantité totale de déchets plastiques en 2030.
|
||||
\item On souhaite déterminer en quelle année la masse totale de ces déchets plastiques aura pour la première fois augmenté de $50$\,\% par rapport à sa valeur de 2017.
|
||||
\begin{enumerate}
|
||||
\item Recopier et compléter l'algorithme ci-dessous pour que la variable $N$ contienne la réponse au problème posé.
|
||||
|
||||
\begin{center}
|
||||
\begin{tabularx}{0.4\linewidth}{|X|}\hline
|
||||
$N = 2017$\\
|
||||
$U = 4$ \\
|
||||
$S = 300 + U$ \\
|
||||
while $S < 450$: \\
|
||||
\hspace{1cm} $N = \ldots$\\
|
||||
\hspace{1cm} $U = \ldots$\\
|
||||
\hspace{1cm} $S = \ldots$\\
|
||||
\hline
|
||||
\end{tabularx}
|
||||
\end{center}
|
||||
\item Que contiennent les variables $S$, $U$ et $N$ après exécution de cet algorithme ?
|
||||
|
||||
Interpréter les résultats dans le contexte de l'exercice.
|
||||
\end{enumerate}
|
||||
\end{enumerate}
|
||||
\end{exercise}
|
||||
|
||||
\begin{solution}
|
||||
\begin{enumerate}
|
||||
\item Une augmentation de $10\,\%$ revient à multiplier la quantité par $1.1$. La suite est donc bien géométrique. Son premier terme est $u_0 = 4$ et sa raison est $q = 1.1$
|
||||
\item
|
||||
\[
|
||||
u_1 = u_0 * 1.1 = 4.4
|
||||
\]
|
||||
\[
|
||||
u_2 = u_0 * 1.1^2 = 4.84
|
||||
\]
|
||||
\item
|
||||
\[
|
||||
u_n = u_0 \times q^n = 4 \times 1.1^n
|
||||
\]
|
||||
\item On calcule la quantité totale déversée entre 2017 et 2030.
|
||||
\[
|
||||
\sum_{n = 0}^{13} u_n = u_0 \times \frac{1-q^{13}}{1-q} = 4 \times \frac{1 - 1.1^{13}}{1 - 1.1} = 98.09
|
||||
\]
|
||||
On en déduit la quantité totale de déchets en 2030
|
||||
\[
|
||||
300 + 98.09 = 398.09000000000003
|
||||
\]
|
||||
\item
|
||||
\begin{enumerate}
|
||||
\item ~
|
||||
\begin{center}
|
||||
\begin{tabularx}{0.4\linewidth}{|X|}\hline
|
||||
$N \gets 2017$\\
|
||||
$U \gets 4$ \\
|
||||
$S \gets 300 + U$ \\
|
||||
Tant que $S < 450$ \\
|
||||
\hspace{1cm} $N \gets N + 1$\\
|
||||
\hspace{1cm} $U \gets U * 1.1$\\
|
||||
\hspace{1cm} $S \gets S + u$\\
|
||||
Fin Tant que\\\hline
|
||||
\end{tabularx}
|
||||
\end{center}
|
||||
\item \textit{Pas de correction automatisé}
|
||||
\end{enumerate}
|
||||
\end{enumerate}
|
||||
\end{solution}
|
||||
|
||||
\end{document}
|
||||
|
||||
%%% Local Variables:
|
||||
%%% mode: latex
|
||||
%%% TeX-master: "master"
|
||||
%%% End:
|
262
TST/DS/DS_21_04_07/TST3/corr_10_210407_DS8.tex
Normal file
262
TST/DS/DS_21_04_07/TST3/corr_10_210407_DS8.tex
Normal file
@ -0,0 +1,262 @@
|
||||
\documentclass[a4paper,10pt]{article}
|
||||
\usepackage{myXsim}
|
||||
|
||||
% Title Page
|
||||
\title{DS8 \hfill HENRIST Maxime}
|
||||
\tribe{TST}
|
||||
\date{\hfillÀ render pour le Mercredi 7 avril}
|
||||
|
||||
\xsimsetup{
|
||||
solution/print = true
|
||||
}
|
||||
|
||||
\begin{document}
|
||||
\maketitle
|
||||
|
||||
\begin{exercise}[subtitle={Automatismes}]
|
||||
\textit{Toutes les questions de cette exercice sont indépendantes et peuvent être répondus séparément}
|
||||
\begin{enumerate}
|
||||
\item De janvier à septembre, une quantité a augmenté de $19\,\%$. Faire un schéma pour représenter la situation puis calculer le taux d'évolution moyen mensuel.
|
||||
\item Une quantité augmente de $19\,\%$ par ans. En 2020, elle est de 133\euro. Quelle était sa valeur en 2019? Faire un schéma pour représenter la situation.
|
||||
\item Déterminer l'équation de la droite \\
|
||||
\begin{tikzpicture}[xscale=0.8, yscale=0.5]
|
||||
\tkzInit[xmin=-5,xmax=5,xstep=1,
|
||||
ymin=-5,ymax=5,ystep=1]
|
||||
\tkzGrid
|
||||
\tkzAxeXY
|
||||
\tkzFct[domain=-5:5,color=red,very thick]%
|
||||
{1.0*\x -2};
|
||||
\end{tikzpicture}
|
||||
\item Résoudre l'équation $7 \times 0.82^x = 19$
|
||||
\end{enumerate}
|
||||
\end{exercise}
|
||||
|
||||
\begin{solution}
|
||||
\begin{enumerate}
|
||||
\item On veut partager cette évolution en 8 évolutions.
|
||||
\[
|
||||
\left(1 + \frac{19}{100}\right)^{\frac{1}{8}} = 1.022
|
||||
\]
|
||||
Donc le taux d'évolution moyen est
|
||||
\[
|
||||
t_m = 1.022 - 1 = 0.02200000000000002
|
||||
\]
|
||||
\item Coefficient multiplicateur pour revenir en arrière
|
||||
\[
|
||||
CM = (1 + \frac{19}{100})^{-1} = 0.8403
|
||||
\]
|
||||
On en déduit la quantité en 2019
|
||||
\[
|
||||
133 * 0.8403 = 111.7599
|
||||
\]
|
||||
\item L'équation de la droite est
|
||||
\[
|
||||
y = 1.0 x -2
|
||||
\]
|
||||
\item Il faut penser à faire la division à par $7$ avant d'utiliser le log car sinon, on ne peut pas utiliser la formule $\log(a^n) = n\times \log(a)$.
|
||||
|
||||
\[x = \frac{\log(2.71)}{\log(0.82)}\]
|
||||
\end{enumerate}
|
||||
\end{solution}
|
||||
|
||||
\begin{exercise}[subtitle={Restaurant}]
|
||||
Un \emph{food truck}, ouvert le midi et le soir, propose deux types de formules :
|
||||
|
||||
\setlength\parindent{10mm}
|
||||
\begin{itemize}
|
||||
\item la formule \emph{Burger} ;
|
||||
\item la formule \emph{Wok}.
|
||||
\end{itemize}
|
||||
\setlength\parindent{0mm}
|
||||
|
||||
\medskip
|
||||
|
||||
Le gérant a remarqué que 20\,\% de ses ventes ont lieu le midi. Le quart des ventes du midi correspondent à la formule \emph{Burger}, alors que 56\,\% des ventes du soir correspondent à la formule \emph{Wok}.
|
||||
|
||||
Le gérant se constitue un fichier en notant, pour chaque vente, la formule choisie et le moment de cette vente (midi ou soir).
|
||||
|
||||
On prélève une fiche de façon équiprobable. On définit les quatre évènements suivants:
|
||||
|
||||
\begin{enumerate}
|
||||
\item $M$ : \og la fiche correspond à une vente du midi\fg{} ;
|
||||
\item $S$ : \og la fiche correspond à une vente du soir\fg {};
|
||||
\item $W$ : \og la fiche correspond à une formule \emph{Wok} \fg{} ;
|
||||
\item $B$ : \og la fiche correspond à une formule \emph{Burger} \fg.
|
||||
\end{enumerate}
|
||||
\setlength\parindent{0mm}
|
||||
|
||||
\medskip
|
||||
|
||||
\begin{enumerate}
|
||||
\item Recopier puis compléter l'arbre pondéré
|
||||
|
||||
\begin{center}
|
||||
\begin{tikzpicture}[sloped]
|
||||
\node {.}
|
||||
child {node {$M$}
|
||||
child {node {$W$}
|
||||
edge from parent
|
||||
node[above] {...}
|
||||
}
|
||||
child {node {$B$}
|
||||
edge from parent
|
||||
node[above] {...}
|
||||
}
|
||||
edge from parent
|
||||
node[above] {...}
|
||||
}
|
||||
child[missing] {}
|
||||
child { node {$S$}
|
||||
child {node {$W$}
|
||||
edge from parent
|
||||
node[above] {...}
|
||||
}
|
||||
child {node {$B$}
|
||||
edge from parent
|
||||
node[above] {...}
|
||||
}
|
||||
edge from parent
|
||||
node[above] {...}
|
||||
} ;
|
||||
\end{tikzpicture}
|
||||
\end{center}
|
||||
|
||||
\item Calculer la probabilité de l'évènement $M \cap W$. Interpréter ce résultat dans le contexte de l'exercice.
|
||||
\item Montrer que la probabilité que la fiche choisie corresponde à une formule \emph{Burger} est égale à $0.394$.
|
||||
\item On a prélevé une fiche correspondant à la formule \emph{Burger}. Quelle est la probabilité, arrondie au millième, que la vente ait eu lieu le soir?
|
||||
\end{enumerate}
|
||||
\end{exercise}
|
||||
|
||||
\begin{solution}
|
||||
\begin{enumerate}
|
||||
\item
|
||||
\begin{center}
|
||||
\begin{tikzpicture}[sloped]
|
||||
\node {.}
|
||||
child {node {$M$}
|
||||
child {node {$W$}
|
||||
edge from parent
|
||||
node[above] {$0.75$}
|
||||
}
|
||||
child {node {$B$}
|
||||
edge from parent
|
||||
node[above] {$0.25$}
|
||||
}
|
||||
edge from parent
|
||||
node[above] {$0.2$}
|
||||
}
|
||||
child[missing] {}
|
||||
child { node {$S$}
|
||||
child {node {$W$}
|
||||
edge from parent
|
||||
node[above] {$0.57$}
|
||||
}
|
||||
child {node {$B$}
|
||||
edge from parent
|
||||
node[above] {$0.43$}
|
||||
}
|
||||
edge from parent
|
||||
node[above] {$0.8$}
|
||||
} ;
|
||||
\end{tikzpicture}
|
||||
\end{center}
|
||||
\item On calcule la probabilité que la vente soit un wok et ait eu lieu à midi
|
||||
\[ P(M\cap W) = P(M) \times P_M(W) = 0.2 \times 0.75 = 0.15 \]
|
||||
\item Probabilité que la vente soit un burger.
|
||||
\[
|
||||
P(B) = P(M\cap B) + P(S\cap B) = 0.2 \times 0.75 + 0.8 \times 0.57 = 0.394
|
||||
\]
|
||||
\item On cherche à calculer la quantité $P_B(S)$. Pour cela on utilise la formule de Bayes
|
||||
\[
|
||||
P_B(S) = \frac{P(B\cap S)}{P(B)} = \frac{P_S(B) \times P(S)}{P(B)} = \frac{0.43\times 0.8}{0.394} = 0.8730964467005077 \approx 0.873
|
||||
\]
|
||||
\end{enumerate}
|
||||
\end{solution}
|
||||
|
||||
\begin{exercise}[subtitle={Continent plastique}]
|
||||
\textit{Les quantités évoqués dans cette exercice sont générés au hasard et sont donc complètement farfelus.}
|
||||
\medskip
|
||||
Le \og continent de plastique\fg{} est la plus grande des plaques de déchets plastiques évoluant sur les océans. Elle occupe actuellement dans l'océan Pacifique une surface dont l'aire est évaluée à plus de $1,6$ million de km$^2$, entre Hawaï et la Californie.
|
||||
|
||||
En 2017, des scientifiques ont estimé qu'il y avait $5$ millions de tonnes de déchets plastiques qui était déversé chaque année dans les océans et que cette quantité augmentait de $16\n\%$ par chaque année.
|
||||
|
||||
On modélise l'évolution de la masse de ces déchets plastiques déversée chaque année, si rien n'est fait pour la réduire, par une suite géométrique $\left(u_n\right)$. L'arrondi au centième du terme $u_n$ représente la masse de ces déchets déversée chaque année, exprimée en million de tonnes, pour l'année $(2017 + n)$.
|
||||
|
||||
\medskip
|
||||
|
||||
\begin{enumerate}
|
||||
\item Expliquer pourquoi la suite $u_n$ est géométrique?
|
||||
\item Calculer $u_1$ et $u_2$.
|
||||
\item Exprimer $u_n$ en fonction de $n$.
|
||||
\item Au début de l'année 2017, il y avait $300$ millions de tonnes de déchets plastique. Calculer la quantité totale de déchets plastiques en 2030.
|
||||
\item On souhaite déterminer en quelle année la masse totale de ces déchets plastiques aura pour la première fois augmenté de $50$\,\% par rapport à sa valeur de 2017.
|
||||
\begin{enumerate}
|
||||
\item Recopier et compléter l'algorithme ci-dessous pour que la variable $N$ contienne la réponse au problème posé.
|
||||
|
||||
\begin{center}
|
||||
\begin{tabularx}{0.4\linewidth}{|X|}\hline
|
||||
$N = 2017$\\
|
||||
$U = 5$ \\
|
||||
$S = 300 + U$ \\
|
||||
while $S < 450$: \\
|
||||
\hspace{1cm} $N = \ldots$\\
|
||||
\hspace{1cm} $U = \ldots$\\
|
||||
\hspace{1cm} $S = \ldots$\\
|
||||
\hline
|
||||
\end{tabularx}
|
||||
\end{center}
|
||||
\item Que contiennent les variables $S$, $U$ et $N$ après exécution de cet algorithme ?
|
||||
|
||||
Interpréter les résultats dans le contexte de l'exercice.
|
||||
\end{enumerate}
|
||||
\end{enumerate}
|
||||
\end{exercise}
|
||||
|
||||
\begin{solution}
|
||||
\begin{enumerate}
|
||||
\item Une augmentation de $16\,\%$ revient à multiplier la quantité par $1.16$. La suite est donc bien géométrique. Son premier terme est $u_0 = 5$ et sa raison est $q = 1.16$
|
||||
\item
|
||||
\[
|
||||
u_1 = u_0 * 1.16 = 5.8
|
||||
\]
|
||||
\[
|
||||
u_2 = u_0 * 1.16^2 = 6.728
|
||||
\]
|
||||
\item
|
||||
\[
|
||||
u_n = u_0 \times q^n = 5 \times 1.16^n
|
||||
\]
|
||||
\item On calcule la quantité totale déversée entre 2017 et 2030.
|
||||
\[
|
||||
\sum_{n = 0}^{13} u_n = u_0 \times \frac{1-q^{13}}{1-q} = 5 \times \frac{1 - 1.16^{13}}{1 - 1.16} = 183.93
|
||||
\]
|
||||
On en déduit la quantité totale de déchets en 2030
|
||||
\[
|
||||
300 + 183.93 = 483.93
|
||||
\]
|
||||
\item
|
||||
\begin{enumerate}
|
||||
\item ~
|
||||
\begin{center}
|
||||
\begin{tabularx}{0.4\linewidth}{|X|}\hline
|
||||
$N \gets 2017$\\
|
||||
$U \gets 5$ \\
|
||||
$S \gets 300 + U$ \\
|
||||
Tant que $S < 450$ \\
|
||||
\hspace{1cm} $N \gets N + 1$\\
|
||||
\hspace{1cm} $U \gets U * 1.16$\\
|
||||
\hspace{1cm} $S \gets S + u$\\
|
||||
Fin Tant que\\\hline
|
||||
\end{tabularx}
|
||||
\end{center}
|
||||
\item \textit{Pas de correction automatisé}
|
||||
\end{enumerate}
|
||||
\end{enumerate}
|
||||
\end{solution}
|
||||
|
||||
\end{document}
|
||||
|
||||
%%% Local Variables:
|
||||
%%% mode: latex
|
||||
%%% TeX-master: "master"
|
||||
%%% End:
|
262
TST/DS/DS_21_04_07/TST3/corr_11_210407_DS8.tex
Normal file
262
TST/DS/DS_21_04_07/TST3/corr_11_210407_DS8.tex
Normal file
@ -0,0 +1,262 @@
|
||||
\documentclass[a4paper,10pt]{article}
|
||||
\usepackage{myXsim}
|
||||
|
||||
% Title Page
|
||||
\title{DS8 \hfill INFANTES Antoine}
|
||||
\tribe{TST}
|
||||
\date{\hfillÀ render pour le Mercredi 7 avril}
|
||||
|
||||
\xsimsetup{
|
||||
solution/print = true
|
||||
}
|
||||
|
||||
\begin{document}
|
||||
\maketitle
|
||||
|
||||
\begin{exercise}[subtitle={Automatismes}]
|
||||
\textit{Toutes les questions de cette exercice sont indépendantes et peuvent être répondus séparément}
|
||||
\begin{enumerate}
|
||||
\item De janvier à septembre, une quantité a augmenté de $18\,\%$. Faire un schéma pour représenter la situation puis calculer le taux d'évolution moyen mensuel.
|
||||
\item Une quantité augmente de $18\,\%$ par ans. En 2020, elle est de 130\euro. Quelle était sa valeur en 2019? Faire un schéma pour représenter la situation.
|
||||
\item Déterminer l'équation de la droite \\
|
||||
\begin{tikzpicture}[xscale=0.8, yscale=0.5]
|
||||
\tkzInit[xmin=-5,xmax=5,xstep=1,
|
||||
ymin=-5,ymax=5,ystep=1]
|
||||
\tkzGrid
|
||||
\tkzAxeXY
|
||||
\tkzFct[domain=-5:5,color=red,very thick]%
|
||||
{2.6666666666666665*\x -4};
|
||||
\end{tikzpicture}
|
||||
\item Résoudre l'équation $3 \times 0.31^x = 15$
|
||||
\end{enumerate}
|
||||
\end{exercise}
|
||||
|
||||
\begin{solution}
|
||||
\begin{enumerate}
|
||||
\item On veut partager cette évolution en 8 évolutions.
|
||||
\[
|
||||
\left(1 + \frac{18}{100}\right)^{\frac{1}{8}} = 1.0209
|
||||
\]
|
||||
Donc le taux d'évolution moyen est
|
||||
\[
|
||||
t_m = 1.0209 - 1 = 0.02089999999999992
|
||||
\]
|
||||
\item Coefficient multiplicateur pour revenir en arrière
|
||||
\[
|
||||
CM = (1 + \frac{18}{100})^{-1} = 0.8475
|
||||
\]
|
||||
On en déduit la quantité en 2019
|
||||
\[
|
||||
130 * 0.8475 = 110.175
|
||||
\]
|
||||
\item L'équation de la droite est
|
||||
\[
|
||||
y = 2.6666666666666665 x -4
|
||||
\]
|
||||
\item Il faut penser à faire la division à par $3$ avant d'utiliser le log car sinon, on ne peut pas utiliser la formule $\log(a^n) = n\times \log(a)$.
|
||||
|
||||
\[x = \frac{\log(5.0)}{\log(0.31)}\]
|
||||
\end{enumerate}
|
||||
\end{solution}
|
||||
|
||||
\begin{exercise}[subtitle={Restaurant}]
|
||||
Un \emph{food truck}, ouvert le midi et le soir, propose deux types de formules :
|
||||
|
||||
\setlength\parindent{10mm}
|
||||
\begin{itemize}
|
||||
\item la formule \emph{Burger} ;
|
||||
\item la formule \emph{Wok}.
|
||||
\end{itemize}
|
||||
\setlength\parindent{0mm}
|
||||
|
||||
\medskip
|
||||
|
||||
Le gérant a remarqué que 33\,\% de ses ventes ont lieu le midi. Le quart des ventes du midi correspondent à la formule \emph{Burger}, alors que 13\,\% des ventes du soir correspondent à la formule \emph{Wok}.
|
||||
|
||||
Le gérant se constitue un fichier en notant, pour chaque vente, la formule choisie et le moment de cette vente (midi ou soir).
|
||||
|
||||
On prélève une fiche de façon équiprobable. On définit les quatre évènements suivants:
|
||||
|
||||
\begin{enumerate}
|
||||
\item $M$ : \og la fiche correspond à une vente du midi\fg{} ;
|
||||
\item $S$ : \og la fiche correspond à une vente du soir\fg {};
|
||||
\item $W$ : \og la fiche correspond à une formule \emph{Wok} \fg{} ;
|
||||
\item $B$ : \og la fiche correspond à une formule \emph{Burger} \fg.
|
||||
\end{enumerate}
|
||||
\setlength\parindent{0mm}
|
||||
|
||||
\medskip
|
||||
|
||||
\begin{enumerate}
|
||||
\item Recopier puis compléter l'arbre pondéré
|
||||
|
||||
\begin{center}
|
||||
\begin{tikzpicture}[sloped]
|
||||
\node {.}
|
||||
child {node {$M$}
|
||||
child {node {$W$}
|
||||
edge from parent
|
||||
node[above] {...}
|
||||
}
|
||||
child {node {$B$}
|
||||
edge from parent
|
||||
node[above] {...}
|
||||
}
|
||||
edge from parent
|
||||
node[above] {...}
|
||||
}
|
||||
child[missing] {}
|
||||
child { node {$S$}
|
||||
child {node {$W$}
|
||||
edge from parent
|
||||
node[above] {...}
|
||||
}
|
||||
child {node {$B$}
|
||||
edge from parent
|
||||
node[above] {...}
|
||||
}
|
||||
edge from parent
|
||||
node[above] {...}
|
||||
} ;
|
||||
\end{tikzpicture}
|
||||
\end{center}
|
||||
|
||||
\item Calculer la probabilité de l'évènement $M \cap W$. Interpréter ce résultat dans le contexte de l'exercice.
|
||||
\item Montrer que la probabilité que la fiche choisie corresponde à une formule \emph{Burger} est égale à $0.6654$.
|
||||
\item On a prélevé une fiche correspondant à la formule \emph{Burger}. Quelle est la probabilité, arrondie au millième, que la vente ait eu lieu le soir?
|
||||
\end{enumerate}
|
||||
\end{exercise}
|
||||
|
||||
\begin{solution}
|
||||
\begin{enumerate}
|
||||
\item
|
||||
\begin{center}
|
||||
\begin{tikzpicture}[sloped]
|
||||
\node {.}
|
||||
child {node {$M$}
|
||||
child {node {$W$}
|
||||
edge from parent
|
||||
node[above] {$0.75$}
|
||||
}
|
||||
child {node {$B$}
|
||||
edge from parent
|
||||
node[above] {$0.25$}
|
||||
}
|
||||
edge from parent
|
||||
node[above] {$0.33$}
|
||||
}
|
||||
child[missing] {}
|
||||
child { node {$S$}
|
||||
child {node {$W$}
|
||||
edge from parent
|
||||
node[above] {$0.13$}
|
||||
}
|
||||
child {node {$B$}
|
||||
edge from parent
|
||||
node[above] {$0.87$}
|
||||
}
|
||||
edge from parent
|
||||
node[above] {$0.67$}
|
||||
} ;
|
||||
\end{tikzpicture}
|
||||
\end{center}
|
||||
\item On calcule la probabilité que la vente soit un wok et ait eu lieu à midi
|
||||
\[ P(M\cap W) = P(M) \times P_M(W) = 0.33 \times 0.75 = 0.2475 \]
|
||||
\item Probabilité que la vente soit un burger.
|
||||
\[
|
||||
P(B) = P(M\cap B) + P(S\cap B) = 0.33 \times 0.75 + 0.67 \times 0.13 = 0.6654
|
||||
\]
|
||||
\item On cherche à calculer la quantité $P_B(S)$. Pour cela on utilise la formule de Bayes
|
||||
\[
|
||||
P_B(S) = \frac{P(B\cap S)}{P(B)} = \frac{P_S(B) \times P(S)}{P(B)} = \frac{0.87\times 0.67}{0.6654} = 0.8760144274120831 \approx 0.876
|
||||
\]
|
||||
\end{enumerate}
|
||||
\end{solution}
|
||||
|
||||
\begin{exercise}[subtitle={Continent plastique}]
|
||||
\textit{Les quantités évoqués dans cette exercice sont générés au hasard et sont donc complètement farfelus.}
|
||||
\medskip
|
||||
Le \og continent de plastique\fg{} est la plus grande des plaques de déchets plastiques évoluant sur les océans. Elle occupe actuellement dans l'océan Pacifique une surface dont l'aire est évaluée à plus de $1,6$ million de km$^2$, entre Hawaï et la Californie.
|
||||
|
||||
En 2017, des scientifiques ont estimé qu'il y avait $12$ millions de tonnes de déchets plastiques qui était déversé chaque année dans les océans et que cette quantité augmentait de $26\n\%$ par chaque année.
|
||||
|
||||
On modélise l'évolution de la masse de ces déchets plastiques déversée chaque année, si rien n'est fait pour la réduire, par une suite géométrique $\left(u_n\right)$. L'arrondi au centième du terme $u_n$ représente la masse de ces déchets déversée chaque année, exprimée en million de tonnes, pour l'année $(2017 + n)$.
|
||||
|
||||
\medskip
|
||||
|
||||
\begin{enumerate}
|
||||
\item Expliquer pourquoi la suite $u_n$ est géométrique?
|
||||
\item Calculer $u_1$ et $u_2$.
|
||||
\item Exprimer $u_n$ en fonction de $n$.
|
||||
\item Au début de l'année 2017, il y avait $300$ millions de tonnes de déchets plastique. Calculer la quantité totale de déchets plastiques en 2030.
|
||||
\item On souhaite déterminer en quelle année la masse totale de ces déchets plastiques aura pour la première fois augmenté de $50$\,\% par rapport à sa valeur de 2017.
|
||||
\begin{enumerate}
|
||||
\item Recopier et compléter l'algorithme ci-dessous pour que la variable $N$ contienne la réponse au problème posé.
|
||||
|
||||
\begin{center}
|
||||
\begin{tabularx}{0.4\linewidth}{|X|}\hline
|
||||
$N = 2017$\\
|
||||
$U = 12$ \\
|
||||
$S = 300 + U$ \\
|
||||
while $S < 450$: \\
|
||||
\hspace{1cm} $N = \ldots$\\
|
||||
\hspace{1cm} $U = \ldots$\\
|
||||
\hspace{1cm} $S = \ldots$\\
|
||||
\hline
|
||||
\end{tabularx}
|
||||
\end{center}
|
||||
\item Que contiennent les variables $S$, $U$ et $N$ après exécution de cet algorithme ?
|
||||
|
||||
Interpréter les résultats dans le contexte de l'exercice.
|
||||
\end{enumerate}
|
||||
\end{enumerate}
|
||||
\end{exercise}
|
||||
|
||||
\begin{solution}
|
||||
\begin{enumerate}
|
||||
\item Une augmentation de $26\,\%$ revient à multiplier la quantité par $1.26$. La suite est donc bien géométrique. Son premier terme est $u_0 = 12$ et sa raison est $q = 1.26$
|
||||
\item
|
||||
\[
|
||||
u_1 = u_0 * 1.26 = 15.120000000000001
|
||||
\]
|
||||
\[
|
||||
u_2 = u_0 * 1.26^2 = 19.0512
|
||||
\]
|
||||
\item
|
||||
\[
|
||||
u_n = u_0 \times q^n = 12 \times 1.26^n
|
||||
\]
|
||||
\item On calcule la quantité totale déversée entre 2017 et 2030.
|
||||
\[
|
||||
\sum_{n = 0}^{13} u_n = u_0 \times \frac{1-q^{13}}{1-q} = 12 \times \frac{1 - 1.26^{13}}{1 - 1.26} = 885.01
|
||||
\]
|
||||
On en déduit la quantité totale de déchets en 2030
|
||||
\[
|
||||
300 + 885.01 = 1185.01
|
||||
\]
|
||||
\item
|
||||
\begin{enumerate}
|
||||
\item ~
|
||||
\begin{center}
|
||||
\begin{tabularx}{0.4\linewidth}{|X|}\hline
|
||||
$N \gets 2017$\\
|
||||
$U \gets 12$ \\
|
||||
$S \gets 300 + U$ \\
|
||||
Tant que $S < 450$ \\
|
||||
\hspace{1cm} $N \gets N + 1$\\
|
||||
\hspace{1cm} $U \gets U * 1.26$\\
|
||||
\hspace{1cm} $S \gets S + u$\\
|
||||
Fin Tant que\\\hline
|
||||
\end{tabularx}
|
||||
\end{center}
|
||||
\item \textit{Pas de correction automatisé}
|
||||
\end{enumerate}
|
||||
\end{enumerate}
|
||||
\end{solution}
|
||||
|
||||
\end{document}
|
||||
|
||||
%%% Local Variables:
|
||||
%%% mode: latex
|
||||
%%% TeX-master: "master"
|
||||
%%% End:
|
262
TST/DS/DS_21_04_07/TST3/corr_12_210407_DS8.tex
Normal file
262
TST/DS/DS_21_04_07/TST3/corr_12_210407_DS8.tex
Normal file
@ -0,0 +1,262 @@
|
||||
\documentclass[a4paper,10pt]{article}
|
||||
\usepackage{myXsim}
|
||||
|
||||
% Title Page
|
||||
\title{DS8 \hfill MAGRO Robin}
|
||||
\tribe{TST}
|
||||
\date{\hfillÀ render pour le Mercredi 7 avril}
|
||||
|
||||
\xsimsetup{
|
||||
solution/print = true
|
||||
}
|
||||
|
||||
\begin{document}
|
||||
\maketitle
|
||||
|
||||
\begin{exercise}[subtitle={Automatismes}]
|
||||
\textit{Toutes les questions de cette exercice sont indépendantes et peuvent être répondus séparément}
|
||||
\begin{enumerate}
|
||||
\item De janvier à septembre, une quantité a augmenté de $11\,\%$. Faire un schéma pour représenter la situation puis calculer le taux d'évolution moyen mensuel.
|
||||
\item Une quantité augmente de $11\,\%$ par ans. En 2020, elle est de 143\euro. Quelle était sa valeur en 2019? Faire un schéma pour représenter la situation.
|
||||
\item Déterminer l'équation de la droite \\
|
||||
\begin{tikzpicture}[xscale=0.8, yscale=0.5]
|
||||
\tkzInit[xmin=-5,xmax=5,xstep=1,
|
||||
ymin=-5,ymax=5,ystep=1]
|
||||
\tkzGrid
|
||||
\tkzAxeXY
|
||||
\tkzFct[domain=-5:5,color=red,very thick]%
|
||||
{3.0*\x -3};
|
||||
\end{tikzpicture}
|
||||
\item Résoudre l'équation $5 \times 0.12^x = 40$
|
||||
\end{enumerate}
|
||||
\end{exercise}
|
||||
|
||||
\begin{solution}
|
||||
\begin{enumerate}
|
||||
\item On veut partager cette évolution en 8 évolutions.
|
||||
\[
|
||||
\left(1 + \frac{11}{100}\right)^{\frac{1}{8}} = 1.0131
|
||||
\]
|
||||
Donc le taux d'évolution moyen est
|
||||
\[
|
||||
t_m = 1.0131 - 1 = 0.01309999999999989
|
||||
\]
|
||||
\item Coefficient multiplicateur pour revenir en arrière
|
||||
\[
|
||||
CM = (1 + \frac{11}{100})^{-1} = 0.9009
|
||||
\]
|
||||
On en déduit la quantité en 2019
|
||||
\[
|
||||
143 * 0.9009 = 128.8287
|
||||
\]
|
||||
\item L'équation de la droite est
|
||||
\[
|
||||
y = 3.0 x -3
|
||||
\]
|
||||
\item Il faut penser à faire la division à par $5$ avant d'utiliser le log car sinon, on ne peut pas utiliser la formule $\log(a^n) = n\times \log(a)$.
|
||||
|
||||
\[x = \frac{\log(8.0)}{\log(0.12)}\]
|
||||
\end{enumerate}
|
||||
\end{solution}
|
||||
|
||||
\begin{exercise}[subtitle={Restaurant}]
|
||||
Un \emph{food truck}, ouvert le midi et le soir, propose deux types de formules :
|
||||
|
||||
\setlength\parindent{10mm}
|
||||
\begin{itemize}
|
||||
\item la formule \emph{Burger} ;
|
||||
\item la formule \emph{Wok}.
|
||||
\end{itemize}
|
||||
\setlength\parindent{0mm}
|
||||
|
||||
\medskip
|
||||
|
||||
Le gérant a remarqué que 37\,\% de ses ventes ont lieu le midi. Le quart des ventes du midi correspondent à la formule \emph{Burger}, alors que 32\,\% des ventes du soir correspondent à la formule \emph{Wok}.
|
||||
|
||||
Le gérant se constitue un fichier en notant, pour chaque vente, la formule choisie et le moment de cette vente (midi ou soir).
|
||||
|
||||
On prélève une fiche de façon équiprobable. On définit les quatre évènements suivants:
|
||||
|
||||
\begin{enumerate}
|
||||
\item $M$ : \og la fiche correspond à une vente du midi\fg{} ;
|
||||
\item $S$ : \og la fiche correspond à une vente du soir\fg {};
|
||||
\item $W$ : \og la fiche correspond à une formule \emph{Wok} \fg{} ;
|
||||
\item $B$ : \og la fiche correspond à une formule \emph{Burger} \fg.
|
||||
\end{enumerate}
|
||||
\setlength\parindent{0mm}
|
||||
|
||||
\medskip
|
||||
|
||||
\begin{enumerate}
|
||||
\item Recopier puis compléter l'arbre pondéré
|
||||
|
||||
\begin{center}
|
||||
\begin{tikzpicture}[sloped]
|
||||
\node {.}
|
||||
child {node {$M$}
|
||||
child {node {$W$}
|
||||
edge from parent
|
||||
node[above] {...}
|
||||
}
|
||||
child {node {$B$}
|
||||
edge from parent
|
||||
node[above] {...}
|
||||
}
|
||||
edge from parent
|
||||
node[above] {...}
|
||||
}
|
||||
child[missing] {}
|
||||
child { node {$S$}
|
||||
child {node {$W$}
|
||||
edge from parent
|
||||
node[above] {...}
|
||||
}
|
||||
child {node {$B$}
|
||||
edge from parent
|
||||
node[above] {...}
|
||||
}
|
||||
edge from parent
|
||||
node[above] {...}
|
||||
} ;
|
||||
\end{tikzpicture}
|
||||
\end{center}
|
||||
|
||||
\item Calculer la probabilité de l'évènement $M \cap W$. Interpréter ce résultat dans le contexte de l'exercice.
|
||||
\item Montrer que la probabilité que la fiche choisie corresponde à une formule \emph{Burger} est égale à $0.5209$.
|
||||
\item On a prélevé une fiche correspondant à la formule \emph{Burger}. Quelle est la probabilité, arrondie au millième, que la vente ait eu lieu le soir?
|
||||
\end{enumerate}
|
||||
\end{exercise}
|
||||
|
||||
\begin{solution}
|
||||
\begin{enumerate}
|
||||
\item
|
||||
\begin{center}
|
||||
\begin{tikzpicture}[sloped]
|
||||
\node {.}
|
||||
child {node {$M$}
|
||||
child {node {$W$}
|
||||
edge from parent
|
||||
node[above] {$0.75$}
|
||||
}
|
||||
child {node {$B$}
|
||||
edge from parent
|
||||
node[above] {$0.25$}
|
||||
}
|
||||
edge from parent
|
||||
node[above] {$0.37$}
|
||||
}
|
||||
child[missing] {}
|
||||
child { node {$S$}
|
||||
child {node {$W$}
|
||||
edge from parent
|
||||
node[above] {$0.32$}
|
||||
}
|
||||
child {node {$B$}
|
||||
edge from parent
|
||||
node[above] {$0.68$}
|
||||
}
|
||||
edge from parent
|
||||
node[above] {$0.63$}
|
||||
} ;
|
||||
\end{tikzpicture}
|
||||
\end{center}
|
||||
\item On calcule la probabilité que la vente soit un wok et ait eu lieu à midi
|
||||
\[ P(M\cap W) = P(M) \times P_M(W) = 0.37 \times 0.75 = 0.2775 \]
|
||||
\item Probabilité que la vente soit un burger.
|
||||
\[
|
||||
P(B) = P(M\cap B) + P(S\cap B) = 0.37 \times 0.75 + 0.63 \times 0.32 = 0.5209
|
||||
\]
|
||||
\item On cherche à calculer la quantité $P_B(S)$. Pour cela on utilise la formule de Bayes
|
||||
\[
|
||||
P_B(S) = \frac{P(B\cap S)}{P(B)} = \frac{P_S(B) \times P(S)}{P(B)} = \frac{0.68\times 0.63}{0.5209} = 0.8224227298905741 \approx 0.822
|
||||
\]
|
||||
\end{enumerate}
|
||||
\end{solution}
|
||||
|
||||
\begin{exercise}[subtitle={Continent plastique}]
|
||||
\textit{Les quantités évoqués dans cette exercice sont générés au hasard et sont donc complètement farfelus.}
|
||||
\medskip
|
||||
Le \og continent de plastique\fg{} est la plus grande des plaques de déchets plastiques évoluant sur les océans. Elle occupe actuellement dans l'océan Pacifique une surface dont l'aire est évaluée à plus de $1,6$ million de km$^2$, entre Hawaï et la Californie.
|
||||
|
||||
En 2017, des scientifiques ont estimé qu'il y avait $16$ millions de tonnes de déchets plastiques qui était déversé chaque année dans les océans et que cette quantité augmentait de $17\n\%$ par chaque année.
|
||||
|
||||
On modélise l'évolution de la masse de ces déchets plastiques déversée chaque année, si rien n'est fait pour la réduire, par une suite géométrique $\left(u_n\right)$. L'arrondi au centième du terme $u_n$ représente la masse de ces déchets déversée chaque année, exprimée en million de tonnes, pour l'année $(2017 + n)$.
|
||||
|
||||
\medskip
|
||||
|
||||
\begin{enumerate}
|
||||
\item Expliquer pourquoi la suite $u_n$ est géométrique?
|
||||
\item Calculer $u_1$ et $u_2$.
|
||||
\item Exprimer $u_n$ en fonction de $n$.
|
||||
\item Au début de l'année 2017, il y avait $300$ millions de tonnes de déchets plastique. Calculer la quantité totale de déchets plastiques en 2030.
|
||||
\item On souhaite déterminer en quelle année la masse totale de ces déchets plastiques aura pour la première fois augmenté de $50$\,\% par rapport à sa valeur de 2017.
|
||||
\begin{enumerate}
|
||||
\item Recopier et compléter l'algorithme ci-dessous pour que la variable $N$ contienne la réponse au problème posé.
|
||||
|
||||
\begin{center}
|
||||
\begin{tabularx}{0.4\linewidth}{|X|}\hline
|
||||
$N = 2017$\\
|
||||
$U = 16$ \\
|
||||
$S = 300 + U$ \\
|
||||
while $S < 450$: \\
|
||||
\hspace{1cm} $N = \ldots$\\
|
||||
\hspace{1cm} $U = \ldots$\\
|
||||
\hspace{1cm} $S = \ldots$\\
|
||||
\hline
|
||||
\end{tabularx}
|
||||
\end{center}
|
||||
\item Que contiennent les variables $S$, $U$ et $N$ après exécution de cet algorithme ?
|
||||
|
||||
Interpréter les résultats dans le contexte de l'exercice.
|
||||
\end{enumerate}
|
||||
\end{enumerate}
|
||||
\end{exercise}
|
||||
|
||||
\begin{solution}
|
||||
\begin{enumerate}
|
||||
\item Une augmentation de $17\,\%$ revient à multiplier la quantité par $1.17$. La suite est donc bien géométrique. Son premier terme est $u_0 = 16$ et sa raison est $q = 1.17$
|
||||
\item
|
||||
\[
|
||||
u_1 = u_0 * 1.17 = 18.72
|
||||
\]
|
||||
\[
|
||||
u_2 = u_0 * 1.17^2 = 21.9024
|
||||
\]
|
||||
\item
|
||||
\[
|
||||
u_n = u_0 \times q^n = 16 \times 1.17^n
|
||||
\]
|
||||
\item On calcule la quantité totale déversée entre 2017 et 2030.
|
||||
\[
|
||||
\sum_{n = 0}^{13} u_n = u_0 \times \frac{1-q^{13}}{1-q} = 16 \times \frac{1 - 1.17^{13}}{1 - 1.17} = 630.46
|
||||
\]
|
||||
On en déduit la quantité totale de déchets en 2030
|
||||
\[
|
||||
300 + 630.46 = 930.46
|
||||
\]
|
||||
\item
|
||||
\begin{enumerate}
|
||||
\item ~
|
||||
\begin{center}
|
||||
\begin{tabularx}{0.4\linewidth}{|X|}\hline
|
||||
$N \gets 2017$\\
|
||||
$U \gets 16$ \\
|
||||
$S \gets 300 + U$ \\
|
||||
Tant que $S < 450$ \\
|
||||
\hspace{1cm} $N \gets N + 1$\\
|
||||
\hspace{1cm} $U \gets U * 1.17$\\
|
||||
\hspace{1cm} $S \gets S + u$\\
|
||||
Fin Tant que\\\hline
|
||||
\end{tabularx}
|
||||
\end{center}
|
||||
\item \textit{Pas de correction automatisé}
|
||||
\end{enumerate}
|
||||
\end{enumerate}
|
||||
\end{solution}
|
||||
|
||||
\end{document}
|
||||
|
||||
%%% Local Variables:
|
||||
%%% mode: latex
|
||||
%%% TeX-master: "master"
|
||||
%%% End:
|
262
TST/DS/DS_21_04_07/TST3/corr_13_210407_DS8.tex
Normal file
262
TST/DS/DS_21_04_07/TST3/corr_13_210407_DS8.tex
Normal file
@ -0,0 +1,262 @@
|
||||
\documentclass[a4paper,10pt]{article}
|
||||
\usepackage{myXsim}
|
||||
|
||||
% Title Page
|
||||
\title{DS8 \hfill MORFIN Chloé}
|
||||
\tribe{TST}
|
||||
\date{\hfillÀ render pour le Mercredi 7 avril}
|
||||
|
||||
\xsimsetup{
|
||||
solution/print = true
|
||||
}
|
||||
|
||||
\begin{document}
|
||||
\maketitle
|
||||
|
||||
\begin{exercise}[subtitle={Automatismes}]
|
||||
\textit{Toutes les questions de cette exercice sont indépendantes et peuvent être répondus séparément}
|
||||
\begin{enumerate}
|
||||
\item De janvier à septembre, une quantité a augmenté de $18\,\%$. Faire un schéma pour représenter la situation puis calculer le taux d'évolution moyen mensuel.
|
||||
\item Une quantité augmente de $18\,\%$ par ans. En 2020, elle est de 119\euro. Quelle était sa valeur en 2019? Faire un schéma pour représenter la situation.
|
||||
\item Déterminer l'équation de la droite \\
|
||||
\begin{tikzpicture}[xscale=0.8, yscale=0.5]
|
||||
\tkzInit[xmin=-5,xmax=5,xstep=1,
|
||||
ymin=-5,ymax=5,ystep=1]
|
||||
\tkzGrid
|
||||
\tkzAxeXY
|
||||
\tkzFct[domain=-5:5,color=red,very thick]%
|
||||
{4.0*\x -4};
|
||||
\end{tikzpicture}
|
||||
\item Résoudre l'équation $3 \times 1.0^x = 32$
|
||||
\end{enumerate}
|
||||
\end{exercise}
|
||||
|
||||
\begin{solution}
|
||||
\begin{enumerate}
|
||||
\item On veut partager cette évolution en 8 évolutions.
|
||||
\[
|
||||
\left(1 + \frac{18}{100}\right)^{\frac{1}{8}} = 1.0209
|
||||
\]
|
||||
Donc le taux d'évolution moyen est
|
||||
\[
|
||||
t_m = 1.0209 - 1 = 0.02089999999999992
|
||||
\]
|
||||
\item Coefficient multiplicateur pour revenir en arrière
|
||||
\[
|
||||
CM = (1 + \frac{18}{100})^{-1} = 0.8475
|
||||
\]
|
||||
On en déduit la quantité en 2019
|
||||
\[
|
||||
119 * 0.8475 = 100.8525
|
||||
\]
|
||||
\item L'équation de la droite est
|
||||
\[
|
||||
y = 4.0 x -4
|
||||
\]
|
||||
\item Il faut penser à faire la division à par $3$ avant d'utiliser le log car sinon, on ne peut pas utiliser la formule $\log(a^n) = n\times \log(a)$.
|
||||
|
||||
\[x = \frac{\log(10.67)}{\log(1.0)}\]
|
||||
\end{enumerate}
|
||||
\end{solution}
|
||||
|
||||
\begin{exercise}[subtitle={Restaurant}]
|
||||
Un \emph{food truck}, ouvert le midi et le soir, propose deux types de formules :
|
||||
|
||||
\setlength\parindent{10mm}
|
||||
\begin{itemize}
|
||||
\item la formule \emph{Burger} ;
|
||||
\item la formule \emph{Wok}.
|
||||
\end{itemize}
|
||||
\setlength\parindent{0mm}
|
||||
|
||||
\medskip
|
||||
|
||||
Le gérant a remarqué que 24\,\% de ses ventes ont lieu le midi. Le quart des ventes du midi correspondent à la formule \emph{Burger}, alors que 57\,\% des ventes du soir correspondent à la formule \emph{Wok}.
|
||||
|
||||
Le gérant se constitue un fichier en notant, pour chaque vente, la formule choisie et le moment de cette vente (midi ou soir).
|
||||
|
||||
On prélève une fiche de façon équiprobable. On définit les quatre évènements suivants:
|
||||
|
||||
\begin{enumerate}
|
||||
\item $M$ : \og la fiche correspond à une vente du midi\fg{} ;
|
||||
\item $S$ : \og la fiche correspond à une vente du soir\fg {};
|
||||
\item $W$ : \og la fiche correspond à une formule \emph{Wok} \fg{} ;
|
||||
\item $B$ : \og la fiche correspond à une formule \emph{Burger} \fg.
|
||||
\end{enumerate}
|
||||
\setlength\parindent{0mm}
|
||||
|
||||
\medskip
|
||||
|
||||
\begin{enumerate}
|
||||
\item Recopier puis compléter l'arbre pondéré
|
||||
|
||||
\begin{center}
|
||||
\begin{tikzpicture}[sloped]
|
||||
\node {.}
|
||||
child {node {$M$}
|
||||
child {node {$W$}
|
||||
edge from parent
|
||||
node[above] {...}
|
||||
}
|
||||
child {node {$B$}
|
||||
edge from parent
|
||||
node[above] {...}
|
||||
}
|
||||
edge from parent
|
||||
node[above] {...}
|
||||
}
|
||||
child[missing] {}
|
||||
child { node {$S$}
|
||||
child {node {$W$}
|
||||
edge from parent
|
||||
node[above] {...}
|
||||
}
|
||||
child {node {$B$}
|
||||
edge from parent
|
||||
node[above] {...}
|
||||
}
|
||||
edge from parent
|
||||
node[above] {...}
|
||||
} ;
|
||||
\end{tikzpicture}
|
||||
\end{center}
|
||||
|
||||
\item Calculer la probabilité de l'évènement $M \cap W$. Interpréter ce résultat dans le contexte de l'exercice.
|
||||
\item Montrer que la probabilité que la fiche choisie corresponde à une formule \emph{Burger} est égale à $0.3792$.
|
||||
\item On a prélevé une fiche correspondant à la formule \emph{Burger}. Quelle est la probabilité, arrondie au millième, que la vente ait eu lieu le soir?
|
||||
\end{enumerate}
|
||||
\end{exercise}
|
||||
|
||||
\begin{solution}
|
||||
\begin{enumerate}
|
||||
\item
|
||||
\begin{center}
|
||||
\begin{tikzpicture}[sloped]
|
||||
\node {.}
|
||||
child {node {$M$}
|
||||
child {node {$W$}
|
||||
edge from parent
|
||||
node[above] {$0.75$}
|
||||
}
|
||||
child {node {$B$}
|
||||
edge from parent
|
||||
node[above] {$0.25$}
|
||||
}
|
||||
edge from parent
|
||||
node[above] {$0.24$}
|
||||
}
|
||||
child[missing] {}
|
||||
child { node {$S$}
|
||||
child {node {$W$}
|
||||
edge from parent
|
||||
node[above] {$0.58$}
|
||||
}
|
||||
child {node {$B$}
|
||||
edge from parent
|
||||
node[above] {$0.42$}
|
||||
}
|
||||
edge from parent
|
||||
node[above] {$0.76$}
|
||||
} ;
|
||||
\end{tikzpicture}
|
||||
\end{center}
|
||||
\item On calcule la probabilité que la vente soit un wok et ait eu lieu à midi
|
||||
\[ P(M\cap W) = P(M) \times P_M(W) = 0.24 \times 0.75 = 0.18 \]
|
||||
\item Probabilité que la vente soit un burger.
|
||||
\[
|
||||
P(B) = P(M\cap B) + P(S\cap B) = 0.24 \times 0.75 + 0.76 \times 0.58 = 0.3792
|
||||
\]
|
||||
\item On cherche à calculer la quantité $P_B(S)$. Pour cela on utilise la formule de Bayes
|
||||
\[
|
||||
P_B(S) = \frac{P(B\cap S)}{P(B)} = \frac{P_S(B) \times P(S)}{P(B)} = \frac{0.42\times 0.76}{0.3792} = 0.8417721518987342 \approx 0.842
|
||||
\]
|
||||
\end{enumerate}
|
||||
\end{solution}
|
||||
|
||||
\begin{exercise}[subtitle={Continent plastique}]
|
||||
\textit{Les quantités évoqués dans cette exercice sont générés au hasard et sont donc complètement farfelus.}
|
||||
\medskip
|
||||
Le \og continent de plastique\fg{} est la plus grande des plaques de déchets plastiques évoluant sur les océans. Elle occupe actuellement dans l'océan Pacifique une surface dont l'aire est évaluée à plus de $1,6$ million de km$^2$, entre Hawaï et la Californie.
|
||||
|
||||
En 2017, des scientifiques ont estimé qu'il y avait $17$ millions de tonnes de déchets plastiques qui était déversé chaque année dans les océans et que cette quantité augmentait de $16\n\%$ par chaque année.
|
||||
|
||||
On modélise l'évolution de la masse de ces déchets plastiques déversée chaque année, si rien n'est fait pour la réduire, par une suite géométrique $\left(u_n\right)$. L'arrondi au centième du terme $u_n$ représente la masse de ces déchets déversée chaque année, exprimée en million de tonnes, pour l'année $(2017 + n)$.
|
||||
|
||||
\medskip
|
||||
|
||||
\begin{enumerate}
|
||||
\item Expliquer pourquoi la suite $u_n$ est géométrique?
|
||||
\item Calculer $u_1$ et $u_2$.
|
||||
\item Exprimer $u_n$ en fonction de $n$.
|
||||
\item Au début de l'année 2017, il y avait $300$ millions de tonnes de déchets plastique. Calculer la quantité totale de déchets plastiques en 2030.
|
||||
\item On souhaite déterminer en quelle année la masse totale de ces déchets plastiques aura pour la première fois augmenté de $50$\,\% par rapport à sa valeur de 2017.
|
||||
\begin{enumerate}
|
||||
\item Recopier et compléter l'algorithme ci-dessous pour que la variable $N$ contienne la réponse au problème posé.
|
||||
|
||||
\begin{center}
|
||||
\begin{tabularx}{0.4\linewidth}{|X|}\hline
|
||||
$N = 2017$\\
|
||||
$U = 17$ \\
|
||||
$S = 300 + U$ \\
|
||||
while $S < 450$: \\
|
||||
\hspace{1cm} $N = \ldots$\\
|
||||
\hspace{1cm} $U = \ldots$\\
|
||||
\hspace{1cm} $S = \ldots$\\
|
||||
\hline
|
||||
\end{tabularx}
|
||||
\end{center}
|
||||
\item Que contiennent les variables $S$, $U$ et $N$ après exécution de cet algorithme ?
|
||||
|
||||
Interpréter les résultats dans le contexte de l'exercice.
|
||||
\end{enumerate}
|
||||
\end{enumerate}
|
||||
\end{exercise}
|
||||
|
||||
\begin{solution}
|
||||
\begin{enumerate}
|
||||
\item Une augmentation de $16\,\%$ revient à multiplier la quantité par $1.16$. La suite est donc bien géométrique. Son premier terme est $u_0 = 17$ et sa raison est $q = 1.16$
|
||||
\item
|
||||
\[
|
||||
u_1 = u_0 * 1.16 = 19.72
|
||||
\]
|
||||
\[
|
||||
u_2 = u_0 * 1.16^2 = 22.8752
|
||||
\]
|
||||
\item
|
||||
\[
|
||||
u_n = u_0 \times q^n = 17 \times 1.16^n
|
||||
\]
|
||||
\item On calcule la quantité totale déversée entre 2017 et 2030.
|
||||
\[
|
||||
\sum_{n = 0}^{13} u_n = u_0 \times \frac{1-q^{13}}{1-q} = 17 \times \frac{1 - 1.16^{13}}{1 - 1.16} = 625.37
|
||||
\]
|
||||
On en déduit la quantité totale de déchets en 2030
|
||||
\[
|
||||
300 + 625.37 = 925.37
|
||||
\]
|
||||
\item
|
||||
\begin{enumerate}
|
||||
\item ~
|
||||
\begin{center}
|
||||
\begin{tabularx}{0.4\linewidth}{|X|}\hline
|
||||
$N \gets 2017$\\
|
||||
$U \gets 17$ \\
|
||||
$S \gets 300 + U$ \\
|
||||
Tant que $S < 450$ \\
|
||||
\hspace{1cm} $N \gets N + 1$\\
|
||||
\hspace{1cm} $U \gets U * 1.16$\\
|
||||
\hspace{1cm} $S \gets S + u$\\
|
||||
Fin Tant que\\\hline
|
||||
\end{tabularx}
|
||||
\end{center}
|
||||
\item \textit{Pas de correction automatisé}
|
||||
\end{enumerate}
|
||||
\end{enumerate}
|
||||
\end{solution}
|
||||
|
||||
\end{document}
|
||||
|
||||
%%% Local Variables:
|
||||
%%% mode: latex
|
||||
%%% TeX-master: "master"
|
||||
%%% End:
|
262
TST/DS/DS_21_04_07/TST3/corr_14_210407_DS8.tex
Normal file
262
TST/DS/DS_21_04_07/TST3/corr_14_210407_DS8.tex
Normal file
@ -0,0 +1,262 @@
|
||||
\documentclass[a4paper,10pt]{article}
|
||||
\usepackage{myXsim}
|
||||
|
||||
% Title Page
|
||||
\title{DS8 \hfill PERES RAMALHO Emeric}
|
||||
\tribe{TST}
|
||||
\date{\hfillÀ render pour le Mercredi 7 avril}
|
||||
|
||||
\xsimsetup{
|
||||
solution/print = true
|
||||
}
|
||||
|
||||
\begin{document}
|
||||
\maketitle
|
||||
|
||||
\begin{exercise}[subtitle={Automatismes}]
|
||||
\textit{Toutes les questions de cette exercice sont indépendantes et peuvent être répondus séparément}
|
||||
\begin{enumerate}
|
||||
\item De janvier à septembre, une quantité a augmenté de $23\,\%$. Faire un schéma pour représenter la situation puis calculer le taux d'évolution moyen mensuel.
|
||||
\item Une quantité augmente de $23\,\%$ par ans. En 2020, elle est de 118\euro. Quelle était sa valeur en 2019? Faire un schéma pour représenter la situation.
|
||||
\item Déterminer l'équation de la droite \\
|
||||
\begin{tikzpicture}[xscale=0.8, yscale=0.5]
|
||||
\tkzInit[xmin=-5,xmax=5,xstep=1,
|
||||
ymin=-5,ymax=5,ystep=1]
|
||||
\tkzGrid
|
||||
\tkzAxeXY
|
||||
\tkzFct[domain=-5:5,color=red,very thick]%
|
||||
{1.3333333333333333*\x -2};
|
||||
\end{tikzpicture}
|
||||
\item Résoudre l'équation $5 \times 0.46^x = 2$
|
||||
\end{enumerate}
|
||||
\end{exercise}
|
||||
|
||||
\begin{solution}
|
||||
\begin{enumerate}
|
||||
\item On veut partager cette évolution en 8 évolutions.
|
||||
\[
|
||||
\left(1 + \frac{23}{100}\right)^{\frac{1}{8}} = 1.0262
|
||||
\]
|
||||
Donc le taux d'évolution moyen est
|
||||
\[
|
||||
t_m = 1.0262 - 1 = 0.0262
|
||||
\]
|
||||
\item Coefficient multiplicateur pour revenir en arrière
|
||||
\[
|
||||
CM = (1 + \frac{23}{100})^{-1} = 0.813
|
||||
\]
|
||||
On en déduit la quantité en 2019
|
||||
\[
|
||||
118 * 0.813 = 95.934
|
||||
\]
|
||||
\item L'équation de la droite est
|
||||
\[
|
||||
y = 1.3333333333333333 x -2
|
||||
\]
|
||||
\item Il faut penser à faire la division à par $5$ avant d'utiliser le log car sinon, on ne peut pas utiliser la formule $\log(a^n) = n\times \log(a)$.
|
||||
|
||||
\[x = \frac{\log(0.4)}{\log(0.46)}\]
|
||||
\end{enumerate}
|
||||
\end{solution}
|
||||
|
||||
\begin{exercise}[subtitle={Restaurant}]
|
||||
Un \emph{food truck}, ouvert le midi et le soir, propose deux types de formules :
|
||||
|
||||
\setlength\parindent{10mm}
|
||||
\begin{itemize}
|
||||
\item la formule \emph{Burger} ;
|
||||
\item la formule \emph{Wok}.
|
||||
\end{itemize}
|
||||
\setlength\parindent{0mm}
|
||||
|
||||
\medskip
|
||||
|
||||
Le gérant a remarqué que 11\,\% de ses ventes ont lieu le midi. Le quart des ventes du midi correspondent à la formule \emph{Burger}, alors que 18\,\% des ventes du soir correspondent à la formule \emph{Wok}.
|
||||
|
||||
Le gérant se constitue un fichier en notant, pour chaque vente, la formule choisie et le moment de cette vente (midi ou soir).
|
||||
|
||||
On prélève une fiche de façon équiprobable. On définit les quatre évènements suivants:
|
||||
|
||||
\begin{enumerate}
|
||||
\item $M$ : \og la fiche correspond à une vente du midi\fg{} ;
|
||||
\item $S$ : \og la fiche correspond à une vente du soir\fg {};
|
||||
\item $W$ : \og la fiche correspond à une formule \emph{Wok} \fg{} ;
|
||||
\item $B$ : \og la fiche correspond à une formule \emph{Burger} \fg.
|
||||
\end{enumerate}
|
||||
\setlength\parindent{0mm}
|
||||
|
||||
\medskip
|
||||
|
||||
\begin{enumerate}
|
||||
\item Recopier puis compléter l'arbre pondéré
|
||||
|
||||
\begin{center}
|
||||
\begin{tikzpicture}[sloped]
|
||||
\node {.}
|
||||
child {node {$M$}
|
||||
child {node {$W$}
|
||||
edge from parent
|
||||
node[above] {...}
|
||||
}
|
||||
child {node {$B$}
|
||||
edge from parent
|
||||
node[above] {...}
|
||||
}
|
||||
edge from parent
|
||||
node[above] {...}
|
||||
}
|
||||
child[missing] {}
|
||||
child { node {$S$}
|
||||
child {node {$W$}
|
||||
edge from parent
|
||||
node[above] {...}
|
||||
}
|
||||
child {node {$B$}
|
||||
edge from parent
|
||||
node[above] {...}
|
||||
}
|
||||
edge from parent
|
||||
node[above] {...}
|
||||
} ;
|
||||
\end{tikzpicture}
|
||||
\end{center}
|
||||
|
||||
\item Calculer la probabilité de l'évènement $M \cap W$. Interpréter ce résultat dans le contexte de l'exercice.
|
||||
\item Montrer que la probabilité que la fiche choisie corresponde à une formule \emph{Burger} est égale à $0.7573$.
|
||||
\item On a prélevé une fiche correspondant à la formule \emph{Burger}. Quelle est la probabilité, arrondie au millième, que la vente ait eu lieu le soir?
|
||||
\end{enumerate}
|
||||
\end{exercise}
|
||||
|
||||
\begin{solution}
|
||||
\begin{enumerate}
|
||||
\item
|
||||
\begin{center}
|
||||
\begin{tikzpicture}[sloped]
|
||||
\node {.}
|
||||
child {node {$M$}
|
||||
child {node {$W$}
|
||||
edge from parent
|
||||
node[above] {$0.75$}
|
||||
}
|
||||
child {node {$B$}
|
||||
edge from parent
|
||||
node[above] {$0.25$}
|
||||
}
|
||||
edge from parent
|
||||
node[above] {$0.11$}
|
||||
}
|
||||
child[missing] {}
|
||||
child { node {$S$}
|
||||
child {node {$W$}
|
||||
edge from parent
|
||||
node[above] {$0.18$}
|
||||
}
|
||||
child {node {$B$}
|
||||
edge from parent
|
||||
node[above] {$0.82$}
|
||||
}
|
||||
edge from parent
|
||||
node[above] {$0.89$}
|
||||
} ;
|
||||
\end{tikzpicture}
|
||||
\end{center}
|
||||
\item On calcule la probabilité que la vente soit un wok et ait eu lieu à midi
|
||||
\[ P(M\cap W) = P(M) \times P_M(W) = 0.11 \times 0.75 = 0.0825 \]
|
||||
\item Probabilité que la vente soit un burger.
|
||||
\[
|
||||
P(B) = P(M\cap B) + P(S\cap B) = 0.11 \times 0.75 + 0.89 \times 0.18 = 0.7573
|
||||
\]
|
||||
\item On cherche à calculer la quantité $P_B(S)$. Pour cela on utilise la formule de Bayes
|
||||
\[
|
||||
P_B(S) = \frac{P(B\cap S)}{P(B)} = \frac{P_S(B) \times P(S)}{P(B)} = \frac{0.82\times 0.89}{0.7573} = 0.9636867819886439 \approx 0.964
|
||||
\]
|
||||
\end{enumerate}
|
||||
\end{solution}
|
||||
|
||||
\begin{exercise}[subtitle={Continent plastique}]
|
||||
\textit{Les quantités évoqués dans cette exercice sont générés au hasard et sont donc complètement farfelus.}
|
||||
\medskip
|
||||
Le \og continent de plastique\fg{} est la plus grande des plaques de déchets plastiques évoluant sur les océans. Elle occupe actuellement dans l'océan Pacifique une surface dont l'aire est évaluée à plus de $1,6$ million de km$^2$, entre Hawaï et la Californie.
|
||||
|
||||
En 2017, des scientifiques ont estimé qu'il y avait $19$ millions de tonnes de déchets plastiques qui était déversé chaque année dans les océans et que cette quantité augmentait de $22\n\%$ par chaque année.
|
||||
|
||||
On modélise l'évolution de la masse de ces déchets plastiques déversée chaque année, si rien n'est fait pour la réduire, par une suite géométrique $\left(u_n\right)$. L'arrondi au centième du terme $u_n$ représente la masse de ces déchets déversée chaque année, exprimée en million de tonnes, pour l'année $(2017 + n)$.
|
||||
|
||||
\medskip
|
||||
|
||||
\begin{enumerate}
|
||||
\item Expliquer pourquoi la suite $u_n$ est géométrique?
|
||||
\item Calculer $u_1$ et $u_2$.
|
||||
\item Exprimer $u_n$ en fonction de $n$.
|
||||
\item Au début de l'année 2017, il y avait $300$ millions de tonnes de déchets plastique. Calculer la quantité totale de déchets plastiques en 2030.
|
||||
\item On souhaite déterminer en quelle année la masse totale de ces déchets plastiques aura pour la première fois augmenté de $50$\,\% par rapport à sa valeur de 2017.
|
||||
\begin{enumerate}
|
||||
\item Recopier et compléter l'algorithme ci-dessous pour que la variable $N$ contienne la réponse au problème posé.
|
||||
|
||||
\begin{center}
|
||||
\begin{tabularx}{0.4\linewidth}{|X|}\hline
|
||||
$N = 2017$\\
|
||||
$U = 19$ \\
|
||||
$S = 300 + U$ \\
|
||||
while $S < 450$: \\
|
||||
\hspace{1cm} $N = \ldots$\\
|
||||
\hspace{1cm} $U = \ldots$\\
|
||||
\hspace{1cm} $S = \ldots$\\
|
||||
\hline
|
||||
\end{tabularx}
|
||||
\end{center}
|
||||
\item Que contiennent les variables $S$, $U$ et $N$ après exécution de cet algorithme ?
|
||||
|
||||
Interpréter les résultats dans le contexte de l'exercice.
|
||||
\end{enumerate}
|
||||
\end{enumerate}
|
||||
\end{exercise}
|
||||
|
||||
\begin{solution}
|
||||
\begin{enumerate}
|
||||
\item Une augmentation de $22\,\%$ revient à multiplier la quantité par $1.22$. La suite est donc bien géométrique. Son premier terme est $u_0 = 19$ et sa raison est $q = 1.22$
|
||||
\item
|
||||
\[
|
||||
u_1 = u_0 * 1.22 = 23.18
|
||||
\]
|
||||
\[
|
||||
u_2 = u_0 * 1.22^2 = 28.2796
|
||||
\]
|
||||
\item
|
||||
\[
|
||||
u_n = u_0 \times q^n = 19 \times 1.22^n
|
||||
\]
|
||||
\item On calcule la quantité totale déversée entre 2017 et 2030.
|
||||
\[
|
||||
\sum_{n = 0}^{13} u_n = u_0 \times \frac{1-q^{13}}{1-q} = 19 \times \frac{1 - 1.22^{13}}{1 - 1.22} = 1059.17
|
||||
\]
|
||||
On en déduit la quantité totale de déchets en 2030
|
||||
\[
|
||||
300 + 1059.17 = 1359.17
|
||||
\]
|
||||
\item
|
||||
\begin{enumerate}
|
||||
\item ~
|
||||
\begin{center}
|
||||
\begin{tabularx}{0.4\linewidth}{|X|}\hline
|
||||
$N \gets 2017$\\
|
||||
$U \gets 19$ \\
|
||||
$S \gets 300 + U$ \\
|
||||
Tant que $S < 450$ \\
|
||||
\hspace{1cm} $N \gets N + 1$\\
|
||||
\hspace{1cm} $U \gets U * 1.22$\\
|
||||
\hspace{1cm} $S \gets S + u$\\
|
||||
Fin Tant que\\\hline
|
||||
\end{tabularx}
|
||||
\end{center}
|
||||
\item \textit{Pas de correction automatisé}
|
||||
\end{enumerate}
|
||||
\end{enumerate}
|
||||
\end{solution}
|
||||
|
||||
\end{document}
|
||||
|
||||
%%% Local Variables:
|
||||
%%% mode: latex
|
||||
%%% TeX-master: "master"
|
||||
%%% End:
|
262
TST/DS/DS_21_04_07/TST3/corr_15_210407_DS8.tex
Normal file
262
TST/DS/DS_21_04_07/TST3/corr_15_210407_DS8.tex
Normal file
@ -0,0 +1,262 @@
|
||||
\documentclass[a4paper,10pt]{article}
|
||||
\usepackage{myXsim}
|
||||
|
||||
% Title Page
|
||||
\title{DS8 \hfill RADOUAA Saleh}
|
||||
\tribe{TST}
|
||||
\date{\hfillÀ render pour le Mercredi 7 avril}
|
||||
|
||||
\xsimsetup{
|
||||
solution/print = true
|
||||
}
|
||||
|
||||
\begin{document}
|
||||
\maketitle
|
||||
|
||||
\begin{exercise}[subtitle={Automatismes}]
|
||||
\textit{Toutes les questions de cette exercice sont indépendantes et peuvent être répondus séparément}
|
||||
\begin{enumerate}
|
||||
\item De janvier à septembre, une quantité a augmenté de $27\,\%$. Faire un schéma pour représenter la situation puis calculer le taux d'évolution moyen mensuel.
|
||||
\item Une quantité augmente de $27\,\%$ par ans. En 2020, elle est de 141\euro. Quelle était sa valeur en 2019? Faire un schéma pour représenter la situation.
|
||||
\item Déterminer l'équation de la droite \\
|
||||
\begin{tikzpicture}[xscale=0.8, yscale=0.5]
|
||||
\tkzInit[xmin=-5,xmax=5,xstep=1,
|
||||
ymin=-5,ymax=5,ystep=1]
|
||||
\tkzGrid
|
||||
\tkzAxeXY
|
||||
\tkzFct[domain=-5:5,color=red,very thick]%
|
||||
{2.0*\x -4};
|
||||
\end{tikzpicture}
|
||||
\item Résoudre l'équation $2 \times 0.88^x = 8$
|
||||
\end{enumerate}
|
||||
\end{exercise}
|
||||
|
||||
\begin{solution}
|
||||
\begin{enumerate}
|
||||
\item On veut partager cette évolution en 8 évolutions.
|
||||
\[
|
||||
\left(1 + \frac{27}{100}\right)^{\frac{1}{8}} = 1.0303
|
||||
\]
|
||||
Donc le taux d'évolution moyen est
|
||||
\[
|
||||
t_m = 1.0303 - 1 = 0.030299999999999994
|
||||
\]
|
||||
\item Coefficient multiplicateur pour revenir en arrière
|
||||
\[
|
||||
CM = (1 + \frac{27}{100})^{-1} = 0.7874
|
||||
\]
|
||||
On en déduit la quantité en 2019
|
||||
\[
|
||||
141 * 0.7874 = 111.0234
|
||||
\]
|
||||
\item L'équation de la droite est
|
||||
\[
|
||||
y = 2.0 x -4
|
||||
\]
|
||||
\item Il faut penser à faire la division à par $2$ avant d'utiliser le log car sinon, on ne peut pas utiliser la formule $\log(a^n) = n\times \log(a)$.
|
||||
|
||||
\[x = \frac{\log(4.0)}{\log(0.88)}\]
|
||||
\end{enumerate}
|
||||
\end{solution}
|
||||
|
||||
\begin{exercise}[subtitle={Restaurant}]
|
||||
Un \emph{food truck}, ouvert le midi et le soir, propose deux types de formules :
|
||||
|
||||
\setlength\parindent{10mm}
|
||||
\begin{itemize}
|
||||
\item la formule \emph{Burger} ;
|
||||
\item la formule \emph{Wok}.
|
||||
\end{itemize}
|
||||
\setlength\parindent{0mm}
|
||||
|
||||
\medskip
|
||||
|
||||
Le gérant a remarqué que 1\,\% de ses ventes ont lieu le midi. Le quart des ventes du midi correspondent à la formule \emph{Burger}, alors que 99\,\% des ventes du soir correspondent à la formule \emph{Wok}.
|
||||
|
||||
Le gérant se constitue un fichier en notant, pour chaque vente, la formule choisie et le moment de cette vente (midi ou soir).
|
||||
|
||||
On prélève une fiche de façon équiprobable. On définit les quatre évènements suivants:
|
||||
|
||||
\begin{enumerate}
|
||||
\item $M$ : \og la fiche correspond à une vente du midi\fg{} ;
|
||||
\item $S$ : \og la fiche correspond à une vente du soir\fg {};
|
||||
\item $W$ : \og la fiche correspond à une formule \emph{Wok} \fg{} ;
|
||||
\item $B$ : \og la fiche correspond à une formule \emph{Burger} \fg.
|
||||
\end{enumerate}
|
||||
\setlength\parindent{0mm}
|
||||
|
||||
\medskip
|
||||
|
||||
\begin{enumerate}
|
||||
\item Recopier puis compléter l'arbre pondéré
|
||||
|
||||
\begin{center}
|
||||
\begin{tikzpicture}[sloped]
|
||||
\node {.}
|
||||
child {node {$M$}
|
||||
child {node {$W$}
|
||||
edge from parent
|
||||
node[above] {...}
|
||||
}
|
||||
child {node {$B$}
|
||||
edge from parent
|
||||
node[above] {...}
|
||||
}
|
||||
edge from parent
|
||||
node[above] {...}
|
||||
}
|
||||
child[missing] {}
|
||||
child { node {$S$}
|
||||
child {node {$W$}
|
||||
edge from parent
|
||||
node[above] {...}
|
||||
}
|
||||
child {node {$B$}
|
||||
edge from parent
|
||||
node[above] {...}
|
||||
}
|
||||
edge from parent
|
||||
node[above] {...}
|
||||
} ;
|
||||
\end{tikzpicture}
|
||||
\end{center}
|
||||
|
||||
\item Calculer la probabilité de l'évènement $M \cap W$. Interpréter ce résultat dans le contexte de l'exercice.
|
||||
\item Montrer que la probabilité que la fiche choisie corresponde à une formule \emph{Burger} est égale à $0.0124$.
|
||||
\item On a prélevé une fiche correspondant à la formule \emph{Burger}. Quelle est la probabilité, arrondie au millième, que la vente ait eu lieu le soir?
|
||||
\end{enumerate}
|
||||
\end{exercise}
|
||||
|
||||
\begin{solution}
|
||||
\begin{enumerate}
|
||||
\item
|
||||
\begin{center}
|
||||
\begin{tikzpicture}[sloped]
|
||||
\node {.}
|
||||
child {node {$M$}
|
||||
child {node {$W$}
|
||||
edge from parent
|
||||
node[above] {$0.75$}
|
||||
}
|
||||
child {node {$B$}
|
||||
edge from parent
|
||||
node[above] {$0.25$}
|
||||
}
|
||||
edge from parent
|
||||
node[above] {$0.01$}
|
||||
}
|
||||
child[missing] {}
|
||||
child { node {$S$}
|
||||
child {node {$W$}
|
||||
edge from parent
|
||||
node[above] {$0.99$}
|
||||
}
|
||||
child {node {$B$}
|
||||
edge from parent
|
||||
node[above] {$0.01$}
|
||||
}
|
||||
edge from parent
|
||||
node[above] {$0.99$}
|
||||
} ;
|
||||
\end{tikzpicture}
|
||||
\end{center}
|
||||
\item On calcule la probabilité que la vente soit un wok et ait eu lieu à midi
|
||||
\[ P(M\cap W) = P(M) \times P_M(W) = 0.01 \times 0.75 = 0.0075 \]
|
||||
\item Probabilité que la vente soit un burger.
|
||||
\[
|
||||
P(B) = P(M\cap B) + P(S\cap B) = 0.01 \times 0.75 + 0.99 \times 0.99 = 0.0124
|
||||
\]
|
||||
\item On cherche à calculer la quantité $P_B(S)$. Pour cela on utilise la formule de Bayes
|
||||
\[
|
||||
P_B(S) = \frac{P(B\cap S)}{P(B)} = \frac{P_S(B) \times P(S)}{P(B)} = \frac{0.01\times 0.99}{0.0124} = 0.7983870967741936 \approx 0.798
|
||||
\]
|
||||
\end{enumerate}
|
||||
\end{solution}
|
||||
|
||||
\begin{exercise}[subtitle={Continent plastique}]
|
||||
\textit{Les quantités évoqués dans cette exercice sont générés au hasard et sont donc complètement farfelus.}
|
||||
\medskip
|
||||
Le \og continent de plastique\fg{} est la plus grande des plaques de déchets plastiques évoluant sur les océans. Elle occupe actuellement dans l'océan Pacifique une surface dont l'aire est évaluée à plus de $1,6$ million de km$^2$, entre Hawaï et la Californie.
|
||||
|
||||
En 2017, des scientifiques ont estimé qu'il y avait $5$ millions de tonnes de déchets plastiques qui était déversé chaque année dans les océans et que cette quantité augmentait de $17\n\%$ par chaque année.
|
||||
|
||||
On modélise l'évolution de la masse de ces déchets plastiques déversée chaque année, si rien n'est fait pour la réduire, par une suite géométrique $\left(u_n\right)$. L'arrondi au centième du terme $u_n$ représente la masse de ces déchets déversée chaque année, exprimée en million de tonnes, pour l'année $(2017 + n)$.
|
||||
|
||||
\medskip
|
||||
|
||||
\begin{enumerate}
|
||||
\item Expliquer pourquoi la suite $u_n$ est géométrique?
|
||||
\item Calculer $u_1$ et $u_2$.
|
||||
\item Exprimer $u_n$ en fonction de $n$.
|
||||
\item Au début de l'année 2017, il y avait $300$ millions de tonnes de déchets plastique. Calculer la quantité totale de déchets plastiques en 2030.
|
||||
\item On souhaite déterminer en quelle année la masse totale de ces déchets plastiques aura pour la première fois augmenté de $50$\,\% par rapport à sa valeur de 2017.
|
||||
\begin{enumerate}
|
||||
\item Recopier et compléter l'algorithme ci-dessous pour que la variable $N$ contienne la réponse au problème posé.
|
||||
|
||||
\begin{center}
|
||||
\begin{tabularx}{0.4\linewidth}{|X|}\hline
|
||||
$N = 2017$\\
|
||||
$U = 5$ \\
|
||||
$S = 300 + U$ \\
|
||||
while $S < 450$: \\
|
||||
\hspace{1cm} $N = \ldots$\\
|
||||
\hspace{1cm} $U = \ldots$\\
|
||||
\hspace{1cm} $S = \ldots$\\
|
||||
\hline
|
||||
\end{tabularx}
|
||||
\end{center}
|
||||
\item Que contiennent les variables $S$, $U$ et $N$ après exécution de cet algorithme ?
|
||||
|
||||
Interpréter les résultats dans le contexte de l'exercice.
|
||||
\end{enumerate}
|
||||
\end{enumerate}
|
||||
\end{exercise}
|
||||
|
||||
\begin{solution}
|
||||
\begin{enumerate}
|
||||
\item Une augmentation de $17\,\%$ revient à multiplier la quantité par $1.17$. La suite est donc bien géométrique. Son premier terme est $u_0 = 5$ et sa raison est $q = 1.17$
|
||||
\item
|
||||
\[
|
||||
u_1 = u_0 * 1.17 = 5.85
|
||||
\]
|
||||
\[
|
||||
u_2 = u_0 * 1.17^2 = 6.8445
|
||||
\]
|
||||
\item
|
||||
\[
|
||||
u_n = u_0 \times q^n = 5 \times 1.17^n
|
||||
\]
|
||||
\item On calcule la quantité totale déversée entre 2017 et 2030.
|
||||
\[
|
||||
\sum_{n = 0}^{13} u_n = u_0 \times \frac{1-q^{13}}{1-q} = 5 \times \frac{1 - 1.17^{13}}{1 - 1.17} = 197.02
|
||||
\]
|
||||
On en déduit la quantité totale de déchets en 2030
|
||||
\[
|
||||
300 + 197.02 = 497.02
|
||||
\]
|
||||
\item
|
||||
\begin{enumerate}
|
||||
\item ~
|
||||
\begin{center}
|
||||
\begin{tabularx}{0.4\linewidth}{|X|}\hline
|
||||
$N \gets 2017$\\
|
||||
$U \gets 5$ \\
|
||||
$S \gets 300 + U$ \\
|
||||
Tant que $S < 450$ \\
|
||||
\hspace{1cm} $N \gets N + 1$\\
|
||||
\hspace{1cm} $U \gets U * 1.17$\\
|
||||
\hspace{1cm} $S \gets S + u$\\
|
||||
Fin Tant que\\\hline
|
||||
\end{tabularx}
|
||||
\end{center}
|
||||
\item \textit{Pas de correction automatisé}
|
||||
\end{enumerate}
|
||||
\end{enumerate}
|
||||
\end{solution}
|
||||
|
||||
\end{document}
|
||||
|
||||
%%% Local Variables:
|
||||
%%% mode: latex
|
||||
%%% TeX-master: "master"
|
||||
%%% End:
|
262
TST/DS/DS_21_04_07/TST3/corr_16_210407_DS8.tex
Normal file
262
TST/DS/DS_21_04_07/TST3/corr_16_210407_DS8.tex
Normal file
@ -0,0 +1,262 @@
|
||||
\documentclass[a4paper,10pt]{article}
|
||||
\usepackage{myXsim}
|
||||
|
||||
% Title Page
|
||||
\title{DS8 \hfill TAY Ummuhan}
|
||||
\tribe{TST}
|
||||
\date{\hfillÀ render pour le Mercredi 7 avril}
|
||||
|
||||
\xsimsetup{
|
||||
solution/print = true
|
||||
}
|
||||
|
||||
\begin{document}
|
||||
\maketitle
|
||||
|
||||
\begin{exercise}[subtitle={Automatismes}]
|
||||
\textit{Toutes les questions de cette exercice sont indépendantes et peuvent être répondus séparément}
|
||||
\begin{enumerate}
|
||||
\item De janvier à septembre, une quantité a augmenté de $14\,\%$. Faire un schéma pour représenter la situation puis calculer le taux d'évolution moyen mensuel.
|
||||
\item Une quantité augmente de $14\,\%$ par ans. En 2020, elle est de 116\euro. Quelle était sa valeur en 2019? Faire un schéma pour représenter la situation.
|
||||
\item Déterminer l'équation de la droite \\
|
||||
\begin{tikzpicture}[xscale=0.8, yscale=0.5]
|
||||
\tkzInit[xmin=-5,xmax=5,xstep=1,
|
||||
ymin=-5,ymax=5,ystep=1]
|
||||
\tkzGrid
|
||||
\tkzAxeXY
|
||||
\tkzFct[domain=-5:5,color=red,very thick]%
|
||||
{1.3333333333333333*\x -2};
|
||||
\end{tikzpicture}
|
||||
\item Résoudre l'équation $6 \times 0.14^x = 14$
|
||||
\end{enumerate}
|
||||
\end{exercise}
|
||||
|
||||
\begin{solution}
|
||||
\begin{enumerate}
|
||||
\item On veut partager cette évolution en 8 évolutions.
|
||||
\[
|
||||
\left(1 + \frac{14}{100}\right)^{\frac{1}{8}} = 1.0165
|
||||
\]
|
||||
Donc le taux d'évolution moyen est
|
||||
\[
|
||||
t_m = 1.0165 - 1 = 0.01649999999999996
|
||||
\]
|
||||
\item Coefficient multiplicateur pour revenir en arrière
|
||||
\[
|
||||
CM = (1 + \frac{14}{100})^{-1} = 0.8772
|
||||
\]
|
||||
On en déduit la quantité en 2019
|
||||
\[
|
||||
116 * 0.8772 = 101.7552
|
||||
\]
|
||||
\item L'équation de la droite est
|
||||
\[
|
||||
y = 1.3333333333333333 x -2
|
||||
\]
|
||||
\item Il faut penser à faire la division à par $6$ avant d'utiliser le log car sinon, on ne peut pas utiliser la formule $\log(a^n) = n\times \log(a)$.
|
||||
|
||||
\[x = \frac{\log(2.33)}{\log(0.14)}\]
|
||||
\end{enumerate}
|
||||
\end{solution}
|
||||
|
||||
\begin{exercise}[subtitle={Restaurant}]
|
||||
Un \emph{food truck}, ouvert le midi et le soir, propose deux types de formules :
|
||||
|
||||
\setlength\parindent{10mm}
|
||||
\begin{itemize}
|
||||
\item la formule \emph{Burger} ;
|
||||
\item la formule \emph{Wok}.
|
||||
\end{itemize}
|
||||
\setlength\parindent{0mm}
|
||||
|
||||
\medskip
|
||||
|
||||
Le gérant a remarqué que 28\,\% de ses ventes ont lieu le midi. Le quart des ventes du midi correspondent à la formule \emph{Burger}, alors que 80\,\% des ventes du soir correspondent à la formule \emph{Wok}.
|
||||
|
||||
Le gérant se constitue un fichier en notant, pour chaque vente, la formule choisie et le moment de cette vente (midi ou soir).
|
||||
|
||||
On prélève une fiche de façon équiprobable. On définit les quatre évènements suivants:
|
||||
|
||||
\begin{enumerate}
|
||||
\item $M$ : \og la fiche correspond à une vente du midi\fg{} ;
|
||||
\item $S$ : \og la fiche correspond à une vente du soir\fg {};
|
||||
\item $W$ : \og la fiche correspond à une formule \emph{Wok} \fg{} ;
|
||||
\item $B$ : \og la fiche correspond à une formule \emph{Burger} \fg.
|
||||
\end{enumerate}
|
||||
\setlength\parindent{0mm}
|
||||
|
||||
\medskip
|
||||
|
||||
\begin{enumerate}
|
||||
\item Recopier puis compléter l'arbre pondéré
|
||||
|
||||
\begin{center}
|
||||
\begin{tikzpicture}[sloped]
|
||||
\node {.}
|
||||
child {node {$M$}
|
||||
child {node {$W$}
|
||||
edge from parent
|
||||
node[above] {...}
|
||||
}
|
||||
child {node {$B$}
|
||||
edge from parent
|
||||
node[above] {...}
|
||||
}
|
||||
edge from parent
|
||||
node[above] {...}
|
||||
}
|
||||
child[missing] {}
|
||||
child { node {$S$}
|
||||
child {node {$W$}
|
||||
edge from parent
|
||||
node[above] {...}
|
||||
}
|
||||
child {node {$B$}
|
||||
edge from parent
|
||||
node[above] {...}
|
||||
}
|
||||
edge from parent
|
||||
node[above] {...}
|
||||
} ;
|
||||
\end{tikzpicture}
|
||||
\end{center}
|
||||
|
||||
\item Calculer la probabilité de l'évènement $M \cap W$. Interpréter ce résultat dans le contexte de l'exercice.
|
||||
\item Montrer que la probabilité que la fiche choisie corresponde à une formule \emph{Burger} est égale à $0.214$.
|
||||
\item On a prélevé une fiche correspondant à la formule \emph{Burger}. Quelle est la probabilité, arrondie au millième, que la vente ait eu lieu le soir?
|
||||
\end{enumerate}
|
||||
\end{exercise}
|
||||
|
||||
\begin{solution}
|
||||
\begin{enumerate}
|
||||
\item
|
||||
\begin{center}
|
||||
\begin{tikzpicture}[sloped]
|
||||
\node {.}
|
||||
child {node {$M$}
|
||||
child {node {$W$}
|
||||
edge from parent
|
||||
node[above] {$0.75$}
|
||||
}
|
||||
child {node {$B$}
|
||||
edge from parent
|
||||
node[above] {$0.25$}
|
||||
}
|
||||
edge from parent
|
||||
node[above] {$0.28$}
|
||||
}
|
||||
child[missing] {}
|
||||
child { node {$S$}
|
||||
child {node {$W$}
|
||||
edge from parent
|
||||
node[above] {$0.8$}
|
||||
}
|
||||
child {node {$B$}
|
||||
edge from parent
|
||||
node[above] {$0.2$}
|
||||
}
|
||||
edge from parent
|
||||
node[above] {$0.72$}
|
||||
} ;
|
||||
\end{tikzpicture}
|
||||
\end{center}
|
||||
\item On calcule la probabilité que la vente soit un wok et ait eu lieu à midi
|
||||
\[ P(M\cap W) = P(M) \times P_M(W) = 0.28 \times 0.75 = 0.21 \]
|
||||
\item Probabilité que la vente soit un burger.
|
||||
\[
|
||||
P(B) = P(M\cap B) + P(S\cap B) = 0.28 \times 0.75 + 0.72 \times 0.8 = 0.214
|
||||
\]
|
||||
\item On cherche à calculer la quantité $P_B(S)$. Pour cela on utilise la formule de Bayes
|
||||
\[
|
||||
P_B(S) = \frac{P(B\cap S)}{P(B)} = \frac{P_S(B) \times P(S)}{P(B)} = \frac{0.2\times 0.72}{0.214} = 0.6728971962616822 \approx 0.673
|
||||
\]
|
||||
\end{enumerate}
|
||||
\end{solution}
|
||||
|
||||
\begin{exercise}[subtitle={Continent plastique}]
|
||||
\textit{Les quantités évoqués dans cette exercice sont générés au hasard et sont donc complètement farfelus.}
|
||||
\medskip
|
||||
Le \og continent de plastique\fg{} est la plus grande des plaques de déchets plastiques évoluant sur les océans. Elle occupe actuellement dans l'océan Pacifique une surface dont l'aire est évaluée à plus de $1,6$ million de km$^2$, entre Hawaï et la Californie.
|
||||
|
||||
En 2017, des scientifiques ont estimé qu'il y avait $7$ millions de tonnes de déchets plastiques qui était déversé chaque année dans les océans et que cette quantité augmentait de $26\n\%$ par chaque année.
|
||||
|
||||
On modélise l'évolution de la masse de ces déchets plastiques déversée chaque année, si rien n'est fait pour la réduire, par une suite géométrique $\left(u_n\right)$. L'arrondi au centième du terme $u_n$ représente la masse de ces déchets déversée chaque année, exprimée en million de tonnes, pour l'année $(2017 + n)$.
|
||||
|
||||
\medskip
|
||||
|
||||
\begin{enumerate}
|
||||
\item Expliquer pourquoi la suite $u_n$ est géométrique?
|
||||
\item Calculer $u_1$ et $u_2$.
|
||||
\item Exprimer $u_n$ en fonction de $n$.
|
||||
\item Au début de l'année 2017, il y avait $300$ millions de tonnes de déchets plastique. Calculer la quantité totale de déchets plastiques en 2030.
|
||||
\item On souhaite déterminer en quelle année la masse totale de ces déchets plastiques aura pour la première fois augmenté de $50$\,\% par rapport à sa valeur de 2017.
|
||||
\begin{enumerate}
|
||||
\item Recopier et compléter l'algorithme ci-dessous pour que la variable $N$ contienne la réponse au problème posé.
|
||||
|
||||
\begin{center}
|
||||
\begin{tabularx}{0.4\linewidth}{|X|}\hline
|
||||
$N = 2017$\\
|
||||
$U = 7$ \\
|
||||
$S = 300 + U$ \\
|
||||
while $S < 450$: \\
|
||||
\hspace{1cm} $N = \ldots$\\
|
||||
\hspace{1cm} $U = \ldots$\\
|
||||
\hspace{1cm} $S = \ldots$\\
|
||||
\hline
|
||||
\end{tabularx}
|
||||
\end{center}
|
||||
\item Que contiennent les variables $S$, $U$ et $N$ après exécution de cet algorithme ?
|
||||
|
||||
Interpréter les résultats dans le contexte de l'exercice.
|
||||
\end{enumerate}
|
||||
\end{enumerate}
|
||||
\end{exercise}
|
||||
|
||||
\begin{solution}
|
||||
\begin{enumerate}
|
||||
\item Une augmentation de $26\,\%$ revient à multiplier la quantité par $1.26$. La suite est donc bien géométrique. Son premier terme est $u_0 = 7$ et sa raison est $q = 1.26$
|
||||
\item
|
||||
\[
|
||||
u_1 = u_0 * 1.26 = 8.82
|
||||
\]
|
||||
\[
|
||||
u_2 = u_0 * 1.26^2 = 11.1132
|
||||
\]
|
||||
\item
|
||||
\[
|
||||
u_n = u_0 \times q^n = 7 \times 1.26^n
|
||||
\]
|
||||
\item On calcule la quantité totale déversée entre 2017 et 2030.
|
||||
\[
|
||||
\sum_{n = 0}^{13} u_n = u_0 \times \frac{1-q^{13}}{1-q} = 7 \times \frac{1 - 1.26^{13}}{1 - 1.26} = 516.25
|
||||
\]
|
||||
On en déduit la quantité totale de déchets en 2030
|
||||
\[
|
||||
300 + 516.25 = 816.25
|
||||
\]
|
||||
\item
|
||||
\begin{enumerate}
|
||||
\item ~
|
||||
\begin{center}
|
||||
\begin{tabularx}{0.4\linewidth}{|X|}\hline
|
||||
$N \gets 2017$\\
|
||||
$U \gets 7$ \\
|
||||
$S \gets 300 + U$ \\
|
||||
Tant que $S < 450$ \\
|
||||
\hspace{1cm} $N \gets N + 1$\\
|
||||
\hspace{1cm} $U \gets U * 1.26$\\
|
||||
\hspace{1cm} $S \gets S + u$\\
|
||||
Fin Tant que\\\hline
|
||||
\end{tabularx}
|
||||
\end{center}
|
||||
\item \textit{Pas de correction automatisé}
|
||||
\end{enumerate}
|
||||
\end{enumerate}
|
||||
\end{solution}
|
||||
|
||||
\end{document}
|
||||
|
||||
%%% Local Variables:
|
||||
%%% mode: latex
|
||||
%%% TeX-master: "master"
|
||||
%%% End:
|
262
TST/DS/DS_21_04_07/TST3/corr_17_210407_DS8.tex
Normal file
262
TST/DS/DS_21_04_07/TST3/corr_17_210407_DS8.tex
Normal file
@ -0,0 +1,262 @@
|
||||
\documentclass[a4paper,10pt]{article}
|
||||
\usepackage{myXsim}
|
||||
|
||||
% Title Page
|
||||
\title{DS8 \hfill VIALON-DUPERRON Victorien}
|
||||
\tribe{TST}
|
||||
\date{\hfillÀ render pour le Mercredi 7 avril}
|
||||
|
||||
\xsimsetup{
|
||||
solution/print = true
|
||||
}
|
||||
|
||||
\begin{document}
|
||||
\maketitle
|
||||
|
||||
\begin{exercise}[subtitle={Automatismes}]
|
||||
\textit{Toutes les questions de cette exercice sont indépendantes et peuvent être répondus séparément}
|
||||
\begin{enumerate}
|
||||
\item De janvier à septembre, une quantité a augmenté de $20\,\%$. Faire un schéma pour représenter la situation puis calculer le taux d'évolution moyen mensuel.
|
||||
\item Une quantité augmente de $20\,\%$ par ans. En 2020, elle est de 137\euro. Quelle était sa valeur en 2019? Faire un schéma pour représenter la situation.
|
||||
\item Déterminer l'équation de la droite \\
|
||||
\begin{tikzpicture}[xscale=0.8, yscale=0.5]
|
||||
\tkzInit[xmin=-5,xmax=5,xstep=1,
|
||||
ymin=-5,ymax=5,ystep=1]
|
||||
\tkzGrid
|
||||
\tkzAxeXY
|
||||
\tkzFct[domain=-5:5,color=red,very thick]%
|
||||
{2.6666666666666665*\x -4};
|
||||
\end{tikzpicture}
|
||||
\item Résoudre l'équation $10 \times 0.26^x = 42$
|
||||
\end{enumerate}
|
||||
\end{exercise}
|
||||
|
||||
\begin{solution}
|
||||
\begin{enumerate}
|
||||
\item On veut partager cette évolution en 8 évolutions.
|
||||
\[
|
||||
\left(1 + \frac{20}{100}\right)^{\frac{1}{8}} = 1.0231
|
||||
\]
|
||||
Donc le taux d'évolution moyen est
|
||||
\[
|
||||
t_m = 1.0231 - 1 = 0.0230999999999999
|
||||
\]
|
||||
\item Coefficient multiplicateur pour revenir en arrière
|
||||
\[
|
||||
CM = (1 + \frac{20}{100})^{-1} = 0.8333
|
||||
\]
|
||||
On en déduit la quantité en 2019
|
||||
\[
|
||||
137 * 0.8333 = 114.16210000000001
|
||||
\]
|
||||
\item L'équation de la droite est
|
||||
\[
|
||||
y = 2.6666666666666665 x -4
|
||||
\]
|
||||
\item Il faut penser à faire la division à par $10$ avant d'utiliser le log car sinon, on ne peut pas utiliser la formule $\log(a^n) = n\times \log(a)$.
|
||||
|
||||
\[x = \frac{\log(4.2)}{\log(0.26)}\]
|
||||
\end{enumerate}
|
||||
\end{solution}
|
||||
|
||||
\begin{exercise}[subtitle={Restaurant}]
|
||||
Un \emph{food truck}, ouvert le midi et le soir, propose deux types de formules :
|
||||
|
||||
\setlength\parindent{10mm}
|
||||
\begin{itemize}
|
||||
\item la formule \emph{Burger} ;
|
||||
\item la formule \emph{Wok}.
|
||||
\end{itemize}
|
||||
\setlength\parindent{0mm}
|
||||
|
||||
\medskip
|
||||
|
||||
Le gérant a remarqué que 87\,\% de ses ventes ont lieu le midi. Le quart des ventes du midi correspondent à la formule \emph{Burger}, alors que 47\,\% des ventes du soir correspondent à la formule \emph{Wok}.
|
||||
|
||||
Le gérant se constitue un fichier en notant, pour chaque vente, la formule choisie et le moment de cette vente (midi ou soir).
|
||||
|
||||
On prélève une fiche de façon équiprobable. On définit les quatre évènements suivants:
|
||||
|
||||
\begin{enumerate}
|
||||
\item $M$ : \og la fiche correspond à une vente du midi\fg{} ;
|
||||
\item $S$ : \og la fiche correspond à une vente du soir\fg {};
|
||||
\item $W$ : \og la fiche correspond à une formule \emph{Wok} \fg{} ;
|
||||
\item $B$ : \og la fiche correspond à une formule \emph{Burger} \fg.
|
||||
\end{enumerate}
|
||||
\setlength\parindent{0mm}
|
||||
|
||||
\medskip
|
||||
|
||||
\begin{enumerate}
|
||||
\item Recopier puis compléter l'arbre pondéré
|
||||
|
||||
\begin{center}
|
||||
\begin{tikzpicture}[sloped]
|
||||
\node {.}
|
||||
child {node {$M$}
|
||||
child {node {$W$}
|
||||
edge from parent
|
||||
node[above] {...}
|
||||
}
|
||||
child {node {$B$}
|
||||
edge from parent
|
||||
node[above] {...}
|
||||
}
|
||||
edge from parent
|
||||
node[above] {...}
|
||||
}
|
||||
child[missing] {}
|
||||
child { node {$S$}
|
||||
child {node {$W$}
|
||||
edge from parent
|
||||
node[above] {...}
|
||||
}
|
||||
child {node {$B$}
|
||||
edge from parent
|
||||
node[above] {...}
|
||||
}
|
||||
edge from parent
|
||||
node[above] {...}
|
||||
} ;
|
||||
\end{tikzpicture}
|
||||
\end{center}
|
||||
|
||||
\item Calculer la probabilité de l'évènement $M \cap W$. Interpréter ce résultat dans le contexte de l'exercice.
|
||||
\item Montrer que la probabilité que la fiche choisie corresponde à une formule \emph{Burger} est égale à $0.2864$.
|
||||
\item On a prélevé une fiche correspondant à la formule \emph{Burger}. Quelle est la probabilité, arrondie au millième, que la vente ait eu lieu le soir?
|
||||
\end{enumerate}
|
||||
\end{exercise}
|
||||
|
||||
\begin{solution}
|
||||
\begin{enumerate}
|
||||
\item
|
||||
\begin{center}
|
||||
\begin{tikzpicture}[sloped]
|
||||
\node {.}
|
||||
child {node {$M$}
|
||||
child {node {$W$}
|
||||
edge from parent
|
||||
node[above] {$0.75$}
|
||||
}
|
||||
child {node {$B$}
|
||||
edge from parent
|
||||
node[above] {$0.25$}
|
||||
}
|
||||
edge from parent
|
||||
node[above] {$0.87$}
|
||||
}
|
||||
child[missing] {}
|
||||
child { node {$S$}
|
||||
child {node {$W$}
|
||||
edge from parent
|
||||
node[above] {$0.47$}
|
||||
}
|
||||
child {node {$B$}
|
||||
edge from parent
|
||||
node[above] {$0.53$}
|
||||
}
|
||||
edge from parent
|
||||
node[above] {$0.13$}
|
||||
} ;
|
||||
\end{tikzpicture}
|
||||
\end{center}
|
||||
\item On calcule la probabilité que la vente soit un wok et ait eu lieu à midi
|
||||
\[ P(M\cap W) = P(M) \times P_M(W) = 0.87 \times 0.75 = 0.6525 \]
|
||||
\item Probabilité que la vente soit un burger.
|
||||
\[
|
||||
P(B) = P(M\cap B) + P(S\cap B) = 0.87 \times 0.75 + 0.13 \times 0.47 = 0.2864
|
||||
\]
|
||||
\item On cherche à calculer la quantité $P_B(S)$. Pour cela on utilise la formule de Bayes
|
||||
\[
|
||||
P_B(S) = \frac{P(B\cap S)}{P(B)} = \frac{P_S(B) \times P(S)}{P(B)} = \frac{0.53\times 0.13}{0.2864} = 0.24057262569832405 \approx 0.241
|
||||
\]
|
||||
\end{enumerate}
|
||||
\end{solution}
|
||||
|
||||
\begin{exercise}[subtitle={Continent plastique}]
|
||||
\textit{Les quantités évoqués dans cette exercice sont générés au hasard et sont donc complètement farfelus.}
|
||||
\medskip
|
||||
Le \og continent de plastique\fg{} est la plus grande des plaques de déchets plastiques évoluant sur les océans. Elle occupe actuellement dans l'océan Pacifique une surface dont l'aire est évaluée à plus de $1,6$ million de km$^2$, entre Hawaï et la Californie.
|
||||
|
||||
En 2017, des scientifiques ont estimé qu'il y avait $14$ millions de tonnes de déchets plastiques qui était déversé chaque année dans les océans et que cette quantité augmentait de $17\n\%$ par chaque année.
|
||||
|
||||
On modélise l'évolution de la masse de ces déchets plastiques déversée chaque année, si rien n'est fait pour la réduire, par une suite géométrique $\left(u_n\right)$. L'arrondi au centième du terme $u_n$ représente la masse de ces déchets déversée chaque année, exprimée en million de tonnes, pour l'année $(2017 + n)$.
|
||||
|
||||
\medskip
|
||||
|
||||
\begin{enumerate}
|
||||
\item Expliquer pourquoi la suite $u_n$ est géométrique?
|
||||
\item Calculer $u_1$ et $u_2$.
|
||||
\item Exprimer $u_n$ en fonction de $n$.
|
||||
\item Au début de l'année 2017, il y avait $300$ millions de tonnes de déchets plastique. Calculer la quantité totale de déchets plastiques en 2030.
|
||||
\item On souhaite déterminer en quelle année la masse totale de ces déchets plastiques aura pour la première fois augmenté de $50$\,\% par rapport à sa valeur de 2017.
|
||||
\begin{enumerate}
|
||||
\item Recopier et compléter l'algorithme ci-dessous pour que la variable $N$ contienne la réponse au problème posé.
|
||||
|
||||
\begin{center}
|
||||
\begin{tabularx}{0.4\linewidth}{|X|}\hline
|
||||
$N = 2017$\\
|
||||
$U = 14$ \\
|
||||
$S = 300 + U$ \\
|
||||
while $S < 450$: \\
|
||||
\hspace{1cm} $N = \ldots$\\
|
||||
\hspace{1cm} $U = \ldots$\\
|
||||
\hspace{1cm} $S = \ldots$\\
|
||||
\hline
|
||||
\end{tabularx}
|
||||
\end{center}
|
||||
\item Que contiennent les variables $S$, $U$ et $N$ après exécution de cet algorithme ?
|
||||
|
||||
Interpréter les résultats dans le contexte de l'exercice.
|
||||
\end{enumerate}
|
||||
\end{enumerate}
|
||||
\end{exercise}
|
||||
|
||||
\begin{solution}
|
||||
\begin{enumerate}
|
||||
\item Une augmentation de $17\,\%$ revient à multiplier la quantité par $1.17$. La suite est donc bien géométrique. Son premier terme est $u_0 = 14$ et sa raison est $q = 1.17$
|
||||
\item
|
||||
\[
|
||||
u_1 = u_0 * 1.17 = 16.38
|
||||
\]
|
||||
\[
|
||||
u_2 = u_0 * 1.17^2 = 19.1646
|
||||
\]
|
||||
\item
|
||||
\[
|
||||
u_n = u_0 \times q^n = 14 \times 1.17^n
|
||||
\]
|
||||
\item On calcule la quantité totale déversée entre 2017 et 2030.
|
||||
\[
|
||||
\sum_{n = 0}^{13} u_n = u_0 \times \frac{1-q^{13}}{1-q} = 14 \times \frac{1 - 1.17^{13}}{1 - 1.17} = 551.66
|
||||
\]
|
||||
On en déduit la quantité totale de déchets en 2030
|
||||
\[
|
||||
300 + 551.66 = 851.66
|
||||
\]
|
||||
\item
|
||||
\begin{enumerate}
|
||||
\item ~
|
||||
\begin{center}
|
||||
\begin{tabularx}{0.4\linewidth}{|X|}\hline
|
||||
$N \gets 2017$\\
|
||||
$U \gets 14$ \\
|
||||
$S \gets 300 + U$ \\
|
||||
Tant que $S < 450$ \\
|
||||
\hspace{1cm} $N \gets N + 1$\\
|
||||
\hspace{1cm} $U \gets U * 1.17$\\
|
||||
\hspace{1cm} $S \gets S + u$\\
|
||||
Fin Tant que\\\hline
|
||||
\end{tabularx}
|
||||
\end{center}
|
||||
\item \textit{Pas de correction automatisé}
|
||||
\end{enumerate}
|
||||
\end{enumerate}
|
||||
\end{solution}
|
||||
|
||||
\end{document}
|
||||
|
||||
%%% Local Variables:
|
||||
%%% mode: latex
|
||||
%%% TeX-master: "master"
|
||||
%%% End:
|
262
TST/DS/DS_21_04_07/TST3/corr_18_210407_DS8.tex
Normal file
262
TST/DS/DS_21_04_07/TST3/corr_18_210407_DS8.tex
Normal file
@ -0,0 +1,262 @@
|
||||
\documentclass[a4paper,10pt]{article}
|
||||
\usepackage{myXsim}
|
||||
|
||||
% Title Page
|
||||
\title{DS8 \hfill ZENAGUI Yanis}
|
||||
\tribe{TST}
|
||||
\date{\hfillÀ render pour le Mercredi 7 avril}
|
||||
|
||||
\xsimsetup{
|
||||
solution/print = true
|
||||
}
|
||||
|
||||
\begin{document}
|
||||
\maketitle
|
||||
|
||||
\begin{exercise}[subtitle={Automatismes}]
|
||||
\textit{Toutes les questions de cette exercice sont indépendantes et peuvent être répondus séparément}
|
||||
\begin{enumerate}
|
||||
\item De janvier à septembre, une quantité a augmenté de $29\,\%$. Faire un schéma pour représenter la situation puis calculer le taux d'évolution moyen mensuel.
|
||||
\item Une quantité augmente de $29\,\%$ par ans. En 2020, elle est de 142\euro. Quelle était sa valeur en 2019? Faire un schéma pour représenter la situation.
|
||||
\item Déterminer l'équation de la droite \\
|
||||
\begin{tikzpicture}[xscale=0.8, yscale=0.5]
|
||||
\tkzInit[xmin=-5,xmax=5,xstep=1,
|
||||
ymin=-5,ymax=5,ystep=1]
|
||||
\tkzGrid
|
||||
\tkzAxeXY
|
||||
\tkzFct[domain=-5:5,color=red,very thick]%
|
||||
{2.0*\x -4};
|
||||
\end{tikzpicture}
|
||||
\item Résoudre l'équation $6 \times 0.41^x = 34$
|
||||
\end{enumerate}
|
||||
\end{exercise}
|
||||
|
||||
\begin{solution}
|
||||
\begin{enumerate}
|
||||
\item On veut partager cette évolution en 8 évolutions.
|
||||
\[
|
||||
\left(1 + \frac{29}{100}\right)^{\frac{1}{8}} = 1.0323
|
||||
\]
|
||||
Donc le taux d'évolution moyen est
|
||||
\[
|
||||
t_m = 1.0323 - 1 = 0.032299999999999995
|
||||
\]
|
||||
\item Coefficient multiplicateur pour revenir en arrière
|
||||
\[
|
||||
CM = (1 + \frac{29}{100})^{-1} = 0.7752
|
||||
\]
|
||||
On en déduit la quantité en 2019
|
||||
\[
|
||||
142 * 0.7752 = 110.0784
|
||||
\]
|
||||
\item L'équation de la droite est
|
||||
\[
|
||||
y = 2.0 x -4
|
||||
\]
|
||||
\item Il faut penser à faire la division à par $6$ avant d'utiliser le log car sinon, on ne peut pas utiliser la formule $\log(a^n) = n\times \log(a)$.
|
||||
|
||||
\[x = \frac{\log(5.67)}{\log(0.41)}\]
|
||||
\end{enumerate}
|
||||
\end{solution}
|
||||
|
||||
\begin{exercise}[subtitle={Restaurant}]
|
||||
Un \emph{food truck}, ouvert le midi et le soir, propose deux types de formules :
|
||||
|
||||
\setlength\parindent{10mm}
|
||||
\begin{itemize}
|
||||
\item la formule \emph{Burger} ;
|
||||
\item la formule \emph{Wok}.
|
||||
\end{itemize}
|
||||
\setlength\parindent{0mm}
|
||||
|
||||
\medskip
|
||||
|
||||
Le gérant a remarqué que 30\,\% de ses ventes ont lieu le midi. Le quart des ventes du midi correspondent à la formule \emph{Burger}, alors que 33\,\% des ventes du soir correspondent à la formule \emph{Wok}.
|
||||
|
||||
Le gérant se constitue un fichier en notant, pour chaque vente, la formule choisie et le moment de cette vente (midi ou soir).
|
||||
|
||||
On prélève une fiche de façon équiprobable. On définit les quatre évènements suivants:
|
||||
|
||||
\begin{enumerate}
|
||||
\item $M$ : \og la fiche correspond à une vente du midi\fg{} ;
|
||||
\item $S$ : \og la fiche correspond à une vente du soir\fg {};
|
||||
\item $W$ : \og la fiche correspond à une formule \emph{Wok} \fg{} ;
|
||||
\item $B$ : \og la fiche correspond à une formule \emph{Burger} \fg.
|
||||
\end{enumerate}
|
||||
\setlength\parindent{0mm}
|
||||
|
||||
\medskip
|
||||
|
||||
\begin{enumerate}
|
||||
\item Recopier puis compléter l'arbre pondéré
|
||||
|
||||
\begin{center}
|
||||
\begin{tikzpicture}[sloped]
|
||||
\node {.}
|
||||
child {node {$M$}
|
||||
child {node {$W$}
|
||||
edge from parent
|
||||
node[above] {...}
|
||||
}
|
||||
child {node {$B$}
|
||||
edge from parent
|
||||
node[above] {...}
|
||||
}
|
||||
edge from parent
|
||||
node[above] {...}
|
||||
}
|
||||
child[missing] {}
|
||||
child { node {$S$}
|
||||
child {node {$W$}
|
||||
edge from parent
|
||||
node[above] {...}
|
||||
}
|
||||
child {node {$B$}
|
||||
edge from parent
|
||||
node[above] {...}
|
||||
}
|
||||
edge from parent
|
||||
node[above] {...}
|
||||
} ;
|
||||
\end{tikzpicture}
|
||||
\end{center}
|
||||
|
||||
\item Calculer la probabilité de l'évènement $M \cap W$. Interpréter ce résultat dans le contexte de l'exercice.
|
||||
\item Montrer que la probabilité que la fiche choisie corresponde à une formule \emph{Burger} est égale à $0.544$.
|
||||
\item On a prélevé une fiche correspondant à la formule \emph{Burger}. Quelle est la probabilité, arrondie au millième, que la vente ait eu lieu le soir?
|
||||
\end{enumerate}
|
||||
\end{exercise}
|
||||
|
||||
\begin{solution}
|
||||
\begin{enumerate}
|
||||
\item
|
||||
\begin{center}
|
||||
\begin{tikzpicture}[sloped]
|
||||
\node {.}
|
||||
child {node {$M$}
|
||||
child {node {$W$}
|
||||
edge from parent
|
||||
node[above] {$0.75$}
|
||||
}
|
||||
child {node {$B$}
|
||||
edge from parent
|
||||
node[above] {$0.25$}
|
||||
}
|
||||
edge from parent
|
||||
node[above] {$0.3$}
|
||||
}
|
||||
child[missing] {}
|
||||
child { node {$S$}
|
||||
child {node {$W$}
|
||||
edge from parent
|
||||
node[above] {$0.33$}
|
||||
}
|
||||
child {node {$B$}
|
||||
edge from parent
|
||||
node[above] {$0.67$}
|
||||
}
|
||||
edge from parent
|
||||
node[above] {$0.7$}
|
||||
} ;
|
||||
\end{tikzpicture}
|
||||
\end{center}
|
||||
\item On calcule la probabilité que la vente soit un wok et ait eu lieu à midi
|
||||
\[ P(M\cap W) = P(M) \times P_M(W) = 0.3 \times 0.75 = 0.225 \]
|
||||
\item Probabilité que la vente soit un burger.
|
||||
\[
|
||||
P(B) = P(M\cap B) + P(S\cap B) = 0.3 \times 0.75 + 0.7 \times 0.33 = 0.544
|
||||
\]
|
||||
\item On cherche à calculer la quantité $P_B(S)$. Pour cela on utilise la formule de Bayes
|
||||
\[
|
||||
P_B(S) = \frac{P(B\cap S)}{P(B)} = \frac{P_S(B) \times P(S)}{P(B)} = \frac{0.67\times 0.7}{0.544} = 0.8621323529411764 \approx 0.862
|
||||
\]
|
||||
\end{enumerate}
|
||||
\end{solution}
|
||||
|
||||
\begin{exercise}[subtitle={Continent plastique}]
|
||||
\textit{Les quantités évoqués dans cette exercice sont générés au hasard et sont donc complètement farfelus.}
|
||||
\medskip
|
||||
Le \og continent de plastique\fg{} est la plus grande des plaques de déchets plastiques évoluant sur les océans. Elle occupe actuellement dans l'océan Pacifique une surface dont l'aire est évaluée à plus de $1,6$ million de km$^2$, entre Hawaï et la Californie.
|
||||
|
||||
En 2017, des scientifiques ont estimé qu'il y avait $4$ millions de tonnes de déchets plastiques qui était déversé chaque année dans les océans et que cette quantité augmentait de $18\n\%$ par chaque année.
|
||||
|
||||
On modélise l'évolution de la masse de ces déchets plastiques déversée chaque année, si rien n'est fait pour la réduire, par une suite géométrique $\left(u_n\right)$. L'arrondi au centième du terme $u_n$ représente la masse de ces déchets déversée chaque année, exprimée en million de tonnes, pour l'année $(2017 + n)$.
|
||||
|
||||
\medskip
|
||||
|
||||
\begin{enumerate}
|
||||
\item Expliquer pourquoi la suite $u_n$ est géométrique?
|
||||
\item Calculer $u_1$ et $u_2$.
|
||||
\item Exprimer $u_n$ en fonction de $n$.
|
||||
\item Au début de l'année 2017, il y avait $300$ millions de tonnes de déchets plastique. Calculer la quantité totale de déchets plastiques en 2030.
|
||||
\item On souhaite déterminer en quelle année la masse totale de ces déchets plastiques aura pour la première fois augmenté de $50$\,\% par rapport à sa valeur de 2017.
|
||||
\begin{enumerate}
|
||||
\item Recopier et compléter l'algorithme ci-dessous pour que la variable $N$ contienne la réponse au problème posé.
|
||||
|
||||
\begin{center}
|
||||
\begin{tabularx}{0.4\linewidth}{|X|}\hline
|
||||
$N = 2017$\\
|
||||
$U = 4$ \\
|
||||
$S = 300 + U$ \\
|
||||
while $S < 450$: \\
|
||||
\hspace{1cm} $N = \ldots$\\
|
||||
\hspace{1cm} $U = \ldots$\\
|
||||
\hspace{1cm} $S = \ldots$\\
|
||||
\hline
|
||||
\end{tabularx}
|
||||
\end{center}
|
||||
\item Que contiennent les variables $S$, $U$ et $N$ après exécution de cet algorithme ?
|
||||
|
||||
Interpréter les résultats dans le contexte de l'exercice.
|
||||
\end{enumerate}
|
||||
\end{enumerate}
|
||||
\end{exercise}
|
||||
|
||||
\begin{solution}
|
||||
\begin{enumerate}
|
||||
\item Une augmentation de $18\,\%$ revient à multiplier la quantité par $1.18$. La suite est donc bien géométrique. Son premier terme est $u_0 = 4$ et sa raison est $q = 1.18$
|
||||
\item
|
||||
\[
|
||||
u_1 = u_0 * 1.18 = 4.72
|
||||
\]
|
||||
\[
|
||||
u_2 = u_0 * 1.18^2 = 5.5696
|
||||
\]
|
||||
\item
|
||||
\[
|
||||
u_n = u_0 \times q^n = 4 \times 1.18^n
|
||||
\]
|
||||
\item On calcule la quantité totale déversée entre 2017 et 2030.
|
||||
\[
|
||||
\sum_{n = 0}^{13} u_n = u_0 \times \frac{1-q^{13}}{1-q} = 4 \times \frac{1 - 1.18^{13}}{1 - 1.18} = 168.87
|
||||
\]
|
||||
On en déduit la quantité totale de déchets en 2030
|
||||
\[
|
||||
300 + 168.87 = 468.87
|
||||
\]
|
||||
\item
|
||||
\begin{enumerate}
|
||||
\item ~
|
||||
\begin{center}
|
||||
\begin{tabularx}{0.4\linewidth}{|X|}\hline
|
||||
$N \gets 2017$\\
|
||||
$U \gets 4$ \\
|
||||
$S \gets 300 + U$ \\
|
||||
Tant que $S < 450$ \\
|
||||
\hspace{1cm} $N \gets N + 1$\\
|
||||
\hspace{1cm} $U \gets U * 1.18$\\
|
||||
\hspace{1cm} $S \gets S + u$\\
|
||||
Fin Tant que\\\hline
|
||||
\end{tabularx}
|
||||
\end{center}
|
||||
\item \textit{Pas de correction automatisé}
|
||||
\end{enumerate}
|
||||
\end{enumerate}
|
||||
\end{solution}
|
||||
|
||||
\end{document}
|
||||
|
||||
%%% Local Variables:
|
||||
%%% mode: latex
|
||||
%%% TeX-master: "master"
|
||||
%%% End:
|
262
TST/DS/DS_21_04_07/TST3/corr_19_210407_DS8.tex
Normal file
262
TST/DS/DS_21_04_07/TST3/corr_19_210407_DS8.tex
Normal file
@ -0,0 +1,262 @@
|
||||
\documentclass[a4paper,10pt]{article}
|
||||
\usepackage{myXsim}
|
||||
|
||||
% Title Page
|
||||
\title{DS8 \hfill VIALON-DUPERRON Victorien}
|
||||
\tribe{TST}
|
||||
\date{\hfillÀ render pour le Mercredi 7 avril}
|
||||
|
||||
\xsimsetup{
|
||||
solution/print = true
|
||||
}
|
||||
|
||||
\begin{document}
|
||||
\maketitle
|
||||
|
||||
\begin{exercise}[subtitle={Automatismes}]
|
||||
\textit{Toutes les questions de cette exercice sont indépendantes et peuvent être répondus séparément}
|
||||
\begin{enumerate}
|
||||
\item De janvier à septembre, une quantité a augmenté de $10\,\%$. Faire un schéma pour représenter la situation puis calculer le taux d'évolution moyen mensuel.
|
||||
\item Une quantité augmente de $10\,\%$ par ans. En 2020, elle est de 120\euro. Quelle était sa valeur en 2019? Faire un schéma pour représenter la situation.
|
||||
\item Déterminer l'équation de la droite \\
|
||||
\begin{tikzpicture}[xscale=0.8, yscale=0.5]
|
||||
\tkzInit[xmin=-5,xmax=5,xstep=1,
|
||||
ymin=-5,ymax=5,ystep=1]
|
||||
\tkzGrid
|
||||
\tkzAxeXY
|
||||
\tkzFct[domain=-5:5,color=red,very thick]%
|
||||
{2.0*\x -3};
|
||||
\end{tikzpicture}
|
||||
\item Résoudre l'équation $6 \times 0.69^x = 39$
|
||||
\end{enumerate}
|
||||
\end{exercise}
|
||||
|
||||
\begin{solution}
|
||||
\begin{enumerate}
|
||||
\item On veut partager cette évolution en 8 évolutions.
|
||||
\[
|
||||
\left(1 + \frac{10}{100}\right)^{\frac{1}{8}} = 1.012
|
||||
\]
|
||||
Donc le taux d'évolution moyen est
|
||||
\[
|
||||
t_m = 1.012 - 1 = 0.01200000000000001
|
||||
\]
|
||||
\item Coefficient multiplicateur pour revenir en arrière
|
||||
\[
|
||||
CM = (1 + \frac{10}{100})^{-1} = 0.9091
|
||||
\]
|
||||
On en déduit la quantité en 2019
|
||||
\[
|
||||
120 * 0.9091 = 109.092
|
||||
\]
|
||||
\item L'équation de la droite est
|
||||
\[
|
||||
y = 2.0 x -3
|
||||
\]
|
||||
\item Il faut penser à faire la division à par $6$ avant d'utiliser le log car sinon, on ne peut pas utiliser la formule $\log(a^n) = n\times \log(a)$.
|
||||
|
||||
\[x = \frac{\log(6.5)}{\log(0.69)}\]
|
||||
\end{enumerate}
|
||||
\end{solution}
|
||||
|
||||
\begin{exercise}[subtitle={Restaurant}]
|
||||
Un \emph{food truck}, ouvert le midi et le soir, propose deux types de formules :
|
||||
|
||||
\setlength\parindent{10mm}
|
||||
\begin{itemize}
|
||||
\item la formule \emph{Burger} ;
|
||||
\item la formule \emph{Wok}.
|
||||
\end{itemize}
|
||||
\setlength\parindent{0mm}
|
||||
|
||||
\medskip
|
||||
|
||||
Le gérant a remarqué que 43\,\% de ses ventes ont lieu le midi. Le quart des ventes du midi correspondent à la formule \emph{Burger}, alors que 68\,\% des ventes du soir correspondent à la formule \emph{Wok}.
|
||||
|
||||
Le gérant se constitue un fichier en notant, pour chaque vente, la formule choisie et le moment de cette vente (midi ou soir).
|
||||
|
||||
On prélève une fiche de façon équiprobable. On définit les quatre évènements suivants:
|
||||
|
||||
\begin{enumerate}
|
||||
\item $M$ : \og la fiche correspond à une vente du midi\fg{} ;
|
||||
\item $S$ : \og la fiche correspond à une vente du soir\fg {};
|
||||
\item $W$ : \og la fiche correspond à une formule \emph{Wok} \fg{} ;
|
||||
\item $B$ : \og la fiche correspond à une formule \emph{Burger} \fg.
|
||||
\end{enumerate}
|
||||
\setlength\parindent{0mm}
|
||||
|
||||
\medskip
|
||||
|
||||
\begin{enumerate}
|
||||
\item Recopier puis compléter l'arbre pondéré
|
||||
|
||||
\begin{center}
|
||||
\begin{tikzpicture}[sloped]
|
||||
\node {.}
|
||||
child {node {$M$}
|
||||
child {node {$W$}
|
||||
edge from parent
|
||||
node[above] {...}
|
||||
}
|
||||
child {node {$B$}
|
||||
edge from parent
|
||||
node[above] {...}
|
||||
}
|
||||
edge from parent
|
||||
node[above] {...}
|
||||
}
|
||||
child[missing] {}
|
||||
child { node {$S$}
|
||||
child {node {$W$}
|
||||
edge from parent
|
||||
node[above] {...}
|
||||
}
|
||||
child {node {$B$}
|
||||
edge from parent
|
||||
node[above] {...}
|
||||
}
|
||||
edge from parent
|
||||
node[above] {...}
|
||||
} ;
|
||||
\end{tikzpicture}
|
||||
\end{center}
|
||||
|
||||
\item Calculer la probabilité de l'évènement $M \cap W$. Interpréter ce résultat dans le contexte de l'exercice.
|
||||
\item Montrer que la probabilité que la fiche choisie corresponde à une formule \emph{Burger} est égale à $0.2899$.
|
||||
\item On a prélevé une fiche correspondant à la formule \emph{Burger}. Quelle est la probabilité, arrondie au millième, que la vente ait eu lieu le soir?
|
||||
\end{enumerate}
|
||||
\end{exercise}
|
||||
|
||||
\begin{solution}
|
||||
\begin{enumerate}
|
||||
\item
|
||||
\begin{center}
|
||||
\begin{tikzpicture}[sloped]
|
||||
\node {.}
|
||||
child {node {$M$}
|
||||
child {node {$W$}
|
||||
edge from parent
|
||||
node[above] {$0.75$}
|
||||
}
|
||||
child {node {$B$}
|
||||
edge from parent
|
||||
node[above] {$0.25$}
|
||||
}
|
||||
edge from parent
|
||||
node[above] {$0.43$}
|
||||
}
|
||||
child[missing] {}
|
||||
child { node {$S$}
|
||||
child {node {$W$}
|
||||
edge from parent
|
||||
node[above] {$0.68$}
|
||||
}
|
||||
child {node {$B$}
|
||||
edge from parent
|
||||
node[above] {$0.32$}
|
||||
}
|
||||
edge from parent
|
||||
node[above] {$0.57$}
|
||||
} ;
|
||||
\end{tikzpicture}
|
||||
\end{center}
|
||||
\item On calcule la probabilité que la vente soit un wok et ait eu lieu à midi
|
||||
\[ P(M\cap W) = P(M) \times P_M(W) = 0.43 \times 0.75 = 0.3225 \]
|
||||
\item Probabilité que la vente soit un burger.
|
||||
\[
|
||||
P(B) = P(M\cap B) + P(S\cap B) = 0.43 \times 0.75 + 0.57 \times 0.68 = 0.2899
|
||||
\]
|
||||
\item On cherche à calculer la quantité $P_B(S)$. Pour cela on utilise la formule de Bayes
|
||||
\[
|
||||
P_B(S) = \frac{P(B\cap S)}{P(B)} = \frac{P_S(B) \times P(S)}{P(B)} = \frac{0.32\times 0.57}{0.2899} = 0.6291824767161089 \approx 0.629
|
||||
\]
|
||||
\end{enumerate}
|
||||
\end{solution}
|
||||
|
||||
\begin{exercise}[subtitle={Continent plastique}]
|
||||
\textit{Les quantités évoqués dans cette exercice sont générés au hasard et sont donc complètement farfelus.}
|
||||
\medskip
|
||||
Le \og continent de plastique\fg{} est la plus grande des plaques de déchets plastiques évoluant sur les océans. Elle occupe actuellement dans l'océan Pacifique une surface dont l'aire est évaluée à plus de $1,6$ million de km$^2$, entre Hawaï et la Californie.
|
||||
|
||||
En 2017, des scientifiques ont estimé qu'il y avait $7$ millions de tonnes de déchets plastiques qui était déversé chaque année dans les océans et que cette quantité augmentait de $11\n\%$ par chaque année.
|
||||
|
||||
On modélise l'évolution de la masse de ces déchets plastiques déversée chaque année, si rien n'est fait pour la réduire, par une suite géométrique $\left(u_n\right)$. L'arrondi au centième du terme $u_n$ représente la masse de ces déchets déversée chaque année, exprimée en million de tonnes, pour l'année $(2017 + n)$.
|
||||
|
||||
\medskip
|
||||
|
||||
\begin{enumerate}
|
||||
\item Expliquer pourquoi la suite $u_n$ est géométrique?
|
||||
\item Calculer $u_1$ et $u_2$.
|
||||
\item Exprimer $u_n$ en fonction de $n$.
|
||||
\item Au début de l'année 2017, il y avait $300$ millions de tonnes de déchets plastique. Calculer la quantité totale de déchets plastiques en 2030.
|
||||
\item On souhaite déterminer en quelle année la masse totale de ces déchets plastiques aura pour la première fois augmenté de $50$\,\% par rapport à sa valeur de 2017.
|
||||
\begin{enumerate}
|
||||
\item Recopier et compléter l'algorithme ci-dessous pour que la variable $N$ contienne la réponse au problème posé.
|
||||
|
||||
\begin{center}
|
||||
\begin{tabularx}{0.4\linewidth}{|X|}\hline
|
||||
$N = 2017$\\
|
||||
$U = 7$ \\
|
||||
$S = 300 + U$ \\
|
||||
while $S < 450$: \\
|
||||
\hspace{1cm} $N = \ldots$\\
|
||||
\hspace{1cm} $U = \ldots$\\
|
||||
\hspace{1cm} $S = \ldots$\\
|
||||
\hline
|
||||
\end{tabularx}
|
||||
\end{center}
|
||||
\item Que contiennent les variables $S$, $U$ et $N$ après exécution de cet algorithme ?
|
||||
|
||||
Interpréter les résultats dans le contexte de l'exercice.
|
||||
\end{enumerate}
|
||||
\end{enumerate}
|
||||
\end{exercise}
|
||||
|
||||
\begin{solution}
|
||||
\begin{enumerate}
|
||||
\item Une augmentation de $11\,\%$ revient à multiplier la quantité par $1.11$. La suite est donc bien géométrique. Son premier terme est $u_0 = 7$ et sa raison est $q = 1.11$
|
||||
\item
|
||||
\[
|
||||
u_1 = u_0 * 1.11 = 7.7700000000000005
|
||||
\]
|
||||
\[
|
||||
u_2 = u_0 * 1.11^2 = 8.6247
|
||||
\]
|
||||
\item
|
||||
\[
|
||||
u_n = u_0 \times q^n = 7 \times 1.11^n
|
||||
\]
|
||||
\item On calcule la quantité totale déversée entre 2017 et 2030.
|
||||
\[
|
||||
\sum_{n = 0}^{13} u_n = u_0 \times \frac{1-q^{13}}{1-q} = 7 \times \frac{1 - 1.11^{13}}{1 - 1.11} = 183.48
|
||||
\]
|
||||
On en déduit la quantité totale de déchets en 2030
|
||||
\[
|
||||
300 + 183.48 = 483.48
|
||||
\]
|
||||
\item
|
||||
\begin{enumerate}
|
||||
\item ~
|
||||
\begin{center}
|
||||
\begin{tabularx}{0.4\linewidth}{|X|}\hline
|
||||
$N \gets 2017$\\
|
||||
$U \gets 7$ \\
|
||||
$S \gets 300 + U$ \\
|
||||
Tant que $S < 450$ \\
|
||||
\hspace{1cm} $N \gets N + 1$\\
|
||||
\hspace{1cm} $U \gets U * 1.11$\\
|
||||
\hspace{1cm} $S \gets S + u$\\
|
||||
Fin Tant que\\\hline
|
||||
\end{tabularx}
|
||||
\end{center}
|
||||
\item \textit{Pas de correction automatisé}
|
||||
\end{enumerate}
|
||||
\end{enumerate}
|
||||
\end{solution}
|
||||
|
||||
\end{document}
|
||||
|
||||
%%% Local Variables:
|
||||
%%% mode: latex
|
||||
%%% TeX-master: "master"
|
||||
%%% End:
|
262
TST/DS/DS_21_04_07/TST3/corr_20_210407_DS8.tex
Normal file
262
TST/DS/DS_21_04_07/TST3/corr_20_210407_DS8.tex
Normal file
@ -0,0 +1,262 @@
|
||||
\documentclass[a4paper,10pt]{article}
|
||||
\usepackage{myXsim}
|
||||
|
||||
% Title Page
|
||||
\title{DS8 \hfill ZENAGUI Yanis}
|
||||
\tribe{TST}
|
||||
\date{\hfillÀ render pour le Mercredi 7 avril}
|
||||
|
||||
\xsimsetup{
|
||||
solution/print = true
|
||||
}
|
||||
|
||||
\begin{document}
|
||||
\maketitle
|
||||
|
||||
\begin{exercise}[subtitle={Automatismes}]
|
||||
\textit{Toutes les questions de cette exercice sont indépendantes et peuvent être répondus séparément}
|
||||
\begin{enumerate}
|
||||
\item De janvier à septembre, une quantité a augmenté de $23\,\%$. Faire un schéma pour représenter la situation puis calculer le taux d'évolution moyen mensuel.
|
||||
\item Une quantité augmente de $23\,\%$ par ans. En 2020, elle est de 135\euro. Quelle était sa valeur en 2019? Faire un schéma pour représenter la situation.
|
||||
\item Déterminer l'équation de la droite \\
|
||||
\begin{tikzpicture}[xscale=0.8, yscale=0.5]
|
||||
\tkzInit[xmin=-5,xmax=5,xstep=1,
|
||||
ymin=-5,ymax=5,ystep=1]
|
||||
\tkzGrid
|
||||
\tkzAxeXY
|
||||
\tkzFct[domain=-5:5,color=red,very thick]%
|
||||
{0.5*\x -1};
|
||||
\end{tikzpicture}
|
||||
\item Résoudre l'équation $3 \times 0.55^x = 39$
|
||||
\end{enumerate}
|
||||
\end{exercise}
|
||||
|
||||
\begin{solution}
|
||||
\begin{enumerate}
|
||||
\item On veut partager cette évolution en 8 évolutions.
|
||||
\[
|
||||
\left(1 + \frac{23}{100}\right)^{\frac{1}{8}} = 1.0262
|
||||
\]
|
||||
Donc le taux d'évolution moyen est
|
||||
\[
|
||||
t_m = 1.0262 - 1 = 0.0262
|
||||
\]
|
||||
\item Coefficient multiplicateur pour revenir en arrière
|
||||
\[
|
||||
CM = (1 + \frac{23}{100})^{-1} = 0.813
|
||||
\]
|
||||
On en déduit la quantité en 2019
|
||||
\[
|
||||
135 * 0.813 = 109.755
|
||||
\]
|
||||
\item L'équation de la droite est
|
||||
\[
|
||||
y = 0.5 x -1
|
||||
\]
|
||||
\item Il faut penser à faire la division à par $3$ avant d'utiliser le log car sinon, on ne peut pas utiliser la formule $\log(a^n) = n\times \log(a)$.
|
||||
|
||||
\[x = \frac{\log(13.0)}{\log(0.55)}\]
|
||||
\end{enumerate}
|
||||
\end{solution}
|
||||
|
||||
\begin{exercise}[subtitle={Restaurant}]
|
||||
Un \emph{food truck}, ouvert le midi et le soir, propose deux types de formules :
|
||||
|
||||
\setlength\parindent{10mm}
|
||||
\begin{itemize}
|
||||
\item la formule \emph{Burger} ;
|
||||
\item la formule \emph{Wok}.
|
||||
\end{itemize}
|
||||
\setlength\parindent{0mm}
|
||||
|
||||
\medskip
|
||||
|
||||
Le gérant a remarqué que 9\,\% de ses ventes ont lieu le midi. Le quart des ventes du midi correspondent à la formule \emph{Burger}, alors que 11\,\% des ventes du soir correspondent à la formule \emph{Wok}.
|
||||
|
||||
Le gérant se constitue un fichier en notant, pour chaque vente, la formule choisie et le moment de cette vente (midi ou soir).
|
||||
|
||||
On prélève une fiche de façon équiprobable. On définit les quatre évènements suivants:
|
||||
|
||||
\begin{enumerate}
|
||||
\item $M$ : \og la fiche correspond à une vente du midi\fg{} ;
|
||||
\item $S$ : \og la fiche correspond à une vente du soir\fg {};
|
||||
\item $W$ : \og la fiche correspond à une formule \emph{Wok} \fg{} ;
|
||||
\item $B$ : \og la fiche correspond à une formule \emph{Burger} \fg.
|
||||
\end{enumerate}
|
||||
\setlength\parindent{0mm}
|
||||
|
||||
\medskip
|
||||
|
||||
\begin{enumerate}
|
||||
\item Recopier puis compléter l'arbre pondéré
|
||||
|
||||
\begin{center}
|
||||
\begin{tikzpicture}[sloped]
|
||||
\node {.}
|
||||
child {node {$M$}
|
||||
child {node {$W$}
|
||||
edge from parent
|
||||
node[above] {...}
|
||||
}
|
||||
child {node {$B$}
|
||||
edge from parent
|
||||
node[above] {...}
|
||||
}
|
||||
edge from parent
|
||||
node[above] {...}
|
||||
}
|
||||
child[missing] {}
|
||||
child { node {$S$}
|
||||
child {node {$W$}
|
||||
edge from parent
|
||||
node[above] {...}
|
||||
}
|
||||
child {node {$B$}
|
||||
edge from parent
|
||||
node[above] {...}
|
||||
}
|
||||
edge from parent
|
||||
node[above] {...}
|
||||
} ;
|
||||
\end{tikzpicture}
|
||||
\end{center}
|
||||
|
||||
\item Calculer la probabilité de l'évènement $M \cap W$. Interpréter ce résultat dans le contexte de l'exercice.
|
||||
\item Montrer que la probabilité que la fiche choisie corresponde à une formule \emph{Burger} est égale à $0.8324$.
|
||||
\item On a prélevé une fiche correspondant à la formule \emph{Burger}. Quelle est la probabilité, arrondie au millième, que la vente ait eu lieu le soir?
|
||||
\end{enumerate}
|
||||
\end{exercise}
|
||||
|
||||
\begin{solution}
|
||||
\begin{enumerate}
|
||||
\item
|
||||
\begin{center}
|
||||
\begin{tikzpicture}[sloped]
|
||||
\node {.}
|
||||
child {node {$M$}
|
||||
child {node {$W$}
|
||||
edge from parent
|
||||
node[above] {$0.75$}
|
||||
}
|
||||
child {node {$B$}
|
||||
edge from parent
|
||||
node[above] {$0.25$}
|
||||
}
|
||||
edge from parent
|
||||
node[above] {$0.09$}
|
||||
}
|
||||
child[missing] {}
|
||||
child { node {$S$}
|
||||
child {node {$W$}
|
||||
edge from parent
|
||||
node[above] {$0.11$}
|
||||
}
|
||||
child {node {$B$}
|
||||
edge from parent
|
||||
node[above] {$0.89$}
|
||||
}
|
||||
edge from parent
|
||||
node[above] {$0.91$}
|
||||
} ;
|
||||
\end{tikzpicture}
|
||||
\end{center}
|
||||
\item On calcule la probabilité que la vente soit un wok et ait eu lieu à midi
|
||||
\[ P(M\cap W) = P(M) \times P_M(W) = 0.09 \times 0.75 = 0.0675 \]
|
||||
\item Probabilité que la vente soit un burger.
|
||||
\[
|
||||
P(B) = P(M\cap B) + P(S\cap B) = 0.09 \times 0.75 + 0.91 \times 0.11 = 0.8324
|
||||
\]
|
||||
\item On cherche à calculer la quantité $P_B(S)$. Pour cela on utilise la formule de Bayes
|
||||
\[
|
||||
P_B(S) = \frac{P(B\cap S)}{P(B)} = \frac{P_S(B) \times P(S)}{P(B)} = \frac{0.89\times 0.91}{0.8324} = 0.9729697260932244 \approx 0.973
|
||||
\]
|
||||
\end{enumerate}
|
||||
\end{solution}
|
||||
|
||||
\begin{exercise}[subtitle={Continent plastique}]
|
||||
\textit{Les quantités évoqués dans cette exercice sont générés au hasard et sont donc complètement farfelus.}
|
||||
\medskip
|
||||
Le \og continent de plastique\fg{} est la plus grande des plaques de déchets plastiques évoluant sur les océans. Elle occupe actuellement dans l'océan Pacifique une surface dont l'aire est évaluée à plus de $1,6$ million de km$^2$, entre Hawaï et la Californie.
|
||||
|
||||
En 2017, des scientifiques ont estimé qu'il y avait $4$ millions de tonnes de déchets plastiques qui était déversé chaque année dans les océans et que cette quantité augmentait de $19\n\%$ par chaque année.
|
||||
|
||||
On modélise l'évolution de la masse de ces déchets plastiques déversée chaque année, si rien n'est fait pour la réduire, par une suite géométrique $\left(u_n\right)$. L'arrondi au centième du terme $u_n$ représente la masse de ces déchets déversée chaque année, exprimée en million de tonnes, pour l'année $(2017 + n)$.
|
||||
|
||||
\medskip
|
||||
|
||||
\begin{enumerate}
|
||||
\item Expliquer pourquoi la suite $u_n$ est géométrique?
|
||||
\item Calculer $u_1$ et $u_2$.
|
||||
\item Exprimer $u_n$ en fonction de $n$.
|
||||
\item Au début de l'année 2017, il y avait $300$ millions de tonnes de déchets plastique. Calculer la quantité totale de déchets plastiques en 2030.
|
||||
\item On souhaite déterminer en quelle année la masse totale de ces déchets plastiques aura pour la première fois augmenté de $50$\,\% par rapport à sa valeur de 2017.
|
||||
\begin{enumerate}
|
||||
\item Recopier et compléter l'algorithme ci-dessous pour que la variable $N$ contienne la réponse au problème posé.
|
||||
|
||||
\begin{center}
|
||||
\begin{tabularx}{0.4\linewidth}{|X|}\hline
|
||||
$N = 2017$\\
|
||||
$U = 4$ \\
|
||||
$S = 300 + U$ \\
|
||||
while $S < 450$: \\
|
||||
\hspace{1cm} $N = \ldots$\\
|
||||
\hspace{1cm} $U = \ldots$\\
|
||||
\hspace{1cm} $S = \ldots$\\
|
||||
\hline
|
||||
\end{tabularx}
|
||||
\end{center}
|
||||
\item Que contiennent les variables $S$, $U$ et $N$ après exécution de cet algorithme ?
|
||||
|
||||
Interpréter les résultats dans le contexte de l'exercice.
|
||||
\end{enumerate}
|
||||
\end{enumerate}
|
||||
\end{exercise}
|
||||
|
||||
\begin{solution}
|
||||
\begin{enumerate}
|
||||
\item Une augmentation de $19\,\%$ revient à multiplier la quantité par $1.19$. La suite est donc bien géométrique. Son premier terme est $u_0 = 4$ et sa raison est $q = 1.19$
|
||||
\item
|
||||
\[
|
||||
u_1 = u_0 * 1.19 = 4.76
|
||||
\]
|
||||
\[
|
||||
u_2 = u_0 * 1.19^2 = 5.6644
|
||||
\]
|
||||
\item
|
||||
\[
|
||||
u_n = u_0 \times q^n = 4 \times 1.19^n
|
||||
\]
|
||||
\item On calcule la quantité totale déversée entre 2017 et 2030.
|
||||
\[
|
||||
\sum_{n = 0}^{13} u_n = u_0 \times \frac{1-q^{13}}{1-q} = 4 \times \frac{1 - 1.19^{13}}{1 - 1.19} = 180.98
|
||||
\]
|
||||
On en déduit la quantité totale de déchets en 2030
|
||||
\[
|
||||
300 + 180.98 = 480.98
|
||||
\]
|
||||
\item
|
||||
\begin{enumerate}
|
||||
\item ~
|
||||
\begin{center}
|
||||
\begin{tabularx}{0.4\linewidth}{|X|}\hline
|
||||
$N \gets 2017$\\
|
||||
$U \gets 4$ \\
|
||||
$S \gets 300 + U$ \\
|
||||
Tant que $S < 450$ \\
|
||||
\hspace{1cm} $N \gets N + 1$\\
|
||||
\hspace{1cm} $U \gets U * 1.19$\\
|
||||
\hspace{1cm} $S \gets S + u$\\
|
||||
Fin Tant que\\\hline
|
||||
\end{tabularx}
|
||||
\end{center}
|
||||
\item \textit{Pas de correction automatisé}
|
||||
\end{enumerate}
|
||||
\end{enumerate}
|
||||
\end{solution}
|
||||
|
||||
\end{document}
|
||||
|
||||
%%% Local Variables:
|
||||
%%% mode: latex
|
||||
%%% TeX-master: "master"
|
||||
%%% End:
|
BIN
TST/DS/DS_21_04_07/TST3/corr_all_210407_DS8.pdf
Normal file
BIN
TST/DS/DS_21_04_07/TST3/corr_all_210407_DS8.pdf
Normal file
Binary file not shown.
@ -20,7 +20,7 @@
|
||||
\item De janvier à septembre, une quantité a augmenté de $\Var{t}\,\%$. Faire un schéma pour représenter la situation puis calculer le taux d'évolution moyen mensuel.
|
||||
%- set valeur = randint(110, 150)
|
||||
\item Une quantité augmente de $\Var{t}\,\%$ par ans. En 2020, elle est de \Var{valeur}\euro. Quelle était sa valeur en 2019? Faire un schéma pour représenter la situation.
|
||||
\item Déterminer l'équation de la droite
|
||||
\item Déterminer l'équation de la droite \\
|
||||
%- set b = randint(-4, -1)
|
||||
%- set denom = randint(2, 4)
|
||||
%- set a = -2*b/denom
|
||||
@ -192,7 +192,7 @@
|
||||
\end{solution}
|
||||
|
||||
\begin{exercise}[subtitle={Continent plastique}]
|
||||
\textit{Les quantités évoqués dans cette exercices sont générés au hasard et sont donc complètement farfelus.}
|
||||
\textit{Les quantités évoqués dans cette exercice sont générés au hasard et sont donc complètement farfelus.}
|
||||
\medskip
|
||||
%- set u0 = randint(2, 20)
|
||||
%- set t = round(0.1 + 0.2*random(), 2)
|
||||
@ -216,14 +216,14 @@
|
||||
|
||||
\begin{center}
|
||||
\begin{tabularx}{0.4\linewidth}{|X|}\hline
|
||||
$N \gets 2017$\\
|
||||
$U \gets \Var{u0}$ \\
|
||||
$S \gets 300 + U$ \\
|
||||
Tant que $S < 450$ \\
|
||||
\hspace{1cm} $N \gets \ldots$\\
|
||||
\hspace{1cm} $U \gets \ldots$\\
|
||||
\hspace{1cm} $S \gets \ldots$\\
|
||||
Fin Tant que\\\hline
|
||||
$N = 2017$\\
|
||||
$U = \Var{u0}$ \\
|
||||
$S = 300 + U$ \\
|
||||
while $S < 450$: \\
|
||||
\hspace{1cm} $N = \ldots$\\
|
||||
\hspace{1cm} $U = \ldots$\\
|
||||
\hspace{1cm} $S = \ldots$\\
|
||||
\hline
|
||||
\end{tabularx}
|
||||
\end{center}
|
||||
\item Que contiennent les variables $S$, $U$ et $N$ après exécution de cet algorithme ?
|
||||
|
Loading…
Reference in New Issue
Block a user